Study Materil 2023-24

Download as pdf or txt
Download as pdf or txt
You are on page 1of 292

अध्ययन सामग्री/STUDY MATERIAL

सत्र/ SESSION: 2023-24


कक्षा/CLASS- बारहवीीं/TWELVE(XII)
गणित/MATHEMATICS
णवषय कोड/Subject Code - 041

सुश्री मीनाक्षी जैन


णनदे शक, केवीएस ण़िएट मैसूरु
Ms. MENAXI JAIN
DIRECTOR, KVS ZIET MYSURU

समन्वित /Co-ordinated by: डि श्रीडिवासुलू , प्रडिक्षण सहायक गडणत


D SREENIVASULU
TRAINING ASSOCIATE(MATHEMATICS)

i
DIRECTOR’S MESSAGE

It is with profound delight and utmost pride that I announce the publication of our
study material for class XII (MATHEMATICS) for the session 2023-24. It’s my
firm belief that access to quality education should know no boundaries,
transcending social and economic constraints. Our collective vision is to
empower all students with the tools for success and intellectual growth.

With their steadfast dedication PGT-MATHEMATICS of Bangalore, Chennai,


Ernakulam & Hyderabad regions of Kendriya Vidyalaya Sangathan have
invested their knowledge, expertise, and passion into meticulously crafting these
study materials to complement the classroom learning experience of the students.
These materials serve as invaluable aids for self-study since they are
comprehensive, well-structured, and presented in a manner that is easy to
comprehend.

It is with pleasure that I place on record my commendation for the commitment


and dedication of the team of teachers which included the Training Associate
(MATHEMATICS) from ZIET Mysore who has been the Coordinator of this
assignment and all the concerned PGT- Mathematics subject experts from the
four feeder regions of ZIET Mysore.

Wishing you all the very best in your academic journey!

MENAXI JAIN
DIRECTOR
ZIET MYSORE

ii
CONTENT DEVELOPMENT TEAM
(PGT-MATHEMATICS)
S.NO NAME OF THE NAME OF THE KV REGION
TEACHER
1 Mr. M. SRINIVASAN KV ASHOKNAGAR
2 Ms. M. LATHA KV NAGERKOIL
3 Ms. D. MRUDULA KV AFS SULUR CHENNAI
4 Mr. P. SENTHIL KV1, TRICHY
KUMARAN
5 Ms. REMA DEVI .M KV RAMAVARMAPURAM
6 Ms. BEENA JOSEPH KV CRPF PALLIPURAM
7 Ms. SATHY.M. N K V INS
DRONACHARYA ERNAKULAM
8 Mr. M. NARASIMH K V NO.1 CPCRI
PRABHU KASARAGOD
9 Mr. V. SIMHADRI KV NO-2
SRIVIJAYANAGAR
10 Mr. E.V.L.N. VAMSI KV NO.1 TIRUPATI, HYDERABAD
KRISHNA S-1
11 Mr. G. KISHORE KV GACHIBOWLI
12 Mr. R RAVI KUMAR KV NO.1 GOLGONDA
13 Mr. V. ESWARAN KV HEBBAL
14 Ms. A. H TELMA KV AFS YELAHANKA
FERNANDEZ BENGALURU
15 Mr. JASEER K.P. KV IISC, BENGALURU
16 Mr. R. MUTHUSWAMY KV NO.2 JALAHALLI

iii
INDEX

S.NO CHAPTER/CONTENT Page Number

1 Curriculum 1–3

2 Relations and Functions 4 – 24

3 Inverse Trigonometric Functions 25 – 33

4 Matrices 34 – 57

5 Determinants 58 - 77

6 Continuity And Differentiability 78 - 102

7 Application of Derivatives 103 – 127

8 Integrals 128 – 137

9 Application of Integrals 138 – 148

10 Differential Equations 149 – 174

11 Vectors 175 – 189

12 Three Dimentional Geometry 190 - 204

13 Linear Programming 205 – 222

14 Probability 223 – 239

15 Sample Question Paper – 01 240 - 252

16 Sample Question Paper -02 253 - 266

17 Sample Question Paper - 03 267 - 288

PLEASE VISIT ZIET MYSURU YOUTUBE CHANNEL


FOR VIDEO LESSONS
https://www.youtube.com/channel/UCFcMLspE4JTudI9T16JwqZQ

iv
SYLLABUS
MATHEMATICS (XII)
(Code No. 041)
Session – 2023-24

No. Units Marks


I. Relations and Functions 08
II. Algebra 10
III. Calculus 35
IV. Vectors and Three - Dimensional Geometry 14
V. Linear Programming 05
VI. Probability 08
TOTAL 80
Internal Assessment 20

Unit-I: Relations and Functions

1. Relations and Functions


Types of relations: reflexive, symmetric, transitive and equivalence
relations. One to one and onto functions.
2. Inverse Trigonometric Functions
Definition, range, domain, principal value branch. Graphs of inverse
trigonometric functions.

Unit-II: Algebra

1. Matrices
Concept, notation, order, equality, types of matrices, zero and identity
matrix, transpose of a matrix, symmetric and skew symmetric matrices.
Operations on matrices: Addition and multiplication and multiplication
with a scalar. Simple properties of addition, multiplication and scalar
multiplication. Non- commutativity of multiplication of matrices and
existence of non-zero matrices whose product is the zero matrix (restrict to
square matrices of order 2). Invertible matrices and proof of the uniqueness
of inverse, if it exists; (Here all matrices will have real entries).
2. Determinants
Determinant of a square matrix (up to 3 × 3 matrices), minors, co-factors
and applications of determinants in finding the area of a triangle. Adjoint
and inverse of a square matrix. Consistency, inconsistency and number of
solutions of system of linear equations by examples, solving system of
linear equations in two or three variables (having unique solution) using
inverse of a matrix.

1
Unit-III: Calculus

1. Continuity and Differentiability

Continuity and differentiability, chain rule, derivative of inverse trigonometric


functions, 𝑙𝑖𝑘𝑒 𝑥, cos −1 𝑥 and tan−1 𝑥, derivative of implicit functions. Concept
of exponential and logarithmic functions.

Derivatives of logarithmic and exponential functions. Logarithmic


differentiation, derivative of functions expressed in parametric forms.
Second order derivatives.
2. Applications of Derivatives
Applications of derivatives: rate of change of quantities,
increasing/decreasing functions, maxima and minima (first derivative test
motivated geometrically and second derivative test given as a provable
tool). Simple problems (that illustrate basic principles and understanding
of the subject as well as real-life situations).
3. Integrals
Integration as inverse process of differentiation. Integration of a variety of
functions by substitution, by partial fractions and by parts, Evaluation of
simple integrals of the following types and problems based on them.
𝑑𝑥 𝑑𝑥 𝑑𝑥 𝑑𝑥
∫ 2 2 , ∫ 2 2 , ∫ 2 2 , ∫ 2 2 , ∫ √𝑥 2 ± 𝑎2 𝑑𝑥 , ∫ √𝑎2 − 𝑥 2 𝑑𝑥,
𝑥 ±𝑎 𝑎 −𝑥 √𝑥 ±𝑎 √𝑎 −𝑥
𝑝𝑥+𝑞 𝑝𝑥+𝑞
∫ √𝑎𝑥 2 + 𝑏𝑥 + 𝑐 𝑑𝑥 ∫ 𝑎𝑥 2+𝑏𝑥+𝑐 𝑑𝑥, ∫ √𝑎𝑥 2+𝑏𝑥+𝑐 𝑑𝑥

Fundamental Theorem of Calculus (without proof). Basic properties of


definite integrals and evaluation of definite integrals.
4. Applications of the Integrals
Applications in finding the area under simple curves, especially lines,
circles/ parabolas/ellipses (in standard form only)
5. Differential Equations
Definition, order and degree, general and particular solutions of a
differential equation. Solution of differential equations by method of
separation of variables, solutions of homogeneous differential equations of
first order and first degree. Solutions of linear differential equation of the
type:
𝑑𝑦
+py=q, where p and q are functions of x or constants.
𝑑𝑥

𝑑𝑥
+ px=q, where p and q are functions of y or constants.
𝑑𝑦

2
Unit-IV: Vectors and Three-Dimensional Geometry

1. Vectors
Vectors and scalars, magnitude and direction of a vector. Direction cosines
and direction ratios of a vector. Types of vectors (equal, unit, zero, parallel
and collinear vectors), position vector of a point, negative of a vector,
components of a vector, addition of vectors, multiplication of a vector by
a scalar, position vector of a point dividing a line segment in a given ratio.
Definition, Geometrical Interpretation, properties and application of scalar
(dot) product of vectors, vector (cross) product of vectors.

2. Three - dimensional Geometry


Direction cosines and direction ratios of a line joining two points. Cartesian
equation and vector equation of a line, skew lines, shortest distance
between two lines. Angle between two lines.

Unit-V: Linear Programming

1. Linear Programming
Introduction, related terminology such as constraints, objective function,
optimization, graphical method of solution for problems in two variables,
feasible and infeasible regions (bounded or unbounded), feasible and
infeasible solutions, optimal feasible solutions (up to three non-trivial
constraints).

Unit-VI: Probability

1. Probability
Conditional probability, multiplication theorem on probability,
independent events, total probability, Bayes’ theorem, Random variable
and its probability distribution, mean of random variable.

3
CHAPTER : RELATIONS AND FUNCTIONS
SYLLABUS: Types of relations: Reflexive, Symmetric, transitive and equivalence relations.

One to one and onto functions.

Definitions and Formulae:

➢ Types of Relations:

• Empty Relation: A relation 𝑅 in a set 𝐴 is called empty relation, if no element of 𝐴


is related to any element of 𝐴, i.e., 𝑅 = ∅ ⊂ 𝐴 × 𝐴.

• Universal Relation: A relation 𝑅 in a set 𝐴 is called universal relation, if each


element of 𝐴 is related to every element of 𝐴, i.e. , 𝑅 = 𝐴 × 𝐴.

• Trivial Relations: Both the empty relation and the universal relation are sometimes
called trivial relations.

• A relation R in a set A is called

a) Reflexive, if (𝑥, 𝑥) ∈ 𝑅 𝑓𝑜𝑟 𝑒𝑣𝑒𝑟𝑦 𝑥 ∈ 𝐴

b) Symmetric, 𝑖𝑓 (𝑥, 𝑦) ∈ 𝑅 𝑖𝑚𝑝𝑙𝑖𝑒𝑠 𝑡ℎ𝑎𝑡 (𝑦, 𝑥) ∈ 𝑅 𝑓𝑜𝑟 𝑎𝑙𝑙 𝑥, 𝑦 ∈ 𝐴

c) Transitive, 𝑖𝑓 (𝑥, 𝑦) ∈ 𝑅 𝑎𝑛𝑑 (𝑦, 𝑧) ∈ 𝑅 implies that (𝑥, 𝑧) ∈ 𝑅 for all

𝑥, 𝑦, 𝑧 ∈ 𝐴

• If A is a finite set with n elements,then


2
i) Total number of relations in A=2𝑛 .
2 −𝑛
ii) Total number of reflexive relations in A = 2𝑛 .
𝑛(𝑛+1)
iii) Total number of symmetric relations in A = 2 2 .

• Equivalence Relation:𝐴 relation 𝑅 in a set 𝐴 is said to be an equivalence relation if


𝑅 is reflexive, symmetric and transitive.

➢ Equivalence Class: Let 𝑅 be an equivalence relation on a non-empty set 𝐴 and 𝑎 ∈ 𝐴.


Then the set of all those elements of 𝐴 which are related to 𝑎, is called the
equivalence class determined by 𝑎 and is denoted by [𝑎].i.e[𝑎] = {𝑥 ∈ 𝐴 ∶ (𝑥, 𝑎) ∈ 𝑅}

➢ Types of Functions:

• One-One (Injective) Function: A function 𝑓 ∶ 𝑋 → 𝑌 is defined to be one-one


(or injective), if the images of distinct elements of X under f are distinct, i.e., for
every 𝑥1 , 𝑥2 ∈ 𝑋, 𝑓(𝑥1 ) = 𝑓( 𝑥2 ) 𝑖𝑚𝑝𝑙𝑖𝑒𝑠 𝑥1 = 𝑥2 . Otherwise, f is called many-
one.

4
• Onto (Surjective) Function: A function 𝑓 ∶ 𝑋 → 𝑌 is said to be onto (or
surjective), if every element of 𝑌 is the image of some element of 𝑋 under f. i.e., for
every 𝑦 ∈ 𝑌, there exists an element 𝑥 𝑖𝑛 𝑋 such that 𝑓(𝑥) = 𝑦.

NOTE: 𝒇: 𝑿 → 𝒀 is onto if and only if Range of 𝒇 = 𝑪𝒐𝒅𝒐𝒎𝒂𝒊𝒏.

• Bijective Function: A function 𝑓 ∶ 𝑋 → 𝑌 is said to be bijective, if 𝑓 is both one-


one and onto.

• If A and B are two finite sets having 𝑚 𝑎𝑛𝑑 𝑛 elements respectively, then

i) Number of functions from A to B is 𝑛𝑚 .

𝑛𝑝𝑚 , 𝑖𝑓 𝑚 ≤ 𝑛
ii) Number of one-one functions from A to 𝐵 = {
0 , 𝑖𝑓 𝑚 > 𝑛

iii) If the function is bijective i. e both one-one and onto then 𝑚 = 𝑛.

MULTIPLE CHOICE QUESTIONS


Q.No QUESTIONS AND SOLUTIONS
1 If 𝐴 = {5,6,7} and
let R = {5,5), (6,6)), (7,7), (5,6), (6,5), (6,7), (7,6)}. Then R is
a) Reflexive, symmetric but not Transitive
b) Symmetric, transitive but not reflexive
c) Reflexive, Transitive but not symmetric
d) an equivalence relation

Answer: A
(5,6) ∈ R and (6,7) ∈ R but (5,7) does not belong to R
2 Let R be a relation defined on Z as follows:
(𝑎, 𝑏) ∈ 𝑅 ⟺ 𝑎2 + 𝑏 2 = 25. 𝑇ℎ𝑒𝑛 𝐷𝑜𝑚𝑎𝑖𝑛 𝑜𝑓 𝑅 𝑖𝑠
a) {3,4,5} b) {0,3,4,5}
c) {0,±3,±4,±5} d) None of these

Answer: C R={( 0, ,±5), (,±5,0),( ±3, ,±4),( ,±4, ±3)}


Domain of R is the set of all first elements of R.
3 The maximum number of equivalence relations on the set 𝐴 = {1 , 2 , 3} is
a) 1 b) 2 c) 3 d)5

Answer: D
Possible equivalence relations are
R1 = { (1,1) , (2,2) , (3,3) }
R2 = { (1,1) , (2,2) , (3,3), (1,2) ,(2,1)}
R3 = { (1,1) , (2,2) , (3,3), (1,3), (3,1)}
R4 = { (1,1) , (2,2) , (3,3), (2,3),(3,2) }
R5 = A×A
4 Consider the set 𝐴 = {1, 2}. The relation on A which is symmetric but neither
transitive nor reflexive is
a) {(1,1) (2,2)} b) { }
c) {(1,2)} d) { (1,2) (2,1) }

Answer: D

5
R is not reflexive since (1,1) and (2,2) are not there in R
R is not transitive since (1,2) and ( 2,1) belong to R but (1,1) does not belong to
R.
5 If 𝐴 = {𝑑, e, f} and
let R = {(d, d), (d, e), (e, d), (𝑒, 𝑒)}. Then R is
a) Reflexive, symmetric but not Transitive
b) Symmetric, transitive but not reflexive
c) Reflexive, Transitive but not symmetric
d) an equivalence relation

Answer: B
R is not reflexive because ( 𝑓, 𝑓) is not present in R
6 Let R be a reflexive relation on a finite set A having n elements and let there be
m,minimum number of ordered pairs in R, then
a) m < n b) m > n
c) m = n d)none of these

Answer: C
A relation on a set A is reflexive if every element of A is related to itself
i.e. (𝑎, 𝑎) ∈ 𝑅, 𝑓𝑜𝑟 𝑎𝑙𝑙 𝑎 ∈ 𝑅
7 The number of elements in set A is 3.The number of possible relations that can be
defined on A is
a) 8 b) 4 c) 64 d) 512

Answer: D
2
The number of possible relations on a set having n elements is 2𝑛 as every
relation is a subset of A× A.
8 The number of elements in Set A is 3.The number of possible reflexive relations
that can be defined in A is
a) 64 b) 8 c) 512 d) 4

Answer: A
If a set has A has n elements then the number of possible reflexive relations on A
is 2𝑛(𝑛−1) .
9 The number of elements in set P is 4.The number of possible symmetric relations
that can be defined on P is
a) 16 b) 32 c) 512 d) 1024

Answer: D
If a set has A has n elements then the number of possible symmetric relations on
𝑛(𝑛+1)
A is 2 2
10 Let R be a relation on the set N of natural numbers defined by 𝑎𝑅𝑏 if and only if
𝑎 𝑑𝑖𝑣𝑖𝑑𝑒𝑠 𝑏.Then R is
a) Reflexive, symmetric but not Transitive
b) Symmetric, transitive but not reflexive
c) Reflexive, Transitive but not symmetric
d) an equivalence relation
Answer C
R is reflexive, since every natural number divides itself.
If a divides b and b divides c then a divides c
So R is transitive
𝑎 𝑑𝑖𝑣𝑖𝑑𝑒𝑠 𝑏 need not imply that 𝑏 𝑑𝑖𝑣𝑖𝑑𝑒𝑠 𝑎.
So R is not symmetric.

6
CHAPTER VIDEO LINK FOR MCQs SCAN QR CODE FOR
VIDEO

RELATIONS AND FUNCTIONS https://youtu.be/hbZIyGzmI0w

EXERCISE
1. 1
Let 𝑓: 𝑅 → 𝑅 𝑏𝑒 𝑑𝑒𝑓𝑖𝑛𝑒𝑑 𝑏𝑦 𝑓(𝑥) = 𝑥 ∀ 𝑥 ∈ 𝑅. 𝑇ℎ𝑒𝑛 𝑓 𝑖𝑠
a) One-one
b) Onto
c) Bijective
d) 𝑓 is not defined

Answer: d
2. Set A has 4 elements and set B has 5 elements. Then the number of bijective
mappings from A to B is
a) 120
b) 20
c) 0
d) 625

Answer: c
3. Set A has 3 elements and set B has 4 elements. Then the number of injective
mappings from A to B is
a) 144
b) 12
c) 24
d) 64

Answer: c
4. The function 𝑓: [ 𝜋, 2𝜋] → 𝑅 defined by 𝑓(𝑥) = 𝑐𝑜𝑠𝑥 is
a) one – one but not onto
b) onto but not one – one
c) many – one function
d) bijective function

Answer: d
5. Let
𝑓: 𝑅 → 𝑅 𝑏𝑒 𝑑𝑒𝑓𝑖𝑛𝑒𝑑 𝑏𝑦 𝑓(𝑥) = 𝑥 3 + 4,then 𝑓 is
a) Injective
b) Surjective
c) Bijective
d) None of these
Answer: c

7
6. Let 𝐴 = {1,2,3}, 𝐵 = {4,5,6,7} and let 𝑓 = {(1,4), (2,5), (3,6)} be a function from
A to B. Based on the given information 𝑓 is best defined as
a) Surjective function
b) Injective function
c) Bijective function
d) Function

Answer: b
7. Let 𝐴 = {1,2,3, . . . , 𝑛}𝑎𝑛𝑑 𝐵 = {𝑝. 𝑞}.Then the number of onto functions from A to
B is
a) 2n
b) 2n−2
c) 2n−1
d) None of these

Answer: b

ASSERTION AND REASONING QUESTIONS


In the following question a statement of Assertion (A) is followed by a statement of
Reason(R).Pick the correct option:
A) Both A and R are true and R is the correct explanation of A.
B) Both A and R are true but R is NOT the correct explanation of A.
C) A is true but R is false.
D) A is false but R is true.
1. Assertion (A): If n (A) =p and n (B) = q then the number of relations from A to B
is 2pq.
Reason(R): A relation from A to B is a subset of 𝐴 × 𝐵.

Answer. A
Solution: Every relation from set A to set B is a subset of 𝐴 × 𝐵.
So R is true
The number of elements in 𝐴 × 𝐵 is 𝑝 × 𝑞.So number of subsets of 𝐴 × 𝐵.i.e no
of relations from A to B is 2pq .
So A is true.
2. Assertion (A): If n (A) =m, then the number of reflexive relations on A is m
Reason(R) : A relation R on the set A is reflexive if (a, a) ∈ 𝑅, ∀a∈ A.

Answer: D
Solution: A relation R is reflexive on the set A iff (a,a) ∈ R ∀ a ∈ A.
So R is true.
2
𝑛(𝐴) = 𝑚 then the number of reflexive relations on A is 2𝑚 −𝑚 .
So A is false.
3. Assertion (A): Domain and Range of a relation 𝑅 = {(𝑥, 𝑦): 𝑥 − 2𝑦 = 0}defined
on the set A = {1,2,3,4} are respectively {1,2,3,4}and {2,4,6,8}
Reason(R): Domain and Range of a relation R are respectively the sets
{𝑎: 𝑎 ∈ 𝐴 𝑎𝑛𝑑 (𝑎, 𝑏) ∈ 𝑅. } 𝑎𝑛𝑑 {𝑏: 𝑏 ∈ 𝐴 𝑎𝑛𝑑 (𝑎, 𝑏) ∈ 𝑅}

Answer: D
Solution: Domain of a relation R is {𝑥: 𝑥 ∈ 𝐴 𝑎𝑛𝑑 (𝑥, 𝑦) ∈ 𝑅. }.
Range of a relation R is {𝑦: 𝑦 ∈ 𝐴 𝑎𝑛𝑑 (𝑥, 𝑦) ∈ 𝑅. }.
So R is true.

8
𝑅 = {(2,1), (4,2)}
So A is false
4. Assertion (A): A relation 𝑅 = { (1,1), (1,2), (2,2), (2,3)(3,3)} defined on the set
𝐴 = {1,2,3} is reflexive.
Reason(R): A relation R on the set A is reflexive if (𝑎, 𝑎) ∈ 𝑅, ∀ 𝑎 ∈ 𝐴

Answer: A
Solution:A relation R on the set A is reflexive if (𝑥, 𝑥) ∈ 𝑅, ∀ 𝑥 ∈ 𝐴
So R is true
For , ∀ 𝑥 ∈ 𝐴 (𝑥, 𝑥) ∈ 𝑅 so R is reflexive and thus A is true.
Therefore answer is A.
5. Assertion (A): A relation 𝑅 = { (1,1), (1,2), (2,2), (2,3)(3,3)} defined on the set
𝐴 = {1,2,3} is symmetric
Reason(R): A relation R on the set A is symmetric if (𝑎, 𝑏) ∈ 𝑅 ⟹ (𝑏, 𝑎) ∈ 𝑅

Answer:D
Solution:A relation R on the set A is symmetric if (𝑥, 𝑦) ∈ 𝑅 ⟹ (𝑦, 𝑥) ∈ 𝑅
So,R is true
(1,2) ∈ 𝑅 𝑏𝑢𝑡 (2,1) does not belong to R so R is not symmetric .
So A is false
6. Assertion (A): A relation 𝑅 = { (1,1), (1,3), (1, 5), (3,1)(3,3), (3,5)} defined on
the set 𝐴 = {1,3,5} is transitive.
Reason(R): A relation R on the set A symmetric if (a, b) ∈ R and (a, c) ∈ R ⟹
(a, c) ∈ R

Answer:C
Solution: A relation R on the set A transitive iff (a, b) ∈ R and (a, c) ∈ R ⟹
(a, c) ∈ R.
So R is false
As the condition of transitivity is satisfied, so A is true.
7. Assertion (A): 𝐴 = {1,2,3}, 𝐵 = {4,5,6,7}, 𝑓 = {(1,4), (2,5), (3,6)} is a function
from A to B.Then 𝑓 𝑖𝑠 𝑜𝑛𝑒 − 𝑜𝑛𝑒
Reason(R): A function 𝑓 𝑖𝑠 𝑜𝑛𝑒 – 𝑜𝑛𝑒 if distinct elements of A have distinct
images in B.

Answer:A
Solution: A function f is one –one if distinct elements of A have distinct images in
B.
So R is true
As distinct elements of Set A have distinct images in the set B.
So, A is true
Thus A and R are true and R is the correct explanation of A.

8. Assertion (A): Consider the function 𝑓: 𝑅 → 𝑅 defined by 𝑓(𝑥) = 𝑥 3. Then 𝑓 is


one-one
Reason(R): Every polynomial function is one-one
Answer: C
Solution: Every polynomial function is not one-one as 𝑓(𝑥) = 𝑥2 is not one one.
So R is false.
A function f is one-one if distinct elements have distinct images
So A is true
Thus A is true but R is false.

9
9. Assertion (A): If 𝑋 = { 0, 1, 2 } and the function 𝑓: 𝑋 → 𝑌 defined by
𝑓(𝑥) = 𝑥 2−2 is surjection then 𝑌 = { −2, −1, 0,2 }
Reason(R): If 𝑓: 𝑋 → 𝑌 is surjective if for all 𝑦 ∈ 𝑌 there exists 𝑥 ∈ 𝑋 such thtat
𝑦 = 𝑓(𝑥)
Answer: D
Solution: A function is surjective or onto if 𝑟𝑎𝑛𝑔𝑒 = 𝑐𝑜 − 𝑑𝑜𝑚𝑎𝑖𝑛,i.e 𝑓: 𝑋 → 𝑌 is
surjective if for all 𝑦 ∈ 𝑌 there exists 𝑥 ∈ 𝑋 such thtat 𝑦 = 𝑓(𝑥)
So R is true.
There is no 𝑥 𝑖𝑛 𝑋 such that 𝑓(𝑥) = 0, so range of 𝑓 is not equal to the codomain,
i.e 𝑓 is not surjective
So, A is false.
Thus A is false but R is true.
10. Assertion (A): A, B are two sets such that 𝑛(𝐴) = 𝑚 𝑎𝑛𝑑 𝑛(𝐵) = 𝑛. The number
of one-one functions from A to B is npm ,if 𝑛 ≥ 𝑚
Reason(R): A function 𝑓 is one –one if distinct elements of A have distinct images
in B
Answer: A
Solution:A function f is one –one if distinct elements of A have distinct images in
B.
So R is true.
For a function from set A to B is one-one iff 𝑛(𝐴) ≤ 𝑛(𝐵)
So A is true.

EXERCISE

1 Assertion (A): A function 𝑓: 𝐴 → 𝐵, cannot be an onto function if 𝑛 (𝐴) < 𝑛 (𝐵).


Reason(R): A function 𝑓 is onto if every element of co-domain has at least one pre-
image in the domain

Answer:A
𝑥
2 Assertion (A): Consider the function 𝑓: 𝑅 → 𝑅 defined by 𝑓(𝑥) = .Then 𝑓 is
𝑥 2 +1
one – one
Reason(R): f(4)=4/17 and f(1/4)=4/17

Answer: D
3 Assertion (A): 𝑛(𝐴) = 5, 𝑛(𝐵) = 5 and 𝑓 ∶ 𝐴 → 𝐵 is one-one then f is bijection
Reason(R): If 𝑛(𝐴) = 𝑛(𝐵) then every one-one function from A to B is onto

Answer: A
4 Assertion (A): Set A has 4 elements and set B has 5 elements. Then the number of
bijective mappings from A to B is 54
Reason(R): A mapping from A to B cannot be bijective if n(A) is not equal to n(B).

Answer: D
5 Assertion (A):The identity relation on a set A is an equivalence relation.
Reason (R): The Universal relation on a set A is an equivalence relation.

Answer: B

10
2 MARKS QUESTIONS

1. The relation R defined by (𝑎, 𝑏)𝑅(𝑐, 𝑑) ⇒ 𝑎 + 𝑑 = 𝑏 + 𝑐 on the A×A. Where


𝐴 = {1,2,3, … . . ,10} is an equivalence relation. Find the equivalence class of the
element (3,4).
Solution: Let (3,4) R (a,b) on 𝐴 × 𝐴 where 𝐴 = {1,2,3, … … .10}
⇒3 + 𝑏 = 4 + 𝑎 → 𝑏 − 𝑎 = 1
[(3,4)]𝑅 = {(1,2). (2,3), (3,4), (4,5), (6,7), (7,8), (8,9), (9,10)}
2. n+1
, if n is odd
Let 𝑓: 𝑁 → 𝑁 be defined by 𝑓(𝑛) = { n2 for all n ∈ N. Is the function 𝑓
, if n is even
2
one-one or not ? Justify your answer.
Solution: Given function is not one-one, because 1 and 2 have the same image.
1+1 2
𝑓(1) = 2 = 1 , 𝑓(2) = 2 = 1

3. Consider f : 𝑅+ → [−9, ∞) given by f(x) = 5x2 + 6x – 9. Show that 𝑓 is one-one.


Solution: One-one:
Let 𝑓(𝑥) = 𝑓(𝑦) ⇒ 5𝑥 2 + 6𝑥 − 9 = 5𝑦 2 + 6𝑦 − 9
⇒ 5(𝑥 − 𝑦)(𝑥 + 𝑦) + 6(𝑥 − 𝑦) = 0
⇒ (𝑥 − 𝑦)(5𝑥 + 5𝑦 + 6) = 0
⇒ 𝑥 = 𝑦. 𝑆𝑖𝑛𝑐𝑒 5𝑥 + 5𝑦 + 6 ≠ 0 𝑓𝑜𝑟 𝑎𝑙𝑙 𝑥, 𝑦 ∈ 𝑅+
𝐺𝑖𝑣𝑒𝑛 𝑓𝑢𝑛𝑐𝑡𝑖𝑜𝑛 𝑖𝑠 𝑜𝑛𝑒 − 𝑜𝑛𝑒.
4. Let R be the relation defined in the set 𝐴 = {1, 2, 3, 4,5} by
𝑅 = {(𝑥, 𝑦): 𝑥, 𝑦 𝜖 𝐴, 𝑥 𝑎𝑛𝑑 𝑦 𝑎𝑟𝑒 𝑒𝑖𝑡ℎ𝑒𝑟 𝑏𝑜𝑡ℎ 𝑜𝑑𝑑 𝑜𝑟 𝑏𝑜𝑡ℎ 𝑒𝑣𝑒𝑛}. Find R
in Roster form.
Solution:
𝑅 = {(1,1), (1,3), (1,5), (3,1), (3,3), (3,5), (5,1), (5,3), (5,5), (2,2), (2,4), (4,2), (4,4)}
5. Check whether the following relation 𝑅 = {(𝑎, 𝑏): 𝑎 ≤ 𝑏} defined on set of real
numbers are reflexive and symmetric or not.
Solution: for each 𝑎 ∈ 𝑅, 𝑎 ≤ 𝑎 is true. Given relation is reflexive.
(2,3)∈ 𝑅 𝑏𝑢𝑡 (3,2) ∉ 𝑅 thus,for each (a,b)∈ 𝑅 ⇏ (𝑏, 𝑎) ∈ 𝑅 Hence given relation is
not symmetric.

6. Prove that the greatest integer function 𝑓: 𝑅 → 𝑅, given by 𝑓(𝑥) = [𝑥], is neither
one-one nor onto.
Solution: 𝑓(1.1) = 1, 𝑓(1.3) = 1, 𝑏𝑢𝑡 1.1 ≠ 1.3, ∴ 𝑓 𝑖𝑠 𝑛𝑜𝑡 𝑜𝑛𝑒 − 𝑜𝑛𝑒.
Range is set of integers only whereas codomain is set of real numbers.
Range ≠ codomain , ∴ 𝑓 is not onto

7. x−2
Let A = R − {1}. If 𝑓: 𝐴 → 𝐴 is a mapping defined by f(x) = x−1, show that f is one-
one.
Solution: One-One:
𝑥−2 𝑦−2
Let 𝑓(𝑥) = 𝑓(𝑦) ⇒ 𝑥−1 = 𝑦−1
(𝑥 − 2)(𝑦 − 1) = (𝑦 − 2)(𝑥 − 1)
⇒ 𝑥𝑦 − 𝑥 − 2𝑦 + 2 = 𝑥𝑦 − 2𝑥 − 𝑦 + 2
⟹𝑥−𝑦 =0
⟹𝑥=𝑦
𝐺𝑖𝑣𝑒𝑛 𝑓𝑢𝑛𝑐𝑡𝑖𝑜𝑛 𝑖𝑠 𝑜𝑛𝑒 − 𝑜𝑛𝑒.
8. x−2
Let A = R − {1}. If 𝑓: 𝐴 → 𝐴 is a mapping defined by f(x) = x−1, show that 𝑓 is onto.
𝑥−2
Solution: 𝑂𝑛𝑡𝑜: Let 𝑥−1 = 𝑦 ⟹ 𝑥 − 2 = 𝑦(𝑥 − 1)

11
⟹ 𝑥 − 2 = 𝑥𝑦 − 𝑦
𝑥(1 − 𝑦) = 2 − 𝑦
2−𝑦
𝑥=
1−𝑦
2−𝑦
𝑓𝑜𝑟 𝑒𝑎𝑐ℎ 𝑦 ∈ 𝐴, 𝑡ℎ𝑒𝑟𝑒 𝑒𝑥𝑖𝑠𝑡 𝑥 = ∈ 𝐴 𝑠𝑢𝑐ℎ 𝑡ℎ𝑎𝑡 𝑓(𝑥) = 𝑦.
1−𝑦
𝐺𝑖𝑣𝑒𝑛 𝑓𝑢𝑛𝑐𝑡𝑖𝑜𝑛 𝑖𝑠 𝑜𝑛𝑡𝑜.
9. A function 𝑓: 𝐴 → 𝐵 defined as 𝑓(𝑥) = 2𝑥, is both one-one and onto.If
𝐴 = {1,2,3,4},then find the set B.
Solution: 𝑓 = {(1,2), (2,4), (3,6), (4,8)}.
Range of 𝑓 = {2,4,6,8}.
As function is onto 𝑟𝑎𝑛𝑔𝑒 = 𝑐𝑜𝑑𝑜𝑚𝑎𝑖𝑛.
So 𝐵 = {2,4,6,8}.
10. Let L be the set of all lines in a plane and R be the relation in L defined as
𝑅 = {(𝐿1 , 𝐿2 ): 𝐿1 is perpendicular to 𝐿2 }. Is the relation R transitive? Justify your
answer.
Solution: R is not transitive
Let (𝐿1 , 𝐿2 ) ∈ 𝑅 𝑎𝑛𝑑 (𝐿2 , 𝐿3 ) ∈ 𝑅
⇒ 𝐿1 𝑖𝑠 𝑝𝑒𝑟𝑝𝑒𝑛𝑑𝑖𝑐𝑢𝑙𝑎𝑟 𝑡𝑜 𝐿2 𝑎𝑛𝑑 𝐿2 𝑖𝑠 𝑝𝑒𝑟𝑝𝑒𝑛𝑑𝑖𝑐𝑢𝑙𝑎𝑟 𝑡𝑜 𝐿3
⇒ 𝐿1 𝑖𝑠 𝑝𝑎𝑟𝑎𝑙𝑙𝑒𝑙 𝑡𝑜 𝐿3 ⇒ 𝐿1 𝑖𝑠 𝑛𝑜𝑡 𝑝𝑒𝑟𝑝𝑒𝑛𝑑𝑖𝑐𝑢𝑙𝑎𝑟 𝑡𝑜 𝐿3
⇒ (𝐿1 , 𝐿3 ) ∉ 𝑅
Hence, R is not transitive.

EXERCISE

1. Is the relation R = {(a, b): a≤ 𝑏 2 } defined on set of real numbers transitive? Justify
your answer.

Answer: No, it is not transitive


(5,3) ∈ R and (3,2) ∈ R but (5,2) ∉ R.
2. Determine whether the relation R defined on the set of all real numbers as
R = {(a, b): a, b and a – b + √3 ∈ S, where S is the set of all irrational numbers}, is
symmetric.

Answer: (4√3, 3√3) ∈ R but (3√3, 4√3) ∉ R.


So, R is not symmetric.
3. Is the relation R defined on the set of all real numbers as
𝑅 = {(𝑎, 𝑏): 𝑎 > 𝑏 }, reflexive,symmetric and trasitive? Justify your answer.

Answer: (𝑎, 𝑎) ∉ 𝑅 .So, R is not reflexive.


Let (𝑎, 𝑏) ∈ 𝑅 ⟹ 𝑎 > 𝑏, 𝑏𝑢𝑡 𝑏 𝑖𝑠 𝑛𝑜𝑡 𝑔𝑟𝑒𝑎𝑡𝑒𝑟 𝑡ℎ𝑎𝑛 𝑎 ⇒ (𝑏, 𝑎) ∉ 𝑅
Therefore R is not symmetric.
𝑎 > 𝑏 𝑎𝑛𝑑 𝑏 > 𝑐 ⟹ 𝑎 > 𝑐
Therefore R is transitive.
4. Show that the relation R defined on the set A of all triangles in a plane as
𝑅 = {(𝑇1 , 𝑇2 ): 𝑇1 𝑖𝑠 𝑠𝑖𝑚𝑖𝑙𝑎𝑟 𝑡𝑜 𝑇2 } is an equivalence relation.
5. Show that the relation R in the set of ℝ real numbers, defined as
𝑅 = {(𝑎, 𝑏): 𝑎 ≤ 𝑏 3 } is not transitive.
Answer: (9,4) ∈ 𝑅 and (4,2) ∈ 𝑅 but (9,2) ∉ 𝑅 .Hence R is not transitive

12
3 MARKS QUESTIONS

1. Show that the relation R in the set {1, 2, 3} given by R = {(1, 1), (2, 2), (3, 3), (1, 2),
(2, 3)} is reflexive but neither symmetric nor transitive.
Solution: Let A = {1, 2, 3}
The relation R is defined on A is given by R = {(1, 1), (2, 2), (3, 3), (1, 2), (2, 3)}
Now, we have to show that R is reflexive but neither symmetric nor transitive.
Reflexive:
Clearly, (𝑎, 𝑎) ∈ 𝑅 𝑓𝑜𝑟 𝑒𝑣𝑒𝑟𝑦 𝑎 ∈ 𝐴
Hence, R is reflexive
Symmetric:
Clearly, (1, 2) ∈ 𝑅, 𝑏𝑢𝑡 (2, 1) ∉ 𝑅
Thus, for every (𝑎, 𝑏) ∈ 𝑅, (𝑏, 𝑎) ∉ 𝑅
Hence, R is not symmetric
Transitive:
For (1, 2) ∈ 𝑅 and (2, 3) ∈ 𝑅 ⇒ (1, 3)∉ 𝑅
Thus, (𝑎, 𝑏) ∈ 𝑅 𝑎𝑛𝑑 (𝑏, 𝑐) ∈ 𝑅 𝑡ℎ𝑒𝑛 (𝑎, 𝑐) ) ∉ 𝑅
Hence, R is not transitive.
Therefore, R is reflexive but neither symmetric nor transitive.
2. Show that the relation R in the set {1, 2, 3, 4} given by
R = {(1, 2), (2, 2), (1, 1), (4,4), (1, 3), (3, 3), (3, 2)} is reflexive and transitive but not
symmetric.
Solution: Let A = {1, 2, 3, 4}
The relation R is defined on A is given by R = {(1, 2), (2, 2), (1, 1), (4, 4), (1, 3), (3,
3), (3, 2)}
Now, we show that the relation R is reflexive and transitive but not symmetric.
Reflexive:
Clearly, (𝑎, 𝑎) ∈ 𝑅 𝑓𝑜𝑟 𝑒𝑣𝑒𝑟𝑦 𝑎 ∈ 𝐴
Hence, R is reflexive.
Symmetric:
Clearly, (1, 2) ∈ 𝑅, 𝑏𝑢𝑡 (2, 1) ∉ 𝑅
Thus, for every (𝑎, 𝑏) ∈ 𝑅, (𝑏, 𝑎) ∉ 𝑅
Hence, R is not symmetric,
Transitive:
For every (𝑎, 𝑏) ∈ 𝑅 𝑎𝑛𝑑 (𝑏, 𝑐) ∈ 𝑅 ⇒ (𝑎, 𝑐) ∈ 𝑅
Hence, R is transitive.
Therefore, R is reflexive and transitive but not symmetric.
3. Check whether the relation R defined in the set A= {1, 2, 3, 4, 5, 6} as

𝑅 = {(𝑎, 𝑏): 𝑏 = 𝑎 + 1, 𝑎, 𝑏 ∈ 𝐴} is reflexive, symmetric or transitive.

Solution: Let A = {1, 2, 3, 4, 5, 6}


R be the relation defined as 𝑅 = {(𝑎, 𝑏): 𝑏 = 𝑎 + 1}
i.e; R = {(1, 2), (2, 3), (3, 4), (4, 5), (5, 6)}
To check R is reflexive:
Clearly, (𝑎, 𝑎) ∉ 𝑅 𝑓𝑜𝑟 𝑒𝑣𝑒𝑟𝑦 𝑎 ∈ 𝐴
Hence, R is not reflexive
To check R is symmetric:
Clearly, (1, 2) ∈ 𝑅, 𝑏𝑢𝑡 (2, 1) ∉ 𝑅

13
Thus, for every (𝑎, 𝑏) ∈ 𝑅, (𝑏, 𝑎) ∉ 𝑅
Hence, R is not symmetric
To check R is Transitive:
Take (1, 2) ∈ 𝑅, 𝑎𝑛𝑑 (2, 3) ∈ 𝑅 𝑏𝑢𝑡 (1, 3) ∉ 𝑅
Thus, (𝑎, 𝑏) ∈ 𝑅 𝑎𝑛𝑑 (𝑏, 𝑐) ∈ 𝑅 𝑡ℎ𝑒𝑛 (𝑎, 𝑐) ) ∉ 𝑅
Hence, R is not transitive.
Therefore, R is neither reflexive, nor symmetric and nor transitive.
4. Determine whether the relation R in the set 𝐴 = {1, 2, 3, … ,13, 14} defined as
𝑅 = {(𝑥, 𝑦): 3𝑥 – 𝑦 = 0, 𝑥, 𝑦 ∈ 𝐴} is reflexive or symmetric or transitive.
Solution: Given 𝐴 = {1, 2, 3, … ,13, 14}
The relation R is defined as R = {(1, 3), (2, 6), (3, 9), (4, 12)}
To check R is reflexive:
Clearly, (𝑎, 𝑎) ∉ 𝑅 𝑓𝑜𝑟 𝑒𝑣𝑒𝑟𝑦 𝑎 ∈ 𝐴
Hence, R is not reflexive
To check R is symmetric:
Clearly, (1, 3) ∈ 𝑅, 𝑏𝑢𝑡 (3, 1) ∉ 𝑅
Thus, For every (𝑎, 𝑏) ∈ 𝑅, (𝑏, 𝑎) ∉ 𝑅
Hence, R is not symmetric
To check R is Transitive:
Take (1, 3) ∈ 𝑅, 𝑎𝑛𝑑 (3, 9) ∈ 𝑅 𝑏𝑢𝑡 (1, 9) ∉ 𝑅
Thus, (𝑎, 𝑏) ∈ 𝑅 𝑎𝑛𝑑 (𝑏, 𝑐) ∈ 𝑅 𝑡ℎ𝑒𝑛 (𝑎, 𝑐) ) ∉ 𝑅
Hence, R is not transitive.
Therefore, R is neither reflexive, nor symmetric and nor transitive.
5. Determine whether the relation R in the set N of natural numbers defined as
𝑅 = {(𝑥, 𝑦): 𝑦 = 𝑥 + 5 𝑎𝑛𝑑 𝑥 < 4} is reflexive, symmetric or transitive.
Solution: Given N = Set of all natural numbers.
The relation R is defined on the set N as 𝑅 = {(𝑥, 𝑦): 𝑦 = 𝑥 + 5 𝑎𝑛𝑑 𝑥 < 4}
i.e; R = {(1, 6), (2, 7), (3, 8)}
To check R is reflexive:
Clearly, (𝑎, 𝑎) ∉ 𝑅 𝑓𝑜𝑟 𝑒𝑣𝑒𝑟𝑦 𝑎 ∈ 𝑁
Hence, R is not reflexive
To check R is symmetric:
Clearly, (1, 6) ∈ 𝑅, 𝑏𝑢𝑡 (6, 1) ∉ 𝑅
Thus, For every (𝑎, 𝑏) ∈ 𝑅, (𝑏, 𝑎) ∉ 𝑅
Hence, R is not symmetric
To check R is Transitive:
For transitive, 𝑤𝑒 ℎ𝑎𝑣𝑒 𝑡𝑜 𝑠ℎ𝑜𝑤 𝑓𝑜𝑟 (𝑎, 𝑏) ∈ 𝑅 𝑎𝑛𝑑 (𝑏, 𝑐) ∈ 𝑅 𝑡ℎ𝑒𝑛 (𝑎, 𝑐) ∈ 𝑅
But in this case, in the relation R, for every order pair (a, b), there exists no order pair
as (b, c).
In such a case, R is also transitive.
Therefore, the relation R is neither reflexive nor symmetric but transitive.

6. Prove that the Greatest Integer Function 𝑓 ∶ 𝑅 → 𝑅, given by 𝑓(𝑥) = [𝑥], is neither
one-one nor onto, where [x] denotes the greatest integer less than or equal to 𝑥.
Solution: f : R → R, given by f(x) = [x]

14
Now, we prove that f is neither one-one nor onto
To Prove f is not one-one:
𝐶𝑙𝑒𝑎𝑟𝑙𝑦, 𝑤𝑒 ℎ𝑎𝑣𝑒 𝑡ℎ𝑎𝑡 𝑓(1.1) = [1.1] = 1
𝑓(1.2) = [1.2] = 1, … … … … … , 𝑓(1.9) = [1.9] = 1
From this, we conclude that different elements in the domain of f have same images
in the co-domain of 𝑓.
Hence, f is not one-one function.
To Prove f is not onto:
𝑤𝑒 𝑘𝑛𝑜𝑤 𝑡ℎ𝑎𝑡 𝑐𝑜𝑑𝑜𝑚𝑎𝑖𝑛 𝑜𝑓 𝑓 = 𝑅 (𝑠𝑒𝑡 𝑜𝑓 𝑎𝑙𝑙 𝑟𝑒𝑎𝑙 𝑛𝑢𝑚𝑏𝑒𝑟𝑠)
𝑅𝑎𝑛𝑔𝑒 𝑜𝑓 𝑓 = 𝑍 (𝑠𝑒𝑡 𝑜𝑓 𝑎𝑙𝑙 𝑖𝑛𝑡𝑒𝑔𝑒𝑟𝑠)
𝐶𝑙𝑒𝑎𝑟𝑙𝑦, 𝑐𝑜𝑑𝑜𝑚𝑎𝑖𝑛 𝑜𝑓 𝑓 ≠ 𝑅𝑎𝑛𝑔𝑒 𝑜𝑓 𝑓
Hence, f is not onto.
Therefore, f is neither one-one nor onto.
7. Show that the Modulus Function f : R → R, given by 𝑓(𝑥) = | 𝑥 |, is neither one-one
nor onto, where | 𝑥 |𝑖𝑠 𝑥, if 𝑥 is positive or 0 and | 𝑥 |𝑖𝑠 – 𝑥, 𝑖𝑓 𝑥 is negative.
𝑥, 𝑖𝑓 𝑥 ≥ 0
Solution: 𝑓: 𝑅 ⟶ 𝑅 𝑖𝑠 𝑔𝑖𝑣𝑒𝑛 𝑏𝑦𝑓(𝑥) = |𝑥| = {
−𝑥, 𝑖𝑓 𝑥 < 0
Now, we prove that f is neither one-one nor onto
To Prove f is not one-one:
𝐶𝑙𝑒𝑎𝑟𝑙𝑦, 𝑤𝑒 ℎ𝑎𝑣𝑒 𝑡ℎ𝑎𝑡 𝑓(1) = 1 = 𝑓(−1)
𝑓(2) = 2 = 𝑓(−2) 𝑎𝑛𝑑 𝑠𝑜 𝑜𝑛.
From this, we conclude that different elements in the domain of 𝑓 have same images in
the co-domain of 𝑓.
Hence, 𝑓 is not one-one function.
To Prove f is not onto:
𝑤𝑒 𝑘𝑛𝑜𝑤 𝑡ℎ𝑎𝑡 𝑐𝑜𝑑𝑜𝑚𝑎𝑖𝑛 𝑜𝑓 𝑓 = 𝑅 (𝑠𝑒𝑡 𝑜𝑓 𝑎𝑙𝑙 𝑟𝑒𝑎𝑙 𝑛𝑢𝑚𝑏𝑒𝑟𝑠)
𝑅𝑎𝑛𝑔𝑒 𝑜𝑓 𝑓 = 𝑅 + (𝑠𝑒𝑡 𝑜𝑓 𝑎𝑙𝑙 𝑛𝑜𝑛 − 𝑛𝑒𝑔𝑎𝑡𝑖𝑣𝑒 𝑟𝑒𝑎𝑙 𝑛𝑢𝑚𝑏𝑒𝑟𝑠)
𝐶𝑙𝑒𝑎𝑟𝑙𝑦, 𝑐𝑜𝑑𝑜𝑚𝑎𝑖𝑛 𝑜𝑓 𝑓 ≠ 𝑅𝑎𝑛𝑔𝑒 𝑜𝑓 𝑓
Hence, f is not onto.
8. 1, 𝑖𝑓 𝑥 > 0
Show that the function 𝑓: 𝑅 ⟶ 𝑅 𝑖𝑠 𝑔𝑖𝑣𝑒𝑛 𝑏𝑦𝑓(𝑥) = { 0, 𝑖𝑓 𝑥 = 0 (this function
−1, 𝑖𝑓 𝑥 < 0
is called signum function) is neither one-one nor onto.
1, 𝑖𝑓 𝑥 > 0
Solution: The function 𝑓: 𝑅 ⟶ 𝑅 𝑖𝑠 𝑔𝑖𝑣𝑒𝑛 𝑏𝑦𝑓(𝑥) = { 𝑖𝑓 𝑥 = 0
0,
−1, 𝑖𝑓 𝑥 < 0
Now, we prove that f is neither one-one nor onto.

15
To Prove f is not one-one:
𝐶𝑙𝑒𝑎𝑟𝑙𝑦, 𝑤𝑒 ℎ𝑎𝑣𝑒 𝑡ℎ𝑎𝑡 𝑓(1) = 1, 𝑓(2) = 1, 𝑓(3) = 1, 𝑎𝑛𝑑 𝑠𝑜 𝑜𝑛.
𝑓(−1) = −1, 𝑓(−2) = −2, 𝑎𝑛𝑑 𝑠𝑜 𝑜𝑛.
From this, we conclude that different elements in the domain of f have same images in
the co-domain of f.
Hence, f is not one-one function.
To Prove f is not onto:
𝑤𝑒 𝑘𝑛𝑜𝑤 𝑡ℎ𝑎𝑡 𝑐𝑜𝑑𝑜𝑚𝑎𝑖𝑛 𝑜𝑓 𝑓 = 𝑅 (𝑠𝑒𝑡 𝑜𝑓 𝑎𝑙𝑙 𝑟𝑒𝑎𝑙 𝑛𝑢𝑚𝑏𝑒𝑟𝑠)
𝑅𝑎𝑛𝑔𝑒 𝑜𝑓 𝑓 = {−1, 0, 1}
𝐶𝑙𝑒𝑎𝑟𝑙𝑦, 𝑐𝑜𝑑𝑜𝑚𝑎𝑖𝑛 𝑜𝑓 𝑓 ≠ 𝑅𝑎𝑛𝑔𝑒 𝑜𝑓 𝑓
Hence, f is not onto.
Therefore, f is neither one-one nor onto.
9. 𝑃𝑟𝑜𝑣𝑒 𝑡ℎ𝑎𝑡 𝑡ℎ𝑒 𝑓𝑢𝑛𝑐𝑡𝑖𝑜𝑛 𝑓: 𝑁 ⟶ 𝑁 𝑑𝑒𝑓𝑖𝑛𝑒𝑑 𝑏𝑦 𝑓(𝑥) = 𝑥 2 + 𝑥 + 1 is one-one
but not onto.
Solution: 𝑇ℎ𝑒 𝑓𝑢𝑛𝑐𝑡𝑖𝑜𝑛 𝒇: 𝑵 ⟶ 𝑵 𝑔𝑖𝑣𝑒𝑛 𝑏𝑦 𝑓(𝑥) = 𝑥 2 + 𝑥 + 1
Now we prove that f is one-one but not onto.
To prove f is one-one:
Let 𝑥1 , 𝑥2 ∈ 𝑁 𝑠𝑢𝑐ℎ 𝑡ℎ𝑎𝑡 𝑓(𝑥1 ) = 𝑓(𝑥2 )
⇒ 𝑥1 2 + 𝑥1 + 1 = 𝑥2 2 + 𝑥2 + 1
⇒ 𝑥1 2 − 𝑥2 2 + 𝑥1 − 𝑥2 = 0
⇒ (𝑥1 2 − 𝑥2 2 ) + (𝑥1 − 𝑥2 ) = 0
⇒ (𝑥1 + 𝑥2 )(𝑥1 − 𝑥2 ) + (𝑥1 − 𝑥2 ) = 0
⇒ (𝑥1 − 𝑥2 )[ (𝑥1 + 𝑥2 ) + 1] = 0
⇒ (𝑥1 − 𝑥2 ) = 0 (∵ [(𝑥1 + 𝑥2 ) + 1] > 0 𝑎𝑠𝑥1 , 𝑥2 𝑖𝑛 𝑑𝑜𝑚𝑎𝑖𝑛 𝑵 )
⇒ 𝑥1 = 𝑥2
Hence, f is one-one
To prove f is onto:
we have 𝑓(1) = 3, 𝑓(2) = 7 and so on.
Thus, 𝑓(𝑥) = 𝑥 2 + 𝑥 + 1 ≥ 3 𝑓𝑜𝑟 𝑒𝑣𝑒𝑟𝑦 𝑥 ∈ 𝑁 (𝑑𝑜𝑚𝑎𝑖𝑛)
Clearly, 𝑓(𝑥)𝑛𝑜𝑡 𝑡𝑎𝑘𝑖𝑛𝑔 𝑣𝑎𝑙𝑢𝑒𝑠 1 𝑎𝑛𝑑 2
Thus, the every element of co-domain in N has no pre-image in the domain N
Hence, 𝑓 is not onto.

10. 𝐿𝑒𝑡 𝑓: 𝑊 → 𝑊𝑏𝑒 𝑑𝑒𝑓𝑖𝑛𝑒𝑑 𝑎𝑠 𝑓(𝑥) = 𝑥 − 1, 𝑖𝑓 𝑥 𝑖𝑠 𝑜𝑑𝑑 𝑎𝑛𝑑


𝑓(𝑥) = 𝑥 + 1, 𝑖𝑓 𝑥 𝑖𝑠 𝑒𝑣𝑒𝑛. Show that 𝑓 is both one-one and onto.
Solution:
𝑓: 𝑊 → 𝑊 𝑏𝑒 𝑑𝑒𝑓𝑖𝑛𝑒𝑑 𝑎𝑠 𝑓(𝑥) = 𝑥 − 1, 𝑖𝑓 𝑥 𝑖𝑠 𝑜𝑑𝑑 and

16
𝑓(𝑥) = 𝑥 + 1, 𝑖𝑓 𝑥 𝑖𝑠 𝑒𝑣𝑒𝑛.
To Prove 𝒇 is one-one:
Case I: when 𝑥1 𝑎𝑛𝑑 𝑥2 are even number
Now, consider 𝑓(𝑥1 ) = 𝑓(𝑥2 )
⇒ 𝑥1 + 1 = 𝑥2 + 1
⇒ 𝑥1 = 𝑥2
Hence, 𝑓 is one-one
Case II: when 𝑥1 𝑎𝑛𝑑 𝑥2 are odd number
Now, consider 𝑓(𝑥1 ) = 𝑓(𝑥2 )
⇒ 𝑥1 − 1 = 𝑥2 − 1
⇒ 𝑥1 = 𝑥2
Hence, 𝑓 is one-one.
Case III: when 𝑥1 𝑖𝑠 𝑜𝑑𝑑 𝑎𝑛𝑑 𝑥2 is even number
Here, 𝑥1 ≠ 𝑥2
Also, in this case 𝑓(𝑥1 ) 𝑖𝑠 𝑒𝑣𝑒𝑛 𝑎𝑛𝑑 𝑓(𝑥2 )𝑖𝑠 𝑜𝑑𝑑.
Hence, 𝑓(𝑥1 ) ≠ 𝑓(𝑥2 )
Therefore, 𝑓 is one-one.
To Prove 𝒇 is onto:
For every even number ‘𝑦’ in co-domain there exists odd number 𝑦 + 1 in domain
and for every odd number ‘𝑦’ in co-domain there exists even number 𝑦 − 1 in
domain such that 𝑓(𝑥) = 𝑦.
Hence, f is onto
Therefore, f is both one-one and onto.

EXERCISE

1. Consider 𝑓: 𝑅 + ⟶ [4, ∞)𝑔𝑖𝑣𝑒𝑛 𝑏𝑦 𝑓(𝑥) = 𝑥 2 + 4. Show that f is both one-one and


onto.
2. 2 2 2x−1
Let A = R − {3} and B = R − {3}. If f: A→ B and f(x) = 3x−2, then
prove that the function f is one-one and onto.
𝑥
3. Show that the function 𝑓: 𝑅 → 𝑅 defined by 𝑓(𝑥) = 𝑥 2 +1 , ∀ 𝑥 ∈ 𝑅 is neither one-
one nor onto.
4. Let R be the relation on set of all integer Z defined by 𝑅 = {(𝑎, 𝑏): |𝑎 − 𝑏| ≤ 3}.
Check whether R is an equivalence relation.
Answer: R is reflexive, symmetic but not transitive

17
5 MARKS QUESTIONS

1. Let R be the relation in the set Z of integers given by


𝑅 = {(𝑎, 𝑏): 2 𝑑𝑖𝑣𝑖𝑑𝑒𝑠 𝑎 – 𝑏}. Show that the relation R equivalence? Write the
equivalence class [0].
Solution :
Given R = {(a, b) : 2 divides a – b}
For equivance relation we have to check :
(i) Reflexive:
If (𝑎 − 𝑏) is divisible by 2 then,
⇒ (𝑎 − 𝑎) = 0 is also divisible by 2
⇒ (𝑎, 𝑎) ∈ 𝑅
Hence R is Reflexive ∀ a,b ∈ Z
(ii) Symmetric:
If (𝑎 − 𝑏) is divisible by 2 then,
⇒ (𝑏 − 𝑎) = −(𝑎 − 𝑏) is also divisible by 2
⇒ (𝑎, 𝑏) ∈ 𝑅 𝑎𝑛𝑑 (𝑏, 𝑎) ∈ 𝑅
Hence R is Symmetric ∀ a,b ∈ Z
(iii) Transitive:
If (𝑎 − 𝑏)𝑎𝑛𝑑 (𝑏 − 𝑐) are divisible by 2 then,
⇒ 𝑎 − 𝑐 = (𝑎 − 𝑏) + (𝑏 − 𝑐) is also divisible by 2
⇒ (a,b) ∈ R, (b,c) ∈ R and (a,c) ∈ R.
Hence R is Transitive ∀ a,b,c ∈ Z
⇒ As Relation R is satisfyingReflexive , Symmetric and Transitive.
Hence R is an equivalence relation.
Now equivalence class [0]
𝑅 = {(𝑎, 𝑏): 2 𝑑𝑖𝑣𝑖𝑑𝑒𝑠 (𝑎 − 𝑏)}⇒ (𝒂 − 𝒃) 𝑖𝑠 𝑎 𝑚𝑢𝑙𝑡𝑖𝑝𝑙𝑒 𝑜𝑓 2.
To find equivalence class 0, put b=0
So, (a−0) is a multiple of 2
⇒ a is a multiple of 2
So, In the set Z of integers, all the multiple of 2 will come in equivalence class [0]
Hence, equivalence class [0] = {2𝑥 ∶ 𝑥 ∈ 𝑍 }
1.2. S Show that the relation R defined by (𝑎, 𝑏)𝑅 (𝑐, 𝑑) ⇔ 𝑎 + 𝑑 = 𝑏 + 𝑐 on the set
N × N is an equivalence relation. Also, find the equivalence classes [(2,3)] and
[(1,3)].
Solution:
Given that, R be the relation in N × N defined by (a, b) R (c, d)
if 𝑎 + 𝑑 = 𝑏 + 𝑐 for (a, b), (c, d) in N × N.
Reflexive:
Let (a, b) R (a, b) ⇒ a + b = b + a
which is true since addition is commutative on N.
⇒ R is reflexive.
Symmetric:
Let (a, b) R (c, d) ⇒ a + d = b + c
⇒b+c=a+d⇒c+b=d+a
⇒ (c, d) R (a, b) ⇒ R is symmetric.
Transitive
⇒ (a, b) R (e, f) for (a, b), (c, d),(e, f) in N × N.
Let (a, b) R (c, d) and (c, d) R (e, f)
⇒ a + d = b + c and c + f = d + e
⇒ (a + d) – (d + e) = (b + c) – (c + f) ⇒ a - e = b - f ⇒ a + f = b + e
⇒ (a, b) R (e, f) ⇒ R is transitive.

18
Hence, R is an equivalence relation.
Equivalence Classes:
The equivalence class of (a, b) is the set of all pairs (c, d) such that a + d = b + c.
⇒𝑎– 𝑏 = 𝑐 − 𝑑
The equivalence classes of [(2,3)] is
Put 𝑎 = 2 𝑎𝑛𝑑 𝑏 = 3
c–d=2–3
d–c=1
[(2,3)] = {(c,d) : d – c = 1 ∀ c,d ∈ Z }
The equivalence classes of [(1,3)] is
Put a=1and b=3
c–d =1–3 ⇒d–c=2
[(1,3)] = {(c,d) : d – c = 2 ∀c,d ∈ Z }
3. Show that the relation R in the set of ℝ real numbers, defined as

𝑅 = {(𝑎, 𝑏): 𝑎 ≤ 𝑏 3 } is neither reflexive nor symmetric nor transitive.

Solution: Given ℝ = 𝑠𝑒𝑡 𝑜𝑓 𝑎𝑙𝑙 𝑟𝑒𝑎𝑙 𝑛𝑢𝑚𝑏𝑒𝑟𝑠


The relation R in the set ℝ defined as 𝑅 = {(𝑎, 𝑏): 𝑎 ≤ 𝑏 3 }
Now we prove that R is neither reflexive nor symmetric nor transitive.
To show R is not reflexive:
We know that 𝑎 ≤ 𝑎3 is not true for any positive real number less than 1.
1 1 1 3 1
For example, 𝑓𝑜𝑟 𝑎 = 2 , 2 ≰ (2) = 8
Thus, clearly, (𝑎, 𝑎) ∉ 𝑅 𝑓𝑜𝑟 𝑒𝑣𝑒𝑟𝑦 𝑎 ∈ ℝ
Hence, R is not reflexive
To show R is not symmetric:
Take a = 1 and b = 2
Now, 𝑎 = 1 ≤ 23 = 𝑏 3 ⇒ 𝑎 ≤ 𝑏 3 ⇒ (𝑎, 𝑏) ∈ 𝑅
But, 𝑏 = 2 ≰ (1)3 = 1 = 𝑎 ⇒ 𝑏 ≰ 𝑎2 ⇒ (𝑏, 𝑎) ∉ 𝑅
Thus, (𝑎, 𝑏) ∈ 𝑅 ⇒ (𝑏, 𝑎) ∉ 𝑅
Hence, R is not symmetric.
To show R is not transitive:
Let us take a = 10, b = 4, c = 2
(𝑎, 𝑏) ∈ 𝑅 = (10, 4) ∈ 𝑅 𝑎𝑠 10 ≤ 43 = 64
(𝑏, 𝑐) ∈ 𝑅 = (4, 2) ∈ 𝑅 𝑎𝑠 4 ≤ 23 = 8
But, (𝑎, 𝑐) ∉ 𝑅 𝑎𝑠 10 ≰ 23 = 8
Thus, (𝑎, 𝑏) ∈ 𝑅 𝑎𝑛𝑑 (𝑏, 𝑐) ∈ 𝑅 ⇒ (𝑎, 𝑐) ∉ 𝑅
Hence, R is not tranitive.
Therefore, the relation R is neither reflexive, nor symmetric nor transitive.

4. Let A = {1, 2, 3, ……., 9} and (𝑎, 𝑏)𝑅 (𝑐, 𝑑)𝑖𝑓


𝑎𝑑 = 𝑏𝑐 𝑓𝑜𝑟 (𝑎, 𝑏), (𝑐, 𝑑) 𝑖𝑛 A×A. Prove that R is an equivalence relation.
Solution: Given A = {1, 2, 3, ……., 9}
The relation R is defined as (𝑎, 𝑏)𝑅 (𝑐, 𝑑)𝑖𝑓 𝑎𝑑 = 𝑏𝑐 𝑓𝑜𝑟 (𝑎, 𝑏), (𝑐, 𝑑)𝑖𝑛 A × A
Now, we prove that R is an equivalence relation.
To show R is reflexive:
Clearly, 𝑎𝑏 = 𝑏𝑎 for every a, b ∈ A
⇒ (𝑎, 𝑏)𝑅(𝑎, 𝑏) 𝑓𝑜𝑟 𝑒𝑣𝑒𝑟𝑦 (𝑎, 𝑏) ∈ A × A
Hence, R is reflexive.
To show R is symmetric:
Let (a, b) R (c, d)

19
⇒ 𝑎𝑑 = 𝑏𝑐
⇒ 𝑑𝑎 = 𝑐𝑏
⇒ 𝑐𝑏 = 𝑑𝑎
⇒ (c, d)R (a, b)
Hence, R is symmetric
To show R is transitive:
Let (a, b) R (c, d) ⇒ 𝑎𝑑 = 𝑏𝑐 …….. (1)
And (c, d) R (e, f) ⇒ 𝑐𝑓 = 𝑑𝑒 ………. (2)
Multiply (1) and (2), we get
(𝑎𝑑) (𝑐𝑓) = (𝑏𝑐)(𝑑𝑒)
⇒ 𝑎𝑓 = 𝑏𝑒
⇒ (𝑎, 𝑏)𝑅(𝑒, 𝑓)
Hence, R is transitive.
Therefore, R is reflexive, symmetric and transitive.
Hence, R is an equivalence relation.
5. 𝜋 𝜋
Consider a function 𝑓: [0, 2 ] ⟶ 𝑅 𝑔𝑖𝑣𝑒𝑛 𝑏𝑦 𝑓(𝑥) = 𝑠𝑖𝑛𝑥 𝑎𝑛𝑑 𝑔: [0, 2 ] ⟶ 𝑅 given

by 𝑔(𝑥) = 𝑐𝑜𝑠𝑥. Show that 𝑓 𝑎𝑛𝑑 𝑔 are one-one but 𝑓 + 𝑔 is not one-one.
Solution:
To prove f is one-one:
𝜋
𝑇ℎ𝑒 𝑓𝑢𝑛𝑐𝑡𝑖𝑜𝑛 𝑓: [0, ] ⟶ 𝑅 𝑔𝑖𝑣𝑒𝑛 𝑏𝑦 𝑓(𝑥) = 𝑠𝑖𝑛𝑥
2
𝜋
Clearly, different elements in the domain [0, 2 ] of ‘𝑓’ have distinct images in the

co-domain of ‘𝑓’
Hence, 𝑓 is one-one.
To prove 𝑔 is one-one:
𝜋
𝑇ℎ𝑒 𝑓𝑢𝑛𝑐𝑡𝑖𝑜𝑛 𝑔: [0, ] ⟶ 𝑅 𝑔𝑖𝑣𝑒𝑛 𝑏𝑦 𝑔(𝑥) = 𝑐𝑜𝑠𝑥
2
𝜋
Clearly, different elements in the domain [0, 2 ] of ‘𝑔’ have distinct images in the

co-domain of ‘𝑔’
Hence, ‘𝑔’ is one-one.
To prove 𝑓 + 𝑔 is one-one:
(𝑓 + 𝑔)(𝑥) = 𝑓(𝑥) + 𝑔(𝑥) = 𝑠𝑖𝑛𝑥 + 𝑐𝑜𝑠𝑥
(𝑓 + 𝑔)(0) = 𝑠𝑖𝑛0 + 𝑐𝑜𝑠0 = 0 + 1 = 1
𝜋 𝜋 𝜋
(𝑓 + 𝑔) ( ) = 𝑠𝑖𝑛 + 𝑐𝑜𝑠 = 1 + 0 = 1
2 2 2
From this, we conclude that different elements in the domain of 𝑓 + 𝑔 have same
images in the co-domain of 𝑓 + 𝑔.
Hence, 𝑓 + 𝑔 is not one-one.

20
EXERCISE

1 Let f : R → R be a function defined as 𝑓(𝑥) = 4𝑥 + 3, then show that 𝑓 is one-one


and onto.
2 A function 𝑓: [−4,4] → [0,4] is given by 𝑓(𝑥) = √16 − 𝑥 2 . Show that 𝑓 is an onto
function but not a one-one function. Further, find all possible values of ‘ 𝑎 ’ for
which 𝑓(𝑎) = √7.
3 A relation R is defined on a set of real numbers ℝ as
𝑅 = {(𝑥, 𝑦): 𝑥𝑦 𝑖𝑠 𝑎𝑛 𝑖𝑟𝑟𝑎𝑡𝑖𝑜𝑛𝑎𝑙 𝑛𝑢𝑚𝑏𝑒𝑟}
Check whether R is reflexive, symmetric and transitive or not.

4 Show that the relation R in the set A = {x ∈ Z: 0 ≤ x ≤ 12}, given by


R = {(a, b): |a – b| is divisible by 3} is an equivalence relation.

CASE STUDY QUESTIONS

I Read the passage given below and answer the questions that follow:
An organisation conducted a bike race under two different categories boys and girls
.Among all the participants finally three from category 1(boys) and two from
category 2(girls) were selected for the final race. Let B={b1,b2,b3},G={g1,g2},
where B represents the set of boys selected and G the set of girls who were selected
for the final race.

1. How many relations are possible from B to G?(1mark)


2. How many functions can be formed from B to G? (1mark)
3. How many one-one functions be formed from B to G? justify your answer.
(2marks)
Solution:
(1) n(B)= 3, n(G)=2
Number of relations from B to G is 26 ,because every relation is a subset of
B×G and there are 3×2=6 elements in B×G.
(2 )Number of functions= 23=8.
(3) Number of one-one functions = 0 as n(B) > n(G).

21
II Priya and Surya are playing monopoly in their house during COVID. While rolling
the dice their mother Chandrika noted the possible outcomes of the throw every
time belongs to the set {1, 2, 3, 4, 5, 6}. Let A denote the set of players and B be the
set of all possible outcomes. Then 𝐴 = {𝑃, 𝑆} 𝐵 = {1, 2, 3, 4, 5, 6}.Then
answer the below questions based on the given information.(each question carries
one mark)

(a) Let 𝑅: 𝐵 → 𝐵 be defined by 𝑅 =


{(𝑎, 𝑏) 𝑏𝑜𝑡ℎ 𝑎 𝑎𝑛𝑑 𝑏 𝑎𝑟𝑒 𝑒𝑖𝑡ℎ𝑒𝑟 𝑜𝑑𝑑 𝑜𝑟 𝑒𝑣𝑒𝑛}, then is R an Equivalence
relation?
(b) Chandrika wants to know the number of functions from 𝐴 to 𝐵. How many
numbers of functions are possible?
(c) Let 𝑅 be a relation on 𝐵 defined by 𝑅 =
{(1, 2), (2, 2), (1, 3), (3, 4), (3, 1), (4, 3), (5, 5)}. Then is R relexive,
symmetric and transitive?
(d) Let 𝑅: 𝐵 → 𝐵 be defined by 𝑅 =
{(1, 1), (1, 2), (2, 2), (3, 3), (4, 4), (5, 5), (6, 6)} then is R symmetric?
Justify
Solution:
(a) Yes, it is equivalence.
The relation is reflexive,symmetric and transitive hence it is Equivalence.
(b) If 𝑛(𝐴) = 𝑚, 𝑛(𝐵) = 𝑛, then the number of functions from A to B is 𝑛m
Ans: 62

(c) As (1,1) ∉ 𝑅 It is not reflexive


As (1,2) ∈ 𝑅 but (2,1) ∉ 𝑅 it is not symmetric
As (1,3) ∈ 𝑅 and (3, 4) ∈ 𝑅 but (1,4) ∉ 𝑅 ,it is not transitive
Hence none.
(d) No, (1,2) ∈ 𝑅 but (2,1) ∉ 𝑅 it is not symmetric

III In two different societies, there are some school going students – including girls as
well as boys. Satish forms two sets with these students, as his college project.
Let 𝐴 = {𝑎1 , 𝑎2 , 𝑎3 , 𝑎4 , 𝑎5 } and 𝐵 = {𝑏1 , 𝑏2 , 𝑏3 , 𝑏4 } where 𝑎𝑖′ 𝑠, 𝑏𝑖′ 𝑠are the school
going students of first and second society respectively.
Using the information given above, answer the following question

(a) Satish wishes to know the number of reflexive relations defined on set 𝐴.
How many such relations are possible? (1mark)

22
(b) Let 𝑅: 𝐴 → 𝐴, 𝑅 = {(𝑥, 𝑦): 𝑥 𝑎𝑛𝑑 𝑦 𝑎𝑟𝑒 𝑠𝑡𝑢𝑑𝑒𝑛𝑡𝑠 𝑜𝑓 𝑠𝑎𝑚𝑒 𝑠𝑒𝑥}. Then
relation is R an equivalence relation. (1 mark)
(c) Let 𝑅: 𝐴 → 𝐵, 𝑅 = {(𝑎1 , 𝑏1 ), (𝑎2 , 𝑏1 ), (𝑎3 , 𝑏3 ), (𝑎4 , 𝑏2 ), (𝑎5 , 𝑏2 )}, then is 𝑅
One-one and onto ? Justify ( 2marks)
Solution:
(a) 220
If 𝑛(𝐴) = 𝑛 then number of reflexive relations that can be
2
defined on A is 2𝑛 −𝑛 .here 𝑛(𝐴) = 5.
(b) Yes the relation is equivalence.
As the relation is reflexive,symmetric and transitive.
(c) No R is not one-one and onto
Distinct elements of A have same image 𝑎1 𝑎𝑛𝑑 𝑎2 are having same
Image 𝑏1 ,so R is not one-one.
𝑏4 is not having pre-image ,so R is not onto.

EXERCISE

Vani and Mani are playing Ludo at home while it was raining outside.While rolling
the dice Vani’s brother Varun observed and noted the possible outcomes of the
throw every time belongs to the set {1,2,3,4,5,6}.Let A be the set of players while B
be the set of all possible outcomes.
A= {Vani, Mani}, B= {1, 2, 3, 4, 5, 6}.
Answer the following questions:

a) Let 𝑅: 𝐵 → 𝐵,be defined by 𝑅 = {(𝑥, 𝑦): 𝑦 𝑖𝑠 𝑑𝑖𝑣𝑖𝑠𝑖𝑏𝑙𝑒 𝑏𝑦 𝑥}.Verify that


whether R is reflexive, symmetric and transitive.(2marks)
b) Is it possible to define an onto function from A to B? Justify.(1mark)
c) Which kind of relation is R defined on B given by
R={(1,2),(2,2),(1,3),(3,4),(3,1),(4,3),(5,5)}?(1mark)
Or
Find the number of possible relations from A to B.

Answer:a) R is reflexive and transitive but not symmetric.


b) No. Because n(B) is greater than n(A)
c) R is neither reflexive nor symmetric nor transitive
or
no. of relations= 212

23
II

Students of class X planned to plant saplings along straight lines,parallel to each


other to one side of the playground ensuring that thy had enough play area.Let us
assume that they planted one of the rows of the saplings along the line 𝑦 = 𝑥 + 4.
Let L be the set of all lines which are parallel on the ground and R be a relation on
L.
i) R = {(𝑙1 , 𝑙2 ): 𝑙1 ⟘ 𝑙2 , 𝑤ℎ𝑒𝑟𝑒 𝑙1 , 𝑙2 ∈ 𝐿}. What is the type of relation R?
(1mark)
ii) R = {(𝑙1 , 𝑙2 ): 𝑙1 ||𝑙2 , 𝑤ℎ𝑒𝑟𝑒 𝑙1 , 𝑙2 ∈ 𝐿}. What is the type of relation R?
(1mark)
iii) Check whether the function 𝑓: 𝑅 → 𝑅 𝑑𝑒𝑓𝑖𝑛𝑒𝑑 𝑏𝑦 𝑓(𝑥) = 𝑥 + 4 is one-
one and onto. (2marks)

Answer: i) R is symmetric but neither reflexive nor transitive.


ii) R is an equivalence relation.
iii)R is both one-one and onto

24
CHAPTER: INVERSE TRIGONOMETRIC FUNCTIONS
SYLLABUS: Definition, range, domain, principal value branch. Graphs of inverse
trigonometric functions.

Definitions and Formulae:


Principal Value Branches:

Range (Principal Value


FUNCTION DOMAIN
Branch)
𝜋 𝜋
sin−1 𝑥 [−1, 1] [− , ]
2 2

cos −1 𝑥 [−1, 1] [0, 𝜋]

𝜋 𝜋
tan−1 𝑥 R (− , )
2 2
𝜋 𝜋
cosec −1 𝑥 𝑅 − (−1, 1) [− , ] − {0}
2 2
𝜋
sec −1 𝑥 𝑅 − (−1, 1) [0, 𝜋] − { }
2

cot −1 𝑥 R (0, 𝜋)

sin-1 (-x) = - sin-1(x) cos-1 (-x) = π - cos-1(x)

cosec-1 (-x) = - cosec-1(x) sec-1 (-x) = π - sec-1(x)

tan-1 (-x) = - tan-1(x) cot-1 (-x) = π - cot-1(x)

−𝜋 𝜋
𝑠𝑖𝑛−1 ( 𝑠𝑖𝑛 𝑥) = 𝑥, 𝑥 ∈ [ , ]
2 2

𝑐𝑜𝑠 −1 ( 𝑐𝑜𝑠 𝑥) = 𝑥, 𝑥 ∈ [0, 𝜋]

𝑠𝑖𝑛( 𝑠𝑖𝑛−1 𝑥) = 𝑥, 𝑥 ∈ [−1, ,1]


25
MULTIPLE CHOICE QUESTIONS

Q.NO QUESTIONS AND SOLUTIONS

1 −1
The principal value of 𝑐𝑜𝑠 −1 ( 2 )`

(a) 2π (b) π (iii) − π (d) − π


3 3 3 6

Solution: We have 𝑐𝑜𝑠 −1 (−𝑥) = 𝜋 − 𝑐𝑜𝑠 −1 (𝑥)


−1 1
𝑐𝑜𝑠 −1 ( 2 )`= 𝜋 − 𝑐𝑜𝑠-1(2).

𝜋 2𝜋
= π– =
3 3

Ans: (a)

2 3𝜋
The principal value of 𝑠𝑖𝑛−1 [𝑠𝑖𝑛 ( 5 )]`

3π 2π − 2π π
(a) (b) (iii) (d)
5 5 5 5

3𝜋 3𝜋
Solution: We have 𝑠𝑖𝑛−1 [𝑠𝑖𝑛 ( 5 )] = 𝑠𝑖𝑛−1 [𝑠𝑖𝑛 (𝜋 − )]
5

2𝜋
= 𝑠𝑖𝑛−1 [𝑠𝑖𝑛 ( 5 )]


=
5

Ans. (b)

3 The value of: 𝑡𝑎𝑛−1 √3 − 𝑠𝑒𝑐 −1( − 2) is

π −π −π
(a) (b) (c) (d) 0
6 6 3

Solution: We have sec-1 (-x) = π - sec-1(x).

 tan
−1
3 − sec −1 (−2) = π − (π − π ) = −𝝅
𝟑
3 3

Ans: (c)

4 𝜋 −1
The value of 𝑠𝑖𝑛 [ 3 − 𝑠𝑖𝑛−1 ( 2 )] is

(a) 0 (b) 1 (c) -1 (d) 2

Solution: We have sin-1 (-x) = - sin-1(x)

26
π  − 1   π  − π  π
 sin  − sin −1   = sin  −   = sin   = 1
3  2   3  6  2

Ans: (b)

5 −7𝜋
The principal value of 𝑐𝑜𝑠 −1 (𝑐𝑜𝑠 ( )) is
3

(a) 7π (b) π (iii) − π (d) − 7π


3 3 3 3

Solution: We have cos(−𝑥) = 𝑐𝑜𝑠𝑥

 − 7π   7π 
 cos −1 cos −1
 = cos cos 
 3   3 

−1  π
= cos cos 2π + 
 3

π
= cos cos  = π
−1

3 3

Ans: (b)

6 The value of 𝑡𝑎𝑛−1 √3 − 𝑐𝑜𝑡 −1 ( − √3) is

π −π −π π
(a) (b) (c) (d)
2 2 3 6

−1 π
−1 π
Solution: cot (− 3 ) = π − cot ( 3 ) = π − = 5
6 6

 tan
−1
3 − cot −1 (− 3 ) = π − 5π = − π
3 6 2

Ans: (b)
𝜋
7 The value of 𝑥 if 𝑡𝑎𝑛−1 √3 + 𝑐𝑜𝑡 −1 𝑥 = 2

1 π
(a) 3 (b) − 3 (c) (d)
3 6

Solution:
π π π π
cot −1 x = − tan −1 3 = − =
2 2 3 6

π
 x = cot = 3
6

Ans: (a)

27
𝜋
8 The value of 𝑥 if 𝑠𝑒𝑐 −1 2 + 𝑐𝑜𝑠 𝑒 𝑐 −1 𝑥 = 2

3
(a) 3 (b) 2 (c) (d) −2
2

Solution:

π π π π
cos ec −1 x = − sec −1 3 = − =
2 2 3 6

π
 x = cos ec =2
6

Ans: (b)

9 If 𝑠𝑖𝑛−1 𝑥 = 𝑦 then the principal value of 𝑦 is:

(a) 0  y  π (b) − π  y  π c) − π  y  π (d) 0  y  π


2 2 2 2

Ans: (b)

10 If 𝑡𝑎𝑛−1 𝑥 = 𝑦 then the principal value of y is:

−π π −π π
(a) 0  y  π (b) y c) y (d) 0  y  π
2 2 2 2

Ans: (c)

CHAPTER VIDEO LINK FOR MCQs SCAN QR CODE FOR


VIDEO

INVERSE TRIGONOMETRIC
FUNCTIONS https://youtu.be/wGsA6WJXvP8

EXERCISE
3
The value of 𝑐𝑜𝑠 (𝑡𝑎𝑛−1 4) is :
1
3 4 3 3
(a) (b) (c) (d)
5 5 4 7

28
4
Answer: (b)
5
2 5𝜋
The principal value of : 𝑡𝑎𝑛−1 [𝑡𝑎𝑛 ( 4 )]`
(a) 5π (b) π (iii) − π (d) 1
4 4 4
𝜋
Answer: (b) 4

3 7
The value of 𝑐𝑜𝑡 (𝑐𝑜𝑠 −1 25) is :
(a) 7 (b) 24 (c) 7 (d) 25
24 25 25 7
7
Answer: (a)
24
4 14𝜋
The value of 𝑐𝑜𝑠 −1 (𝑐𝑜𝑠 ( )) is :
3

(a) 14π (b) π (c) 2π (d) 4π


3 3 3 3

Answer: (c)
3

5 1
The value of 2 𝑠𝑖𝑛−1 (2) + 𝑐𝑜𝑡 −1(1)
7π 3π 2π π
(a) (b) (c) (d)
12 4 3 4

Answer:(a)
12

ASSERTION-REASON BASED QUESTIONS

In the following questions, a statement of assertion (A) is followed by a statement of


Reason (R). Choose the correct answer out of the following choices.
(a) Both A and R are true and R is the correct explanation of A.
(b) Both A and R are true but R is not the correct explanation of A.
(c) A is true but R is false.
(d) A is false but R is true.
1 7𝜋 5𝜋
ASSERTION (A): Principal value of 𝑐𝑜𝑠 −1 𝑐𝑜𝑠 ( 6 ) 𝑖𝑠 6
REASON (R): Range of principal branch of cos is [ 0, π] and cos-1(cosx) = x
-1

if xϵ [ 0, π].
Ans: (a)

2 13𝜋 𝜋
ASSERTION (A): Principal value of 𝑠𝑖𝑛−1 𝑠𝑖𝑛 ( ) 𝑖𝑠
6 6
REASON (R): sin-1 (-x) = - sin-1(x)
Ans:(b)
−𝜋
3 ASSERTION (A): Principal value of 𝑠𝑖𝑛−1(−1) = 2
REASON (R): sin-1 (-x) = - sin-1(x)
Ans: (a)

29
4 3𝜋 3𝜋
ASSERTION (A): Principal value of 𝑠𝑖𝑛−1 𝑠𝑖𝑛 ( 5 ) = 5
− π π 
REASON (R): sin −1 sin (x ) = x, x   , 
 2 2
Ans: (d)

5 −1 1
ASSERTION (A): The principal value of 𝑐𝑜𝑠 −1 ( ) = 𝑐𝑜𝑠 −1 ( )`
√2 √2
REASON (R): cosine function is an even function, therefore cos(−𝑥) = 𝑐𝑜𝑠𝑥.
Ans: (d)

6 −1 1
ASSERTION (A): The principal value of 𝑐𝑜𝑠 −1 ( 2 ) = 𝜋 − 𝑐𝑜𝑠 −1 (2)`
REASON (R): Range of cos-1x is [0, π]
Ans: (b)

7 −𝜋 −𝜋
ASSERTION (A): The principal value of 𝑡𝑎𝑛−1 [𝑠𝑖𝑛 ( 2 )] = `
2
REASON (R): tan-1 (-x) = tan-1(x)
Ans: (d)

8 −𝜋 −𝜋
ASSERTION (A): The principal value of 𝑡𝑎𝑛−1 𝑡𝑎𝑛 ( 4 ) = 4
`
−𝜋 𝜋 −𝜋 𝜋
REASON (R): Range of tan-1 x is ( 2 , 2 ), tan-1 (tanx) = x if xϵ ( 2 , 2 )
Ans: (a)

9 ASSERTION(A): One branch of 𝑐𝑜𝑠-1𝑥 other than the principal value


branch is [𝜋, 2𝜋]
−𝜋
REASON (R): 𝑐𝑜𝑠 ( 2 ) = −1
Ans: (c)

10 ASSERTION (A): One branch of 𝑠𝑖𝑛-1𝑥 other than the principal value branch is
𝜋 3𝜋
[2 , 2 ]
3𝜋 𝜋
REASON (R): 𝑠𝑖𝑛 ( 2 ) = 𝑠𝑖𝑛 ( 2 ) = 1
Ans: (c)

EXERCISE

1 ASSERTION (A): 𝑠𝑖𝑛(𝑐𝑜𝑠 −1 𝑥) = 𝑐𝑜𝑠(𝑠𝑖𝑛−1 𝑥) = √1 − 𝑥 2 , |𝑥| ≤ 1


REASON (R): Because 𝑠𝑖𝑛2 𝜃 + 𝑐𝑜𝑠 2 𝜃 = 1
Answer: (b)
2 −𝜋 −𝜋
ASSERTION (A): The principal value of 𝑐𝑜𝑠 −1 [𝑐𝑜𝑠 ( )] =
4 4
REASON (R): Range of cos-1 𝑥 is [0, 𝜋] cos-1 (𝑐𝑜𝑠𝑥) = 𝑥 if 𝑥𝜖 [0, 𝜋]
Answer: (d)
3 𝟐
ASSERTION (A): The principal value of 𝑠𝑖𝑛[𝑐𝑜𝑡 −1 (𝑐𝑜𝑠(𝑡𝑎𝑛−1 1))] = √𝟑

30
−π π
REASON (R): Range of tan-1 x is  ,  tan-1 (𝑡𝑎𝑛𝑥) = 𝑥 if 𝑥𝜖
 2 2 ,
−π π
 , 
 2 2
Answer: (b)
4 −𝜋 𝜋
ASSERTION (A): The principal value of 𝑐𝑜𝑠 −1 𝑐𝑜𝑠 ( ) = `
4 4
REASON (R): Cosine function is an even function, therefore cos(−𝑥) = 𝑐𝑜𝑠𝑥.
and cos-1 (𝑐𝑜𝑠𝑥) = 𝑥 if 𝑥𝜖 [0, 𝜋]

Answer: (a)
5 3 3
ASSERTION (A): The value of 𝑠𝑖𝑛 [2 𝑠𝑖𝑛−1 (4)] = 4
REASON (R): 𝑠𝑖𝑛( 𝑠𝑖𝑛−1 𝑥) = 𝑥, 𝑥 ∈ [−1, ,1]
Answer: (d)

2 MARK QUESTIONS

1 2𝜋 2𝜋
Find the value of 𝑠𝑖𝑛−1 [𝑠𝑖𝑛 ( 3 )] + 𝑐𝑜𝑠 −1 [𝑐𝑜𝑠 ( 3 )].

2𝜋 𝜋
Solution: We have 𝑠𝑖𝑛−1 𝑠𝑖𝑛 ( 3 ) = 𝑠𝑖𝑛−1 𝑠𝑖𝑛 (𝜋 − 3 )

𝜋
= 𝑠𝑖𝑛−1 𝑠𝑖𝑛 (3 )
𝜋
= 3

2𝜋 2𝜋
𝑉𝑎𝑙𝑢𝑒 𝑜𝑓 𝑐𝑜𝑠 −1 𝑐𝑜𝑠 ( 3 ) = 3

−1 2𝜋 2𝜋 𝜋 2𝜋
 𝑠𝑖𝑛 𝑠𝑖𝑛 ( 3 ) + 𝑐𝑜𝑠 −1 𝑐𝑜𝑠 ( 3 ) = 3 + =𝜋
3

2 1
Find the value of: 𝑡𝑎𝑛−1 [2 𝑐𝑜𝑠( 𝑠𝑖𝑛−1 (2))]

Solution:

1 𝜋 √3 𝜋
𝑡𝑎𝑛−1 [2 𝑐𝑜𝑠( 𝑠𝑖𝑛−1 (2))]=𝑡𝑎𝑛−1 [2 𝑐𝑜𝑠( 6 )]=𝑡𝑎𝑛−1 [2 × ]=𝑡𝑎𝑛−1 √3= 3
2

3 Find the value of: 𝑡𝑎𝑛−1 [2 𝑠𝑖𝑛( 2 𝑐𝑜𝑠 −1 ( 2 ))]


√3

Solution:
√3 𝜋
𝑡𝑎𝑛−1 [2 𝑠𝑖𝑛( 2 𝑐𝑜𝑠 −1 ( 2 ))]=𝑡𝑎𝑛−1 [2 𝑠𝑖𝑛( 2 × 6 )]

√3 𝜋
= 𝑡𝑎𝑛−1 [2 × ]=𝑡𝑎𝑛−1 √3=3
2

4 1
If 𝑐𝑜𝑡 −1 (5) = 𝑥, 𝑡ℎ𝑒𝑛 𝑓𝑖𝑛𝑑 𝑡ℎ𝑒 𝑣𝑎𝑙𝑢𝑒 𝑜𝑓 sin 𝑥
,

31
1
Solution: 𝑐𝑜𝑡𝑥 = 5
5
 𝑠𝑖𝑛 𝑥 =
√26

5 −1
Find the value of 𝑠𝑖𝑛−1 ( 2 ) + 2 𝑐𝑜𝑠 −1 (
−√3
)
2

−1 1 𝜋
Solution: 𝑠𝑖𝑛−1 ( 2 ) = − 𝑠𝑖𝑛−1 (2) = − 6

−√3 √3 𝜋 5𝜋
𝑐𝑜𝑠 −1 ( ) = 𝜋 − 𝑐𝑜𝑠 −1 ( 2 ) = 𝜋 − 6 =
2 6

−1 −1 −√3 −𝜋 5𝜋 3𝜋
 𝑠𝑖𝑛 (2) + 2 𝑐𝑜𝑠 −1 ( )= +2× =
2 6 6 2
𝑥
6 Show that for |𝑥| < 1, 𝑠𝑖𝑛(𝑡𝑎𝑛−1 𝑥) = √1+𝑥 2

Solution:

Let 𝑡𝑎𝑛−1 𝑥 = 𝑦

 𝑡𝑎𝑛𝑦 = 𝑥
𝑥
 L H S = 𝑠𝑖𝑛 𝑦 = √1+𝑥 2 =𝑅 𝐻 𝑆

7 1
Prove that: 𝑡𝑎𝑛 (2 𝑠𝑖𝑛−1 4) =
3 4−√7
3

3
Solution: Let 𝑠𝑖𝑛−1 4 = 𝑥
3
 𝑠𝑖𝑛 𝑥 = 4

√7
 𝑐𝑜𝑠 𝑥 = 4

√7
𝑥 1−𝑐𝑜𝑠 𝑥 1− 4−√7
4
L H S = 𝑡𝑎𝑛 2 = √1+𝑐𝑜𝑠 𝑥 = √ √7
= = 𝑅𝐻𝑆
1+ 3
4

8 1 𝜋
Find the value of 𝑡𝑎𝑛 (2 𝑡𝑎𝑛−1 5 − 4 )
1
Solution: Let 𝑡𝑎𝑛−1 5 = 𝑥
1
 𝑡𝑎𝑛 𝑥 = 5
𝜋
1 𝜋 𝜋 𝑡𝑎𝑛 2𝑥−𝑡𝑎𝑛 −7
 𝑡𝑎𝑛 (2 𝑡𝑎𝑛−1 5 − 4 ) = 𝑡𝑎𝑛 (2𝑥 − 4 ) = 4
𝜋 =
1+𝑡𝑎𝑛 2𝑥 𝑡𝑎𝑛 17
4

1
2 𝑡𝑎𝑛 𝑥 2× 5
5
where, 𝑡𝑎𝑛 2 𝑥 = 1−𝑡𝑎𝑛2 𝑥 = 1 2
= 12
1−( )
5

32
9 −1 −1
Find the value of 𝑠𝑖𝑛−1 ( 2 ) + 2 𝑐𝑜𝑠 −1 ( 2 ) + 𝑡𝑎𝑛−1 (1)

−1 1 𝜋
Solution: 𝑠𝑖𝑛−1 ( 2 ) = − 𝑠𝑖𝑛−1 (2) = − 6

−1 1 𝜋 2𝜋
𝑐𝑜𝑠 −1 ( 2 ) = 𝜋 − 𝑐𝑜𝑠 −1 (2) = 𝜋 − 3 = 3
𝜋
𝑡𝑎𝑛−1 (1) == 4

−1 −1 −1 −𝜋 2𝜋 𝜋 3𝜋
 𝑠𝑖𝑛 (2) + 2 𝑐𝑜𝑠 −1 ( 2 ) + 𝑡𝑎𝑛−1 (1) = + +4=
6 3 4

10 3
Find the value of 𝑠𝑖𝑛 (2 𝑠𝑖𝑛−1 5)

3
Solution: Let 𝑠𝑖𝑛−1 (5) = 𝜃
3
 𝑠𝑖𝑛 𝜃 = 5

3
 𝑠𝑖𝑛 (2 𝑠𝑖𝑛−1 5) = 𝑠𝑖𝑛 2 𝜃 = 2 𝑠𝑖𝑛 𝜃 𝑐𝑜𝑠 𝜃
3 4 24
= 2 × 5 × 5 = 25

2 MARKS

1 5
Find the value of sin [2 cot −1 (− 12)]
120
Answer: − 169
2 𝜋 1
Find the value of 𝑡𝑎𝑛 [ 6 − 𝑡𝑎𝑛−1 ( )]
√3
Answer: 0
3 Find the value of 𝑠𝑖𝑛 (𝑐𝑜𝑡 −1 𝑥) in terms of 𝑥
1
Answer: √1+𝑥 2
4 4
Find the value of 𝑠𝑖𝑛 [𝑐𝑜𝑡 −1 (3)]
3
Answer: 5

5 7𝜋 7𝜋
Find the principal of 𝑡𝑎𝑛−1 𝑡𝑎𝑛 ( 6 ) + 𝑐𝑜𝑡 −1 𝑐𝑜𝑡 ( 6 )
𝜋
Answer: 3

33
CHAPTER: MATRICES
SYLLABUS: Concept, notation, order, equality, types of matrices, zero and identity matrix,
transpose of a matrix, symmetric and skew symmetric matrices. Operations on matrices:
Addition and multiplication and multiplication with a scalar. Simple properties of addition,
multiplication and scalar multiplication. Non- commutativity of multiplication of matrices and
existence of non-zero matrices whose product is the zero matrix (restrict to square matrices of
order 2). Invertible matrices and proof of the uniqueness of inverse, if it exists; (Here all
matrices will have real entries).

Definitions and Formulae:

• Matrix representation and order of matrix


• Types of Matrices
• Operations on Matrices
• Transpose of a Matrix
• Symmetric and Skew Symmetric Matrices
• Invertible Matrices
Order of Matrix:
If a matrix has 𝑚 𝑟𝑜𝑤𝑠 𝑎𝑛𝑑 𝑛 𝑐𝑜𝑙𝑢𝑚𝑛𝑠, then it is known as the matrix of order 𝑚 × 𝑛.
Representation of matrix
A general matrix of order 𝑚 × 𝑛 can be written as

= [𝑎𝑖𝑗 ]𝑚×𝑛 , 𝑤ℎ𝑒𝑟𝑒 𝑖 = 1,2, … 𝑚 𝑎𝑛𝑑 𝑗 = 1,2, … 𝑛

Types of Matrices:

Depending upon the order and elements, matrices are classified as:

• Column matrix
• Row matrix
• Square matrix
• Diagonal matrix
• Scalar matrix
• Identity matrix
• Zero matrix

34
Type of Matrix Definition Example
COLUMN A matrix is said to be a column 2
[ ] 0𝑟𝑑𝑒𝑟2 × 1
−9
MATRIX matrix if it has only one column
−√5
[ 0 ] 𝑜𝑟𝑑𝑒𝑟 3 × 1
−12
ROW MATRIX A matrix is said to be a row [14 26]order1×2
matrix if it has only one row [0 √7 12] order1×3
SQUARE MATRIX A matrix in which the number of 2 4
[ ]
6 −8
rows is equal to the number of
5 0 −8
columns, is said to be a square [0 1 14 ]
matrix. 7 −8 4

DIAGONAL A square matrix 𝐴 is said to be a 6 0 0


[0 √6 0]
MATRIX diagonal matrix if all its non-
0 0 9
diagonal elements are zero
SCALAR MATRIX A diagonal matrix is said to be a 5 0 0
[0 5 0 ]
scalar matrix if its diagonal
0 0 5
elements are equal
IDENTITY A square matrix in which all the 1 0 0
[0 1 0 ]
MATRIX elements in the diagonal are all
0 0 1
equal to one and rest are all zero 1 0
[ ]
is called an identity matrix. And 0 1

generally it is denoted by I.
ZERO MATRIX A matrix is said to be zero matrix 0 0 0
A=[ ]
0 0 0
or null matrix if all its elements
0 0 0
are zero. B=[0 0 0]
0 0 0

➢ OPERATION OF MATRICES:

ADDITION OF MATRICES: Let 𝐴 = [𝑎𝑖𝑗 ]𝑚×𝑛 𝑎𝑛𝑑 𝐵 = [𝑏𝑖𝑗 ]𝑚×𝑛 be two

matrices of the same order. Then 𝐴 + 𝐵 is defined to be the matrix of order of 𝑚 × 𝑛


obtained by adding corresponding elements of 𝐴 𝑎𝑛𝑑 𝐵

i.e 𝐴 + 𝐵 = [𝑎𝑖𝑗 + 𝑏𝑖𝑗 ]𝑚×𝑛

35
DIFFERENCE OF MATRICES: Let 𝐴 = [𝑎𝑖𝑗 ]𝑚×𝑛 𝑎𝑛𝑑 𝐵 = [𝑏𝑖𝑗 ]𝑚×𝑛 be two

matrices of the same order. Then 𝐴 − 𝐵 is defined to be the matrix of order of 𝑚 × 𝑛


obtained by subtracting corresponding elements of 𝐴 𝑎𝑛𝑑 𝐵

i.e 𝐴 − 𝐵 = [𝑎𝑖𝑗 − 𝑏𝑖𝑗 ]𝑚×𝑛

MULTIPLICATION OF MATRICES: The product of two matrices 𝐴 and 𝐵 is


defined if the number of columns of 𝐴 is equal to the number of rows of 𝐵.

Let = [𝑎𝑖𝑗 ]𝑚×𝑛 𝑎𝑛𝑑 𝐵 = [𝑏𝑗𝑘 ]𝑛×𝑝 . Then the product of the matrices 𝐴 and 𝐵

is the matrix 𝐶 of order 𝑚 × 𝑝

4 3
2 3 5
Example: Let 𝐴 = [ ] 𝑎𝑛𝑑 𝐵 = [6 9]
1 6 8
5 8

2×4+3×6+5×5 2×3+3×9+5×8
𝐴𝐵 = [ ]
1×4+6×6+8×5 1×3+6×9+8×8

8 + 18 + 25 6 + 27 + 40 51 73
=[ ]=[ ]
4 + 36 + 40 3 + 54 + 64 80 121

MULTIPLICATION OF A MATRIX BY A SCALAR:

Let 𝐴 = [𝑎𝑖𝑗 ]𝑚×𝑛 and 𝑘 is a scalar, then 𝑘𝐴 = 𝑘[𝑎𝑖𝑗 ]𝑚×𝑛 = [𝑘. 𝑎𝑖𝑗 ]𝑚×𝑛

𝟐 𝟒 −𝟓 𝟑(𝟐) 𝟑(𝟒) 𝟑(−𝟓)


Example: 𝑨 = [ ] ⟹ 𝟑𝑨 = [ ]=
𝒚 𝒛 𝒙 𝟑𝒚 𝟑𝒛 𝟑𝒙
𝟔 𝟏𝟐 −𝟏𝟓
[ ]
𝟑𝒚 𝟑𝒛 𝟑𝒙

➢ TRANSPOSE OF A MATRIX: If 𝐴 = [𝑎𝑖𝑗 ]𝑚×𝑛 be an 𝑚 × 𝑛 matrix, then the

matrix obtained by interchanging the rows and columns of 𝐴 is called the transpose of
𝐴. Transpose of the matrix 𝐴 is denoted by 𝐴′ or 𝐴T.

If 𝐴 = [𝑎𝑖𝑗 ]𝑚×𝑛 , then 𝐴′ = [𝑎𝑗𝑖 ]𝑛×𝑚

1 2 3 1 4 5
𝑇
Example:𝐴 = [4 7 9] ⟹ 𝐴 = [2 7 1]
5 1 0 3 9 0

➢ SYMMETRIC MATRIX: A square matrix If 𝐴 = [𝑎𝑖𝑗 ] is said to be symmetric if


𝐴𝑇 = 𝐴

36
2 5 12
Example:𝐴 = [ 5 7 3]
12 3 6

➢ SKEW-SYMMETRIC MATRIX: A square matrix 𝐴 = [𝑎𝑖𝑗 ] is said to be skew


symmetric matrix if 𝐴𝑇 = −𝐴.

0 5 −12
Example:𝐴 = [−5 0 −3 ]
12 3 0

➢ INVERTIBLE MATRICES: If A is a square matrix of order m, and if there exists


another square matrix B of the same order m, such that AB = BA = I, then B is called
the inverse matrix of A and it is denoted by A– 1. In that case A is said to be invertible.

➢ PROPERTIES OF MATRICES:

• 𝐴+𝐵 =𝐵+𝐴

• 𝐴−𝐵 ≠𝐵−𝐴

• 𝐴𝐵 ≠ 𝐵𝐴

• (𝐴𝐵)𝐶 = 𝐴(𝐵𝐶)

• (𝐴′ )′ = 𝐴

• 𝐴𝐼 = 𝐼𝐴 = 𝐴

• 𝐴𝐵 = 𝐵𝐴 = 𝐼, 𝑡ℎ𝑒𝑛 𝐴−1 = 𝐵 𝑎𝑛𝑑 𝐵 −1 = 𝐴

• 𝐴𝐵 = 0 ⟹ 𝑖𝑡 𝑖𝑠 𝑛𝑜𝑡 𝑛𝑒𝑐𝑒𝑠𝑠𝑎𝑟𝑦 𝑡ℎ𝑎𝑡 𝑜𝑛𝑒 of 𝑡ℎ𝑒 𝑚𝑎𝑡𝑟𝑖𝑥 𝑖𝑠 𝑧𝑒𝑟𝑜.

• 𝐴(𝐵 + 𝐶) = 𝐴𝐵 + 𝐴𝐶

• Every square matrix can possible to express as the sum of symmetric and
skew-symmetric matrices.

1 1 1
𝐴 = 2 (𝐴 + 𝐴′ ) + 2 (𝐴 − 𝐴′ ), where 2 (𝐴 + 𝐴′ ) is symmetric matrix and
1
(𝐴 − 𝐴′ ) is skew-symmetric matrices.
2

37
MULTIPLE CHOICE QUESTIONS
Q.NO QUESTIONS AND SOLUTIONS
1 A is 2×2 matrix and A=[𝑎𝑖𝑗 ] where 𝑎𝑖𝑗 = (𝑖 + 𝑗)2, then A is
4 9 9 4
(a) [ ] (b) [ ]
9 16 16 9
2 3
(c) [ ] (d) 𝑛𝑜𝑛𝑒 𝑜𝑓 𝑡ℎ𝑒 𝑎𝑏𝑜𝑣𝑒
3 2
Solution:
𝑎11 = 4 , 𝑎12 = 9, 𝑎21 = 9 , 𝑎22 = 16
4 9
Option (a) [ ]
9 16

2 A and B are two matrices such that 𝐴𝐵 = 𝐴 𝑎𝑛𝑑 𝐵𝐴 = 𝐵 then B2 is


(a) A (b) B (c) 0 (d) I
Solution:
B2 = 𝐵𝐵 = (𝐵𝐴)𝐵 = 𝐵(𝐴𝐵) = 𝐵𝐴 = 𝐵
Option: (b) B

3 x 1
If A = [ ] and A2 is unit matrix, then what is value of x?
1 0
(a) 1 (b) 2 (c) 0 (d) -1
Solution:
x 1 x 1 2
A2 = [ ][ ] = [x + 1 x ]
1 0 1 0 x 1
x 2 + 1 = 1 .So 𝑥 = 0
Option (c)0

4 1 1
A= [ ] , then A10 is
1 1
(a) 10A (b)9A (c) 29 A (d)210 A
Solution:
1 1 1 1 2 2
A2 = [ ] [ ] = [ ] =2I, A3 =4I=22I….. and so A10 = 29 I
1 1 1 1 2 2
Option: (c) 29 A
5 A is a 3 × 4 matrix. A matrix B is such that A'B and BA' are defined. Then the
order of B is
(a) 4 × 3 (b) 3 × 3 (c) 4 × 4 (d) 3 × 4
Solution:
Let 𝑂(𝐵) = 𝑚 × 𝑛 . A'B is defined. So 𝑚 = 3. BA' is defined. So 𝑛 = 4.
Option: (d) 3 × 4

6 2 3 −5
If 𝐴 = [𝑎𝐼𝐽 ] = [1 4 9 ] , then find a12 a21 + a22
0 7 −2
(a) 4 (b)12 (c) −4 (d) 7
Solution:
a12 a21 + a22 = 3 × 1 + 4 = 7
Option: (d) 7
7 0 0 4
The matrix A= [0 4 0] is a
4 0 0
(a) diagonal matrix (b)square matrix

38
(c) unit matrix (d) None of these
Solution:
Ans: (b) square marix
8 A and B are symmetric matrices of same order , then ABT − BAT is always
(a) symmetric matrix (b) skew symmetric matrix
(c) zero matrix (d) unit matrix
Solution: ( AB − BA ) = (BT )T AT − (AT )T BT = BAT − ABT
T T T

Option: (b)skew symmetric matrix


9 1 2
If A=[ ], then A2 − 3I is
2 1

(a) 2A (b)3A
(c) zero matrix (d) 2I
2 1 2 1 2 3 0 2 4
Solution: A = [ ][ ]−[ ]=[ ]
2 1 2 1 0 3 4 2
Option: (a)2A
10 5 x
If 𝐴 = [ ] and 𝐴 = 𝐴𝑇 , then
y 0
(a) 𝑥 = 0, 𝑦 = 5 (b) x + y = 5
(c) 𝑥 = 𝑦 (d) none of these
Solution:
5 x 5 y
[ ]=[ ]
y 0 x 0
Option: (c) 𝑥 = 𝑦
11 The number of all possible matrices of order 3 × 3 with each entry 0 or 1 is
(a) 32 (b) 64 (c) 512 (d) none of these
Solution:
There are nine places. Each can be filled in two ways. 29 ways
Option: (c) 512
12 A is a 2 × 2 matrix whose elements are given by
1 if i ≠ j
aij = {
0 , if i = j
Then value of A2 is

0 1 0 0 1 0 0 1
(a)[ ] (b) [ ] (c) [ ] (d) [ ]
1 0 0 1 0 1 1 1
0 1 0 1 0 1 1 0
Solution: In A=[ ] , A2 = [ ][ ]= [ ]
1 0 1 0 1 0 0 1
1 0
Option: (c) [ ]
0 1

13 P and Q are two different matrices order 3 × 𝑛 and 𝑛 × 𝑝, then the order of
matrix P×Q is
(a) 3 × 𝑝 (b) 𝑝 × 3 (c) 𝑛 × 𝑛 (d) 3×3
Solution:
By property of order of matrix, order of matrix P×Q is 3 × 𝑝
Option: (a) 3 × 𝑝
14 1 0 0 x 1
−y
If [0 1 0] [ ] = [2] , find value of 𝑥 + 2𝑦 + 3𝑧.
0 0 1 z 1
(a) 1 (b) 2 (c) 0 (d) −1
Solution:

39
x 1
[−y].= [2] 𝑥 = 1. 𝑦 = −2, 𝑧 = 1
z 1
Option: (c ) 0
15 A and B are two matrices such that AB exists, then which is true
(a) 𝐴𝐵 = 𝐵𝐴 (b) BA does not exist.
(c) BA may or may not exists. (d) none of these
Solution:
If O(A)=3 × 2 and O(B) is 2×3, ,then both AB and BA possible
If O(A)=1×2 and O(B) is 2×3, ,then AB possible ,but not BA
Option: (c) BA may or may not exist.
i
16 For a 3 × 3 matrix A = [aij ] whose elements defined by aij = j , then write
a12 + a21
2 1 5
(a) 5 (b) 2 (c) 1 (d) 2
Solution:
1 2 5
a11 = 2 a21 = 1. A12 + a21 = 2

5
Option: (d) 2
17 A is a matrix of order 2×3 and B is a matrix of order 3×2. 𝐶 = 𝐴𝐵 𝑎𝑛𝑑 𝐷 =
𝐵𝐴, then order of CD is
(a) 3×3 (b) 2×2 (c) 3×2 (d) CD not defined
Solution:
O(C )=2×2 and O(D )=3×3. The number of columns of A not equal to number
of rows of B. Therefor CD not defined
Option: (d) CD not defined
18 α β
𝐴=[ ] and A2 = I , then which of the following is correct?
γ −α
(a) 1 + α2 + βγ=0 (b) 1 − α2 + βγ=0
(c) 1 − α2 − βγ=0 (d) 1 + α2 − βγ=0
α β α β 1 0
Solution: [ ][ ] = [ ]
γ −α γ −α 0 1
α2 + βγ 0 1 0
[ 2 ]=[ ] α2 + βγ=1
0 α + βγ 0 1
Ans: (c) 1 − α2 − βγ=0

CHAPTER VIDEO LINK FOR MCQs SCAN QR CODE FOR


VIDEO

MATRICES https://youtu.be/DuFAHDTsDms

40
EXERCISE

1 Given that A and B are matrices of order 3 × 𝑛 and 𝑚 × 5 respectively, the order of
matrix 𝐶 = 5𝐴 + 3𝐵 is
(a) 3 × 5 (b) 5×3 (c )3×3 (d) 5×5

2 cosα −sinα
If A= [ ] is identity then value of 𝛼
sinα cosα
𝜋 𝜋 𝜋
(a) 4 (b) 2 (c ) 0 (d) 3
3 The elements of 3 × 3 matrix are given by
1
aij = (−3i + j)
2
Write the element a32
(a) -7/2 (b) 18 (c )27 (d) 3
4 4 3 −4
If A = [ ] and B=[ ], AB=
1 2 3
−7 2 7 2
(a)[ ] (b) (a)[ ] (c ) (a)[ ] (dα) (a)[ ]
2 7 2 2
5 cosα −sinα
A=[ ] and 𝐴 + AT = I .find value of α
sinα cosα

a) ᴨ/6 (b) ᴨ/3 (c) ᴨ (d) 3ᴨ/2

−7
Answers: 1. (a) 3 × 5 2. (c) 0 . 3. ( a) -7/2 4. (a)[ ]
2

5. (b) ᴨ/3

ASSERTION - REASON BASED QUESTIONS

Q.NO QUESTION AND ANSWER


In the following questions from 1 to 20 , a statement of Assertion(A) is followed
by a statement of Reason ( R).
Choose the correct answer out of the following choices.
(a) Both( A) and (R ) are true and (R ) is the correct explanation of (A).
(b) Both( A) and (R ) are true and (R ) is not the correct explanation of (A)
(c) (A) is true and (R ) is false
(d) (A) is false and (R ) is true
1 1
Assertion (A): matrix [2] is a column matrix
4

Reason (R): Any matrix of order 𝑛 × 1 is called column matrix

Solution:
1
Matrix[2] is of order 3×1. Both are correct and second is explanation for first
4
option (a)

41
2 Assertion (A): The sum of two square matrices is always commutative.
Reason (R): If A and B are two 𝑚 × 𝑛 matrices, then 𝐴 + 𝐵 = 𝐵 + 𝐴
Solution:
𝐴 + 𝐵 = 𝐵 + 𝐴. Both are correct and second is correct explanation for first.
option (a)

3 Assertion (A): For two matrices A and B , 𝐴𝐵 ≠ 𝐵𝐴


Reason (R): Matrix multiplication follows the commutative property.
Solution:
3 3
AB and BA are different.. e. g [1 2] [ ] = [8] and[ ][1 2], So first statement is
4 4
correct and second is wrong
option (c )

4 Assertion (A): For any two matrices of the same order, (𝐴 + 𝐵)𝑇 = 𝐴𝑇 + 𝐵 𝑇 .
Reason (R): For any two matrices such that AB is defined, then (𝐴. 𝐵)𝑇 = 𝐴𝑇 . 𝐵 𝑇
Solution:
By definition assertion is true and reason is false.
Option: (c)
5 1 0 1 0
Assertion (A): 𝐴 = [ ]. Then A10 =[ ].
0 1 0 1
Reason (R): A is unit matrix and A× 𝐴 = 𝐴
Solution:
Given matrix is unit matrix. By property of unit matrx, 𝐼 × 𝐼 = 𝐼.
Both are correct. and second is correct explanation for first

Option (a)
6 Assertion (A): If A is a matrix of order 3 × 3 and B is a matrix of order
2 × 3, Then order of AB is 3 × 3.

Reason (R): Product AB is possible if number of columns of A and number of


rows of B are equal .
Solution:
Here number of column of matrix is 3 and number of rows of B is 2. So AB not
possible. First statement is not correct. Second statement is correct
Option: (d)
7 Assertion (A):If A is a square matrix and A2 = A, then(A + I)3 − 7A = I
Reason (R): 𝐴𝐼 = 𝐼 = 𝐴𝐼 where I is unit matrix.
Solution:
(𝐼 + 𝐴)3 − 7A =I 3 + 3A2 + 3A + A3 − 7A = I + 3A + 3A + A − 7A = I
Since A2 = A and A3 = A
So both the statements are correct. Second statement is reason for first.
Option (a)

8 Assertion (A): If a matrix is skew symmetric, then its diagonal elements must be
zero.

Reason (R): A matrix A is skew symmetric if AT = A


Solution:
By definition of skew symmetric matrix 𝑎𝑖𝑗 = −𝑎𝑗𝑖 . So 𝑎𝑗𝑗 = −𝑎𝑗𝑗 . ℎ𝑒𝑛𝑐𝑒 𝑎𝑗𝑗 = 0
Both the statements are true and second is not the reason for first
Option (c)

42
9 1 2 5
Assertion (A): The matrix [2 −2 6] is symmetric matrix
5 6 0

Reason (R): A matrix A is symmetric if AT = A


Solution:
Here transpose of the given matrix is same matrix, So symmetric. Both the
statements are correct.
Option (a)

10 1 2 3
Assertion (A): The matrix [5 −7 8] can be expressed as sum of a symmetric
0 −1 9
and a skew symmetric matrices

Reason (R): If A and B, are skew symmetric matrices of same order, then AB is
symmetric if 𝐴𝐵 = 𝐵𝐴
Solution:
Assertion is correct, since any matrix A can be written as 𝐴 = 𝑃 + 𝑄
𝐴+𝐴𝑇 𝐴−𝐴𝑇
where 2 𝑎𝑛𝑑 𝑄 = 2 where P symmetric and Q skew symmetrices.
Reason is also correct, since (𝐴𝐵)𝑇 = 𝐵 𝑇 𝐴𝑇 = −𝐵 × −𝐴 = 𝐵𝐴.
AB is symmetric if 𝐴𝐵 = 𝐵𝐴 But second statement is not correct reason for first

Option: (b)

11 Assertion (A): The product of a matrix and the identity matrix is always the
original matrix.

Reason (R): Identity matrix is a square matrix in which all diagonal elements are
zeros and all other elements are unity

Solution:
Assertion(A) is true by property of unit matrix.
Reason( R) is wrong. Identity matrix is a square matrix in which all diagonal
elements are unity and all other elements are zeros
Option ( c)

12 Assertion (A): If two matrices have the same order, their addition is always
defined.

Reason (R): Matrix multiplication is defined only for matrices with the same
order..

Solution:
Assertion (A) is true, by condition for addition.
Reason (R) is not correct since
1 2 3 6 5 4
[ ][ ] not possible
4 5 6 3 2 1

Option ( c)
13 Assertion (A): The product of two non-square matrices can not be a square
matrix.

43
Reason (R): Matrix multiplication is defined for matrices if the number of
columns of first matrix is equal to number of rows of second matrix

Solution:
Assertion (A) not correct. Let A and B are matrices of orders 2 × 3 and 3×1
respectively, then AB is of order 2 ×1 which is not square matrix. Reason (R) is
correct.
Option: (d)

14 Assertion (A):If A and B are symmetric matrices of same order, 𝐴𝐵 − 𝐵𝐴 is


skew symmetric matrix
Reason (R): If A and B are symmetric matrices of same order, 𝐴𝐵 + 𝐵𝐴 is
symmetric matrix

Solution:
Both the statements are correct. . But second statement is not the reason for first
statement.
(𝐴𝐵 + 𝐵𝐴)𝑇 =(𝐴𝐵)𝑇 + (𝐵𝐴)𝑇 = 𝐵 𝑇 𝐴𝑇 + 𝐴𝑇 𝐵 𝑇 = 𝐵𝐴 + 𝐴𝐵 and
(𝐴𝐵 − 𝐵𝐴)𝑇 =(𝐴𝐵)𝑇 − (𝐵𝐴)𝑇 = 𝐵 𝑇 𝐴𝑇 − 𝐴𝑇 𝐵 𝑇 = 𝐵𝐴 − 𝐴𝐵 = −(𝐴𝐵 − 𝐵𝐴)
Option: (b)
15 2 0 5 0
Assertion (A): A= [ ] and B= [ ] , Then (𝐴 + 𝐵)2 = 𝐴2 + 2𝐴𝐵 + 𝐵 2
0 3 0 0

Reason (R): A and B for two matrices(A + B)2 = A2 + 2AB + B2 if AB=BA


Solution:
(A + B)2 = (A + B)(A + B) = A2 + AB + BA + B2
= A2 + 2AB + B2 only if AB = BA
both the statements are true and second statement is reason for first.
Option: (a)
EXERCISE

1 Assertion (A): product of a matrix and the identity matrix is always the original
matrix.

Reason (R): The identity matrix serves as the multiplicative identity for matrices.

2 Assertion (A): Two matrices A and B are of order 2 × 3 and 3 × 2 respectively.


Then order of AB is 2 × 2 and order of BA is 3 × 3.
Reason (R): Order of a matrix is 𝑚 × 𝑛 where m is the number of rows and m is
the number of columns
3 Assertion (A): Transpose of the matrix [1 −4 5] is a column matrix

Reason (R): Row matrix is of order 1 × 𝑚.

4 0 2 6
Assertion (A): The element 𝑎12 in the matrix[1 2 −1] is 1
2 2 3

Reason (R): 𝑎𝑖𝑗 is the element in ith row and jth column.

5 y 4 1 4
Assertion (A): [ ]=[ ], then 𝑥 = 8 𝑎𝑛𝑑 𝑦 = 1
3 8 3 x

44
Reason (R): Two matrices are equal , if they are of same order and elements in
same places are equal.

Answers
1. Option (a) 2.Option (b) 3.Option (b) 4. Option (d) 5.Option (a)

2 MARK QUESTIONS

1 1
A=[2] and B=[2 2 0] Find AB and BA
3

Solution:
2 2 0
AB=[4 4 0] and BA=[6]
6 6 0
2 𝑥+3 4 5 4
If [ ]=[ ] Find 𝑥 𝑎𝑛𝑑 𝑦
𝑦−4 𝑥+𝑦 3 9

Solution: 𝑥 + 3 = 5, 𝑦 − 4 = 3, 𝑥+𝑦 =9
𝑥 = 2 𝑎𝑛𝑑 𝑦 = 7
3 Construct a matrix of order 3 × 3 whose elements are given by
aij = 1 if i≠ j and aij = 0 if I = j.

Solution: a11= a22= a33= 0 . and other elements zero


0 1 1
[1 0 1]
1 1 0
4 0 2b −2
Matrix[ 3 1 3 ] is symmetric, then find a and b.
3a 3 −1

0 2b −2 0 3 3a
Solution: [ 3 1 3 ] = [ 2b 1 3 ]
3a 3 −1 −2 3 −1
T 3 2
A=A . 𝑏 = 2 𝑎𝑛𝑑 𝑎 = − 3
5 cos ∝ −sin ∝
If A=[ ] and A+AT=I. then find ∝
sin ∝ cos ∝

Solution: A+AT=I
cos ∝ −sin ∝ cos ∝ sin ∝ 1 0
[ ]+[ ]=[ ]
sin ∝ cos ∝ −sin ∝ cos ∝ 0 1
π
∝= 3
6 p 0 1 0
Let 𝐴 = [ ] 𝐵=[ ]. Find the value of 𝑝, if A2 = 𝐵.
1 1 5 1
p 0 p 0 1 0
Solution: [ ][ ]=[ ]
1 1 1 1 5 1
2
p 0 1 0
[ ]=[ ] . p2 = 1 and p + 1 = 5. So 𝑝 has no common value.
p+1 1 5 1

45
7 3
7 1 2 4
Evaluate [ ] [4] + 2 [ ]
9 2 1 2
5
3
7 1 2 35 4 8
Solution: [ ] [4] = [ ] and 2[ ] = [ ]
9 2 1 40 2 4
5
3
7 1 2 4 43
Therefore [ ] [4] + 2 [ ] = [ ]
9 2 1 2 44
5
8 1 −1
A=[ ] and A 2 = 𝑘𝐴. Find the value of 𝑘.
−1 1

1 −1 1 −1 2 −2 k −k
Solution: [ ][ ]=[ ]=[ ]
−1 1 −1 1 −2 2 −k k
𝑘=2
9 2
A=[3] and B=[1 2 6] Find 𝐴𝑇 𝐵 𝑇 .
6
1
Solution: [2 3 6] [2] =[2+6+36]=[44]
6
10 Construct a 2 × 3 matrix whose elements in ith row and jthcolumn are given by
i − j , if i ≥ j
aij = {
i + j, if i < 𝑗

Solution: a11 = 0, a12=3, ……….. a32 = 5


0 3 4
Ans:[ ]
1 0 5
11 A and B are matrices of order 3 × 4 and 4 × 3 respectively. Write the order of
𝐴𝑇 𝐵 𝑇 .

Solution: 1 order of AT is 4 × 3
order of BT is 3 × 4
Ans: Order of 𝐴𝑇 𝐵 𝑇 𝑖𝑠 4 × 4
12 If A and B is symmetric matrices of same order, show that AB is symmetric iff
𝐴𝐵 = 𝐵𝐴

Solution:
(AB)T =BT AT = BA
(AB)T = AB iff AB = BA
13 1 2 𝑥 1 −2 𝑦
If A=[0 1 0] and B=[0 1 0] and AB is identity matrix of order 3×3,then
0 0 1 0 0 1
find 𝑥 + 𝑦.
1 2 x 1 −2 y 1 0 0
Solution: [0 1 0] [0 1 0] = [0 1 0]
0 0 1 0 0 1 0 0 1
1 0 x+y 1 0 0
[0 1 0 ] = [0 1 0]
0 0 1 0 0 1
𝑥+𝑦 =0

46
14 2a + b a − 2b 4 −3
[ ]=[ ] find 𝑎 + 𝑏 − 𝑐 + 2𝑑
5c − d 4c + 3d 11 24

Solution:
2a + b = 4,
a − 2b = −3
5c − d = 11
4c + 3d = 24
𝑎 = 1, 𝑏 = 2, 𝑐 = 3, 𝑑 = 4. Ans: 𝑎 + 𝑏 − 𝑐 + 2𝑑 = 8
15 1 2 −2
A=[0 1 6 ] Find a12 a21 + a13 a31
3 4 1

Solution:
a12 a21 + a13 a31 = 2 × 0 ± 2 × 3 = −6

EXERCISE

1 1 2 3 0 2 −1
P=[ 2 3 4] and Q=[1 3 4 ] Find PQ and QP
−1 1 2 0 −2 −3
2 2 3
A=[ ] , Find A + AT and check whether A + AT is symmetric or not
4 5
3 α β
Given A=[ ] and A2 = 3I . Find the value of 3- α2 − βγ
γ α
4 6 5 11 0
Find a matrix B such that [ ] B=[ ]
5 6 0 11
5 a
c
[ ] [
Evaluate a b [ ] + a b c [ ] ] b
d
c
Answers:
2 2 −2 5 5 6
(1) PQ=[3 5 −2] and QP = [ 3 15 23 ]
1 −3 −1 −1 −9 −14
T 4 7
(2) A+ A =[ ]. Yes, it is symmetric. (3) zero
7 10
6 −5
(4) B=[ ] (5) [ac+bd+a2+b2+c2]
−5 6

3 MARK QUESTIONS

1 1 2 2
A=[ 2 1 x ] and AAT =9I. Find 𝑥
−2 2 −1
Solution:
AAT =9I
9 4 + 2x 0 9 0 0
[4 + 2x 5 + x 2 −2 − x] = [0 9 0]
0 −2 − x 9 0 0 9
𝑥 = −2

47
2 0 a −3
A=[2 0 −1] is a skew symmetric matrix, find a and b
b 1 0

Solution:
AT = −A
0 2 b 0 a −3
[ a 0 1 ]=− [ 2 0 −1]
−3 −1 0 b 1 0
𝑎 = −2 𝑎𝑛𝑑 𝑏 = 3

3 3 −2 −4
Express [ 3 −2 −5] as the sum of a symmetric and skew symmetric matrix
−1 1 2

Solution:
1 5
3 −2
2
A+AT 1
P= = −2 −2
2 2
5
[− 2 −2 2 ]
5
0 − 2 −4
A−AT
Q= 2 = [ 5 0 −3]
2
−1 3 0
1 5 5 3
3 −2 0 −2 −2
2 3 −2 −4
1 5
−2 −2 + 0 −3 =[ 3 −2 −5]
2 2
5 3 −1 1 2
[− 2 −2 2] [2 3 0]
P+Q=A
4 2 −3
Show that 𝐴 = [ ] satisfies the equation x 2 − 6x + 17I = 0
3 4

Solution:
−5 −18
A2 = [ ]
18 7
12 −18 17 0
6A= [ ]and 17I=[ ]
18 24 0 17
−5 −18 12 −18 17 0 0 0
X 2 − 6x + 17 = [ ]−[ ]+[ ]=[ ]=𝟎
18 7 18 24 0 17 0 0
5 A trust fund has Rs 30,000 that must be invested in two different types of bonds. The
first bond pays 5% interest per year, and the second bond pays 7% interest per year.
Using matrix multiplication, determine how to divide Rs 30,000 among the two
types of bonds,if the trust fund must obtain an annual total interest of Rs 1,800.

Solution:
Let Investment in 1st bond = Rs x
So, Investment in 2nd bond = Rs 30,000−x
5

[x 30000 − x] [100
7 ] = 1800
100
x=15000

48
6 7 0 3 0
𝑋+𝑌 =[ ] and 𝑋 − 𝑌 = [ ] Find matrices 𝑋 𝑎𝑛𝑑 𝑌.
2 5 0 3

Solution: Add both, we get


10 0
2X=[ ]
2 8
5 0
X= [ ]
1 4
2 0
Y= [ ]
1 1
7 2 4 5 4
A=[ ] ,B = [ ]. Verify (AB)T =BT AT
3 5 3 2

Solution:
22 16 22 30
AB=[ ] , (AB)T=[ ]
30 22 16 22
5 3 2 3 22 30
BT AT = [ ][ ]=[ ]
4 2 4 5 16 22
8 4 2
A= [ ] , then show that (𝐴 − 2𝐼)(𝐴 − 3𝐼) = 0
−1 1

Solution:
2 2 1 2
𝐴 − 2𝐼 = [ ] , 𝐴 − 3𝐼 = [ ]
−1 −1 −1 −2
0 0
(𝐴 − 2𝐼)(𝐴 − 3𝐼) = [ 2 2
][
1 2
]=[ ]
−1 −1 −1 −2 0 0
9 Let A and B be symmetric matrices of the same order,then show that
(i)𝐴 + 𝐵 is symmetric
(ii)𝐴𝐵 − 𝐵𝐴 is skew symmetric
(iii) 𝐴𝐵 + 𝐵𝐴 is symmetric

Solution:
(A + B)T = AT + BT = 𝐴 + 𝐵
(AB − BA)T = (AB)T − (BA)T =BT AT − AT BT = BA − AB = −(AB − BA)
(AB + BA)T = (AB)T + (BA)T =BT AT + AT BT = BA + AB = (AB + BA)

10. Find the matrix X such that:


1 2 3 −7 −8 −9
X[ ]=[ ]
4 5 6 2 4 6

Solution:
a b 1 2 3 −7 −8 −9
[ ][ ]=[ ]
c d 4 5 6 2 4 6
𝑎 + 4𝑏 = −7 , 2𝑎 + 5𝑏 = −8, 𝑐 + 4𝑑 = 2, 2𝑐 + 5𝑑 = 4.
Solving 𝑎 = 1, 𝑏 = −2, 𝑐 = 2 𝑎𝑛𝑑 𝑑 = 0
1 −2
𝑋=[ ]
2 0

EXERCISE

1 Find a matrix A such that 2𝐴 − 3𝐵 + 5𝐶 = 0 where


−2 2 0 2 0 1
𝐵=[ ] and C = [ ]
3 1 2 7 1 6

49
2 Find the values of 𝑥, 𝑦, 𝑎 𝑎𝑛𝑑 𝑏 when
2x + 3y a − 2b 3 8
[ ]=[ ]
2a + b 3x − 2y 6 11
3 1 −1 x 1
If A=[ ] and B = [ ] and (A + B)2 = A2 + B2 . Find 𝑥
2 −1 4 −1
4 0
If A=[1] , B = [1 5 7], then verify (AB)T = BT AT
2

5 x 2 −3 3 −1 2 5 3 3
[5 y 2 ] [4 2 5] = [19 −5 16]. Find 𝑥 𝑎𝑛𝑑 𝑦
1 −1 1 2 0 3 1 −3 0

Answers:
5
(1) A=[ −8 3 ] −2 (2) 𝑎 = 4 , 𝑥 = 3, 𝑦 = −1𝑎𝑛𝑑 𝑏 = −2
−13 −1 −12
(3 ) 𝑥 = 1 (5) 𝑥 = 1 𝑦 = 0

5 MARK QUESTIONS

1 1 0 2
1.If A=[0 2 1] and 𝐴3 − 6𝐴2 +7A+k𝐼 = 0. Find k.
2 0 3

Solution
1 0 2 5 0 8
𝐴 = [0 2 1] and 𝐴2 = [2 4 5 ]
2 0 3 8 0 13
Sub in eqn 𝐴3 − 6𝐴2 +7A+k𝐼
21 0 34 5 0 8 1 0 2 𝑘 0 0 0 0 0
= [12 8 23] − 6 [2 4 5 ] + 7 [0 2 1] + [ 0 𝑘 0 ] = [ 0 0 0]
34 0 55 8 0 13 2 0 3 0 0 𝑘 0 0 0
−2 0 0 𝑘 0 0
[ 0 −2 0 ] + [0 𝑘 0] = [0]
0 0 −2 0 0 𝑘
𝑘=2
2 1 0 2 𝑥
Find the value of x, . if [𝑥 −5 −1] [0 2 1] [4] = 0. Hence
2 0 3 1
𝑥
find[𝑥 −5 −1 4]
] [
1

Solution:
1 0 2
[𝑥 −5 −1] [0 2 1]=[𝑥 − 2 −10 2𝑥 − 8]
2 0 3
𝑥
[𝑥 − 2 −10 2𝑥 − 8] [4] =[x2-48]
1
[x2-48]=0
x=±4√3
𝑥
[𝑥 −5 −1] [4]=[48-20-1]=[27]
1

50
3 2
−1 −1 −8 −10
Find a matrix A such that[ 1 0 ] 𝐴 = [ 1 −2 −5 ]
−3 4 9 22 15

Solution:
𝑥 𝑦 𝑧
Let A=[ ]
𝑎 𝑏 𝑐
2𝑥 − 𝑎 2𝑦 − 𝑏 2𝑧 − 𝑐 −1 −8 −10
Getting [ 𝑥 𝑦 𝑧 ] = [ 1 −2 −5 ]
−3𝑥 + 4𝑎 −3𝑦 + 4𝑏 −3𝑧 + 4𝑐 9 22 15
Equating corresponding elements, getting equations in variable.
2𝑥 − 𝑎 = −1, 𝑥 = 1, −3𝑥 + 4𝑎 = 9, 2𝑦 − 𝑏=-8, y=-2, −3𝑦 + 4𝑏 = 22
2𝑧 − 𝑐 = −10, z=-5, −3𝑧 + 4𝑐 = 15
1 −2 −5
A=[ ]
3 4 0
𝛼
4 0 −𝑡𝑎𝑛 2
If A=[ 𝛼 ] and I is the identity matrix of matrix of order2, show that
𝑡𝑎𝑛 2 0
𝑐𝑜𝑠𝛼 −𝑠𝑖𝑛𝛼
𝐼 + 𝐴 = (𝐼 − 𝐴) [ ]
𝑠𝑖𝑛𝛼 𝑐𝑜𝑠𝛼

Solution:
𝛼 1−𝑡 2 2𝑡
Let 𝑡𝑎𝑛 2 =t, then 𝑐𝑜𝑠𝛼 = 1+𝑡 2 and 𝑠𝑖𝑛𝛼 =1+𝑡 2
1 0 0 −𝑡 1 −𝑡
I+A =[ ]+[ ]=[ ]
0 1 𝑡 0 𝑡 1 2
1−𝑡 2𝑡 1−𝑡 2 2𝑡
1 0 0 −𝑡 2 − 1+𝑡 2 1 𝑡 1+𝑡 2 − 1+𝑡 2
RHS= ([ ]−[ ] ) [1+𝑡 1−𝑡 2
] =[ ] [ 2𝑡 1−𝑡 2
]
0 1 𝑡 0 2𝑡 −𝑡 1
1+𝑡 2 1+𝑡 2 1+𝑡 2 1+𝑡 2

1 −𝑡
=[ ]
𝑡 1

EXERCISE

1 3 −2 1 0
If A=[ ] and I=[ ], find 𝑘 such that A2= 𝐾𝐴 − 2𝐼
4 −2 0 1

2 2 3 2 −2
2𝑋 + 3𝑌 = [ ] 𝑎𝑛𝑑 3𝑋 + 2𝑌 = [ ]. Find the matrices X and Y
4 0 −1 5
3 5 3 4 −1 2 1
A=[2 1] B=[ ] C=[ ] Prove that A(B+C)=AB+AC
8 7 6 1 0 2

Answers:
2 12 2 13

(1) 𝑘 = 1 (2) 𝑋 = [ 511 5
] 𝑎𝑛𝑑 𝑌 = 3
[14 5
]
− 3 −2
5 5

51
CASE BASED QUESTIONS

1 A manufacture produces three types of emergency lamps which he sells in two


markets. Their Monthly sales are indicated below
Type 1 Type 2 Type 3
Market A 100 100 50
Market B 80 100 100
If the unit Sale price of the three types of emergency lights are 2000, 3000 and
2500 respectively, and unit cost of the above three commodities are Rs.
1500,2200, and Rs. 2000 respectively, then based on the above information
answer the following.
(i) Which of the following gives the total revenue of market A
2000 1500
(a) [100 100 50] [3000] (b) [100 100 50] [ 2200 ]
2500 2000
100 100
(c) [100] 2000 3000 2500 (d) [100] [1500 2200 2000]
[ ]
50 50
Solution:
total revenue of market A
=100 × 2000 + 100 × 3000 + 50 × 2500
2000
Which is in matrix form[100 100 50] [3000]
2500
2000
option: (a) [100 100 50] [3000]
2500

(ii) Total revenue of market B


(a) ₹470,000 (b) ₹155,000
(c) ₹520,000 (d) ₹ 710,000
Solution:
Total revenue of market B= 80× 2000 + 100 × 3000 + 100 × 2500
= 710000
Option :(d) ₹ 7,10,000

(iii)Total profit of market B is …


(a) 470,000 (b) 155,000
(c) 625000 (d) 170,000
Solution:
Total profit of market B is 80× 500 + 100 × 800 + 100 × 500 =170000
Option :(d) ₹170,000

(iv) Which of the following gives the total profit of market A.


100 80
(a) [100] [500 800 500] (b) [ 100 ] [1500 2200 2000]
50 100
500 500
(c) [100 100 50] [ 800 ] (d) [80 100 100] [800]
500 500
Solution:

52
Total profit of market A =100× 500 + 100 × 800 + 50 × 500 which can be
500
represented in matrix form as [100 100 50] [ 800 ]
500
500
Option :(c) [100 100 50] [800]
500
OR
Find gross profit in both market
(a) Rs.325,000 (b ) Rs. 90,000 (c) Rs. 696000 (d) None of the above
Profit of market A=155000
Profit of market B=170000
Obtion:(a) ₹325,000
2 Ashish wants to purchase a rectangular plot from his neighbour to construct a
house. He asked about the dimensions of the plot , his neighbour told that if the
length is decreased by 50m and the breadth is increased by 50m, then the area
will remain the same, but If the length is decreased by 20m and breadth is
increased by 30 m ,the area will increase by 1400m2 .
Based on the information given above, answer the following questions
(i) Let 𝑥 𝑎𝑛𝑑 𝑦 denote the length and breadth of the plot, then equations in terms
of 𝑥 𝑎𝑛𝑑 𝑦 are.
(a) 𝑥 + 𝑦 = 50; 3𝑥 + 2𝑦 = 200 (b) 𝑥 − 𝑦 = 50; 3𝑥 − 2𝑦 = 200
(c) 𝑥 + 𝑦 = 50; 3𝑥 − 2𝑦 = 200 (d) 𝑥 − 𝑦 = 50; 3𝑥 + 2𝑦 = 200
Solution:
(𝑥 − 50)(𝑦 + 50) = 𝑥𝑦 𝑎𝑛𝑑 (𝑥 − 20)(𝑦 + 30) = 𝑥𝑦 + 1400 on simplification
we get
𝑥 − 𝑦 = 50 𝑎𝑛𝑑; 3𝑥 − 2𝑦 = 200
Option :(b) 𝑥 − 𝑦 = 50; 3𝑥 − 2𝑦 = 200

(ii) Which of the following matrix equations is represented by the given


information?
1 −1 x 200 3 −2 x 50
(a)[ ] [y] = [ ] (b)[ ][ ] = [ ]
3 −2 50 1 −1 y 200
3 −2 y 200 3 −2 x 200
(c)[ ][ ] = [ ] (d)[ ] [y] = [ ]
1 −1 x 50 1 −1 50
Solution:
3 −2 x 200
3𝑥 − 2𝑦 = 200 𝑎𝑛𝑑 𝑥 − 𝑦 = 50 So [ ][ ] = [ ] is matrix form
1 −1 y 50
3 −2 x 200
Option :(d)[ ][ ] = [ ]
1 −1 y 50
3 −2
(iii) If A=[ ] , Find AAT
1 −1
Solution:
3 −2 3 1 13 5
AAT = [ ][ ]=[ ]
1 −1 −2 −1 5 2
13 5
Option : [ ]
5 2
1 −1 200
(iv) If P=[ ] and Q=[ ] Find PQ and QP
3 −2 50
Solution:
1 −1 200 150
PQ=[ ][ ]=[ ].
3 −2 50 500

53
150
Option : PQ= [ ]
500
Number of columns in Q is not equal to number rows in P. So QP not possible
3 Number of girls and boys of three sections of XI are given below
Section No of Boys No of Girls
11A 20 25
11B 25 23
11C 23 24
Fee for girls per month is ₹1000 and for boys ₹1200.
Based on the above the information answer the following questions
(i) Which of the following gives the total amount of fee of paid by class 11A?
1200 1200
(a)[20 25] [ ] (b)[25 23] [ ]
1000 1000
1000 20
(c)[23 24] [ ] (d)[ ] [1000 1200]
1200 25
Solution
Total amount of fee paid by class 11A= 20× 1200 + 25 × 1000.
1200
Matrix form is[20 25] [ ]
1000
1200
Option : (a)[20 25] [ ]
1000

(ii) Which of the following will not give the total fee paid by sections A and B
together is
45 48
(a)[1200 1000] [ ] (b)[1000 1200] [ ]
48 45
1200 1000
(c)[45 48] [ ] (d)[45 48] [ ]
1000 1200
Solution:
The total fee paid by sections A and B together is 45× 1200 + 48 × 1000
1000
Option :(b)[45 48] [ ]
1200

(iii) What is the total fee paid by all boys of class11?


OR
What is the total fee paid by students of class 11C?
Solution
(iii)20× 1200 + 25 × 1200 + 23 × 1200 = 24000+30000+27600 = ₹81600
OR
23 × 1200 + 24 × 1000 = ₹51600
4 In farewell party ,students of class12 A,B and C decided to have some dress
code. Girls of 12 A ,B and C selected red ,green and blue saries of rate
2000,2500 and 2500 respectively from a shop and boys of each section
purchased the dress of same colour matching with girls of the same section and
of rate 3000,2800 and3000 respectively. Number of girls and boys of three
sections of XI are given below
Section No of Boys No of Girls
A 20 20
B 25 20
C 20 25
Based on the above data, answer the following questions

(i) Which of the following product gives the total amount paid by girls of all
sections

54
3000 2000
(a) [20 25 20] [ 2500 ] (b)[20 20 25] [ 2500 ]
2500 2500
2000 3000
(c) [20 25 20] [ 2500 ] (d)[20 20 25] [ 2500 ]
3000 2500
Solution:
the total amount paid by girls of all sections=
20 × 2000 + 20 ×2500+25× 2500 which is same as
2000
[20 20 25] [ 2500 ]
2500
3000
Option : (b) [20 20 25] [ 2500 ]
2500

(ii) Which of the following product gives the total amount paid by boys of all
sections
3000 2000
[ ]
(a) 20 25 20 [2800] ]
(b)[20 20 25 [2800]
3000 3000
2000 3000
(c) [20 25 20] [2800] (d)[20 20 25] [2800]
3000 2500
Solution:
the total amount paid by boys of all sections=
20 × 3000 + 25 ×2800+20× 3000 which is obtained by
3000
[20 25 20] [2800]
3000
3000
Option (a) [20 25 20] [ 2800 ]
3000

(iii) Amount paid by students of A


3000 2500
(a)[20 20] [ ] (b)[25 20] [ ]
2000 2500
2500 20
(c)[20 25] [ ] (b)[2500 2500] [ ]
3000 25
Solution:
Amount paid by students of A=20 × 2000 + 20 ×3000
3000
Which is represented by[20 20] [ ]
2000
3000
Option : (a)[20 20] [ ]
2000
3000 2000
(iv) A=[2500 2500] Find a11 + a32
2500 3000
(a) 4500 (b) 5000 (c ) 5500 (d) 6000
Solution:
a11 + a32 = 3000 + 3000
Option -:(d) 6000
5 Sushama owns a P.G for girls. One day she went to market purchase the food
items. She bought 4kg onion,3 kg wheat, and 2kg rice for Rs 560.Next day she
bought 2kg onion,4kgwheat and 6 kg rice. It cost her Rs:780. Another day she
bought 6 kg onion,2kg wheat and3 kg rice which cost Rs: 640.

55
(i) Convert the given condition above in matrix equation of the form AX=B
Solution:
4 3 2 x 560
[2 4 6] [y]=[780]
6 2 3 z 640

(ii) Find A+AT . Is it symmetric?


Solution:
4 3 2 4 2 6 8 5 8
[2 4 6] + [3 4 2]=[5 8 8].It is symmetric
6 2 3 2 6 3 8 8 6

Find a matrix P such that P = A2 − 5A


OR
3
(iii) Find A
Solution:
4 3 2 4 3 2 4 3 2
A2 − 5A=[2 4 6] [2 4 6] − 5 [2 4 6]
6 2 3 6 2 3 6 2 3
34 28 32 20 15 10 14 13 22
=[52 34 46]—[10 20 30]=[42 14 16]
46 32 33 30 10 15 16 22 18
OR
34 26 32 4 3 2 348 278 332
A3 =[52 34 46] [2 4 6] = [552 384 446]
46 32 33 6 2 3 446 332 383

EXERCISE

1 Consider 2 families A and B. Suppose there are 4 men, 4 women and 4 children
in family A and 2 men, 2 women and 2 children in family B. The recommend
daily amount of calories is 2400 for a man, 1900 for a woman, 1800 for a
children and 45 grams of proteins for a man, 55 grams for a woman and 33
grams for children.

(i) Requirement of calories of family A is


(a) 24000 (b) 24400 (c) 15000 (d) 15800
(ii) Requirement of proteins for family B is
(a) 560 grams (b) 332 grams
(c) 266 grams (d) 300 grams
2400 45
4 4 4
(iii) If A=[ ] , B=[1900] and C=[55] Then Find A(B+C)
2 2 2
1800 33

56
(iv) Which of the following product gives the total calories required for
children in both the families
1800 1900 1900
(a)[4 2] [ ] (b) [4 2] [ ] (c)[2 4] [ ] (d) none of
1800 1800 1800
the above.
2 Three schools DPS, CVC and KVS have decided to organize a fair for
collecting money for helping the flood victims. They sold handmade fans, mats
and plates from recycled material at a cost of Rs. 25, Rs.100 and Rs. 50 each
respectively. The numbers of articles sold are given
School /Article DPS CVC KVS
Handmade fans 40 25 35
Mats 50 40 50
Plates 20 30 40
Based on the information given above, answer the following questions
(i) What is the total money (in Rupees) collected by the school DPS?
40 50 20 25
(ii) If A=[25 40 30] and B=[100] Find AB.
35 50 40 50
OR
T
Find AA
(iii) What is the total amount of money collected by all three schools
DPS, CVC and KVS?

Answers:
24932 1800
1. (i) (b) 24400 (ii). (c) 266 grams (iii). [ ] (iv) .(𝑎)[4 2] [ ]
12466 1800
7000 4500 3600 4700
2. (i) ₹7000 (ii)𝐴𝐵 = [6125] OR AAT = [3600 3125 4075]
7875 4700 4075 5325

(iii) ₹21000

57
CHAPTER: DETERMINANTS
SYLLABUS: Determinant of a square matrix (up to 3 x 3 matrices), minors, co-factors and
applications of determinants in finding the area of a triangle. Adjoint and inverse of a square
matrix. Consistency, inconsistency and number of solutions of system of linear equations by
examples, solving system of linear equations in two or three variables (having unique solution)
using inverse of a matrix.

Definitions and Formulae:


To every square matrix we can assign a number called determinant

➢ Determinant:

𝑎 𝑏
• 𝐿𝑒𝑡 𝐴 = [ ] , 𝑡ℎ𝑒𝑛 𝑑𝑒𝑡(𝐴) = |𝐴| = 𝑎𝑑 − 𝑏𝑐
𝑐 𝑑
𝑎 𝑏 𝑐
𝑑 𝑓 𝑑 𝑒
• 𝐿𝑒𝑡 𝐴 = [𝑑 𝑒 𝑓 ] , 𝑡ℎ𝑒𝑛 |𝐴| = 𝑎 |𝑒 𝑓
|−𝑏| |+𝑐| |
ℎ 𝑘 𝑔 𝑘 𝑔 ℎ
𝑔 ℎ 𝑘

• For easier calculations, we shall expand the determinant along that row or
column which contains maximum number of zeros

• The area of a triangle whose vertices are (x1 , y1 ), (x2 , y2) and (x3 , y3), is

𝑥1 𝑦1 1
1
Δ = 2 | 𝑥2 𝑦2 1|
𝑥3 𝑦3 1

Since area is a positive quantity, we always take the absolute value of the
determinant

• The area of the triangle formed by three collinear points is zero.

• Equation of line joining the points (x1 , y1 ) and (x2 , y2) is


𝑥 𝑦 1
| 𝑥1 𝑦1 1 |=0
𝑥2 𝑦2 1
➢ Minors: Minor of an element 𝑎𝑖𝑗 of a determinant is the determinant obtained by
deleting its ith row and jth column in which element 𝑎𝑖𝑗 lies. Minor of an element 𝑎𝑖𝑗
is denoted by 𝑀𝑖𝑗 .

➢ Co-Factors: Cofactor of an element 𝑎𝑖𝑗 , denoted by 𝐴𝑖𝑗 is defined by

𝐴𝑖𝑗 = (−1)𝑖+𝑗 . 𝑀𝑖𝑗 , where 𝑀𝑖𝑗 is minor of 𝑎𝑖𝑗

➢ The value of a determinant Δ = sum of the product of elements of any row (or
column) with their corresponding cofactors.

58
➢ If elements of a row (or column) are multiplied with cofactors of any other row (or
column), then their sum is zero. For example a11A31+a12A32+a13A33 =0

➢ Adjoint of a Matrix: The adjoint of a square matrix 𝐴 = [𝑎𝑖𝑗 ] is defined as the


transpose of the matrix [𝐴𝑖𝑗 ] , where 𝐴𝑖𝑗 is the cofactor of the element 𝑎𝑖𝑗 .

Adjoint of the matrix A is denoted by adj A.

➢ To find adjoint of a 2×2 matrix interchange the diagonal elements and change the sign
of non – diagonal elements.

➢ Inverse of a Matrix: Let A be a square matrix.

1
𝐴−1 = 𝑎𝑑𝑗𝐴
|𝐴|

➢ Solution of system of linear equations by using matrix method:

Let the system of linear equations be

𝑎1 𝑥 + 𝑏1 𝑦 + 𝑐1 𝑧 = 𝑑1

𝑎2 𝑥 + 𝑏2 𝑦 + 𝑐2 𝑧 = 𝑑2

𝑎3 𝑥 + 𝑏3 𝑦 + 𝑐3 𝑧 = 𝑑3

These equations can be written as

𝑎1 𝑏1 𝑐1 𝑥 𝑑1
[𝑎 2 𝑏2 𝑦
𝑐2 ] [ ] = [𝑑2 ]
𝑎3 𝑏3 𝑐3 𝑧 𝑑3

𝐴𝑋 = 𝐵

𝑋 = 𝐴−1 𝐵

• 𝐴−1 𝑒𝑥𝑖𝑠𝑡𝑠, 𝑖𝑓 |𝐴| ≠ 0 𝑖. 𝑒 𝑡ℎ𝑒 𝑠𝑜𝑙𝑢𝑡𝑖𝑜𝑛 𝑒𝑥𝑖𝑠𝑡𝑠 𝑎𝑛𝑑 𝑖𝑡 𝑖𝑠 𝑢𝑛𝑖𝑞𝑢𝑒.

• 𝑇ℎ𝑒 𝑠𝑦𝑠𝑡𝑒𝑚 𝑜𝑓 𝑒𝑞𝑢𝑎𝑡𝑖𝑜𝑛𝑠 𝑖𝑠 𝑠𝑎𝑖𝑑 𝑡𝑜 𝑏𝑒 𝑐𝑜𝑛𝑠𝑖𝑠𝑡𝑒𝑛𝑡 𝑖𝑓 𝑡ℎ𝑒 𝑠𝑜𝑙𝑢𝑡𝑖𝑜𝑛 𝑒𝑥𝑖𝑠𝑡𝑠.

• 𝑖𝑓 |𝐴| = 0 , 𝑡ℎ𝑒𝑛 𝑤𝑒 𝑐𝑎𝑙𝑐𝑢𝑙𝑎𝑡𝑒 (𝑎𝑑𝑗𝐴)𝐵.

• 𝐼𝑓 |𝐴| = 0 𝑎𝑛𝑑 (𝑎𝑑𝑗𝐴)𝐵 ≠ 𝑂 , (O being zero matrix), then solution does not
exist and the system of equations is called inconsistent.

• 𝐼𝑓 |𝐴| = 0 𝑎𝑛𝑑 (𝑎𝑑𝑗𝐴)𝐵 = 𝑂, then system may be either consistent or


inconsistent according as the system have either infinitely many solutions or
no solution.

➢ Important notes:

• The matrix A is singular if |𝐴| = 0

59
• A square matrix A is said to be non-singular if |𝐴| ≠ 0

• If A and B are nonsingular matrices of the same order, then AB and BA are also
nonsingular matrices of the same order

• If A is an invertible matrix, then |𝐴| ≠ 0 and (A-1) T = (AT)-1

• |𝜆𝐴| = 𝜆𝑛 |𝐴|, 𝑤ℎ𝑒𝑟𝑒 𝑛 = 𝑜𝑟𝑑𝑒𝑟 𝑜𝑓 𝑚𝑎𝑡𝑟𝑖𝑥 𝐴

• 𝐴(𝑎𝑑𝑗𝐴) = (𝑎𝑑𝑗𝐴)𝐴 = |𝐴|𝐼

• |𝑎𝑑𝑗𝐴| = |𝐴|𝑛−1 , 𝑤ℎ𝑒𝑟𝑒 𝑛 = 𝑜𝑟𝑑𝑒𝑟 𝑜𝑓 𝑚𝑎𝑡𝑟𝑖𝑥 𝐴

• |𝐴(𝑎𝑑𝑗𝐴)| = |𝐴|𝑛 , 𝑤ℎ𝑒𝑟𝑒 𝑛 = 𝑜𝑟𝑑𝑒𝑟 𝑜𝑓 𝑚𝑎𝑡𝑟𝑖𝑥 𝐴

• |𝐴𝐵| = |𝐴||𝐵|

• (𝐴𝐵)−1 = 𝐵 −1𝐴−1

• |𝐴−1 | = |𝐴|−1

• |𝐴𝑇 | = |𝐴|

• If A and B are square matrices of the same order, then 𝑎𝑑𝑗(𝐴𝐵) = (𝑎𝑑𝑗𝐵). ( 𝑎𝑑𝑗 𝐴)

MULTIPLE CHOICE QUESTIONS


Q.No QUESTIONS WITH SOLUTIONS
1 If A is a square matrix of order 3 such that |A| = - 5 , then value of |−A| is

(a) 125 (b) - 125 (c) 5 (d) – 5

Solution:-
|−𝐴| =(-1)3|𝐴|
= - (-5)
=5
Correct option: c

2 𝑐𝑜𝑠15 𝑠𝑖𝑛15
Evaluate | |
𝑠𝑖𝑛75 𝑐𝑜𝑠75
√3 1
(a) 1 (b) 0 (c) 2 (d) 2
𝑐𝑜𝑠15 𝑠𝑖𝑛15
Solution:- | |=cos15cos75-sin15sin75
𝑠𝑖𝑛75 𝑐𝑜𝑠75
=cos(15+75)
=cos900
=0
Correct option: b

3 What positive value of x makes the following pair of determinants equal


2𝑥 3 16 3
| |,| |
5 𝑥 5 2

60
(a) 4 (b) 8 (c) 2 (d) ±4

Solution:-
2𝑥 3 16 3
| |=| |
5 𝑥 5 2
2x2-15 = 32-15
=17
2
2x =32
x2=16
x=±4
The positive value of x is 4

Correct option: a

4 If A is a square matrix of order 3 such that |𝑎𝑑𝑗𝐴|=64, then what is the value of
|𝐴|
(a)64 (b) 8 (c) −8 (d) ±8

Solution:-|𝑎𝑑𝑗𝐴|=|𝐴|2 =64
|𝐴|= ±8
Correct option : d
5 If for a square matrix A, 𝐴2−3𝐴 + 𝐼 = 0 and 𝐴−1 = 𝑥𝐴 + 𝑦𝐼, then the value of
𝑥 + 𝑦 is
(a) -2 (b) 2 (c) 3 (d) -3
Solution:-
A2-3A+I=0
Multiply by A-1
We get
A-3I+A-1=0
A-1= -A+3I
Hence x= -1,y=3
x+y=2
Correct option : b
6 1−𝑥 2 3
If |0 𝑥 0| =0, then its roots are
0 0 𝑥

a) 0 (b)1 (c) 0,1 (d) 0,1, −1


Solution:-
1−𝑥 2 3
|0 𝑥 0|=0
0 0 𝑥
On expanding the determinant, we get
(1-x)x2=0
Hence x=0,1
Correct option: c
7 If 𝐴 (3,4), 𝐵(−7,2), 𝐶(𝑥, 𝑦) are collinear, then which of the following is true?
(a) 𝑥 + 5𝑦 + 17 = 0 (b) 𝑥 + 5𝑦 + 13 = 0
(c) 𝑥 − 5𝑦 + 17 = 0 (d) 𝑥 − 5𝑦 − 17 = 0
Solution:- If A (3,4), B(-7,2), C(x, y) are collinear, then area of triangle ABC=0
3 4 1
1
i.e 2 |−7 2 1|=0
𝑥 𝑦 1

61
i.e
3 4 1
|−7 2 1|=0
𝑥 𝑦 1
𝑖. 𝑒 3(2 − 𝑦) − 4(−7 − 𝑥) + 1(−7𝑦 − 2𝑥) = 0
i.e x-5y+17=0
Correct option: c
8 If A is an invertible matrix of order 2, then det(𝐴−1 )=
1
a) 𝑑𝑒𝑡𝐴 (b) 0 (c) 1 (d) det(A)
Solution:-
𝑎𝑑𝑗(𝐴)
det(A-1)=| 𝐼𝐴𝐼 |
1
= |𝐴|2 |𝑎𝑑𝑗(𝐴)|
1
= |𝐴|2 |𝐴|2-1
1
= |𝐴|
1
= 𝑑𝑒𝑡𝐴
Correct option: a
9 If A is a square matrix such that A2=I , then A-1 is equal to:
a) 2A (b) O (c) A (d) A+I
Solution:-
A2=I
Multiply by A-1 on both sides
A-1A2=A-1I
A=A-1
A-1= A
Correct option:c
10 𝑥+𝑦 𝑦+𝑧 𝑧+𝑥
The value of | 𝑧 𝑥 𝑦 | is equal to
1 1 1
(a) 0 (b) 1 (c) 𝑥 + 𝑦 + 𝑧 (d) 2(𝑥 + 𝑦 + 𝑧)
Solution:-
𝑥+𝑦 𝑦+𝑧 𝑧+𝑥
|𝑧 𝑥 𝑦 |=(x+y)(x-y)-(y+z)(z-y)+(z+x)(z-x)
1 1 1
=( x2-y2)-(z2-y2)+(z2-x2)
=0
Correct option: a

CHAPTER VIDEO LINK FOR MCQs SCAN QR CODE FOR


VIDEO

DETERMINANTS https://youtu.be/nrquFxCgHsE

62
EXERCISE
1 1 2 1
If [ 2 3 1 ] is a non-singular matrix and 𝑎 ∈ 𝐴, then set A is
3 𝑎 1

a) R (b){0} (c) {4} (d) R-{4}


Correct option: d
2 If |𝐴|=|𝑘𝐴|, where A is a square matrix of order 2, then sum of all possible values of
𝑘 is

a) 1 (b) -1 (c) 2 (d) 0


Correct option: d
3 If A and B are square matrix of order of 3 such that |𝐴| = −1 and |𝐵| = 3 then
what is the value of |3𝐴𝐵| ?
a) −9 (b) −27 (c) −81 (d) 81
Correct option: c

4 1 2 3
The value of the determinant | 0 𝑠𝑖𝑛𝑥 𝑐𝑜𝑠𝑥 | is
0 𝑐𝑜𝑠𝑥 𝑠𝑖𝑛𝑥
a) 1 (b) −1 (c) – 𝑐𝑜𝑠2𝑥 (d) 𝑐𝑜𝑠2𝑥

Correct option: c
5 𝑥 0 0
If A=[0 𝑥 0] then the value of |𝑎𝑑𝑗(𝐴)| is
0 0 𝑥

a) x3 (b) x6 (c) x9 (d) x27


Correct option: b

ASSERTION REASONING QUESTIONS

Two statements are given, one labelled Assertion(A) and the other labelled Reason(R).Select
the correct answer from the codes (a),(b),(c) and (d) as given below

(a) Both Assertion (A) and Reason(R) are true and Reason(R) is the correct explanation of the
Assertion(A)

(b) Both Assertion (A) and Reason(R) are true but Reason(R) is not the correct explanation of
the Assertion(A)

(c) Assertion (A) is true and Reason (R) is false.

(d) Assertion (A) is false and Reason (R) is true.

1 𝑥 𝑥+1
Assertion(A):The value of | | is equal to 1
𝑥−1 𝑥
Reason(R):The value of the determinant of a matrix A order 2 × 2,
𝑎 𝑏
where A=[ ] is 𝑎𝑑 − 𝑏𝑐
𝑐 𝑑

63
𝑥 𝑥+1
Solution: | | = x (x) – (x + 1) (x – 1) = x2 – (x2 – 1) = x2 – x2 + 1 = 1
𝑥−1 𝑥
So Assertion A is true.
𝑎 𝑏
| |= 𝑎𝑑 − 𝑏𝑐
𝑐 𝑑
So Reason R is true.
Hence Reason(R) is the correct explanation of the Assertion(A)
Correct option: a
2 Assertion(A): If A is an invertible matrix of order 2, and det A= 3 then det(
1
A-1 )is equal to 3
Reason(R): If A is an invertible matrix of order 2 then det (A-1) =det(A)
1 1
Solution:-Since det(A-1)= =
𝑑𝑒𝑡𝐴 3
Hence A is true but R is false.
Correct option: c
3 Assertion(A): In a square matrix of order 3 the minor of an element
a22 is 3 then cofactor of a22 is −3.
Reason(R): Cofactor an element aij = Aij = ( -1)i+j Mij
Solution:- Cofactor an element aij = Aij = ( -1)i+j Mij
Cofactor an element a22 = A22 = ( -1)2+2(3)
=3
Hence Assertion (A) is false but Reason (R) is true
Correct option:d
4 1 0 1
Assertion(A):If A=[0 1 2] then|3𝐴|=27|𝐴|
0 0 4
Reason(R): If A = kB where A and B are square matrices of order n,
then | A| = kn| B |, where n = 1, 2, 3
Solution:-
Since |kA| = kn| A |
|3𝐴|=33|𝐴|
=27|𝐴|
Hence Reason(R) is the correct explanation of the Assertion(A)
Correct option :a
5 Assertion(A): If A=[2 3] then Adj(A)= [−2 1]
1 4 3−4
𝑎11 𝑎12
Reason(R):If A=[ 𝑎
21 𝑎22 ] then Adjoint(A) can be obtained by interchanging a11
and a22 and by changing signs of a12 and a21
Solution:-
4 −3
Adj(A)= [ ].
−1 2
Hence A is false but R is true
Correct option:d
6 Assertion(A): If A is a square natrix of order 3, then |2𝐴|=8|𝐴|
Reason(R): Let A be a square matrix of order n. Then | adj A| = |𝐴|n-1
Solution:-A is true since |2𝐴|=23|𝐴|=8|𝐴|
R is also true, but R is not the correct explanation of A
Correct option:b
7 3 7 6 8 1 61 − 87
Assertion(A): if A=[ ] and B=[ ] then (AB)-1= − 2 [ ]
2 5 7 9 −47 67

64
Reason(R): For any 2 matrix A and B , (AB)-1=B-1A-1
Solution:-
𝑎𝑑𝑗(𝐴𝐵) 1 61 − 87
(AB)-1= |𝐴𝐵| = − [ ] = B-1A-1
2 −47 67

(AB)-1=B-1A-1
Hence Both Assertion (A) and Reason(R) are true and Reason(R) is the correct
explanation of the Assertion(A)
Correct option: a
8 1 2
Assertion(A): Value of 𝑥 for which the matrix[ ] is singular is 4
2 𝑥

Reason(R): A square matrix is singular if |A | =0


Solution:-
A square matrix is singular if |A | =0
1 2
| | =0
2 𝑥
𝑥−4=0
𝑥=4
Correct option: a
9 Assertion(A): The system of equations 2𝑥 + 5𝑦 = 1; 3𝑥 + 2𝑦 = 7 are consistent
Reason(R): A system of equations is said to be consistent if they have one or more
solution.
Solution:-
2 5 𝑥
The system of equations can be written in the form AX = B, where A=[ ],X=[𝑦]
3 2
1
B=[ ]
7
On solving these system of equations by matrix method|𝐴|= −11 ≠ 0, Hence, A is non
singular matrix and so has a unique solution. Hence they are consistent.
Correct option:a
10 Assertion(A): For two matrices A and B of order 3, |𝐴|=2 |𝐵| = − 3, then |𝐴𝐵| = 6
Reason(R): The determinant of the product of matrices is equal to product of their
respective determinants, that is, |𝐴𝐵| = |𝐴||𝐵| where A and B are square matrices of
the same order
Solution:- |𝐴𝐵| = |𝐴||𝐵|
= 2(−3)
=−6
Hence A is false but R is true.
Correct option:d

EXERCISE
1 3 𝑥 3 4
Assertion(A): The value of 𝑥 for which | |=| | is ±4
𝑥 1 4 1
Reason(R) : The determinant of a matrix A order 2 × 2,
𝑎 𝑏
A=[ ] 𝑖𝑠 𝑎𝑑 − 𝑏𝑐
𝑐 𝑑
Correct option: a
2 Assertion(A): For two matrices A and B of order 3, |𝐴|= 1,|𝐵| = −4 then |2𝐴𝐵| is
−32.
Reason(R): For a square matrix A, A(adj A)=(adj A)A=|A | I
Correct option: b

65
3 Assertion(A): The equation of the line joining (1,2) and (3,6) using determinants is 𝑦 +
2𝑥 = 0.
Reason(R): The area of ∆ PAB is zero if P(x, y) is a point on the line joining two points
A and B.
Correct option: d
4 1 1 1
1
Assertion(A):The maximum value of |1 1 + sin 𝐴 1 | is 2
1 1 1 + cos 𝐴
𝜋 𝜋
Reason(R): Principal value branch of sin-1A is [- , ]
2 2
Correct option:b
5 2 𝑥 −3
−8
Assertion(A): A=[ 0 2 5 ], then A-1 exists if 𝑥 = 5
1 1 3

Reason(R): A square matrix A has inverse if and only if A is non-


singular.
Correct option: d

2 MARK QUESTIONS
Q. No QUESTIONS WITH SOLUTIONS
1 1 2 1 −4
If A=[ ] and B=[ ] find |𝐴𝐵|
3 −1 3 −2
Solution:-
1 2 1 −4
AB=[ ] ×[ ]
3 −1 3 −2

7 −8
=[ ]
0 − 10

|𝐴𝐵| = −70

2 3
If A=[ ] write A-1 in terms of A
5−2
2 Solution:-
𝐴𝑑𝑗(𝐴)
A-1= |𝐴|
−1 −2 − 3
= [ ]
19−5 2
1 2 3
=19 [ ]
5 − 2
1
=19 A
What positive value of 𝑥 makes the following pair of determinants equal
2𝑥 3 10 3
| |,| |
3 5 𝑥 5 5
Solution:-
2𝑥2−15 = 50 − 15
2𝑥 2= 50
𝑥 = ±5

66
4 For what value of 𝑥, is the following matrix singular?
3 − 2𝑥 𝑥+1
[ ]
2 4
Solution:-
A matrix is singular if |𝐴|=0
(3 − 2𝑥)4 − (𝑥 + 1)2 = 0
On solving we get 𝑥 = 1
5 8 0
If for any 2 × 2 square matrix A, A(AdjA)= [ ] , then write the value of |𝐴|
0 8
Solution:-
A (adj A) = |A| I
8 0 1 0
[ ]=|A|[ ]
0 8 0 1
8 0 |𝐴| 0
[ ]=[ ]
0 8 0 |𝐴|
|𝐴|=8
6 If A is a nonsingular matrix of order 3 and |𝐴| = −4, find |𝐴. 𝑎𝑑𝑗𝐴|
Solution:-
|𝐴. 𝑎𝑑𝑗𝐴|= |𝐴||𝑎𝑑𝑗𝐴|
=|𝐴||𝐴| 2
=|𝐴|3
= (−4)3
= −64
7 Find the equation of the line joining A ( 1 , 3) and B ( 0, 0) using determinants
Solution:-
Let 𝑝(𝑥, 𝑦) be any point on the line AB
Then area of ∆𝑃𝐴𝐵 = 0
1 3 1
|0 0 1 |=0
𝑥 𝑦 1
Equation of line AB is 𝑦 = 3 𝑥

2 −3 5
If Aij is the cofactor of the element aij of the determinant| 6 0 4 | , find the value of
1 5 −7
8 a32 A32
Solution:- a32 A32 = 5 × − (8 − 30)
= 5(22)
= 110
9 1 3
If A = [ ] , find the value of | 𝐴2 −2 𝐴 |
2 1
Solution:-
7 6 2 6
A2-2A=[ ]-[ ]
4 7 4 2
5 0
=[ ]
0 5
2
| 𝐴 −2 𝐴 | = 25
10 If A = [ aij ] is a matrix of order 2 ×2 , such that |𝐴| = − 15 and Aij is the cofactor
of the element aij then find a 21 A21 + a 22 A22
Solution:-
a 21 A21 + a 22 A22
= |𝐴|
= − 15

67
EXERCISE
1 2 −3
Given A= [ ] , compute A-1 and show that 2A-1=9I-A
−4 7
2 1 𝑘 3
Find 𝑘 if the matrix[1 3 3] is the adjoint of matrix A and |𝐴| = 4
2 4 4
Answer: 𝑘 = 11

3 𝑥+1 𝑥−1 4 −1
If | |=| | then find the value of 𝑥
𝑥−3 𝑥+2 1 3
Answer:-
𝑥=2
4 1 0 0
Find the inverse of the matrix [0 𝑐𝑜𝑠𝑥 𝑠𝑖𝑛𝑥 ]
0 𝑠𝑖𝑛𝑥 −𝑐𝑜𝑠𝑥
1 0 0
Answer:- [0 𝑐𝑜𝑠𝑥 𝑠𝑖𝑛𝑥 ]
0 𝑠𝑖𝑛𝑥 −𝑐𝑜𝑠𝑥
5 𝑐𝑜𝑠𝐴𝑐𝑜𝑠𝐵 𝑐𝑜𝑠𝐴𝑠𝑖𝑛𝐵 − 𝑠𝑖𝑛𝐴
Evaluate | −𝑠𝑖𝑛𝐵 𝑐𝑜𝑠𝐵 0 |
𝑠𝑖𝑛𝐴𝑐𝑜𝑠𝐵 𝑠𝑖𝑛𝐴𝑠𝑖𝑛𝐵 𝑐𝑜𝑠𝐴

Answer: 1

3 MARK QUESTIONS
Q.No QUESTIONS WITH SOLUTIONS
1 A coaching institute of Mathematics conduct classes in two batches, I and II and fees
for rich and poor children are different. In batch I, it has 20 poor and 5 rich children
and total monthly collection of Rs.9000/-, where as in batch II 5 poor and 25 rich
children and the monthly collection is Rs.26,000/-. Using matrix method finds the
monthly fees paid by each child of the two types.
Solution:-
Let 𝑥 𝑎𝑛𝑑 𝑦 be the fees paid by rich and poor children respectively.
According to the question,
5x+20y=9000
25x+5y=26000
5 20 𝑥 9000
Which can be written as 𝐴𝑋 = 𝐵, where A=[ ] ,X= [𝑦],B=[ ]
25 5 26000
|𝐴| = −475 ≠ 0
5 −20
adj A= [ ]
−25 5
−1 5 −20
∴ A−1 = 475 [ ]
−25 5

68
x 1000
X= [y], A−1 B=[ ]
200
𝑥 = 1000, 𝑦 = 200
2 1 2 3
If A=[3 − 2 1], then show that A3-23A-40I=0
4 2 1
Solution:-
19 4 8
A2=[ 1 12 8 ]
14 6 15

63 46 69
A3=[69 − 6 23]
92 46 63

A3-23A-40I=
63 46 69 23 46 69 40 0 0 0 0 0
[69 − 6 23] - [69 − 46 23] -[0 40 0] =[ 0 0 0 ]=0
92 46 63 92 46 23 0 0 40 0 0 0
3 Using determinants, find the area of ∆PQR with vertices P(3,1),Q(9,3)and R(5,7).
Also find the equation of line PQ using determinants.
Solution:-
3 1 1
1
Area =2 |9 3 1| = 16 sq.units
5 7 1
𝑥 𝑦 1
Equation of PQ is |3 1 1| =0
9 3 1
−2𝑥 + 6𝑦 = 0
OR
𝑥 − 3𝑦 = 0
4 3 0 0
If A.(adjA)=[0 3 0],then find the value of |𝐴| +|𝑎𝑑𝑗𝐴|
0 0 3
Solution:-

|𝐴. 𝑎𝑑𝑗𝐴|=|𝐴||𝑎𝑑𝑗𝐴|=|𝐴||𝐴|2 =|𝐴|3


27=|𝐴|3
|𝐴|=3
|𝑎𝑑𝑗𝐴|=32=9
|𝐴| +|𝑎𝑑𝑗𝐴|=12

69
5 𝑥 3 7
𝐼𝑓 𝑥 = −9 is a root of |2 𝑥 2|=0 then find the other 2 roots
7 6 𝑥
Solution:-
𝑥 3 7
|2 𝑥 2|=0
7 6 𝑥
𝑥3 − 67𝑥 + 126 = 0
(𝑥 + 9)(𝑥 − 7)(𝑥 − 2) = 0
𝑥 = − 9,7,2
Hence the other two roots are 7 𝑎𝑛𝑑 2

6 2 3 1 −2
If A=[ ] and B=[ ] then verify that (AB)-1 =B-1A-1
1 −4 −1 3
Solution:-
−1 5
AB=[ ]
5 − 14
|𝐴𝐵| = −11 ≠ 0 , Therefore (AB)-1 exists

1 14 5
(AB)-1= 11 [ ]
5 1
1 −4 − 3 3 2
A-1= - [ ] B-1 = [ ]
11 −1 2 1 1
1 14 5
B-1A-1 =11 [ ]=(AB)-1
5 1
Hence proved
7 2 3
Show that the matrix A =[ ] satisfies the equation A2 – 4A + I = O,
1 2
where I is 2 × 2 identity matrix and O is 2 × 2 zero matrix. Using this equation, find
A-1.
Solution:-
7 12 8 12 1 0 0 0
A2 – 4A + I=[ ]-[ ]+[ ]=[ ]=0
4 7 4 8 0 1 0 0
A2 – 4A + I = O
AA-4A+I=0
AA-4A= -I
Multiplying by A-1,we get

AI – 4I = −A-1
A-1= 4𝐼 − 𝐴

70
4 0 2 3
=[ ]-[ ]
0 4 1 2
2 −3
=[ ]
−1 2
8 If A is a skew symmetric matrix of order 3, then prove that 𝑑𝑒𝑡 𝐴 = 0
Solution:-
If A is a skew symmetric matrix of order 3,then A= - AT
|𝐴| =|− 𝐴𝑇 |
= - |𝐴𝑇 |
= - |𝐴| (Since |𝐴𝑇 | =|𝐴|)
2|𝐴| =0
Hence|𝐴|=0
9 1 𝑠𝑖𝑛𝑥 1
If 𝐴 = |−𝑠𝑖𝑛𝑥 1 𝑠𝑖𝑛𝑥|, where 0 ≤ x ≤ 2π. Then prove that |𝐴|ϵ[2,4]
−1 −𝑠𝑖𝑛𝑥 1
Solution:-
|𝐴|=2+2sin2x
We know that 0 ≤ sin2x ≤ 1
i.e. 0 ≤ 2sin2x ≤ 2
i.e 2 ≤ 2+2sin2x ≤ 4
i.e. 2 ≤ |𝐴| ≤ 4
Hence |𝐴|ϵ[2,4]
10 If the points (a1, b1), (a2, b2) and (a1 + a2, b1 + b2) are collinear, then prove that
a1b2=a2b1
Solution:-
If the points (a1, b1), (a2, b2) and (a1 + a2, b1 + b2) are collinear,
𝑎1 𝑏1 1
then|𝑎2 𝑏2 1|= 0
𝑎1 + 𝑎2 𝑏1 + 𝑏2 1
On expanding we get a2b1-a1b2=0
Hence a2b1 = a1b2
i.e. a1b2=a2b1

71
EXERCISE
1 𝑥 𝑠𝑖𝑛𝐴 𝑐𝑜𝑠𝐴
Show that the determinant|−𝑠𝑖𝑛𝐴 − 𝑥 1| is independent of A
𝑐𝑜𝑠𝐴 1 𝑥
2 Show that points A (a, b + c), B (b, c + a), C (c, a + b) are collinear.
3 𝑥 0 0
If x, y, z are nonzero real numbers and A=[0 𝑦 0] then prove that
0 0 𝑧
𝑥 −1 0 0
A-1=[ 0 𝑦 −1 0 ]
0 0 𝑧 −1
4 6 5
If A=[ ] , Show that A2-12A+I=0, hence find A-1
7 6
6 − 5
Answer:- A-1= [ ]
−7 6
5 3 −1 1
If A=[−15 6 − 5] then prove that A-1A=I
5 −2 2

5 MARK QUESTIONS
Q. No QUESTIONS WITH SOLUTIONS
1 Solve the following system of equations by matrix method
3𝑥 − 2𝑦 + 3𝑧 = 8
2𝑥 + 𝑦 − 𝑧 = 1
4𝑥 − 3𝑦 + 2𝑧 = 4
Solution:-

The given system of equations can be written as AX=B


|𝐴|= -17≠0
−1 – 5 − 1
-1
A = -1/17[ −8 – 6 9 ]
−10 1 7

𝑥 = 𝐴-1𝐵

𝑥 = 1, 𝑦 = 2, 𝑧 = 3

1 2 −2 3 −1 1
2 If A=[ −1 3 0 ] and B-1=[−15 6 − 5 ], find (AB)-1
0 −2 1 5 −2 2
Solution:-
(AB)-1 = B-1A-1
|𝐴|= 1≠0

72
3 2 6
Adj(A)=[1 1 2]
2 2 5
3 2 6
-1 1
A =1 [1 1 2]
2 2 5
Therefore (AB)-1 = B-1A-1
3 −1 1 3 2 6
=[−15 6 − 5 ] [1 1 2]
5 −2 2 2 2 5
10 7 21
=[−49 − 34 − 103]
17 12 36

3 Using the matrix method, solve the following system of linear equations :
2 3 10 4 6 5 6 9 20
+ + = 4, - + =1 , + - =2
𝑋 𝑌 𝑍 𝑋 𝑌 𝑍 𝑋 𝑌 𝑍

Solution:-

The given system of equations can be written in the form AX=B,


1
2 3 10 𝑥 4
1
Where A=[ 4 − 6 5 ] 𝑋 = 𝑦
and B=[1]
6 9 − 20 1 2
[𝑧]
|𝐴|= 1200 ≠ 0 , A exists -1

75 150 75
Adj(A)=[110 − 100 30]
72 0 − 24

75 150 75
1
Hence A-1 = 1200 [110 − 100 30]
72 0 − 24
Since AX=B, X=A-1B
75 150 75 4
1
=1200 [110 − 100 30] [1]
72 0 − 24 2

600
1
=1200 [400]
240
1 1
𝑥 2
1 1
𝑦
= 3
1 1
[𝑧] [5]
Hence 𝑥 = 2, 𝑦 = 3, 𝑧 = 5

73
EXERCISE

1 3 1
If A=[ ] show that A2-5A+7I=0, hence find A-1
−1 2
Answer:-

1 2 −1
A-1=7 [ ]
1 3
2 1 1 2 2 0 1
Use the product [0 2 3] [9 2 3] to solve the system of equations:-
3 2 4 6 1 2
𝑥 − 𝑦 + 2𝑧 = 1
2𝑦 − 3𝑧 = 1
3𝑥 − 2𝑦 + 4𝑧 = 2
Answer:-
𝑥 = 0, 𝑦 = 5, 𝑧 = 3

3 2 −3 5
If A=[ 3 2 − 4 ], find A-1. Use it to solve the s
1 1 −2
ystem of equations
2𝑥 − 3𝑦 + 5𝑧 = 11
3𝑥 + 2𝑦 − 4𝑧 = −5
𝑥 + 𝑦 − 2𝑧 = −3
Answer-
𝑥 = 1, 𝑦 = 2, 𝑧 = 3

CASE BASED QUESTIONS

Q. No QUESTIONS WITH SOLUTIONS


1 Manjit wants to donate a rectangular plot of land for a school in his
village.
When he was asked to give dimensions of the plot, he told that if its
length is decreased by 50 m and breadth is increased by 50m, then its
area will remain same, but if length is decreased by 10m and breadth
is decreased by 20m, then its
area will decrease by 5300 m2

74
1. Based on the information given above, form equations in terms of x and y
2. Write down matrix equation represented by the given information
3. How much is the area of rectangular field?
Solution:-
1) (x-50)(y+50)=xy

x-y=50--------(1)

(x-10)(y-20)=xy-5300

2x+y=550---------(2)

1 −1 𝑥 50
2) [ ] [ ]=[ ]
2 1 𝑦 550

3) On solving

We get, x=200 m

y=150 m

Area =200 × 150 = 30, 000 sq .m

2 Ram buys 5 pens, 3 bags 1 instrument box and pays a sum of Rs. 160. From the
same shop, Madhav buys 2 pens, 1 bag and 3 instrument boxes and pays a sum of
Rs.190.Also Ankit buys 1 pen, 2 bags and 4 instrument boxes and pays a sum of
Rs.250.
Based on the above information, answer the following questions:
1) Convert the given above situation into a matrix equation of the form AX=B
2) Find |𝐴|
3) Find A-1
OR
Determine P=A2-5A
Solution:-
5 3 1 𝑥 160
1) Matrix equation AX=B, where A=[2 1 .3] X=[𝑦] B=[190] where x is the
1 2 4 𝑧 250
number of pens bought, y the number of bags and z the number of instrument boxes.
2) |𝐴|= -22
−2 − 10 8
3) Adj(A)=[−5 19 − 13]
3 −7 −1

75
−2 − 10 8
-1 1
A =−22 [−5 19 − 13]
3 −7 −1

OR

7 5 13
2
P=A -5A= [ 5 8 2 ]
8 3 3
3 The management committee of a residential colony decided to award some of the
students of their colony (say x ) for honesty, some (say y) for helping others and
some others (say z)for supervising the workers to keep the colony neat and clean.
The sum of all the awardees is 12.Three times the sum of awardees for cooperation
and supervision added to two times the number of awardees for honesty is 33.If the
sum of the number of awardees for honesty and supervision is twice the number of
awardees for helping others.

Based on the above information, answer the following questions:


1)Convert the given above situation into a matrix equation of the form AX=B
2)Find A-1
3)Find the number of awardees of each category
Solution:-
1)
x+y+z=12----(1)
3(y+z)+2x=33-----(2)
x+z=2y-------(3)
i.e
x+y+z=12
2x+3y+3z=33
x-2y+z=0
1 1 1 𝑥 12
Matrix equation AX=B, where A=[ 2 3 3 ] X=[𝑦] B=[33]
1 −2 1 𝑧 0
2) |𝐴| =3
9 −3 0
1
A-1=3 [ 1 0 − 1 ]
−7 3 1
3)
X=A-1B
Hence x=3,y=4,z=5

76
EXERCISE
A company produces three products every day. Their production on certain day is 45
1 tons. It is found that the production of third product exceeds the production of first
product by 8 tons while the total production of first and third product is twice the
production of second product.
Using the concepts of matrices and determinants, answer the following questions.
1. If x, y and z respectively denotes the quantity (in tons) of first, second and third
product produced, then convert the given above situation into a matrix equation of the
form AX=B
2. Find A-1
3. Find x:y:z
Answer:-

1 1 1 𝑥 45
1)[1 0 − 1] [𝑦]=[−8]
1 −2 1 𝑧 0

2 3 1
1
2) A-1 = 6
[2 0 − 2]
2 −3 1

3)11:15:19

2 Each triangular face of the Pyramid is made up of 25 smaller equilateral triangles as


shown in the figure.

Using the above information and concept of determinants, answer the following
questions
1) If the vertices of one of the smaller equilateral triangle are
(0, 0), (3, √3) and(3, −√3) then find the area of such triangle
1 1
2) Let A (a, 0), B (0, b) and C (1, 1) be three points such that 𝑎
+ 𝑏
= 1 then prove that
the 3 points are collinear.
Answer:-

1) 3√3

77
CHAPTER: CONTINUITY AND DIFFERENTIABILITY

SYLLABUS: Continuity and differentiability, Chain rule, Derivative of inverse trigonometric


functions, Derivative of implicit functions, Concept of exponential and logarithmic functions,
Derivatives of logarithmic and exponential functions, Logarithmic differentiation, derivative
of functions expressed in parametric forms. Second order derivatives.

Definitions and Formulae:

• Continuous function - A real valued function f(x) is said to be continuous, if it is


continuous at every point in the domain of f (x) .

Continuity of a function at a point – A real valued function f(x) is said to be


continuous at

𝑥 = 𝑎 if

LHL= RHL= f (a)

lim 𝑓(𝑥) = lim+ 𝑓(𝑥) = f (a)


𝑥→𝑎− 𝑥→𝑎

• Derivative of a function- The derivative of a function f(x) is defined by


𝑓(𝑥+ℎ)−𝑓(𝑥)
𝑓 ′ (𝑥) = lim
ℎ→0 ℎ

𝑓(𝑎−ℎ)−𝑓(𝑎)
• Left Hand Derivative (LHD)= L𝑓 ′ (a)= lim
ℎ→0 −ℎ

𝑓(𝑎+ℎ)−𝑓(𝑎)
• Right Hand Derivative (RHD)= R𝑓 ′ (a)= lim
ℎ→0 ℎ

• A real valued function f(x) is said to be differentiable at x= a if its LHD and RHD at
x=a exist and both are equal

Standard Derivatives

Sl. No Function Derivative

1 𝑥𝑛 n𝑥 𝑛−1
2 K (constant) 0
3 √𝑥 1
2√𝑥
4 𝑠𝑖𝑛 𝑥 𝑐𝑜𝑠 𝑥
5 𝑐𝑜𝑠 𝑥 −𝑠𝑖𝑛 𝑥
6 𝑡𝑎𝑛 𝑥 𝑠𝑒𝑐 2 𝑥
7 𝑠𝑒𝑐 𝑥 𝑠𝑒𝑐 𝑥 𝑡𝑎𝑛 𝑥

78
8 𝑐𝑜𝑠𝑒𝑐 𝑥 −𝑐𝑜𝑠𝑒𝑐 𝑥 𝑐𝑜𝑡 𝑥
9 𝑐𝑜𝑡 𝑥 -𝑐𝑜𝑠𝑒𝑐 2 𝑥
10 𝑒𝑥 𝑒𝑥
11 log 𝑒 𝑥 1
𝑥
12 𝑠𝑖𝑛−1 𝑥 1
√1 − 𝑥 2

13 𝑐𝑜𝑠 −1 𝑥 −1
√1 − 𝑥 2

14 𝑡𝑎𝑛−1 𝑥 1
1 + 𝑥2
15 𝑠𝑒𝑐 −1 𝑥 1
𝑥√𝑥 2 − 1

16 𝑐𝑜𝑠𝑒𝑐 −1 𝑥 −1
𝑥√𝑥 2 − 1
17 𝑐𝑜𝑡 −1 𝑥 −1
1 + 𝑥2
18 𝑎𝑥 𝑎 𝑥 log 𝑒 𝑎

• Product Rule-
𝑑𝑦 𝑑𝑣 𝑑𝑢
If y= u v then 𝑑𝑥 = u 𝑑𝑥 + v 𝑑𝑥

• Quotient Rule-
𝑑𝑢 𝑑𝑣
𝑢 𝑑𝑦 𝑣 −𝑢
𝑑𝑥 𝑑𝑥
If y= then =
𝑣 𝑑𝑥 𝑣2

• Chain Rule-
𝑑𝑦 𝑑𝑦 𝑑𝑡
If 𝑦 = 𝑓 (𝑡), then = .
𝑑𝑥 𝑑𝑡 𝑑𝑥

• Derivative of implicit functions- Let 𝑓(𝑥, 𝑦) = 0 be an implicit function of 𝑥, then to


𝑑𝑦
find , first differentiate both sides of the equation w.r.t x and then take all the
𝑑𝑥
𝑑𝑦 𝑑𝑦
terms containing 𝑑𝑥 to LHS and remaining terms to the right , then find 𝑑𝑥 .

• Logarithmic Differentiation-

Used to differentiate functions of the form 𝑢(𝑥)𝑣(𝑥)

• Parametric Differentiation-

79
𝑑𝑦
𝑑𝑦 𝑑𝑡
If x=f (t) and y= g (t) then 𝑑𝑥 = 𝑑𝑥
𝑑𝑡

• Second order derivative-


𝑑2 𝑦 𝑑 𝑑𝑦
If y= f(x) then the second order derivative is 𝑑𝑥 2 = 𝑑𝑥 (𝑑𝑥 )

MULTIPLE CHOICE QUESTIONS

Q.NO QUESTIONS AND SOLUTION


1 𝑑2 𝑦
If y= a sin mx + b cos mx ,then 𝑑𝑥 2 is
(a) 𝑚2 𝑦 (b) −𝑚2 𝑦 (c) my (d) –my

SOLUTION: Option (b)

𝒅𝒚
= am cos mx – bm sin mx
𝒅𝒙

𝑑2 𝑦
= -a𝑚2 sin mx - b𝑚2 cos mx
𝑑𝑥 2

= -𝑚2 (a sin mx + b cos mx)

= -𝑚2 y

2 𝑑
𝑙𝑜𝑔(x+ √𝑥 2 + 1) is equal to
𝑑𝑥

(a) √𝑥 2 + 1 (b) 𝑥√𝑥 2 + 1


𝑥 1
(c) √𝑥 2 (d) √𝑥 2
+1 is +1

SOLUTION: Option (d)


𝑑𝑦 1 2𝑋
= [1+ ]
𝑑𝑥 x+ √𝑥 2 +1 2√𝑥 2 +1

1 x+ √𝑥 2 +1
= .
x+ √𝑥 2 +1 √𝑥 2 +1

1
= √𝑥 2
+1
3 −1 2𝑥 2𝑥 𝑑𝑢
If u = sin 1+𝑥 2 and v = 𝑡𝑎𝑛−1 1−𝑥 2 , then 𝑑𝑣 is

1 1−𝑥 2
(a) (b) x (c) 1+𝑥 2 (d) 1
2

SOLUTION: Option (d)


Put x= tan𝜃

2𝑡𝑎𝑛𝜃
u = sin−1 1+𝑡𝑎𝑛2𝜃 = sin−1 𝑠𝑖𝑛2𝜃 =2𝜃= 2 tan−1 𝑥

80
2𝑡𝑎𝑛𝜃
v= 𝑡𝑎𝑛−1 1−𝑡𝑎𝑛2 𝜃 = 𝑡𝑎𝑛−1 𝑡𝑎𝑛2𝜃 = 2𝜃 = 2 tan−1 𝑥

𝑑𝑢
𝑑𝑢 𝑑𝑥
= 𝑑𝑣
𝑑𝑣
𝑑𝑥

=1

4 The points of discontinuity of the function

2𝑥 + 3 , 𝑖𝑓 𝑥 ≤ 2
f(x) = { are
2𝑥 − 3 , 𝑖𝑓 𝑥 > 2

(a) -2 (b) 2 (c) ±2 (d) (-2,2)

SOLUTION: Option (b)

At x =2
LHL = 2(2)+3 =7
RHL = 2(2)-3 = 1
LHL ≠ 𝑅𝐻𝐿
So f(x) is not continuous at x=2

X = 2 is the point of discontinuity.


5
Derivative of 𝑥 2 with respect to 𝑥 3 is
1 3 2 3𝑥
(a) (b) (c) (d)
𝑥 2𝑥 3𝑥 2

SOLUTION: Option (c)

u= 𝑥 2 v = 𝑥3
𝑑𝑢
𝑑𝑢 𝑑𝑥 2𝑥 2
= 𝑑𝑣 = 3𝑥 2 = 3𝑥
𝑑𝑣
𝑑𝑥

6
3𝑥 − 5 , 𝑥≤4
If f(x) = { is continuous at x=4 then k is
2𝑘 , 𝑥>4
7 2 −7 −2
(a) (b) (c) (d)
2 7 2 7

SOLUTION: Option (a)

At x= 4
LHL = 3(4)-5 =7
RHL = 2k
LHL = RHL
7=2k
7
k= 2

81
7 𝑑2 𝑦
If x = 𝑡 2 y= 𝑡 3 then 𝑑𝑥 2 is equal to

3 3 3 3𝑡
(a) 2
(b) 4𝑡
(c) 2𝑡
(d) 2

SOLUTION: Option (b)


𝒅𝒚 𝒅𝒚/𝒅𝒕 𝟑𝒕𝟐 𝟑𝒕
= 𝒅𝒙/𝒅𝒕 = =
𝒅𝒙 𝟐𝒕 𝟐

𝑑2 𝑦 𝟑 𝟏 3
= 𝟐 . 𝟐𝒕 = 4𝑡
𝑑𝑥 2

8 The function f(x) =[x] is continuous at

(a) 4 (b) -2 (c) 1 (d) 1.5

SOLUTION: Option (d)


Greatest integer function is not continuous at integral values.

9 𝑑𝑦 1 1
For the curve √𝑥 + √𝑦 = 1 , 𝑑𝑥 at (4 , 4 ) is

1
(a) 1 (b) (c) -1 (d) none of these
2

SOLUTION: Option (c)

√𝑥 + √𝑦 = 1

1 1 𝑑𝑦
+2 =0
2√𝑥 √𝑦 𝑑𝑥

𝑑𝑦 −√𝑦
=
𝑑𝑥 √𝑥

𝑑𝑦 1 1
at (4 , 4 ) = -1
𝑑𝑥

10 log 𝑥
If y = , then 𝑦2 =
𝑥

3−2𝑙𝑜𝑔𝑥 2𝑙𝑜𝑔𝑥−3
(𝑎) (b)
𝑥3 𝑥3
2𝑙𝑜𝑔𝑥−3
(c) (d) 𝑛𝑜𝑛𝑒 𝑜𝑓 𝑡ℎ𝑒𝑠𝑒
𝑥4

SOLUTION: Option (b)

log 𝑥
y= 𝑥
𝟏
𝒅𝒚 𝒙( )−𝒍𝒐𝒈𝒙
𝒙
=
𝒅𝒙 𝒙𝟐

𝟏−𝒍𝒐𝒈𝒙
= 𝒙𝟐

−𝑥2
− (1−𝑙𝑜𝑔𝑥)2𝑥 −𝑥−2𝑥+2𝑥𝑙𝑜𝑔𝑥 2𝑙𝑜𝑔𝑥−3
𝑥
𝑦2 = = =
𝑥4 𝑥4 𝑥3

82
CHAPTER VIDEO LINK FOR MCQs SCAN QR CODE FOR
VIDEO

CONTINUITY AND https://youtu.be/fIuPO6ThkfQ


DIFFERENTIABILITY

EXERCISE

1 𝑑2 𝑦
If y = a𝑒 𝑚𝑥 + b𝑒 −𝑚𝑥 , then 𝑑𝑥 2 is
(a) 𝑚2 𝑦 (b) −𝑚2 𝑦 (c) my (d) –my
Ans: (a)

𝑠𝑖𝑛𝑥
+ 𝑐𝑜𝑠𝑥, 𝑥≠0
The function f(x) = { 𝑥 , is continuous at 𝑥 = 0, then value of 𝑘 is
𝑘 , 𝑖𝑓 𝑥 = 0

(a) 3 (b) 2 (c) 1 (d) 1.5


Ans: (b)

3 𝑑𝑦
If x= a𝑐𝑜𝑠 3 𝜃 , y= a𝑠𝑖𝑛3 𝜃 then √1 + (𝑑𝑥 )2
(a) 𝑡𝑎𝑛2 𝜃 (b) 𝑠𝑒𝑐 2 𝜃 (c) sec𝜃 (d) | sec𝜃|
Ans: (c )
4 The points of discontinuity of the function
3𝑥 + 5 , 𝑖𝑓 𝑥 ≥ 2
f(x) = { are
𝑥 2 , 𝑖𝑓 𝑥 < 2

(a) −2 (b) 2 (c) ±2 (d) (-2,2)


Ans: (b)

5 Find the value of k for which the given function is continuous

2𝑥+2 −16
𝑖𝑓 𝑥 ≠ 2
f(x)= { 4𝑥 −16
𝑘 𝑖𝑓 𝑥 = 2
1 −1
(a) − 2 (b) 2 (c) (d)
2 2

Ans: (c )

83
ASSERTION & REASONING QUESTIONS

In the following questions, a statement of Assertion (A) is followed by a statement of


Reason (R). Choose the correct answer out of the following choices.
(a) Both A and R true and R is the correct explanation of A.
(b) Both A and R true and R is not the correct explanation of A
(c) A is true but R is false
(d) A is false but R is true

1 𝑑2 𝑦
Assertion(A) : If 𝑥 = 2𝑎𝑡 , 𝑦 = 𝑎𝑡 2 , then 𝑑𝑥 2 is constant for all t

𝑑2 𝑦 𝑓 ′ (𝑡)𝑔′′ (𝑡)−𝑔′ (𝑡)𝑓′′(𝑡)


Reason (R ) : If 𝑥 = 𝑓(𝑡) , 𝑦 = 𝑔(𝑡) , then =
𝑑𝑥 2 (𝑓 ′ (𝑡))2

SOLUTION: Option (c)

Explanation:

𝑑𝑦 𝑑𝑦/𝑑𝑡 2𝑎𝑡
= = =t
𝑑𝑥 𝑑𝑥/𝑑𝑡 2𝑎

𝑑2 𝑦 1
= 1(2𝑎) which is a constant.
𝑑𝑥 2

So A is true

𝑑2 𝑦 𝑑 𝑑𝑦 𝑑𝑡
= 𝑑𝑡 (𝑑𝑥 ) 𝑑𝑥 so R is false
𝑑𝑥 2

A is true but R is false


2
Assertion(A) : If the function f(x) = {𝑎√𝑥 + 1 , 0 ≤ 𝑥 ≤ 3 is differentiable, then
𝑏𝑥 + 2, 3<𝑥≤5
2𝑎 = 3𝑏 + 2

Reason (R): Every continouou function is differentiable.

SOLUTION: Option (a)

Explanation:
LHL = lim 𝑓(3 − ℎ) = lim 𝑎√3 − ℎ + 1 = 2a
ℎ→0 ℎ→0

RHL =lim 𝑓(3 + ℎ)= lim 𝑏(3 + ℎ) + 2 = 3b+2


ℎ→0 ℎ→0
Since 𝑓 is differentiable, it is continuous so LHL = RHL
2𝑎 = 3𝑏 + 2
So A is true & R is also not true

3 Assertion (A): The function f(x) = |x| is everywhere continuous.

Reason (R): Every differentiable function is continuous.

SOLUTION: Option (b)

84
Explanation:

LHL = lim 𝑓(𝑎 − ℎ) = lim |𝑎 − ℎ|=a


ℎ→0 ℎ→0

RHL =lim 𝑓(𝑎 + ℎ)= lim |𝑎 + ℎ| = a


ℎ→0 ℎ→0

f( a) = |a| =a

LHL= RHL= f (a) so f is continuous.

So A is true & R is true

Both A and R true and R is not the correct explanation of A

4
Assertion(A) : If 𝑓(𝑥) 𝑎𝑛𝑑 𝑔(𝑥) are two continuous functions such that
𝑓(0) = 3 , 𝑔( 0) = 2, then lim{𝑓(𝑥) + 𝑔(𝑥)} =5
𝑥→0

Reason (R ) : If f(x) and g(x) are two continuous functions at x=a then
lim {𝑓(𝑥) + 𝑔(𝑥)} = lim 𝑓(𝑥) + lim 𝑔(𝑥)
𝑥→𝑎 𝑥→𝑎 𝑥→𝑎

SOLUTION: Option (a)


Explanation:

By using algebra of limits


lim{𝑓(𝑥) + 𝑔(𝑥)} = lim 𝑓(𝑥) + lim 𝑔(𝑥) = 𝑓(0) + 𝑔(0) = 3 + 2 = 5
𝑥→0 𝑥→0 𝑥→0
So A is true and R is also true

Both A and R true and R is the correct explanation of A.

5 𝑑3 𝑦
Assertion (A): If 𝑦 = 𝑠𝑖𝑛 𝑥 , then 𝑑𝑥 3 = −1 𝑎𝑡 𝑥 = 0

𝑑𝑦
Reason (R ) : If 𝑦 = 𝑓(𝑥). 𝑔(𝑥) then 𝑑𝑥 = 𝑓(𝑥) 𝑔’(𝑥) + 𝑔(𝑥) 𝑓’(𝑥)

SOLUTION: Option (b)

Explanation:
y= sin x
𝒅𝒚
= cos x
𝒅𝒙

𝑑3 𝑦
= -sin x
𝑑𝑥 3

𝑑3 𝑦
= - cos x
𝑑𝑥 3

𝑑3 𝑦
=-1 at x=0
𝑑𝑥 3

85
So A is true & R is product rule which is also true

Both A and R true and R is not the correct explanation of A

6
Assertion (A): If 𝑓(𝑥) = sin−1 𝑥 + cos −1 𝑥 + 2 , then 𝑓’(1) = 0
𝑑
Reason (R ) : 𝑑𝑥 (sin 𝑥) = cos x

SOLUTION: Option (b)


Explanation:

f(x) = sin−1 𝑥 + cos−1 𝑥 + 2


1 −1
f’(x) = √1−𝑥2 +√1−𝑥 2 +0 =0

f’(1) = 0 so A is true & R is also tue.

Both A and R true and R is not the correct explanation of A


7
|𝑥|
Assertion (A): The function f(x) = is continuous at 𝑥 = 0
𝑥

|𝑥| |𝑥|
Reason (R ) ∶ lim and lim+ are −1 𝑎𝑛𝑑 1 respectively
𝑥→0− 𝑥 𝑥→0 𝑥

SOLUTION: Option (d)

Explanation:

|𝑥| |𝑥|
LHL = lim = -1 , RHL = lim+ =1
𝑥→0− 𝑥 𝑥→0 𝑥

LHL≠ RHL
So A is false.
A is false but R is true
8
sin 5𝑥
𝑖𝑓 𝑥 ≠ 0
Assertion (A): If f(x) = { 𝑘 𝑥 is continuous at 𝑥 = 0 𝑡ℎ𝑒𝑛 𝑘 = 15
𝑖𝑓 𝑥 = 0
3
Reason (R ) ∶ If 𝑓(𝑥) is continuous at a point 𝑥 = 𝑎 in its domain ,
then lim 𝑓(𝑥) = 𝑓(𝑎)
𝑥→𝑎

SOLUTION: Option (a)


Explanation:

sin 5(0−ℎ) sin −5ℎ


LHL =lim 𝑓(𝑎 − ℎ) = lim = lim5. =5
ℎ→0 ℎ→0 0−ℎ ℎ→0 −5ℎ

𝑘
f(a)= f(0) = 3

Since f is continuous LHL = RHL= f (a)

86
𝑘/3 = 5
𝑘 = 15
Both A and R true and R is the correct explanation of A

9 𝑠𝑖𝑛𝑥+𝑐𝑜𝑠𝑥 𝑑𝑦
Assertion (A): If y= tan−1(𝑐𝑜𝑠𝑥−𝑠𝑖𝑛𝑥) , then 𝑑𝑥 =1

𝑠𝑖𝑛𝑥+𝑐𝑜𝑠𝑥 𝜋
Reason (R ) ∶ = tan ( 4 + 𝑥)
𝑐𝑜𝑠𝑥−𝑠𝑖𝑛𝑥

SOLUTION: Option (a)

Explanation:

𝑠𝑖𝑛𝑥+𝑐𝑜𝑠𝑥
y= tan−1 (𝑐𝑜𝑠𝑥−𝑠𝑖𝑛𝑥)

1+tan 𝑥
y= tan−1 (1−𝑡𝑎𝑛𝑥)

𝜋 𝜋
y= tan−1 (tan (4 + 𝑥)) = 4 + 𝑥

𝑑𝑦
=1
𝑑𝑥

Both A and R true and R is the correct explanation of A

10 Assertion (A): The real valued function 𝑓(𝑥) = 3𝑥 2 − 2𝑥 + 7 is continuous at 𝑥 = 2


Reason (R ) ∶ Every polynomial function is continuous.

SOLUTION: Option (a)


Both A & R are true
Both A and R true and R is the correct explanation of A.

EXERCISE

1 Assertion (A): Every continuous function is differentiable

Reason (R ) ∶ Every differentiable function is continuous.

Ans: (d)
2 Assertion (A): The function f(x) =[x], greatest integer function, is not differentiable at
integer points

Reason (R ) ∶ The greatest integer function is not continuous at integer points


Ans: (b)
3 𝑑𝑦 𝑎𝜋
Assertion (A): If y = sin a𝑥 0 , then 𝑑𝑥 = 180 cos a𝑥 0

Reason (R ) ∶ 𝜋 𝑐 = 1800
Ans: (a )
4 2𝑥
Assertion (A): If f(x) = 2tan−1 𝑥 + sin−1 1+𝑥 2 , 𝑓’(2) = 𝑓’(3)

87
2𝑥
Reason (R ): sin−1 1+𝑥 2 = 2tan−1 𝑥 for all 𝑥.
Ans: (d)
5 Assertion (A): If 𝑓(𝑥) 𝑑𝑖𝑓𝑓𝑒𝑟𝑒𝑛𝑡𝑖𝑎𝑏𝑙𝑒 𝑎𝑡 𝑥 = 𝑎 , then lim 𝑓(𝑥) = f(a)
𝑥→𝑎
Reason (R ) ∶ Every differentiable function is continuous.
Ans: (a)
2 MARKS QUESTIONS

Q.NO QUESTIONS WITH SOLUTIONS


1 For what value of 𝑘, is the following function continuous at 𝑥 = 0
1 − cos 4 x , when, x  0

f(x)=  8 x 2
 k , when, x = 0
SOLUTION:
1 − cos 4 x
lim f (x ) = lim
x →0 x →0 8x 2
2 sin 2 2 x
= lim
x→0 8x 2
sin 2 x sin 2 x
= lim  lim
x →0 2x 2 x →0 2 x

=1 1 = 1
f (0) = k
f ( x ) is continuous at x=0 if lim f (x ) = f (0)
x →0
k =1 ,

2 𝑑𝑦 𝜋
If 𝑥 = 𝑎 (𝜃 − sin 𝜃)𝑎𝑛𝑑 𝑦 = 𝑎 (1 − cos 𝜃), 𝑓𝑖𝑛𝑑 𝑎𝑡 𝜃 =
𝑑𝑥 2

SOLUTION:
𝑑𝑦
𝑑𝑦 𝑑𝜃
= 𝑑𝑥
𝑑𝑥
𝑑𝜃

𝑑𝑦 𝑎 sin 𝜃
= 𝑎(1−cos 𝜃)
𝑑𝑥

𝜃 𝜃
2𝑠𝑖𝑛 𝑐𝑜𝑠
2 2
= 𝜃
2𝑠𝑖𝑛2
2

𝜃
= cot 2
𝑑𝑦 𝜋
at 𝜃 =
𝑑𝑥 2

𝜋
= cot 4 = 1
3 𝑑𝑦
If 𝑥 = 𝑎(𝑐𝑜𝑠𝜃 + 𝜃𝑠𝑖𝑛𝜃) , 𝑦 = 𝑎(𝑠𝑖𝑛𝜃 − 𝜃𝑐𝑜𝑠𝜃),find .
𝑑𝑥

SOLUTION:

88
𝑑𝑦
𝑑𝑦 𝑑𝜃
= 𝑑𝑥
𝑑𝑥
𝑑𝜃

𝑎(𝑐𝑜𝑠𝜃−(−𝜃𝑠𝑖𝑛𝜃+𝑐𝑜𝑠𝜃)
= a(-sin𝜃+𝜃𝑐𝑜𝑠𝜃+sin𝜃)

𝜃𝑠𝑖𝑛𝜃
= 𝜃𝑐𝑜𝑠𝜃

= tan𝜃

4 If y= (tan−1 𝑥)2 ,show that (𝑥 2 + 1)2 𝑦2 +2x (𝑥 2 + 1)𝑦1=2

SOLUTION:

y= (tan−1 𝑥)2

2 tan−1 𝑥
𝑦’ = 1+𝑥 2

(1 + 𝑥 2 )𝑦’ = 2 tan−1 𝑥
2
(1 + 𝑥 2 )𝑦’’ + 2𝑥𝑦’ = 1+𝑥 2

(𝑥 2 + 1)2 𝑦2 +2x (𝑥 2 + 1)𝑦1=2


5 𝑑𝑦 𝑐𝑜𝑠2 (𝑎+𝑦)
If 𝑐𝑜𝑠 𝑦 = 𝑥 𝑐𝑜𝑠 (𝑎 + 𝑦) , Prove that 𝑑𝑥 = sin 𝑎

SOLUTION:

𝑐𝑜𝑠 𝑦 = 𝑥 𝑐𝑜𝑠 (𝑎 + 𝑦)
cos 𝑦
𝑥 = cos(𝑎+𝑦)

𝑑𝑥 −cos(𝑎 + 𝑦) 𝑠𝑖𝑛𝑦 + 𝑐𝑜𝑠𝑦𝑠𝑖𝑛(𝑎 + 𝑦)


=
𝑑𝑦 𝑐𝑜𝑠 2 (𝑎 + 𝑦)

𝑑𝑥 sin 𝑎
= 𝑐𝑜𝑠2 (𝑎+𝑦)
𝑑𝑦

𝑑𝑦 𝑐𝑜𝑠2 (𝑎+𝑦)
=
𝑑𝑥 sin 𝑎

6 𝑐𝑜𝑠𝑥−𝑠𝑖𝑛𝑥 𝑑𝑥
If 𝑦 = tan−1 (𝑐𝑜𝑠𝑥+𝑠𝑖𝑛𝑥) , find 𝑑𝑦
SOLUTION:
𝑐𝑜𝑠𝑥−𝑠𝑖𝑛𝑥
y= tan−1 (𝑐𝑜𝑠𝑥+𝑠𝑖𝑛𝑥)

Dividing by cos x
1−𝑡𝑎𝑛𝑥
y= tan−1 (1+𝑡𝑎𝑛𝑥)

89
𝜋
y= tan−1 tan( 4 − 𝑥)

𝜋
y=4−𝑥

𝑑𝑦
= −1
𝑑𝑥

7 Find all the points of discontinuity of the function


𝑥10 − 1 , 𝑖𝑓 𝑥 ≤ 1
𝑓(𝑥) = { 2
𝑥 𝑖𝑓 𝑥 > 1

SOLUTION:

At x = 1
LHL= lim− 𝑓(𝑥)
𝑥→𝑎

= lim 𝑓(1 − ℎ)
ℎ→0

= lim (1 − ℎ)10 − 1
ℎ→0

=0

RHL = lim+ 𝑓(𝑥)


𝑥→𝑎

= lim 𝑓(1 + ℎ)
ℎ→0

= lim (1 + ℎ)2
ℎ→0

=1
Since LHL ≠RHL
𝑓(𝑥) 𝑖𝑠 𝑛𝑜𝑡 𝑐𝑜𝑛𝑡𝑖𝑛𝑢𝑜𝑢𝑠 𝑎𝑡 𝑥 = 1

𝑆𝑜 𝑥 = 1 𝑖𝑠 𝑡ℎ𝑒 𝑝𝑜𝑖𝑛𝑡 𝑜𝑓 𝑑𝑖𝑠𝑐𝑜𝑛𝑡𝑖𝑛𝑢𝑖𝑡𝑦.


8 2𝑥 𝑑𝑥
If y= cos −1 1+𝑥 2 , find 𝑑𝑦

SOLUTION:
2𝑥
y= cos −1 1+𝑥 2

Put x= tan θ
2𝑥
y= cos −1 1+𝑥 2

2𝑡𝑎𝑛𝜃
y= cos −1 1+𝑡𝑎𝑛2 𝜃

y= cos −1 𝑠𝑖𝑛2𝜃

90
𝜋
y= cos −1 cos( 2 − 2𝜃)

𝜋 𝜋
y= 2 − 2𝜃 = 2 − 2 tan−1 𝑥

−2
y = 1+𝑥 2

9 𝑑𝑦
If 𝑦 = tan−1 𝑥, then find in terms of 𝑦 alone
𝑑𝑥

SOLUTION:

y=tan−1 𝑥
𝑑𝑦 1
= 1+𝑥 2
𝑑𝑥

1 1
= 1+𝑡𝑎𝑛2 𝑦 = 𝑠𝑒𝑐 2 𝑦 = 𝑐𝑜𝑠 2 𝑦

10
Differentiate log (1+𝑥 2 ) with respect to tan−1 𝑥.

SOLUTION:

Let u = log (1+𝑥 2 ) and v= tan−1 𝑥

𝑑𝑢 𝑑𝑢/𝑑𝑥
=
𝑑𝑣 𝑑𝑣/𝑑𝑥

2𝑥
= 1+𝑥 2 .1+𝑥 2 = 2𝑥

EXERCISE

1 Find the value of 𝑘 for which


√1+𝑘𝑥−√1−𝑘𝑥
, −1 ≤ 𝑥 < 0
f(x) = {2𝑥+1 𝑥
𝑥−1
,0 ≤ 𝑥 ≤ 1
is continuous at 𝑥 = 0.
Ans: 𝑘 = −1
2 Show that the function 𝑓(𝑥) = |𝑥 + 2 | is continuous at every x∈ 𝑅, but fails to be
differentiable at 𝑥 = −2

3 3𝑥 + 5 , 𝑥 ≥ 2
Check whether the function f(x) = { is continuous at 𝑥 = 2.
𝑥2, 𝑥<2
Ans: Not continuous
4 1 + 𝑥, 𝑥 ≤ 2
Prove that 𝑓(𝑥) = { is not differentiable at 𝑥 = 2.
5 − 𝑥, 𝑥 > 2
5 𝑑2 𝑦
If 𝑦 = 5𝑐𝑜𝑠 𝑥 – 3 𝑠𝑖𝑛 𝑥 , prove that 𝑑𝑥 2 +y = 0

91
3 MARK QUESTIONS

Q.NO QUESTIONS WITH SOLUTIONS


1 𝑡
If 𝑥 = a(cost + log(tan2)) , 𝑦 = 𝑎 𝑠𝑖𝑛 𝑡
𝑑2 𝑦 𝜋
find the value of at t=
𝑑𝑥 2 4
SOLUTION:
𝑑𝑦
𝑑𝑦 𝑑𝑡
= 𝑑𝑥
𝑑𝑥
𝑑𝑡

𝑎𝑐𝑜𝑠𝑡
= 𝑡 1
1.𝑠𝑒𝑐2
𝑎(−𝑠𝑖𝑛𝑡+ 2 2)
𝑡
𝑡𝑎𝑛
2

𝑎𝑐𝑜𝑠𝑡
= 𝑡
1./𝑐𝑜𝑠2
𝑎(−𝑠𝑖𝑛𝑡+ 2 )
𝑡 𝑡
2 𝑠𝑖𝑛 /𝑐𝑜𝑠
2 2

𝑎𝑐𝑜𝑠𝑡
= 𝑡
1./𝑐𝑜𝑠2
𝑎(−𝑠𝑖𝑛𝑡+ 2 )
𝑡 𝑡
2 𝑠𝑖𝑛 /𝑐𝑜𝑠
2 2
𝑎𝑐𝑜𝑠𝑡
= 1
𝑎(−𝑠𝑖𝑛𝑡+ )
𝑠𝑖𝑛𝑡
𝑐𝑜𝑠𝑡
= 1
(−𝑠𝑖𝑛𝑡+ )
𝑠𝑖𝑛𝑡
𝑐𝑜𝑠𝑡𝑠𝑖𝑛𝑡
= 1−𝑠𝑖𝑛2 𝑡
𝑐𝑜𝑠𝑡𝑠𝑖𝑛𝑡
= 𝑐𝑜𝑠2 𝑡
= tan t
𝑑2 𝑦 1
= 𝑠𝑒𝑐 2 𝑡 .𝑎 sect
𝑑𝑥 2
𝜋
= (1/a) 𝑠𝑒𝑐 3 ( 4 )
1
= 𝑎 2√2

2 dy 1− y2
If 1 − x 2 + 1 − y 2 = a ( x – y ) , prove that =
dx 1− x2
SOLUTION:
Put x = sin α, y = sin β
cosα + cos β = a ( sin α – sin β )
𝛼+𝛽 𝛼−𝛽 𝛼+𝛽 𝛼−𝛽
2 Cos ( ) Cos ( ) = a 2 Cos ( )𝑆𝑖𝑛 ( )
2 2 2 2

 − 
cot  = a
 2 
𝛼−𝛽
( 2 ) = cot-1a
α-β = 2 cot-1a
sin-1 x – sin-1 y = 2 cot-1a
1 1 𝑑𝑦
√1−𝑥 2
- 2 𝑑𝑥
=0
√1−𝑦

92
dy 1− y2
=
dx 1 − x2

3 −1 𝑥 𝑑2 𝑦 𝑑𝑦
If y = 𝑒 𝑎𝑐𝑜𝑠 ,-1≤ x ≤ 1 , show that (1 - 𝑥 2 )𝑑2 𝑥 – x 𝑑𝑥 - 𝑎2 y = 0

SOLUTION:
−1
y = 𝑒 𝑎𝑐𝑜𝑠 𝑥
−1 𝑥
𝑑𝑦 −𝑎𝑒 𝑎𝑐𝑜𝑠
= √1−𝑥 2
𝑑𝑥
𝑑𝑦 −1 𝑥
√1 − 𝑥 2 𝑑𝑥 = −𝑎𝑒 𝑎𝑐𝑜𝑠

𝑑2 𝑦 𝑑𝑦 −2𝑥 −1 𝑥 1
√1 − 𝑥 2 + 𝑑𝑥 .√1−𝑥 2 = 𝑎2 𝑒 𝑎𝑐𝑜𝑠 .√1−𝑥 2
𝑑𝑥 2

𝑑2 𝑦 𝑑𝑦
(1 - 𝑥 2 )𝑑2 𝑥 – x 𝑑𝑥 - 𝑎2 y = 0

4 If x √1 + 𝑦 + y√1 + 𝑥 = 0,for,-1<x<1,prove that


𝑑𝑦 −1
= (1+𝑥)2
𝑑𝑥

SOLUTION:

x √1 + 𝑦 + y√1 + 𝑥 = 0

x √1 + 𝑦 = y√1 + 𝑥

𝑥 2 (1 + 𝑦) = 𝑦 2 (1+x)

𝑥 2 (1 + 𝑦) = 𝑦 2 (1+x)

𝑥 2 +𝑥 2 y = 𝑦 2 +𝑦 2 x

𝑥 2 -𝑦 2 +𝑥 2 y-𝑦 2 x=0

(x + y)(x-y)+x y (x-y)=0

( x – y )[x + y + x y ] = 0

[x + y + x y ] = 0

y(1+x) =-x
−𝑥
y = 1+𝑥

𝑑𝑦 (1+𝑥)(−1)−(−𝑥)
=
𝑑𝑥 (1+𝑥)2

−1
= (1+𝑥)2

93
5 3𝑎𝑥 + 𝑏 , 𝑥 > 1
If the function f(x) = { 11, 𝑥 = 1
5𝑎𝑥 − 2𝑏 , 𝑥 < 1
is continuous at x=1.Find the value of a & b.

SOLUTION:

At x=1
LHL= lim− 𝑓(𝑥)
𝑥→𝑎

= lim 𝑓(1 − ℎ)
ℎ→0

= lim 5𝑎(1 − ℎ) − 2𝑏
ℎ→0

= 5a-2b

RHL = lim+ 𝑓(𝑥)


𝑥→𝑎

= lim 𝑓(1 + ℎ)
ℎ→0

= lim 3𝑎(1 + ℎ) + 𝑏
ℎ→0

= 3a + b

Since f (x) is continuous LHL=RHL=f(1)

5a-2b = 3a+b = 11

On solving a= 3 b= 2
6 𝑑𝑦 𝑑2 𝑦
If x= a (cost +t sin t) and y = a (sin t -t cos t) . Find 𝑑𝑥 . & . 𝑑𝑥 2
SOLUTION:
𝑑𝑥
= a (-sin t + t cos t + sin t) = at cos t
𝑑𝑡

𝑑𝑦
= a ( cos t –(-t sin t +cos t) ) = at sin t
𝑑𝑡

𝑑𝑦 𝑑𝑦/𝑑𝑡
= = tan t
𝑑𝑥 𝑑𝑥/𝑑𝑡
𝑑2 𝑦 1 𝑠𝑒𝑐 3 𝑡
= 𝑠𝑒𝑐 2 𝑡 . 𝑎𝑡 cos 𝑡 =
𝑑𝑥 2 𝑎𝑡

7 𝑑𝑦 cos 𝑥
If y = √𝑠𝑖𝑛𝑥 + √𝑠𝑖𝑛𝑥 + √𝑠𝑖𝑛𝑥 + ⋯ … 𝑡𝑜 ∞. Prove that 𝑑𝑥 = 2𝑦−1
SOLUTION:

y = √𝑠𝑖𝑛𝑥 + √𝑠𝑖𝑛𝑥 + √𝑠𝑖𝑛𝑥 + ⋯ … 𝑡𝑜 ∞

y= √sin 𝑥 + 𝑦

94
𝑦 2 = sin x + y
𝑑𝑦 𝑑𝑦
2y = cos x +
𝑑𝑥 𝑑𝑥

𝑑𝑦
(2y-1) =cos x
𝑑𝑥

𝑑𝑦 cos 𝑥
= 2𝑦−1
𝑑𝑥
8 𝑑2 𝑦 𝑑𝑦
If y= log ( x+ √𝑥 2 + 𝑎2 ) ,prove that (𝑥 2 + 𝑎2 ) 𝑑𝑥 2 + x 𝑑𝑥 =0

SOLUTION:
y= log ( x+ √𝑥 2 + 𝑎2 )
𝑑𝑦 1 2𝑥
= [1+ ]
𝑑𝑥 x+ √𝑥 2 +𝑎2 2√𝑥 2 +𝑎2

1 x+ √𝑥2 +𝑎2
= .
x+ √𝑥 2 +𝑎2 √𝑥 2 +𝑎2

1
= √𝑥 2
+𝑎2
𝑑𝑦
𝑑𝑥
√𝑥 2 + 𝑎2 =1
𝑑2 𝑦 𝑑𝑦 2𝑋
√𝑥 2 + 𝑎2 𝑑𝑥 2
+ 𝑑𝑥 . =0
2√𝑥 2 +𝑎2

𝑑2 𝑦 𝑑𝑦
(𝑥 2 + 𝑎2 ) 𝑑𝑥 2 + x 𝑑𝑥 =0

9 𝑑𝑦
If x y = 𝑒 𝑥−𝑦 . Find 𝑑𝑥 .

SOLUTION:
x y = 𝑒 𝑥−𝑦

log x y = log 𝑒 𝑥−𝑦

log x y = x-y
𝑑𝑦
𝑥 +𝑦 𝑑𝑦
𝑑𝑥
= 1- 𝑑𝑥
𝑥𝑦

𝑑𝑦 𝑑𝑦
𝑥 𝑑𝑥 + 𝑦 = x y – x y 𝑑𝑥

𝑑𝑦
(x + x y) = (x y -y)
𝑑𝑥

𝑑𝑦 (x y −y)
=
𝑑𝑥 (x + x y)

10 𝑑2 𝑦 𝑑𝑦
If y= sin−1 𝑥 , prove that (1-𝑥 2 ) 𝑑𝑥 2 -x 𝑑𝑥 =0
SOLUTION:
y= sin−1 𝑥

95
𝑑𝑦 1
= √1−𝑥 2
𝑑𝑥

𝑑𝑦
√1 − 𝑥 2 =1
𝑑𝑥

𝑑2 𝑦 𝑑𝑦 −2𝑥
√1 − 𝑥 2 + 𝑑𝑥 . =0
𝑑𝑥 2 √√1−𝑥2

𝑑2 𝑦 𝑑𝑦
(1-𝑥 2 ) 𝑑𝑥 2 -x 𝑑𝑥 =0

EXERCISE

1 1−𝑐𝑜𝑠4𝑥
𝑖𝑓 𝑥 < 0
𝑥2
Let 𝑓(𝑥) = 𝑎 𝑖𝑓 𝑥 = 0
√𝑥
𝑖𝑓 𝑥 > 0
{√16+√𝑥−4
Determine a so that f(x) is continuous at x = 0
Ans: 𝑎 = 8
2 𝑑𝑦
If (cos 𝑥)𝑦 = (sin 𝑦)𝑥 , 𝑓𝑖𝑛𝑑 .
𝑑𝑥
𝑦𝑡𝑎𝑛𝑥+𝑙𝑜𝑔𝑐𝑜𝑠𝑦
Ans: 𝑥𝑡𝑎𝑛𝑦+𝑙𝑜𝑔𝑐𝑜𝑠𝑥
3 𝜋 𝑑2 𝑥 𝑑2 𝑦 𝑑2 𝑦
If x = a (cost + t sin t) and y = a (sin t – t cost), 0 < t < 2 , then find , 𝑎𝑛𝑑
𝑑𝑡 2 𝑑𝑡 2 𝑑𝑥 2
𝑠𝑒𝑐 3 𝑡
Ans: -at sin t+ a cost , at cos t+ a sin t, 𝑎𝑡
1
4 𝑥
If y= (𝑥𝑐𝑜𝑠𝑥 ) + (𝑥𝑠𝑖𝑛 𝑥) .Find 𝑑𝑥 𝑥
𝑑𝑦

𝑥𝑐𝑜𝑡𝑥+1−𝑙𝑜𝑔𝑥𝑠𝑖𝑛𝑥
Ans: (𝑥𝑐𝑜𝑠𝑥)𝑥 [1-tanx+log(xcosx)+(𝑥𝑠𝑖𝑛𝑥)1/𝑥 [ ]
𝑥2
5 𝑑2 𝑦 𝑑𝑦
If 𝑥 = 𝑠𝑖𝑛 𝑡 𝑎𝑛𝑑 𝑦 = 𝑠𝑖𝑛 𝑝𝑡, prove that (1-𝑥 2 ) 𝑑𝑥 2 -x𝑑𝑥 +𝑝2 𝑦 = 0

5 MARK QUESTIONS

Q.NO QUESTIONS WITH SOLUTIONS


1 Differentiate the function f(x) = 𝑥 𝑠𝑖𝑛𝑥 + 𝑠𝑖𝑛𝑥 𝑐𝑜𝑠𝑥 with respect to x.

SOLUTION:
Y = 𝑥 𝑠𝑖𝑛𝑥 + 𝑠𝑖𝑛𝑥 𝑐𝑜𝑠𝑥
Y=u+v
𝑑𝑦 𝑑𝑢 𝑑𝑣
= 𝑑𝑥 + 𝑑𝑥 -------(1)
𝑑𝑥

u= 𝒙𝒔𝒊𝒏𝒙
log u = log 𝒙𝒔𝒊𝒏𝒙

log u = sin x log x

96
1 𝑑𝑢 sin 𝑥
= +cos x log x
𝑢 𝑑𝑥 𝑥

𝑑𝑢 sin 𝑥
= 𝒙𝒔𝒊𝒏𝒙 [ +cos x log ] ------(2)
𝑑𝑥 𝑥

V= 𝒔𝒊𝒏𝒙𝒄𝒐𝒔𝒙

log v = log 𝒔𝒊𝒏𝒙𝒄𝒐𝒔𝒙

log v = cos x log sinx

1 𝑑𝑣 𝑐𝑜𝑠 2 𝑥
= − sin 𝑥 log sin 𝑥
𝑣 𝑑𝑥 sin 𝑥
𝑑𝑣 𝑐𝑜𝑠2 𝑥
= sinx cosx [ sin 𝑥 − sin 𝑥 log sin 𝑥 ]------(3)
𝑑𝑥

Substituting in (1)

𝑑𝑦 sin 𝑥 𝑐𝑜𝑠2 𝑥
= 𝒙𝒔𝒊𝒏𝒙 [ +cos x log ] + 𝒔𝒊𝒏𝒙𝒄𝒐𝒔𝒙 [ sin 𝑥 − sin 𝑥 log sin 𝑥 ]
𝑑𝑥 𝑥

2 2𝑥+1
Differentiate the function f(x) = 𝑠𝑖𝑛−1 (1+4𝑥 ) with respect to x.
SOLUTION:
𝟐𝒙+𝟏
y = 𝑠𝑖𝑛−1 (𝟏+𝟒𝒙 )

𝟐𝒙 .𝟐
= 𝒔𝒊𝒏−𝟏 ( 𝟐 )
𝟏+(𝟐𝒙)

Put 2𝑥 = tan 𝜃

2𝑡𝑎𝑛 𝜃
Y = 𝑠𝑖𝑛−1 (1+𝑡𝑎𝑛2 𝜃)

Y = sin−1 𝑠𝑖𝑛 2𝜃

Y = 2𝜃

Y = 2tan−1 2𝑥

𝒅𝒚 1 𝑑
= 2 1+(2𝑥 )2 . 𝑑𝑥(2𝑥 )
𝒅𝒙

𝑑𝑦 2
= 1+(4𝑥 ) . 2𝑥 log 2
𝑑𝑥

2𝑥+1
= 1+(4𝑥 ) log 2

97
3 Determine the values of a, b and c for which the function
sin(𝑎 + 1) 𝑥 + sin 𝑥
𝑓𝑜𝑟 𝑥 < 0
𝑥
𝑓(𝑥) = 𝑐, 𝑓𝑜𝑟 𝑥 = 0
√𝑥 + 𝑏𝑥 2 − √𝑥
3 ,𝑥 > 0
{ 𝑏𝑥 2
Is continuous at x=0.
SOLUTION:
LHL = lim− 𝑓(𝑥)
𝑥→0
= lim 𝑓(0 − ℎ)
ℎ→0

sin(𝑎+1)(0−ℎ)+sin(0−ℎ)
= lim
ℎ→0 0−ℎ

− sin(𝑎+1)ℎ sin(−ℎ)
= lim +
ℎ→0 −ℎ −ℎ

= a+1 +1

=a+2

RHL = lim+ 𝑓(𝑥)


𝑥→0

=lim 𝑓(0 + ℎ)
ℎ→0

√𝑥+𝑏𝑥 2 −√𝑥 √𝑥+𝑏𝑥 2 +√𝑥


= lim 3 . √𝑥+𝑏𝑥 2 +√𝑥
ℎ→0 𝑏𝑥 2

𝑥+𝑏𝑥 2 −𝑥
= lim 3
ℎ→0 𝑏𝑥 2 (√𝑥+𝑏𝑥 2 +√𝑥)

√𝑥
= lim
ℎ→0 √𝑥(√1+𝑏𝑥+1)

1
=2
=
f (0) = c

Since f (x) is continuous LHL=RHL=f(o)


1
a+2 = 2 = c

−3 1
a= , c=2 , b any real number other than 0.
2

98
EXERCISE

1 𝑑𝑦 √1+𝑥 2 +√1−𝑥 2
Find 𝑑𝑥 if y=tan−1 [√1+𝑥 2 ], 0 <|𝑥| < 1.
−√1−𝑥 2

−𝑥
Ans: √1−𝑥 4
𝑦
2 𝑑2 𝑦 𝑎
If (a+ bx) 𝑒 𝑥 = x then prove that x. 𝑑𝑥 2 = (𝑎+𝑏𝑥)2

3 𝑑𝑦 𝑦
If 𝑥 𝑝 𝑦 𝑞 = (𝑥 + 𝑦)𝑝+𝑞 , Prove that 𝑑𝑥 = 𝑥

CASE BASED QUESTIONS (4 MARKS)

1 Sonia was noticing the path traced by a crawling insect and she observed that the
path traced is given by
x= a𝑡 2 , y = 2at
Based on the above information, answer the following questions.
𝑑𝑥
(i) Find 𝑑𝑡
𝑑𝑦
(ii) Find
𝑑𝑥
𝑑2 𝑦
(iii) Find 𝑑𝑥 2
at t =4
OR
𝑑2 𝑥
Find 𝑑𝑦 2 at t =4

SOLUTION:

x= a𝑡 2 , y = 2at
𝑑𝑥
(i) 𝑑𝑡
= 2at

𝑑𝑦 𝑑𝑦/𝑑𝑡 2𝑎 1
(ii) 𝑑𝑥
= 𝑑𝑥/𝑑𝑡 = 2𝑎𝑡 = 𝑡

𝑑2 𝑦 −1 1 −1
(iii) 𝑑𝑥 2
= 𝑡2
x 2𝑎𝑡 = 2𝑎𝑡 3

−1
At t = 4 = 128𝑎

OR

𝑑2 𝑥 1
𝑎𝑡 𝑡 = 4 = 2𝑎
𝑑𝑦 2

2 Let f(x) be a real valued function. Then its


𝑓(𝑎−ℎ)−𝑓(𝑎)
• Left Hand Derivative (LHD): L𝑓 ′ (a)= lim −ℎ
ℎ→0
𝑓(𝑎+ℎ)−𝑓(𝑎)
• Right Hand Derivative (RHD): R𝑓 ′ (a)= lim ℎ
ℎ→0

99
Also a function f(x) is said to be differentiable at x= a if its LHD and RHD at
x=a exist and both are equal.
|𝒙 − 𝟑|, 𝑥 ≥ 1
For the function 𝑓(𝑥) = {𝑥 2 3𝑥 13
− 2 + 4 𝑥<1
4
Answer the following questions.
(i) What is RHD of f(x) at x= 1?
(ii) What is LHD of f(x) at x=1?
(iii) (a) Check if the function f(x) is differentiable at x=1
OR
(b) Find 𝑓 (2) and 𝑓 ′ (−1)

SOLUTION:

𝑓(1+ℎ)−𝑓(1)
(i) RHD of f(x) at x= 1 = lim ℎ
ℎ→0
|1+ℎ−3|−|−2|
= lim
ℎ→0 ℎ

2−ℎ−2
= lim = -1
ℎ→0 ℎ

𝑓(1−ℎ)−𝑓(1)
(ii) LHD of f(x) at x=1 = lim −ℎ
ℎ→0

−1 (1−ℎ)2 3(1−ℎ) 13
= lim [ − + − 2]
ℎ→0 ℎ 4 2 4

ℎ2 −2ℎ+1−6+6ℎ+13−8
= lim
ℎ→0 −4ℎ

ℎ2 +4ℎ
= lim = -1
ℎ→0 −4ℎ

(iii) (a) Since LHD of f(x) at x=1 = RHD of f(x) at x=1, f(x) is differentiable at
x=1

OR

𝑥 − 3, 𝑥 ≥ 3
(b) 𝑓(𝑥) = { 23 − 𝑥, 1 ≤ 𝑥 < 3
𝑥 3𝑥 13
− 2 + 4 𝑥<1
4

[𝑓 ′ (𝑥)] at x=2 = 0-1=-1

2(−1) 3
[𝑓 ′ (𝑥)] at x=-1= − 2 = -2
4

3 A potter made a mud vessel, where the shape of the pot is based on

f(x)= |x-3|+|x-2|, where f(x) represents the height of the pot.


Based on the above information answer the following questions.
(i) Find the value of [f (2.3)] where [a] represent the greatest integer ≤ a.

100
(ii) What is the value of 𝑓 ′ (𝑥) at x=4
(iii) Show that f is continuous at x=2
OR
Show that f is not differentiable at x=3

SOLUTION:
(i) f(x) = |x-3|+|x-2|

f(2.3) = |2.3-3|+|2.3-2| = 0.7+0.3 =1

[f(2.3)] =[1] =1

5 − 2𝑥, 𝑥 < 2
(ii) 𝑓(𝑥) = { 1, 2 ≤ 𝑥 ≤ 3
2𝑥 − 5, 𝑥 > 3

𝑓 ′ (4) = 2

(iii) LHL = lim−|𝑥 − 3| + |𝑥 − 2|


𝑥→2

=lim |2 − ℎ − 3| + |2 − ℎ − 2| =1
ℎ→0

RHL = lim+ |𝑥 − 3| + |𝑥 − 2|
𝑥→2

=lim |2 + ℎ − 3| + |2 + ℎ − 2| =1
ℎ→0

f(2) = |2-3|+|2-2| = 1

Since LHL=RHL=f(2), f is continuous at x=2.

OR

5 − 2𝑥, 𝑥 < 2
𝑓(𝑥) = { 1, 2 ≤ 𝑥 ≤ 3
2𝑥 − 5, 𝑥>3

L𝑓 ′ (3) =0, and R𝑓 ′ (3) =2


Since L𝑓 ′ (3) ≠ R𝑓 ′ (3), f is not differentiable at x=3.
EXERCISE

1 If a relation between x & y is such that y cannot be expressed in terms of x, then y


is called an implicit function of x. When a given relation expresses y as an
𝑑𝑦
implicit function of x and we want to find 𝑑𝑥 , then we differentiate every term of
the given relation w.r.t x, remembering that a term in y is first differentiated w.r.t
𝑑𝑦
y and then multiplied by 𝑑𝑥 .
𝑑𝑦
Based on the above information, find the value of 𝑑𝑥 in each of the following
(i) 𝑥 3 + 𝑥 2 𝑦 + 𝑥𝑦 2 + 𝑦 3 =81
(ii) 𝑥 𝑦 = 𝑒 𝑥−𝑦
(iii) 𝑒 𝑠𝑖𝑛𝑦 = 𝑥𝑦
OR

101
𝑠𝑖𝑛2 𝑥 + 𝑐𝑜𝑠 2 𝑦 = 1
−(3𝑥 2 +2𝑥𝑦+𝑦 2 ) 𝑥−𝑦 1/𝑥 𝑠𝑖𝑛2𝑥
Ans: (𝑖) ,(ii) 𝑥(𝑙𝑜𝑔𝑥+1) , (𝑖𝑖𝑖) 1 𝑜𝑟
(𝑥 2 +2𝑥𝑦+3𝑦 2 ) (𝑐𝑜𝑠𝑦− ) 𝑠𝑖𝑛2𝑦
𝑦

2 If y= f (u) is a differentiable function of u and u= g(x) is a differentiable function


𝑑𝑦 𝑑𝑦 𝑑𝑢
of x ,then y= f(g(x))is a differentiable function of x and = 𝑋 . This
𝑑𝑥 𝑑𝑢 𝑑𝑥
rule is known as CHAIN RULE.
𝑑𝑦
Based on the above information find the value of 𝑑𝑥 in each of the following
(i) cos√𝑥
1
(ii) 7𝑥+𝑥
1 𝑥 1 𝑥
(iii) 𝑏
tan−1 𝑏 +𝑎 tan−1 𝑎

OR
𝑥
sec −1 𝑥 + 𝑐𝑜𝑠𝑒𝑐 −1 √𝑥 2
−1
1
−𝑠𝑖𝑛√𝑥 𝑥+ 1
Ans: (i) (ii) 7 𝑥 log7 (1- 𝑥 2 )
2 √𝑥
1 1 2
(iii) 𝑥 2 +𝑏2 + or
𝑥 2 +𝑎2 𝑥√𝑥 2 −1

102
CHAPTER: APPLICATION OF DERIVATIVES
SYLLABUS: Applications of derivatives: rate of change of quantities, increasing/decreasing
functions, maxima and minima (first derivative test motivated geometrically and second
derivative test given as a provable tool). Simple problems (that illustrate basic principles and
understanding of the subject as well as real-life situations).

Definations and Formulae:

Derivative as Rate of Change

𝑑𝑦
• Let 𝑦 = 𝑓(𝑥) be a function. Then 𝑑𝑥 denotes the rate of change of 𝑦 𝑤. 𝑟. 𝑡 𝑥.
𝑑𝑦 𝑑𝑦
• The value of at 𝑥 = 𝑥0 i.e (𝑑𝑥 ) i.e. represents the rate of change of 𝑦 𝑤. 𝑟. 𝑡 𝑥 𝑎𝑡
𝑑𝑥 𝑥=𝑥0
𝑥 = 𝑥0
• If two variables x and y are varying with respect to another variable t, i.e., if 𝑥 = 𝑓(𝑡) and
𝑑𝑦
𝑑𝑦𝑑𝑡 𝑑𝑥
y= g(t), then by Chain Rule 𝑑𝑥 = ( 𝑑𝑥 ), provided ≠0
𝑑𝑡
𝑑𝑡
𝑑𝑦
• is positive if 𝑦 𝑖𝑛𝑐𝑟𝑒𝑎𝑠𝑒𝑠 𝑎𝑠 𝑥 increases and is negative if
𝑑𝑥
𝑦 𝑑𝑒𝑐𝑟𝑒𝑎𝑠𝑒𝑠 𝑎𝑠 𝑥 𝑖𝑛𝑐𝑟𝑒𝑎𝑠𝑒𝑠.

Increasing and Decreasing Functions

• A function y = f (x) is said to be increasing on an interval (a, b) if x 1 < x 2 in (a, b) f(x1) ≤


f(x2) for all 𝑥1 , 𝑥2 ∈ (𝑎, 𝑏)
Alternatively, a function y = f (x) is said to be increasing if f ′(x) ≥ 0 for each x in (a, b)
(a) strictly increasing on an interval (a, b) if x 1 < x 2 in (a, b) f(x1) < f(x2) for all x 1 , x 2
(a, b).
Alternatively, a function y = f (x) is said to be strictly increasing if f ′(x) > 0 for each x in
(a, b)
(b) decreasing on (a, b) if x 1 < x 2 in (a, b) f(x1) ≥ f(x 2 ) for all x 1 , x 2 (a, b). Alternatively,
a function y = f (x) ) is said to be decreasing if f ′(x) ≤ 0 for each x in (a, b)
(c) strictly decreasing on (a, b) if x 1 < x 2 in (a, b) f(x 1 ) > f(x 2 ) for all x 1 , x 2 (a, b).

Alternatively, a function y = f (x) is said to be strictly decreasing if f ′(x)<0 for each x in


(a, b)

(d) constant function in (a, b), if f (x) = c for all x (a, b), where c is a constant.
Alternatively , f(x) is a constant function if f ′ (x )= 0.

A point c in the domain of a function f at which either f ′(c) = 0 or f is not differentiable is


called a critical point of f.

103
Maxima and Minima

Definition: Let f be a function defined on an interval I. Then


1) f is said to have a maximum value in I, if there exists a point c in I such that f(c) > f(x), for all
x I. The number f(c) is called the maximum value of f in I and the point c is called a point of
maximum value of f in I.
2) f is said to have a minimum value in I, if there exists a point c in I such that f (c) < f (x), for all
x I. The number f (c), in this case, is called the minimum value of f in I and the point c, in
this case, is called a point of minimum value of f in I.
3) f is said to have an extreme value in I if there exists a point c in I such that f (c) is either a
maximum value or a minimum value of f in I. The number f (c), in this case, is called an extreme
value of f in I and the point c is called an extreme point.

Local Maxima and Local Minima

Definition: Let f be a real valued function and let c be an interior point in the domain of f.
Then
(a) c is called a point of local maxima if there is an h > 0 such that 𝑓 (𝑐) ≥ 𝑓 (𝑥), for all x in
(𝑐 – ℎ, 𝑐 + ℎ), 𝑥 ≠ 𝑐. The value f(c) is called the local maximum value of f.
(b) c is called a point of local minima if there is an h > 0 such that f (c) ≤ f (x), for all x in (c – h,
c + h). The value f(c) is called the local minimum value of f.

Geometrically, the above definition states that if x = c is a point of local maxima of f, then the
graph of f around c will be as shown in Fig.(a) below. Note that the function f is increasing
(i.e., f ′(x) > 0) in the interval (c – h, c) and decreasing (i.e., f ′(x) < 0) in the interval (c, c + h).
This suggests that f ′(c) must be zero,

Similarly ,if x = c is a point of local minima of f, then the graph of f around c will be as shown
in Fig.(b) above. Note that the function f is decreasing (i.e., f ′(x) < 0) in the interval (c – h,

104
c) and increasing (i.e., f ′(x) >0) in the interval (c, c + h). This again suggests that f ′(c) must
be zero,

Theorem: Let f be a function defined on an open interval I. Suppose c I be any point. If f has
a local maxima or a local minima at x = c, then either f ′(c) = 0 or f is not differentiable at c.

Definition: A point c in the domain of a function f at which either f ′(c) = 0 or f is not


differentiable is called a critical point of f.

Theorem: (First Derivative Test) Let f be a function defined on an open interval I. Let f be
continuous at a critical point c in I. Then
1) If f ′(x) changes sign from positive to negative as x increases through c, i.e.,
if f ′(x) > 0 at every point sufficiently close to and to the left of c, and f ′(x) < 0 at every point
sufficiently close to and to the right of c, then c is a point of local maxima.
2) If f ′(x) changes sign from negative to positive as x increases through c, i.e., if f ′(x) < 0 at every
point sufficiently close to and to the left of c, and f ′(x) > 0 at every point sufficiently close to
and to the right of c, then c is a point of local minima.
3) If f ′(x) does not change sign as x increases through c, then c is neither a point of local maxima
nor a point of local minima. In fact, such a point is called point of inflection.

Theorem: (Second Derivative Test) Let f be a function defined on an interval I and c I. Let
f be twice differentiable at c. Then
1) x = c is a point of local maxima if f ′(c) = 0 and f ″(c) < 0 The value f (c) is local maximum
value of f.
2) x = c is a point of local minima if f ′(c) 0 = and f ″(c) > 0 In this case, f (c) is local minimum
value of f.
3) The test fails if f ′(c) = 0 and f ″(c) = 0. In this case, we go back to the first derivative test and
find whether c is a point of local maxima, local minima or a point of inflexion.
Working rule for finding absolute maximum value and/or absolute minimum value
Step 1: Find all critical points of f in the interval, i.e., find points x where either f ′(x) = 0 or f
is not differentiable.
Step 2: Take the end points of the interval.
Step 3: At all these points (listed in Step 1 and 2), calculate the values of f.
Step 4: Identify the maximum and minimum values of f out of the values calculated in Step 3.

105
This maximum value will be the absolute maximum value of f and the minimum value

will be the absolute minimum value of f.

MULTIPLE CHOICE QUESTIONS

S.NO QUESTIONS WITH SOLUTIONS


1 The edge of a cube is increasing at the rate of 0.3 cm/s, the rate of change of its
surface area when edge is 3 cm is
(a) 10.8cm (b) 10.8 cm2 (c) 10.8 cm2/s (d) 10.8 cm/s

dx
Solution: (c) as dt = 0.3 cm/s ,x is edge of a cube.
Surface area S = 6x 2
ds dx dS
Then dt = 12x ⋅ dt = 12x × 0.3 dt = 3.6x
dS
And dt at x = 3 is 3.6 × 3 = 10.8 cm2/s
2 The total revenue in ₹ received from the sale of x units of an article is given by
R(x) =3x2+36x+5. The marginal revenue when x=15 is (in ₹)
(a) 126 (b) 116 (c) 96 (d) 90

Solution: (a), as Rʹ (x)=6x+36


Rʹ(15)=90+36=126
3 The point on the curve 𝑦 = 𝑥 2 where the rate of change of 𝑥 −coordinate is equal
to the rate of change of 𝑦 −coordinate is
1 1 1 1
(a) 2 (b) 4 (c) (2 , 4) (d) (1,1)

𝑑𝑦 𝑑𝑥 𝑑𝑦 𝑑𝑥
Sol: (C), as y=x2⇒ 𝑑𝑡 = 2𝑥 𝑑𝑡 ; given 𝑑𝑡 = 𝑑𝑡
1
1 = 2𝑥 ⇒ 𝑥 =
2
1 1
Substituting in the equation of curve, we get point as (2 , 4)

4 The interval on which the function f(x)=2x3+9x2+12x-1 is decreasing ,is


(a) (−1, ∞) (b) (−2, −1)
(c) (−∞, −2) (d) [−1,1]

Sol: (b) We have, f(x)=2x3+9x2+12x-1


fʹ(x)=6x2+18x+12=6(x2+3x+2)= 6(𝑥 + 2)(𝑥 + 1)
for f(x) to be decreasing, we must have
𝑓ʹ(𝑥) < 0 ⟹ 6(𝑥 + 2)(𝑥 + 1) ≤ 0
⟹ (𝑥 + 2)(𝑥 + 1) ≤ 0
−2 ≤ 𝑥 ≤ −1.
Hence, f(x) is decreasing on (−2, −1).

5 If at 𝑥 = 1, the function 𝑓(𝑥) = 𝑥 4 − 62𝑥 2 + 𝑎𝑥 + 9 attains its maximum


value on the interval [0, 2]. Then the value of 𝑎 is
(a) 124 b) −124 c) 120 d) −120

Sol: (c)
As fʹ(x) = 4x3-124x+a,

106
Given x=1 is point of maximum
⇒ fʹ(1)=0 ⇒ 4 – 124 + a = 0 ⇒ a = 120
6 The function f(x) = 4 sin3 x – 6 sin2 x + 12 sinx + 100 is strictly
3π π
(a) increasing in (π, 2 ) b) decreasing in ( 2 , π)
π π π
c) decreasing in(− 2 , 2 ) d) decreasing in (0, 2 )

Sol: (b) we have, f(x) = 4 sin3 x – 6 sin2 + 12 sin +100


1 2 3
⇒ fʹ(x)= 12(sin2x-sin x + 1) cos x = {(sin 𝑥 − 2) + 4} cos x
⇒ Sign of fʹ(x) is same as that of cos x
π π
⇒ fʹ(x) < 0 on (0, 2 ) ⇒ f(x) is decreasing on ( 2 , π)

7 Which of the following functions is decreasing in (0, π/2).


(a) 𝑠𝑖𝑛 2𝑥 b) 𝑡𝑎𝑛 𝑥 c) 𝑐𝑜𝑠 𝑥 d) 𝑐𝑜𝑠 3𝑥
𝑑
Sol :(c) We find that 𝑑𝑥 (𝑐𝑜𝑠 𝑥) = −𝑠𝑖𝑛 𝑥 < 0 for all
𝑥 ∈ (0, 𝜋/2)
So, 𝑐𝑜𝑠 𝑥 𝑖𝑠 𝑑𝑒𝑐𝑟𝑒𝑎𝑠𝑖𝑛𝑔 𝑜𝑛 (0, 𝜋/2)

8 The function f(x)=2x3-3x2-12x+4 has


(a) two points of local maximum
(b) two points of local minimum
(c) one maximum and one minimum
(d)no maximum, no minimum

Sol: (c) We have, f(x)=2x3-3x2-12x+4


⇒ fʹ(x)=6x2+18x+12 and fʹʹ (x)=12x-6
At points of local maximum or minimum, we have
⇒ fʹ(x)=0 ⇒ 6(x2-x-2)=0 ⇒ (x-2) (x+1) = 0 ⇒ x = -1, 2
At x = -1, we obtain : fʹʹ (-1) = -18<0. So, x = -1 is a point of local maximum.
At x = 2, we obtain : fʹʹ (2) = 24 - 6 = 18>0. So, x = 2 is a point of local
minimum.
9 The interval on which the function f(x)=x3+6x2+6 is strictly increasing is
(a) (-∞,-4) ∪ (0, ∞) (b) (-∞,-4)
(c) (-4,0) (d) (-∞,0) ∪ (4, ∞)

Sol: (a) We have, f(x) = x3+6x2+6


⇒ fʹ(x) =3x2+12x=3x(x+4)
For f(x) to be increasing, we must have
fʹ(x)>0 ⇒ 3x(x+4)>0 ⇒ x(x+4) > 0 ⇒ x<-4 or, x>0
Hence, f(x) is increasing on (-∞,-4) ∪ (0, ∞).
10 The rate of change of the area of a circle with respect to its radius 𝑟 𝑎𝑡 𝑟 = 6𝑐𝑚
is:
(a) 10π cm2/cm (b) 12π cm2/cm
(c) 8π cm /cm
2
(d)11π cm2/cm

Sol: (b) Area of circle (𝐴) = 𝜋𝑟 2


𝑑𝐴
⇒ 𝑑𝑟 = 2 𝜋r
𝑑𝐴
⇒ 𝑑𝑟 ]r = 6
= 2π x 6 = 12π

107
11 The total revenue in Rupees received from the sale of x units of a product is
given by R(x) = 3x2+36x+5. The marginal revenue, when 𝑥 = 15 is:
(a) 116 b) 96 c) 90 d) 126

Sol: (d) Total revenue R(x) = 3x2+36x+5


𝑑
Marginal revenue = 𝑑𝑥 𝑅(𝑥) = 6𝑥 + 36 = 6 × 15 + 36 =126

12 The maximum value of the function 𝑓(𝑥) = 5 + 𝑠𝑖𝑛2𝑥 is


(a) 1 b) 6 c) 4 d) −1

Sol: (b)
−1 ≤ 𝑠𝑖𝑛 2𝑥 ≤ 1
⇒ 5 − 1 ≤ 5 + 𝑠𝑖𝑛 2𝑥 ≤ 5 + 1
⇒ 4 ≤ 𝑓(𝑥) ≤ 6
Maximum value of 𝑓(𝑥) = 6

13 The function 𝑓(𝑥) = 𝑥 − 𝑠𝑖𝑛 𝑥 decreases for


(a) 𝑎𝑙𝑙 𝑥 (b) 𝑥 < 𝜋/2
(c) 0 < 𝑥 < 𝜋/4 (d) 𝑛𝑜 𝑣𝑎𝑙𝑢𝑒 𝑜𝑓 𝑥

Sol: (d)
We have, 𝑓(𝑥) = 𝑥 − 𝑠𝑖𝑛 𝑥
⇒ fʹ(x) = 1 – cos 𝑥 ≥ 0 for all 𝑥 , since −1 ≤ 𝑐𝑜𝑠𝑥 ≤ 1

⇒ 𝑓(𝑥) is increasing for all 𝑥 ∈ 𝑅 ⇒ 𝑓(𝑥) decreases for no value of 𝑥.


14 The absolute maximum value of f(x) = x3 - 3x + 2 in 0 ≤ 𝑥 ≤ 2 is
(a) 4 b) 6 c) 2 d) 0

Sol: (a)
As fʹ(x) = 3x2-3, fʹ(x) = 0 ⇒ x = ±1.
f(0) = 2, f(1) =1 – 3 + 2 =0,
f(-1) = -1 + 3 + 2 = 4, f(2) = 8 – 6 + 2 = 4

CHAPTER VIDEO LINK FOR MCQs SCAN QR CODE FOR


VIDEO

APPLICATION OF DERIVATIVES https://youtu.be/0Zk2RUMBcWU

108
EXERCISE

1 For the function y = x3+21, the value of x, when y increases 75 times as fast as x,
is
(a) ±3 (b) ±5√3 c) ±5 d) none of these
Answer: c
2 1 𝑥
The maximum value of (𝑥) is
1
1
𝑒 1 𝑒
(a) 𝑒 (b) 𝑒 c) 𝑒 𝑒 d) (𝑒 )
Answer: c
3 The function f(x) = cos x -2px is monotonically decreasing for
1 1
(a) p < 2 b) p > 2 c) p < 2 d) p > 2
Answer: b
4 The maximum value of xy, subject to x+y = 8 is
(a) 8 b) 16 c) 20 d) 24
Answer: b
5 2
A particle moves along the curve 𝑦 = 3 x 3 + 1. The x-coordinates of the points on
the
curve at which y-coordinate is changing twice as fast as x-coordinate is
5 1
(a) 1 b) ±1 c) 3 d) 3
Answer: b

ASSERTION AND REASONING QUESTIONS

In the following questions, a statement of assertion (A) is followed by a statement of


reason(R). Choose the correct answer out of the following choices.(a) Both (A) and (R)
are true and (R) is the correct explanation of (A)
(b) Both (A) and (R) are true and (R) is not the correct explanation of (A)
(c) (A) is true but (R) is false
(d) (A) is false but (R) is true

1 𝑑
Let f(x) be a polynomial function in a degree 6 such that 𝑑𝑥(f(x)) = (𝑥 − 1)3(𝑥 −
3)2 , then
Assertion (A): 𝑓(𝑥) ℎ𝑎𝑠 𝑎 𝑚𝑖𝑛𝑖𝑚𝑢𝑚 𝑎𝑡 𝑥 = 1.
𝑑 𝑑
Reason (R): When 𝑑𝑥(f(x))<0, ∀ 𝑥 ∈ (𝑎 − ℎ, 𝑎) and 𝑑𝑥 (f(x))>0, ∀ 𝑥 ∈ (𝑎, 𝑎 +
ℎ); where ‘h’ is an infinitesimally small positive quantity, then f(x) has a
minimum at x=a, provided f(x) is continuous at x=a.
Sol : (a)
𝑑
(f(x)) = (𝑥 − 1)3(𝑥 − 3)2
𝑑𝑥
Assertion : f(x) has a minimum at x=1 is true as
𝑑 𝑑
(f(x))<0, ∀ 𝑥 ∈ (1 − ℎ, 1) and (f(x))>0, ∀ 𝑥 ∈ (1,1 + ℎ); where, ‘h’ is an
𝑑𝑥 𝑑𝑥
infinitesimally small positive quantity, which is in accordance with the Reason
statement.
2 Let C be the circumference and A be the area of a circle.
Assertion(A): The rate of change of the area with respect to radius is equal to C.
𝐶
Reason(R): The rate of change of the area with respect to diameter is 2.
Sol: (b)

109
Let r be the radius of the circle. Then,
𝑑𝐴
A = 𝜋𝑟 2 and C = 2 𝜋r ⇒ 𝑑𝑟 = 2 𝜋r = C.
So, (A) is true.
Let x be the diameter of the circle. Then,
𝑥 𝑥 𝑑𝐴 𝜋𝑥 𝑑𝐴 𝐶
A = (2)2= 4 𝑥 2 and C = 𝜋𝑥 ⇒ 𝑑𝑥 = 2 ⇒ 𝑑𝑥 = 2
So, (R) is also true but (R) is not a correct explanation for (A).

3 Let the radius, surface area and volume of sphere be r, S and V respectively.
Assertion(A): The rate of change of volume of sphere with respect to its radius is
equal to S.
𝑟
Reason(R): The rate of change of volume of sphere with respect to S is 2.
Sol: (b)
We have,
4 𝑑𝑉 𝑑𝑆 𝑑𝑉
V = 𝜋𝑟 3 and S = 4 𝜋𝑟 2 ⇒ = 4 𝜋𝑟 2and = 8 𝜋r ⇒ =
3 𝑑𝑟 𝑑𝑟 𝑑𝑟
𝑑𝑉 𝑑𝑉/𝑑𝑟 4 𝜋𝑟 2 𝑟
𝑆 , (𝐴)𝑖𝑠 𝑡𝑟𝑢𝑒 𝑎𝑛𝑑 = = = ,(R)is true
𝑑𝑆 𝑑𝑆/𝑑𝑟 8 𝜋r 2
Thus, both (A) & (R) are true but (R) is not a correct explanation for (A).

4 Assertion(A): If the area of a circle increases at a uniform rate, then its perimeter
varies inversely as its radius.
Reason(R): The rate of change of area of a circle with respect to its perimeter is
equal to the radius.

Sol: (a)
Let r be the radius, P be the perimeter and A be the area of a circle. Then,
𝑑𝐴 𝑑𝑃
(𝐴) = 𝜋𝑟 2 and P =2 𝜋r ⇒ = 2 𝜋r and = 2 𝜋
𝑑𝑟 𝑑𝑟
𝑑𝐴 𝑑𝐴/𝑑𝑟 2𝜋r
⇒ 𝑑𝑃 =𝑑𝑃/𝑑𝑟 = =r
2𝜋
So, (R) is true.
𝑑𝐴 𝑑𝐴/𝑑𝑡 𝑑𝑃 1 𝑑𝐴
Now, 𝑑𝑃 = r ⇒ 𝑑𝑃/𝑑𝑡 = 𝑟 ⇒ 𝑑𝑡 = 𝑟 𝑑𝑡
𝑑𝐴
If = constant (=k,say). Then,
𝑑𝑡
𝑑𝑃 𝑘 𝑑𝑃 1
= 𝑟 ⇒ 𝑑𝑡 ∝ 𝑟 ⇒Perimeter varies inversely as the radius.
𝑑𝑡
So, (A) is also true and (R) is a correct explanation for (A).

5 Let f(x) =1-x3- x5


Assertion(A): f(x) is an increasing function
Reason(R): 3x2+5x4>0, for all x ≠ 0.
Sol: (d)
f(x) = 1-x3- x5
⇒ fʹ(x)= -3x2-5x4 = - (3x2+5x4)<0
⸪3x2+5x4>0
⸫ f(x) is decreasing [(A) is false]
x2+x4 always >0 for all x>0
⸫ 3x2+5x4>0, for all x≠0 [R is true]

6 𝐿𝑒𝑡 𝑓(𝑥) = 2𝑠𝑖𝑛3𝑥 + 3𝑐𝑜𝑠3𝑥


5𝜋
Assertion(A): f(x) does not have a maximum or minimum at x = 6
5𝜋
Reason(R): fʹ( 6 ) =0

110
Sol: (c)
f(x) = 2sin3x+3cos3x
fʹ(x)= 6cos3x-9sin3x
for maximum or minimum fʹ(x)=0
5𝜋 5𝜋 5𝜋
fʹ( 6 )= 6cos 2 - 9sin 2
= 0-9 = -9 ≠ 0
5𝜋
⸫ f(x) does not have a maximum at x = 6
⇒ (A) is true
5𝜋
fʹ( 6 ) = −9 ≠ 0 ⸫ (R) is false
⸫ (A) is true but (R) is false.
7 Let f(x) = 2x3-3x2-12x+4
Assertion(A): x = -1 is a point of local maximum
Reason(R): fʹʹ (-1) >0

Sol: (c)
f(x) = 2x3-3x2-12x+4
fʹ(x)= 6x2-6x-12
= 6(x2-x-2)
=6(x-2) (x+1)
fʹ(x) = 0 ⇒ x=2 or x=-1
= fʹʹ (x) = 6(2x-1)
fʹʹ(1) = 6(-2-1) = -18<0
⸫ x = -1 is a point of local maximum, [(A) is true.]
fʹʹ(-1) = -18<0
⸫ (R) is false.

8 𝐿𝑒𝑡 𝑓(𝑥) = 𝑥 + 𝑐𝑜𝑠𝑥


Assertion(A): f(x) is an increasing function on R
Reason(R): −1 ≤ 𝑠𝑖𝑛𝑥 ≤ 1
Sol: (a)
f(𝑥) = 𝑥 + 𝑐𝑜𝑠𝑥
fʹ(x)= 1-sinx ≥0, for all x (⸪-1≤sinx≤1)
⇒ f is an increasing function.
(A)is true
−1 ≤ 𝑠𝑖𝑛𝑥 ≤ 1 ⇒(R)is true
Thus, both (A) & (R) are true and(R) is the correct explanation for (A).

9 Assertion(A) : The function f(x) = x3+5x+1, 𝑥 ∈ ℝ is always increasing


Reason(R): fʹ(x)>0 for 𝑥 ∈ ℝ, for increasing function
Sol: (a)
f(x) = x3+5x+1
fʹ(x)= 3x2+5>0 for all 𝑥 ∈ ℝ
⸫ f(x) is always increasing ((A)is true)
(R)is also true and R is the correct explanation of A.

10 𝐿𝑒𝑡 𝑓(𝑥) = 𝑠𝑖𝑛𝑥


𝜋
Assertion(A) : f(x) is increasing in (0, 2 )

𝜋
Reason(R): cos𝜃 is positive for all 𝜃 ∈ (0, 2 )
Sol: (a)

111
𝜋
f(x) = sinx ⇒ fʹ(x)= cosx>0, for all 𝑥 ∈ (0, 2 )
𝜋
⇒ f is increasing in (0, 2 ) [(A)is true]
(R)is also true and R is the correct explanation of A.

EXERCISE

In the following questions, a statement of assertion (A) is followed by a statement of


reason(R). Choose the correct answer out of the following choices.
(a) Both (A) and (R) are true and (R) is the correct explanation of (A)
(b) Both (A) and (R) are true and (R) is not the correct explanation of (A)
(c) (A) is true but (R) is false
(d) (A) is false but (R) is true

1 Assertion (A) :The minimum value of 𝑥 2 – 8x + 17 is 4.


Reason (R) : A function f(x) is minimum at x=c if f ′(𝑐) = 0 and f ′ ′(𝑐) is
positive.
Answer: d
2 Assertion (A) :The absolute minimum value of 𝑥 3 – 18x2 -96x in [0,9] is 0.
Reason (R) : A function always attains absolute minimum in the interval
[𝑎, 𝑏] 𝑎𝑡 𝑥 = 𝑎
Answer: c
3 Assertion (A) : Let f(x)=𝑒 𝑥 is an increasing function ∀𝑥 ∈ 𝑅.
Reason (R) : If f ′(𝑥) ≤ 0 then f(x) is an increasing function.
Answer: c
4 Assertion (A) : f(x) =logx is defined for all 𝑥 ∈ (0, ∞).
Reason (R) : If f ′(𝑥) > 0 then f(x) is strictly increasing function.
Answer: b
5 Assertion (A) : f(x) = sin 2x+3 is defined for all real values of x.
Reason (R) : Minimum value of f(x) is 2 and Maximum value is 4.
Answer: b

2 MARK QUESTIONS

1 Radius of variable circle is changing at the rate of 5cm/s. What is the radius of the
circle at a time when its area is changing at the rate of 100cm2 /s ?

Solution: The area A of a circle with radius r is given by A = 𝜋𝑟 2


𝑑𝐴 𝑑𝑟 10
= 2 𝜋r 𝑑𝑡 ⇒ 100 = 2 𝜋r x 5 ⇒ r = 𝜋 cm.
𝑑𝑡
𝑑𝑟 𝑑𝐴
[𝑑𝑡 = 5𝑐𝑚/𝑠, = 100cm2 /s]
𝑑𝑡
10
Hence, radius of the circle is r = 𝜋 cm.
2 The side of an equilateral triangle is increasing at the rate of 0.5 cm/s. Find the
rate of increase of its perimeter.
𝑑𝑎
Sol: Let the side of the triangle be ‘a’ then 𝑑𝑡 = 0.5 𝑐𝑚/𝑠
Perimeter of the triangle, P =3a
𝑑𝑃 𝑑𝑎
⇒ 𝑑𝑡 = 3 𝑑𝑡 =3 x 0.5 cm/s = 1.5 cm/s

112
3 If the rate of change of volume of a sphere is equal to the rate of change of its
radius, then find the radius.
𝑑𝑉 𝑑𝑟
Sol: Given, =
𝑑𝑡 𝑑𝑡
𝑑 4 𝑑𝑟
⇒ 𝑑𝑡 (3 𝜋𝑟 3 ) = 𝑑𝑡
𝑑𝑟 𝑑𝑟
⇒ 4𝜋𝑟 2 . 𝑑𝑡 = 𝑑𝑡
⇒ 4𝜋𝑟 2 = 1
1 1
⇒ 𝑟 2 = 4𝜋 ⇒ 𝑟 = 2 𝜋 units

4 3
A balloon which always remain spherical has a variable diameter (2𝑥 + 1).
2
Find the rate of change of its volume with respect to x.
3
Sol: Diameter of the balloon = 2 (2𝑥 + 1)
3
⸫ r = radius of the balloon = 4 (2𝑥 + 1)
4 3
Volume of the balloon, 𝑉 = 3 𝜋[4 (2𝑥 + 1)]3
9
= 16 𝜋(2𝑥 + 1)3
𝑑𝑉 9 27
⇒ 𝑑𝑥 = 16 𝜋.3(2𝑥 + 1)2 . 2 = 8
𝜋(2𝑥 + 1)2

5 x and y are the sides of two squares such that 𝑦 = 𝑥 − 𝑥 2 . Find the rate of change
of the area of Second Square with respect to the area of the first square.
Sol: The area 𝐴1 of square of side x is given by 𝐴1 = 𝑥 2
and area 𝐴2 of square of side y is given by 𝐴2 = y 2 =(𝑥 − 𝑥 2 )2
𝑑𝐴1 𝑑𝐴2
= 2𝑥, = 2(𝑥 − 𝑥 2 ) (1 − 2𝑥)
𝑑𝑥 𝑑𝑥

𝑑𝐴2 𝑑𝐴2 𝑑𝐴1 2(𝑥−𝑥 2 )(1−2𝑥)


= ÷ =
𝑑𝐴1 𝑑𝑥 𝑑𝑥 2𝑥
= (1 − 𝑥) (1 − 2𝑥) = 1 − 3𝑥 − 2𝑥 2

6 Find the intervals in which 𝑓(𝑥) = 𝑥 2 − 2𝑥 + 15 is strictly increasing or strictly


decreasing.
Sol: We have,
𝑓(𝑥) = −𝑥 2 − 2𝑥 + 15
⇒ 𝑓ʹ(𝑥)= −2𝑥 − 2 = -2(𝑥 + 1)
For 𝑓(𝑥) to be increasing, we must have
𝑓ʹ(𝑥)>0
⇒ -2(𝑥 + 1)>0
⇒ 𝑥 + 1<0
⇒ 𝑥 < −1 ⇒ 𝑥 ∈ (−∞, −1)
Thus, 𝑓(𝑥) is increasing on the interval (−∞, −1).
For 𝑓(𝑥) to be decreasing, we must have
𝑓ʹ(𝑥) < 0
⇒ -2(𝑥 + 1)<0
⇒ 𝑥 + 1>0
⇒ 𝑥 > −1 ⇒ 𝑥 ∈ (−1, ∞)
So, 𝑓(𝑥) is decreasing on (−1, ∞).
7 Show that the function 𝑓(𝑥) = (𝑥 3 − 6𝑥 2 + 12𝑥 + 18) is an increasing function
on R.
Sol: 𝑓(𝑥) = (𝑥 3 − 6𝑥 2 + 12𝑥 + 18)
⇒ 𝑓ʹ(𝑥) = 3𝑥 2 + 12𝑥 + 12
= 3(𝑥 2 + 4𝑥 + 4) = 3(𝑥 − 2)2 ≥ 0 for all 𝑥 ∈ 𝑅.
Thus, 𝑓ʹ(𝑥) ≥ 0 for all 𝑥 ∈ 𝑅.
Hence, 𝑓(𝑥) is an increasing function on R.

113
8 Find the intervals on which the function 𝑓(𝑥) = (5 + 36𝑥 + 3𝑥 2 − 2𝑥 3 ) is
increasing.
Sol: 𝑓(𝑥) = (5 + 36𝑥 + 3𝑥 2 − 2𝑥 3 )
⇒ 𝑓ʹ(𝑥) = 36 + 6𝑥 − 6𝑥 2
= -6(𝑥 2 − 𝑥 − 6) = -6(x+2)(x-3)
𝑓(𝑥) is increasing
⇒ 𝑓ʹ(𝑥) ≥ 0
⇒ -6(𝑥 + 2)(𝑥 − 3) ≥ 0
⇒ (𝑥 + 2)(𝑥 − 3) ≤ 0
⇒ −2 ≤ 𝑥 ≤ 3
⇒ 𝑥 ∈ [−2,3].
⸫ 𝑓(𝑥) is increasing on[−2,3].

9 Find the maximum and the minimum values of the function 𝑓(𝑥) = 𝑥 + 2, 𝑥 ∈
(0,1).
Sol: 𝑓(𝑥) = 𝑥 + 2
𝑓ʹ(𝑥) = 1
so for no value of x, 𝑓ʹ(𝑥) = 0.
So 𝑓(𝑥) has no critical points.
Hence, 𝑓(𝑥) has neither local maximum nor local minimum.

10 Amongst all pairs of positive numbers with sum 24, find those whose product is
maximum.
Sol: Let the numbers be x and (24 − 𝑥).
Let 𝑃 = 𝑥(24 − 𝑥) = (24𝑥 − 𝑥 2 )
𝑑𝑃 𝑑2 𝑃
Then, 𝑑𝑥 = (24 − 2𝑥)and 𝑑𝑥 2 = -2.
𝑑𝑃
Now, 𝑑𝑥 = 0 ⇒ (24 − 2𝑥) = 0 ⇒ x = 12.
𝑑2 𝑃
Thus,{𝑑𝑥 2 } = -2<0.
𝑥=12
⸫ x=12 is a point of maximum.
Hence, the required numbers are 12 and 12.

11 Find the local maxima and local minima, if any of the function f, given by f(x) =
𝜋
sin x + cos x, 0 < x < 2 .
Sol: f(x) = sin x + cos x
𝑓ʹ(𝑥) = cos x – sin x,
for points of local maxima or minima
𝜋
𝑓ʹ(𝑥) = 0 ⇒ cos x – sin x =0⇒ tan x = 1 ⇒ 𝑥 = 4
𝜋
fʹʹ (x) = – sin x – cos x, fʹʹ ( 4 )<0
𝜋
⸫𝑥 = is a point of local maximum
4
𝜋 𝜋 𝜋
Local maximum value = f( 4 ) = 𝑠𝑖𝑛 4 + 𝑐𝑜𝑠 4
1 1
= + = √2
√2 √2
12 Find the interval/s in which the function 𝑓 ∶ ℝ → ℝ defined by (𝑥) = 𝑥𝑒 𝑥 , is
increasing.

Sol: 𝑓(𝑥) = 𝑥𝑒 𝑥 ⇒ 𝑓ʹ(𝑥) = 𝑒 𝑥 (𝑥 + 1)


For 𝑓(𝑥) to be increasing, 𝑓ʹ(𝑥)≥0
⇒ 𝑒 𝑥 (𝑥 + 1) ≥ 0 ⇒ 𝑥 ≥ −1 as 𝑒 𝑥 > 0, ∀ 𝑥 ∈ ℝ

114
Hence, 𝑓(𝑥) increases in [−1, ∞).
13 1
If 𝑓(𝑥) = 4𝑥 2 +2𝑥+1; 𝑥 ∈ ℝ, then find the maximum value of 𝑓(𝑥).
1
Sol: We have 𝑓(𝑥) = 4𝑥 2 +2𝑥+1
1 1 3
Let 𝑔(𝑥) = 4𝑥 2 + 2𝑥 + 1 = 4(𝑥 2 + 2𝑥 4 + 16) + 4
1 2 3 3
= 4(𝑥 + 4) + 4 ≥ 4
3
⇒minimum value of 𝑔(𝑥) = 4.

4
⸫ maximum value of 𝑓(𝑥) = 3.
14 Find the maximum profit that a company can make, if the profit function is given
by 𝑃(𝑥) = 72 + 42𝑥 − 𝑥 2 , where x is the number of units and P is the
profit in rupees.

Sol: 𝑃(𝑥) = 72 + 42𝑥 − 𝑥 2


𝑃ʹ(𝑥) =42 – 2x , 𝑃ʹʹ(x) = −2
For maxima or minima,
𝑃ʹ(𝑥) = 0 ⇒ 42 – 2x = 0 ⇒ x = 21
𝑃ʹʹ(x) = −2 < 0
So, 𝑃(𝑥) is maximum at x = 21.
The maximum value of 𝑃(𝑥)
= 𝑃(21) =72+(42 x 21)- (21)2 = 513
i.e., the maximum profit is Rs.513.
15 Check whether the function 𝑓 ∶ ℝ → ℝ defined by 𝑓(𝑥) = 𝑥 3 + 𝑥, has any
critical point/s or not. If yes, then find the point/s.

Sol: 𝑓(𝑥) = 𝑥 3 + 𝑥, for all 𝑥 ∈ ℝ.


𝑓ʹ(𝑥) = 3𝑥 2 + 1 > 0 for all 𝑥 ∈ ℝ, (𝑥 2 ≥ 0 )
⇒ 𝑓ʹ(𝑥) ≠ 0
Hence, no critical point exists.

EXERCISE

1 The total cost C(x) associated with the production of x units of an item is given by
C(x) = = 0.005𝑥 3 − 0.02𝑥 2 − 30𝑥 + 5000.
Find the marginal cost when 3 units are produced, where by marginal cost we
mean the instantaneous rate of change of total cost at any level of output.
Answer: 29.985
2 Find the intervals in which the function 𝑓(𝑥) = 2𝑥 3 − 3𝑥 2 − 36𝑥 + 7 is strictly
increasing & decreasing.
Answer: Increasing. in (−∞, −𝟐) ∪ (𝟑, ∞). Decreasing. in (−𝟐 , 𝟑).

3 A ladder, 5-meter-long, standing on a horizontal floor, leans against a vertical


wall. If the top of the ladder slides downwards at the rate of 10cm/sec, find the
rate at which the angle between the floor and the ladder is decreasing when lower
end of ladder is 2 meters from the wall.
𝟏
Answer: 𝟐𝟎 radians/sec
4 Let x and y be the radii of two circle such that 𝑦 = 𝑥 2 + 1. Find the rate of
change of circumference of second circle w.r.t the circumference of the first
circle.
Answer: 𝟐𝒙

115
5 Find the least value of the function 𝑓(𝑥) = 𝑥 3 − 18𝑥 2 + 96𝑥 in the interval
[0,9].
Answer: 0

3 MARK QUESTIONS

1 A man 1.6 m tall walks at the rate of 0.5 m/s away from a lamp post, 8 meters
high. Find the rate at which his shadow is increasing and the rate with which the
tip of shadow is moving away from the pole.
Solution: Let AB be the lamp post and CD the height of the man.
Let distance of the man from the lamp post be x m
and from tip of shadow be y m. A
𝑑𝑥
= 0.5 m/s
𝑑𝑡
In similar triangles ABO ans CDO
8 𝑥+𝑦
=
1.6 𝑦
1 8
⇒ 5y = x+y ⇒ y = 4 𝑥 m
𝑑𝑦 1 𝑑𝑥 0.5
⸫ 𝑑𝑡 = 4 𝑑𝑡 = 4 = 0.125 m/s C
1
⸫ Rate at which shadow is increasing 0.125 m/s .
B
𝑑 x 6 Dy O
Rate of change of tip of shadow = (𝑥 + 𝑦) m m m
𝑑𝑡
𝑑𝑥 𝑑𝑦
= 𝑑𝑡 + 𝑑𝑡
= 0.5+0.125 = 0.625 m/s

2 The area of an expanding rectangle is increasing at the rate of 48𝑐𝑚2 /𝑠. The
length of the rectangle is always equal to square of breadth. At what rate, the
length is increasing at the instant when breadth is 4.5 cm?
Solution: Let the length of the rectangle be l and its breadth b.
3
Then 𝑙 = 𝑏 2 ⇒ 𝐴 = 𝑙. √𝑙 = 𝑙 2
𝑑𝐴 3 𝑑𝑙
= √𝑙.
𝑑𝑡 2 𝑑𝑡

3 𝑑𝑙
⇒ 48 = 2 x 4.5 x 𝑑𝑡 …… ( b= √𝑙 )
𝑑𝑙 320 64
⇒ 𝑑𝑡] = = = 7.11 cm/s
𝑏=4.5 𝑐𝑚 45 9

3 Sand is pouring from a pipe at the rate of 12 cm3/s. The falling sand forms a cone
on the ground in such a way that the height of the cone is always one-sixth of the
radius of the base. How fast is the height of the sand cone increasing when the
height is 4 cm?
Solution: Let h be the height, V the volume and r the radius of the base of cone at
the time t.
1
Given ℎ = 6 𝑟
⇒ 𝑟 = 6ℎ
𝑑𝑉
𝑑𝑡
= 12 𝑐𝑚3 /𝑠
1
Volume of the cone, 𝑉 = 𝜋𝑟 2 h h
3
1
= 3 𝜋(6ℎ)2 h = 12 𝜋ℎ3
𝑑𝑉 𝑑ℎ
⸫ 𝑑𝑡 = 12 𝜋 . 3ℎ2 . 𝑑𝑡 r
𝑑ℎ 𝑑ℎ 1
⇒ 12 = 36 𝜋h2 𝑑𝑡 ⇒ 𝑑𝑡 = 3𝜋h2

116
𝑑ℎ 1 1
⇒ ] =
𝑑𝑡 ℎ=4 3𝜋𝑋16
= 48𝜋 𝑐𝑚/𝑠.

4 Find the intervals in which the function 𝑓(𝑥) = 2𝑥 3 − 9𝑥 2 + 12𝑥 + 15


is strictly increasing.

Solution:We have,
𝑓(𝑥) = 2𝑥 3 − 9𝑥 2 + 12𝑥 + 15
⇒ 𝑓ʹ(𝑥) = 6𝑥 2 − 18𝑥 + 12 = 6(𝑥 2 − 3𝑥 + 2)
(i) For 𝑓(𝑥) to be increasing, we must have
𝑓ʹ(𝑥) > 0
⇒ 6(𝑥 2 − 3𝑥 + 2) > 0
⇒ 𝑥 2 − 3𝑥 + 2 > 0 [∵ 6 > 0 ∴ 6(𝑥 2 − 3𝑥 + 2) > 0 ⇒ 𝑥 2 − 3𝑥 + 2 > 0]
⇒ (𝑥 − 1) (𝑥 − 2) > 0 (See fig)
⇒ 𝑥 < 1 or 𝑥 > 2
⇒ 𝑥 ∈ (−∞, 1) ∪ (2, ∞).
So, 𝑓(𝑥) is increasing on (−∞, 1) ∪ (2, ∞).
+ +
- 1 2 ∞
∞ Fig. Signs of 𝑓ʹ(𝑥) for different values of x

5 Show that 𝑓(𝑥) = 𝑡𝑎𝑛−1 (𝑐𝑜𝑠𝑥 + 𝑠𝑖𝑛𝑥) is a strictly increasing function on the
𝜋
interval (0, 4 ).
Solution: 𝑓(𝑥) = 𝑡𝑎𝑛−1 (𝑐𝑜𝑠𝑥 + 𝑠𝑖𝑛𝑥)
1 𝑑
⇒ 𝑓ʹ(𝑥) = 1+(𝑐𝑜𝑠𝑥+𝑠𝑖𝑛𝑥)2 . 𝑑𝑥 (𝑐𝑜𝑠𝑥 + 𝑠𝑖𝑛𝑥)

(−𝑠𝑖𝑛𝑥+cos 𝑥)
=
1+𝑐𝑜𝑠2 x+𝑠𝑖𝑛2 x+2 sin x cos x

𝑐𝑜𝑠 𝑥−𝑠𝑖𝑛 𝑥
=
(2+sin 2𝑥)
𝜋
Now, when 0 <x<4 , we have 𝑐𝑜𝑠𝑥 > 𝑠𝑖𝑛𝑥 &𝑠𝑖𝑛2𝑥 > 0
⸫ (𝑐𝑜𝑠𝑥 − 𝑠𝑖𝑛𝑥) > 0 and (2 + 𝑠𝑖𝑛2𝑥) > 0.
𝜋
⸫ 𝑓ʹ(𝑥) > 0 for all x when 0 <x<4
𝜋
Hence, 𝑓(𝑥) is strictly increasing in (0, 4 ).

𝜋
6 Separate [0, 2 ] into subintervals in which 𝑓(𝑥) = sin 3x is (a) increasing (b)
decreasing.
Solution: 𝑓(𝑥) = sin 3x ⇒ 𝑓ʹ(𝑥) = 3cos 3x
𝜋 3𝜋
Also, 0 ≤ 𝑥 ≤ 2 ⇒ 0 ≤ 3𝑥 ≤ 2
(a) 𝑓(𝑥) is increasing
⇒ 𝑓ʹ(𝑥) ≥ 0
⇒ 3 cos 3𝑥 ≥ 0 ⇒ cos 3𝑥 ≥ 0
𝜋
⇒ 0 ≤ 3𝑥 ≤ 2
𝜋
⇒0≤𝑥≤ 6
𝜋
⇒ 𝑥 ∈ [0, 6 ].
𝜋
⸫ 𝑓(𝑥) is increasing on [0, 6 ].
(b) 𝑓(𝑥) is decreasing
⇒ 𝑓ʹ(𝑥) ≤ 0
⇒ 3 cos 3𝑥 ≤ 0 ⇒ cos 3𝑥 ≤ 0

117
𝜋 3𝜋
⇒ 2 ≤ 3𝑥 ≤ 2
𝜋 𝜋
⇒6≤𝑥≤ 2
𝜋 𝜋
⇒𝑥 ∈ [ 6 , 2 ].

7 Show that 𝑓(𝑥) = 2𝑥 + 𝑐𝑜𝑡 −1 𝑥 + 𝑙𝑜𝑔(√1 + 𝑥 2 − 𝑥) is increasing in R.

Solution: We have,
𝑓(𝑥) = 2𝑥 + 𝑐𝑜𝑡 −1 𝑥 + 𝑙𝑜𝑔(√1 + 𝑥 2 − 𝑥)
−1 1 1
⇒ 𝑓ʹ(𝑥) = 2 + (1+𝑥 2 ) + √1+𝑥 2 ( 2
.2𝑥 − 1)
−𝑥 2√1+𝑥
1 1 𝑥−√1+𝑥2
= 2 - 1+𝑥 2 + √1+𝑥 2 . √1+𝑥 2
−𝑥
1 1
= 2 - 1+𝑥 2 - √1+𝑥 2
2+2𝑥 2 −1−√1+𝑥 2 1+2𝑥 2 −√1+𝑥 2
= =
1+𝑥 2 1+𝑥 2
For increasing function, 𝑓ʹ(𝑥) ≥ 0
1+2𝑥 2 −√1+𝑥 2
⇒ ≥0
1+𝑥 2
2
⇒ 1 + 2𝑥 ≥ √1 + 𝑥 2
⇒ (1 + 2𝑥 2 )2 ≥ 1 + 𝑥 2
⇒ 1 + 4𝑥 4 + 4𝑥 2 ≥ 1 + 𝑥 2
⇒ 4𝑥 4 + 3𝑥 2 ≥ 0
⇒ 𝑥 2 (4𝑥 2 + 3) ≥ 0
which is true for any real value of x.
Hence, 𝑓(𝑥) is increasing in R.

𝜋
8 Prove that 𝑓(𝑥) = sin x + √3 cos 𝑥 has maximum, value at 𝑥 = 6 .
Solution:We have, 𝑓(𝑥) = sin x + √3 cos 𝑥
⸫ 𝑓ʹ(𝑥) = cos 𝑥 + √3(−𝑠𝑖𝑛𝑥)
= cos 𝑥 − √3𝑠𝑖𝑛𝑥
For 𝑓ʹ(𝑥) = 0, ⇒ cos 𝑥 − √3𝑠𝑖𝑛𝑥=0
1 𝜋
⇒ cos 𝑥 = √3𝑠𝑖𝑛𝑥 ⇒ tan 𝑥 = 3 = tan 6

𝜋
⇒𝑥= 6
Again, differentiating 𝑓ʹ(𝑥), we get
𝑓ʹʹ(𝑥) = −sin 𝑥 - √3 cos 𝑥
𝜋 𝜋 𝜋
𝑓ʹʹ (6 ) = −sin 6 - √3 cos 6
1 √3
= − 2 − √3 . 2
1 3
= − 2 − 2 = −2 < 0
𝜋
Hence 𝑥 = is the point of local maxima.
6

9 Find the local maximum and the local minimum values of the function f(x) =
−3 4 3 45 2
𝑥 −8𝑥 − 𝑥 + 105
4 2
Solution: 𝑓ʹ(𝑥) = −3𝑥 3 − 24𝑥 2 − 45𝑥
= −3𝑥(𝑥 2 + 8𝑥 + 15)
= −3𝑥(𝑥 + 3)(𝑥 + 5)
𝑓ʹʹ(𝑥) = −9𝑥 2 − 48𝑥 − 45
𝑓ʹ(𝑥) = 0 ⇒ −3𝑥(𝑥 + 3)(𝑥 + 5) = 0
⇒ 𝑥 = 0, 𝑥 = −3, 𝑥 = −5

118
𝑓ʹʹ(0) = -45<0
So x=0 is a point of local maxima
. 𝑓ʹʹ(−3)=+18 > 0
So x=-3 is a point of local minima
𝑓ʹʹ(−5)=−30 < 0
So x=-5 is a point of local maxima

10 A telephone company in a town has 500 subscribers on its list and collects
fixed charges of Rs.300 per subscriber per year. The company proposes to
increase the annual subscription and it is believed that for every increase of
Rs.1 per one subscriber will discontinue the service. Find what increase will
bring maximum profit?
Solution: Consider that company increases the annual subscription by Rs. x
So, x subscribers will discontinue the service
⸫ Total revenue of company after the increment is given by
R(x) = (500-x)(300+x)
=15 x 104 + 500𝑥 − 300𝑥 − 𝑥 2
= −𝑥 2 + 200𝑥 + 150000
On differentiating both sides w.r.t x, we get
Rʹ (x) = −2𝑥 + 200
Now, Rʹ (x) = 0
⇒ 2x = 200 ⇒ x=100
⸫ Rʹʹ (x) = -2<0
So, R(x) is maximum when x=100
Hence, the company should increase the subscription fee by Rs.100, so that it
has maximum profit.

EXERCISE
1 Find two positive numbers whose sum is 16 and sum of whose cubes is minimum.
Answer: 8, 8
2 2𝑥
Show that y = log(1+x) - 2+𝑥 , , x> −1 is an increasing function of x throughou its
domain .
3 Show that the function 𝑓(𝑥) = 𝑥 3 − 3𝑥 2 + 3𝑥, 𝑥 ∈ 𝑅 is increasing on R.
4 Find the intervals in which the function 𝑓(𝑥)=2𝑥 3 − 9𝑥 2 + 12𝑥 − 15 is
(i) increasing. (ii) decreasing

Answer: : (−∞, 𝟏) ∪ (𝟐, ∞)


5 The total revenue received from the sale of 𝑥 units of a product is given by
R(x) = 3𝑥 2 + 36𝑥 + 5 in rupees. Find the marginal revenue when x=5, where by
marginal revenue we mean the rate of change of total revenue with respect to the
number of items sold at an instant. At what value of x is R(x) minimum

Answer: 66

5 MARK QUESTIONS

1 Find the intervals in which the function f(x) =3x4 -4x3-12x2+5 is increasing or
decreasing.
Solution:

119
𝑓(𝑥) = 3𝑥 4 − 4𝑥 3 − 12𝑥 2 + 5
𝑓 ′ (𝑥) = 12𝑥 3 − 12𝑥 2 − 24𝑥
= 12𝑥(𝑥 2 − 𝜘 − 2)
=12𝑥(𝑥 − 2)(𝑥 + 1)
𝑓 1 (𝑥) = 0 ⇒ 𝑥 = 0,2, −1

Intervals Sign of f’(x) Nature of f(x)


(-∞,-1) -ve decreasing

(-1,0) +ve increasing


(0,2) -ve decreasing
(2,∞) +ve increasing

Hence f is increasing in (−1,0) ∪ (2, ∞) and decreasing in (−∞, −1) ∪ (0,2).

2 Show that the surface area of a closed cuboid with a square base and given volume
is minimum, when it is a cube.

Solution: Let x be side of the square base and y be the height of the cuboid.
Volume (V)= x.x.y=x2y …….. (i)
𝑉
y=𝑥 2
Surface area (S)=2(x.x+x.y+x.y)
𝑣
=2𝑥 2 + 4𝜘𝑦 = 2𝑥 2 + 4𝑥 𝑥 2
4𝑣 𝑑𝑠 4𝑣
S=2𝑥 2 + 𝑥 ⇒ 𝑑𝑥 = 4𝑥 − 𝑥 2
For minimum surface area,
𝑑𝑠 4𝑣
= 0 ⇒ 4𝑥 − 2 = 0 ⇒ 𝑥 3 = 𝑣
𝑑𝑥 𝑥
3
x= √𝑣
𝑑2𝑠 8𝑣
= 4 +
𝑑𝑥 2 𝑥3
𝑑2𝑆 8𝑣
= 4 + >0
𝑑𝑥 2 𝑣
3
For x= √𝑣, surface area is minimum
x3 = V
x3 = x2y [from (i)]
x=y cuboid is a cube.

3 Prove that the volume of the largest cone that can be inscribed in a sphere of
8
radius R is 27 of the volume of the sphere.
Solution: Let a cone of base radius x and height y be inscribed in a sphere of
radius R.
R2=(y-R)2 +x2
x2 = 2Ry-y2 [in right triangle OAB]…(i)
1
Volume of the cone, V=3 π x2y

120
1
𝜋𝑦(2𝑅𝑦 − 𝑦 2 )
3
1
= 𝜋(2𝑅𝑦 2 − 𝑦 3 ) [from (i)]……(ii)
3
𝑑𝑉 1
= 𝜋(4𝑅𝑦 − 3𝑦 2 )
𝑑𝑦 3
For maximum volume,
𝑑𝑉
=0
𝑑𝑦
⇒4Ry=3y2
4𝑅
⇒y = 3
𝑑2𝑣 𝜋
= (4𝑅 − 6𝑦)
𝑑𝑦 2 3
𝑑2𝑣 4𝑅
< 0, for y =
𝑑𝑦 2 3
1 4𝑅 2 4𝑅 3
Vmax. = π[2𝑅 ( ) −( ) ]
3 3 3
3 3
1 32𝑅 64𝑅
= 𝜋[ − ]
3 9 27
32𝜋𝑅 3
= 𝑐𝑚3
81
8 4
= ( 𝛱𝑅 3 )
27 3
8
(𝑉𝑜𝑙𝑢𝑚𝑒 𝑜𝑓 𝑡ℎ𝑒 𝑠𝑝ℎ𝑒𝑟𝑒)
27
4 Show that the height of the cylinder of maximum volume that can be inscribed in
2𝑅
a sphere of radius R is . Also, find the maximum volume.
√3
Solution:
Let x be radius of base and y height of a cylinder which is inscribed in a sphere of
radius R.
4𝑥 2 + 𝑦 2 = 4𝑅 2 …….(i)
Volume of cylinder
4𝑅 2 −𝑦 2
V=𝜋𝑥 2 𝑦 = 𝜋𝑦 ( )
4
𝜋
= 4 (4𝑅 2 𝑦 − 𝑦 3 )
[from(i)] ……(ii)
𝑑𝑣 𝜋
= (4𝑅 2 − 3𝑦 2 )
𝑑𝑦 4
𝑑𝑉
For maximum volume, 𝑑𝑦 = 0
2𝑅
Y=
√3
𝑑 2 𝑣 −3𝜋𝑦
=
𝑑𝑦 2 2
𝑑2 𝑣 2𝑅
< 0 for y=
𝑑𝑦 2 √3
Now substituting the value of y in equation (ii), we get maximum volume
4𝜋𝑅 3
= 3√3 cubic units.
5 Two men A and B start with velocities v at the same time from the junction of two
roads inclined at 450 to each other. If they travel by different roads, then find the
rate at which they are being separated.

121
Solution: Let two men start from the point C with velocity v each at the same
time.
Also, ∠𝐵𝐶𝐴 = 450
Since, A and B are moving with same velocity v, so they will cover same distance
in same time.
Therefore, 𝛥𝐴𝐵𝐶 is an isosceles triangle with
AC=BC.
Now, draw CD⊥AB.
Let at any instant t, the distance between them is AB.
Let AC=BC=x and AB=y
In ∆ACD and ∆DCB,
∠CAD=∠ CBD
∠CDA=∠ CDB=900
∠ACD=∠ DCB
1
∠ACD= 2 𝑋∠ ACB
1
∠ ACD= 2 𝑋450
𝜋
∠ ACD= 8
𝜋 𝐴𝐷
Sin 8 = 𝐴𝐶
𝜋 𝑌/2
Sin 8 = 𝑥
𝑦
= x sin 𝜋8
2
𝜋
Y=2x.sin8
Now, differentiating both sides w.r.t. t ,we get
𝑑𝑦
𝑑𝑡
= 2.sin 𝜋8.𝑑𝑥
𝑑𝑡

𝜋 𝑑𝑥
=2.sin .v …………. [v= ]
8 𝑑𝑡
√2−√2 𝜋 √2−√2
=2v. …………. [sin 8 = ]
2 2
=√2 − √2 v unit/s
Which is the rate at which A and B are being separated.

EXERCISE

1 𝑥4
Find the intervals in which the function f(x)= 4 − 𝑥 3 − 5𝑥 2 + 24𝑥 + 12 is
(i) strictly increasing (ii) strictly decreasing.
Answer: (i) (−𝟑, 𝟐) ∪ (𝟒, ∞). (ii) (−∞, −𝟑) ∪ (𝟐, 𝟒).
2 The length of the sides of an isosceles triangle are 9+x2 , 9+x2 and 18-2x2 units.
Calculate the area of the triangle in terms of x and find the value of x which makes
the area maximum.
Answer: 𝑥 = √3
3 A rectangle is inscribed in a semicircle of radius r with one of its sides on the
diameter of the semicircle. Find the dimensions of the rectangle, so that its area is
maximum. Also find maximum area.
𝑟
Answer: √2r ,
√2
4 If the sum of a side and the hypotenuse of a right- angled triangle be given, show
that the area of the triangle will be maximum if the angle between the given side
and the hypotenuse be 600.

122
5 Show that the semi-vertical angle of a right circular cone of given total surface
1
area and maximum volume is 𝑠𝑖𝑛−1 3.
6 Show that the surface area of a closed cuboid with a square base and given volume
is minimum, when it is a cube.
7 If the length of three sides of a trapezium other than the base are equal to 10 cm ,
then find the maximum area of the trapezium .
8 Find the maximum area of an isosceles triangle inscribed in the ellipse
𝑥2 𝑦2
+ = 1 with its vertex at one end of the major axis.
16 9
Answer: 9√3 sq.units

CASE STUDY QUESTIONS

Read the following and answer the questions given below.

1 Dr. Ritham residing in Delhi went to see an apartment of 3BHK in Noida. The
window
of the house was in the form of a rectangle surmounted by a semicircular
opening
having a perimeter of the window 10 m as shown in figure.

(i). If x and y represent the length and breadth of the rectangular region, then the
relation between the variable is
𝑥 𝑥
a) x + y + 2 = 10 b) x + 2y + 2 = 10
𝑥
c) x + 2y + π 2 = 10 d) 2x+ 2y = 10
(ii) The area of the window (A) expressed as a function of x is
𝜋𝑥 3 𝑥2 −𝑥 2 𝜋𝑥 2
a) 𝐴 = 𝑥 − − b) 𝐴 = 5𝑥 − −
8 2 2 8
𝑥2 3𝑥 2 𝑥2 𝜋𝑥 2
c) A= 5 𝑥 − 2 − 8 d) A= 5x + 2 + 8
(iii) Dr. Ritam is interested in maximizing the area of the whole window. For this
to
Happen the value of x should be
20 20 20 20
(a) 𝜋 b) 4−𝜋 c ) 2+𝜋 d) 4+𝜋
Solution:
(i) Since perimeter of window = x + y + y + perimeter of semicircle
1 𝑥 𝑥
= x+ 2y + 2 x 2 π x 2 [𝐻𝑒𝑟𝑒 𝑟𝑎𝑑𝑖𝑢𝑠 𝑜𝑓𝑠𝑒𝑚𝑖𝑐𝑖𝑟𝑐𝑙𝑒 𝑖𝑠 2]
𝜋𝑥
= x + 2y + 2

123
Option ( c ) is correct.

1 𝑥 2
(ii) A = x x y + 2 π (2)

𝜋𝑥 2 𝑥 𝜋𝑥 𝜋𝑥 2
= 𝑥𝑦 + = 𝑥 (5 − 2 − )+
8 4 8

𝑥2 𝜋𝑥 2 𝜋𝑥 2 𝑥2 𝜋𝑥 2
= 5𝑥- - + = 5𝑥 - -
2 4 8 2 8

Option ( b ) is correct.

(iii) For maximum value of A


𝑑𝐴
= 0
𝑑𝑥

𝜋𝑥 𝜋𝑥
⇒5 – 𝑥 - =0 ⇒𝑥+ =5
4 4

⇒4𝑥 + π 𝑥 = 20 ⇒ 𝑥(4 + π) = 20
20
⇒ 𝑥 = 4+𝜋

Option (d) is correct

2 Read the following passage and answer the questions given below:
The relation between the height of the plant (‘y” in cm ) with respect to its
exposure
1
to the sunlight is governed by the following equation y = 4x - 2 x 2 , where ‘ x ‘ is
the
number of days exposed to the sunlight for x ≤ 3.

(i) Find the rate of growth of the plant with respect to the number of days
exposed to the sunlight.
(ii) Does the rate of growth of the plant increase or decrease in the first three
days? What will be the height of the plant after 2 days?
Solution:
(i)The rate of growth of the plant with respect to the number of days exposed to
𝑑𝑦
sunlight is given by 𝑑𝑥 = 4 – x.
𝑑𝑦
(ii) Let rate of growth be represented by the function g (x) = 𝑑𝑥 .
𝑑 𝑑𝑦
Now, g ՜՜(x) = 𝑑𝑥 (. 𝑑𝑥 ) = -1 < 0
g(x ) decreases.
So the rate of growth of the plant decreases for the first three days.
1
Height of the plant after 2 days is y = 4 x 2 - 2 (2) 2 = 6 cm.

124
3 A rectangular hall is to be developed for a meeting of farmers in an agriculture
college to aware them for new techniques in cultivation. It is given that the floor
has a fixed
perimeter P as shown below. And if x & y represents the length & breadth of the
rectangular region.

Answer the following question.


(i) The area of the rectangular region ‘A’ expressed as a function of x is
1 1 1
(a) 2 ( P + x2 ) (b) 2 ( P x – 2x2 ) (c) 2 ( P x + 2x2 ) (d) P x
– 2x2
(ii) Principal of agriculture college is interested in maximizing the area of floor
‘A’ . For this to happen the value of x should be
𝑃 2𝑃 𝑃
(a) P (b) 2 (c) 3
(d) 4
Solution:
(i) A = x.y
𝑃−2𝑥
= x. 2
𝑠𝑖𝑛𝑐𝑒, 𝑔𝑖𝑣𝑒𝑛 𝑃𝑒𝑟𝑖𝑚𝑒𝑡𝑒𝑟 = 𝑃
𝑃𝑥−2𝑥 2 2( 𝑥 + 𝑦 ) =
= [ ]
2 𝑃 𝑃−2𝑥
𝑌 = 2 –𝑥 = 2

Option ( b) is correct.
𝑃𝑥−2𝑥 2 𝑑𝐴 𝑃−4𝑥
(ii) A = 2 ⇒ = 2
𝑑𝑥
For maximum or minimum value of x
𝑑𝐴 𝑃−4𝑥
=0 ⇒ =0
𝑑𝑥 2
𝑃
⇒P – 4x =0 ⇒ x = 4

𝑑2 𝐴 𝑃
Also , 𝑑𝑥 2 = −2 < 0 𝑎𝑡 𝑥 = 4
∴ 𝐴 𝑖𝑠 𝑚𝑎𝑥𝑖𝑚𝑢𝑚
Option (d) is correct.

EXERCISE

1 Q1.Read the following and answer the questions given :


On the request of villagers, a construction agency designs a tank with the
help of an architect. Tank consists of rectangular base with rectangular sides,
open at the top so that its depth is 2 m and volume is 8 m 3 as shown below:

125
(i) If x and y represents the length and breadth of its rectangular base, then the
relation between the variables is
𝑥
(a) x + y = 8 (b) x . y = 4 (c) x + y = 4 (d) 𝑦 = 4
(ii) If construction of tank cost ₹70 per sq. meter for the base and ₹45 per square
meter or sides, then making cost ‘ C’ expressed as a function of x is
4 4
(a ) C = 80 + 80 ( 𝑥 + 𝑥 ) (b) C = 280x +280 ( 𝑥 + 𝑥 )
4 𝑥
( c ) C = 280 + 180( 𝑥 + ) (d) C = 70x + 70 (𝑥 + )
𝑥 4
(iii) The owner of a construction agency is interested in minimizing the cost ‘C’ of
whole tank, for this to happen the value of x should be
(a) 4 m (b) 3 m (c) 1 m (d) 2 m
Answer: (𝒊)𝒃 (𝒊𝒊) 𝒄 (𝒊𝒊𝒊) 𝒅

2 Read the following text and answer the following questions on the basis of the
same:
In a residential society comprising of 100 houses, there were 60 children between
the ages of 10 – 15 years. They were inspired by their teachers to start composting
to ensure that biodegradable waste is recycle, For this purpose, instead of each child
doing it for only his/her house, children convinced the Residents welfare association
to do it as a society initiative. For this they identified a square area in the local park.
Local authorities charged amount of ₹ 50 per square meter for space so that there is
no misuse of the space and Resident welfare association takes it seriously.
Association hired a labourer for digging out 250 m3 and he charged ₹400 x (depth)2.
Association will like to have minimum cost.

i).Let side of square plot is x m and its depth is h meters, then cost C for the pit is
50 12500 250 250
(a) ℎ + 400h2 (b) ℎ + 400h2 (c) ℎ + h2 (d) ℎ +400h2
𝑑𝑐
ii).Value of h (in m ) for which 𝑑ℎ = 0 is
(a) 1.5 (b) 2 (c) 2.5 (d) 3
iii). Value of x (in m) for minimum cost is
5
(a)5 (b)10√3 (c)5√5 (d) 10
Answer: (i) b (ii) c (iii) d

126
3 Read the following text and answer the following questions, on the basis of the
same:
P(𝑥) = -5x2 +125x + 37500 is the total profit function of a company, where 𝑥 is
the production of the company.

(i) What will be the production when the profit is maximum?


(a) 37,500 (b) 12.5 (c) -12.5 (d) -37,500
(ii) What will be the maximum profit?
(a) ₹38,28,125 (b) ₹38,28,.25 (c) ₹39,000 (d) None of these
(iii) Check in which interval the profit is strictly increasing.
(a) (12.5, ∞ ) (b) for all real numbers
(c) for all positive real numbers (d) (0,12.5)
Answer: (i) b (ii) b (iii) d

127
CHAPTER: INTEGRALS
SYLLABUS:

Integration as inverse process of differentiation. Integration of a variety of functions by substitution, by


partial fractions and by parts, Evaluation of simple integrals of the following types and problems based
on them.
𝑑𝑥 𝑑𝑥 𝑑𝑥 𝑑𝑥
∫ 𝑥 2 ±𝑎2 , ∫ 𝑎2 −𝑥 2 , ∫ √ , ∫ √𝑎2 , ∫ √𝑥 2 ± 𝑎2 𝑑𝑥 , ∫ √𝑎2 − 𝑥 2 𝑑𝑥,
𝑥 2 ±𝑎2 −𝑥 2
𝑝𝑥+𝑞 𝑝𝑥+𝑞
∫ √𝑎𝑥 2 + 𝑏𝑥 + 𝑐 𝑑𝑥 ∫ 𝑑𝑥, ∫ 𝑑𝑥
𝑎𝑥 2 +𝑏𝑥+𝑐 √𝑎𝑥 2 +𝑏𝑥+𝑐

Fundamental Theorem of Calculus (without proof). Basic properties of definite integrals and
evaluation of definite integrals.

Formulae and Definitions:

Indefinite Integrals

1. ∫ 1𝑑𝑥 = 𝑥 + 𝑐
2. ∫ 𝑥 𝑑𝑥 = 𝑥 2 + 𝑐
3. ∫ 𝑠𝑖𝑛𝑥 𝑑𝑥 = −𝑐𝑜𝑠𝑥 + 𝑐
4. ∫ 𝑐𝑜𝑠𝑥𝑑𝑥 = 𝑠𝑖𝑛𝑥 + 𝑐
5. ∫ 𝑡𝑎𝑛𝑥 𝑑𝑥 = log 𝑠𝑒𝑐𝑥 + 𝑐
6. ∫ 𝑐𝑜𝑠𝑒𝑐𝑥 𝑑𝑥 = log|(𝑐𝑜𝑠𝑒𝑐𝑥 − 𝑐𝑜𝑡𝑥)| + 𝑐
7. ∫ 𝑠𝑒𝑐𝑥 𝑑𝑥 = log|𝑠𝑒𝑐𝑥 + 𝑡𝑎𝑛𝑥| + 𝑐
8. ∫ 𝑐𝑜𝑡𝑥𝑑𝑥 = 𝑙𝑜𝑔|𝑠𝑖𝑛𝑥| + 𝑐
9. ∫ 𝑠𝑒𝑐 2 𝑥𝑑𝑥 = 𝑡𝑎𝑛𝑥 + 𝑐
10. ∫ 𝑐𝑜𝑠𝑒𝑐 2 𝑥𝑑𝑥 = −𝑐𝑜𝑡𝑥 + 𝑐
11. ∫ 𝑠𝑒𝑐𝑥. 𝑡𝑎𝑛𝑥𝑑𝑥 = 𝑠𝑒𝑐𝑥 + 𝑐
12. ∫ 𝑐𝑜𝑠𝑒𝑐𝑥. 𝑐𝑜𝑡𝑥𝑑𝑥 = −𝑐𝑜𝑠𝑒𝑐𝑥 + 𝑐
13. ∫ 𝑒 𝑥 𝑑𝑥 = 𝑒 𝑥 + 𝑐
𝑑𝑥
14. ∫ = log 𝑥 + 𝑐
𝑥

𝑎𝑥
15. ∫ 𝑎 𝑥 𝑑𝑥 = log 𝑎 + 𝑐

1 1 𝑥−𝑎
16. ∫ 𝑥 2 −𝑎2 𝑑𝑥 = 2𝑎 log |𝑥+𝑎| + 𝑐
1 1 𝑎+𝑥
17. ∫ 𝑎2 −𝑥 2 𝑑𝑥 = 2𝑎 log |𝑎−𝑥| + 𝑐
1 1 𝑥
18. ∫ 𝑥 2 +𝑎2 𝑑𝑥 = 𝑎 tan−1 (𝑎) + 𝑐

128
1
19. ∫ √𝑥 2 𝑑𝑥 = log|𝑥 + √𝑥 2 − 𝑎2 | + 𝑐
−𝑎2
1
20. ∫ √𝑥 2 𝑑𝑥 = log|𝑥 + √𝑥 2 + 𝑎2 | + 𝑐
+𝑎2

1 𝑥
21. ∫ 𝑑𝑥 = sin−1 + 𝑐
√𝑎2 −𝑥 2 𝑎

𝑥 𝑎2
22. ∫ √𝑥 2 − 𝑎2 𝑑𝑥 = √𝑥 2 − 𝑎2 − log|𝑥 + √𝑥2 − 𝑎2 | + 𝑐
2 2
𝑥 𝑎2
23. ∫ √𝑥 2 + 𝑎2 𝑑𝑥 = 2 √𝑥2 + 𝑎2 + 2 log|𝑥 + √𝑥 2 + 𝑎2 | + 𝑐

𝑥 𝑎2 𝑥
24. ∫ √𝑎2 − 𝑥 2 𝑑𝑥 = √𝑎2 − 𝑥 2 + sin−1 + 𝑐
2 2 𝑎

25. ∫ 𝑒 𝑥 (𝑓(𝑥) + 𝑓 𝑙 (𝑥))𝑑𝑥 = 𝑒 𝑥 𝑓(𝑥) + 𝑐

26. ∫ 𝑢. 𝑣𝑑𝑥 = 𝑢 ∫ 𝑣 𝑑𝑥 − ∫ 𝑢𝑙 [∫ 𝑣 𝑑𝑥]𝑑𝑥


Partial fractions
P(x)
• The rational function is said to be proper if the degree of P(x) is less than
Q(x)
the degree of Q(x)
• Partial fractions can be used only if the integrand is proper rational function

Definite Integrals
S.No Form of rational function Form of Partial fraction

1 1 𝐴 𝐵
+
(𝑥 − 𝑎)(𝑥 − 𝑏) (𝑥 − 𝑎) (𝑥 − 𝑏)
2 𝑝𝑥 + 𝑞 𝐴 𝐵
+
(𝑥 − 𝑎)(𝑥 − 𝑏) (𝑥 − 𝑎) (𝑥 − 𝑏)
2 𝐴 𝐵 𝐶
3 𝑝𝑥 + 𝑞𝑥 + 𝑐
+ +
(𝑥 − 𝑎)(𝑥 − 𝑏)(𝑥 − 𝑐) (𝑥 − 𝑎) (𝑥 − 𝑏) (𝑥 − 𝑐)
4 1 𝐴 𝐵 𝐶
+ +
(𝑥 − 𝑎)(𝑥 − 𝑏)(𝑥 − 𝑐) (𝑥 − 𝑎) (𝑥 − 𝑏) (𝑥 − 𝑐)
5 1 𝐴 𝐵 𝐶
2
+ 2
+
(𝑥 − 𝑎) (𝑥 − 𝑏) (𝑥 − 𝑎) (𝑥 − 𝑎) (𝑥 − 𝑏)
6 𝑝𝑥 + 𝑞 𝐴 𝐵 𝐶
2 + +
(𝑥 − 𝑎) (𝑥 − 𝑏) (𝑥 − 𝑎) (𝑥 − 𝑎)2 (𝑥 − 𝑏)
2 𝐴 𝐵𝑥 + 𝐶
𝑝𝑥 + 𝑞𝑥 + 𝑟
+ 2
8 (𝑥 − 𝑎)(𝑥 2 + 𝑏𝑥 + 𝑐) (𝑥 − 𝑎) 𝑥 + 𝑏𝑥 + 𝑐
2
where 𝑥 + 𝑏𝑥 + 𝑐 cannot be factorized further

Properties of Definite Integrals


𝑎
1. ∫𝑎 𝑓(𝑥)𝑑𝑥=0
𝑏 𝑏
2. ∫𝑎 𝑓(𝑥)𝑑𝑥 =∫𝑎 𝑓(𝑡)𝑑𝑡
𝑏 𝑎
3. ∫𝑎 𝑓(𝑥)𝑑𝑥 =− ∫𝑏 𝑓(𝑥)𝑑𝑥

129
𝑏 𝑐 𝑏
4. ∫𝑎 𝑓(𝑥)𝑑𝑥=∫𝑎 𝑓(𝑥)𝑑𝑥+∫𝑐 𝑓(𝑥)𝑑𝑥, where a<c<b
𝑏 𝑏
5. ∫𝑎 𝑓(𝑥)𝑑𝑥 =∫𝑎 𝑓(𝑎 + 𝑏 − 𝑥)𝑑𝑥
𝑎 𝑎
6. ∫0 𝑓(𝑥)𝑑𝑥 =∫0 𝑓(𝑎 − 𝑥)𝑑𝑥
2𝑎 𝑎 𝑎
7. ∫0 𝑓(𝑥)𝑑𝑥=∫0 𝑓(𝑥)𝑑𝑥+∫0 𝑓(2𝑎 − 𝑥)𝑑𝑥
𝑎
2𝑎 2∫ 𝑓(𝑥)𝑑𝑥 𝑖𝑓 𝑓(2𝑎 − 𝑥) = 𝑓(𝑥)
8. ∫0 𝑓(𝑥)𝑑𝑥x={ 0
0 𝑖𝑓 𝑓(2𝑎 − 𝑥) = −𝑓(𝑥)
𝑎
𝑎 2 ∫0 𝑓(𝑥)𝑑𝑥 𝑖𝑓 𝑓(−𝑥) = 𝑓(𝑥)
9. ∫−𝑎 𝑓(𝑥)𝑑𝑥 ={
0 𝑖𝑓 𝑓(−𝑥) = −𝑓(𝑥)
MULTIPLE CHOICE QUESTIONS
Q.NO QUESTIONS AND SOLUTION S
1. 1
Evaluate ∫ dx ?
𝑥− √𝑥
a) 2 log √𝑥 + C b) log (√𝑥 - 1 ) + C c) 2 log (√𝑥 - 1 ) + C d ) None of the
above
1
Ans. I = ∫ 𝑥( 𝑥 −1) dx
√ √
Put √𝑥 − 1 = t then
I = 2 logt
= 2 log (√𝑥 − 1) + C (c)
2. 𝑠𝑒𝑐 2 𝑥
Evaluate ∫ dx ?
√𝑡𝑎𝑛2 𝑥 +4
a) log|√𝑡𝑎𝑛2 𝑥 + 4 | + 𝐶 b) log |tanx + √𝑡𝑎𝑛2 𝑥 + 4 | + 𝐶
1
c) 2 log |𝑡𝑎𝑛 𝑥 | + 𝐶 d) tanx + C
Ans. Sub tan x = t then
𝑑𝑡
I = ∫ √𝑡 2 2
+2
=log |tanx + √𝑡𝑎𝑛2 𝑥 + 4 | + 𝐶 (b)
3. Evaluate ∫ 𝑐𝑜𝑠 3 𝑥 . 𝑒 log 𝑠𝑖𝑛𝑥 dx ?
1 1 1
𝑎) 3 sin3 x + C b) − 2 cos4 x + C c) − 4 cos4 x + C
1
d) − 3 sin3 x + C
Ans. Here elog sinx = sinx then
I = ∫ 𝑐𝑜𝑠 3 𝑥 . 𝑠𝑖𝑛𝑥 𝑑𝑥
Let cosx = t
1
Ans : − 4 cos4 x + C (c)
𝜋
4 𝜋
∫0 𝑠𝑒𝑐 2 (𝑥 − 6 ) 𝑑𝑥 is equal to :
6

1 1
(a) (b) − (c) √3 (d) −√3
√3 √3

𝜋 𝜋
2 𝜋 𝜋 6 𝜋 𝜋 1
Ans: ∫0 𝑠𝑒𝑐 (𝑥 − 6 ) 𝑑𝑥 = 𝑡𝑎𝑛 (𝑥 − 6 )] = tan 0 − 𝑡𝑎𝑛 (− 6 )= 𝑡𝑎𝑛 ( 6 ) =
6
0 √3
Option: a

130
5. 𝑑𝑥
Evaluate ∫
𝑒𝑥 − 1

a) log |ex – 1| +C b) log|1 – 𝑒 −𝑥 |+C


c) log|1 – 𝑒 𝑥 |+C d) log| 𝑒 −𝑥 − 1|+C
Ans. Dividing the numerator and denominator with ex
𝑒 −𝑥
I = ∫ 1− 𝑒 −𝑥 dx = log|1 – 𝑒 −𝑥 |+C (b)

6. 1 2𝑥
Evaluate ∫0 𝑠𝑖𝑛−1 ( 1 +𝑥 2 ) dx
𝜋
𝑎) 2 - log2 b) log 2 c) 0 d) 1
𝜋
Ans. Let x = tant then t from 0 to 4 .and apply sin2x formula then 𝐼=
𝜋
𝜋
∫0 2𝑡. 𝑠𝑒𝑐 2 𝑡 𝑑𝑡= 2 - log2
4 (a)

7. 𝑑
If ( f(x) ) = logx , then f(x) equals :
𝑑𝑥
1 1
𝑎) 𝑥 +C b) x ( logx + x )+C c) x ( log x -1 ) +C d) - 𝑥 +C
Ans : x ( log x -1 )+C (c)

8. 1 |𝑥−2|
Evaluate ∫−1 𝑥−2 dx ?
a)1 b) -1 c) 2 d) -2
Ans. Apply |𝑥 − 2 | property to get the -2 (d)

9. Evaluate ∫ 𝑒 𝑥 ( 𝑐𝑜𝑠𝑥 − 𝑠𝑖𝑛𝑥) dx ?


a) – ex sinx +C b) ex sinx +C
x
c) e cosx+C d) - ex cosx+C
Ans. Apply ∫ ex [ f(x) + f 1 (x) ] dx = ex f(x) formula I= ex cosx
(c)

10. 5
Find ∫−5 𝑓(𝑥) 𝑑𝑥 where f(x) = |𝑥 − 2|
a) 25 b) 29 c) 15 d) 20
5 2
Q. Apply modulus function definition to get ∫−5 |𝑥 − 2| dx = ∫−5 |𝑥 − 2| dx +
5
∫2 |𝑥 − 2| dx=29 (b)

CHAPTER VIDEO LINK FOR MCQs SCAN QR CODE FOR


VIDEO

INTEGRALS https://youtu.be/VBMyRVKOvck

131
EXERCISE
𝑐𝑜𝑠𝑥
1. Q. Evaluate ∫ +𝑠𝑖𝑛𝑥 dx ?
√1
𝑎) 1 + 𝑠𝑖𝑛𝑥 b) 1 − 𝑠𝑖𝑛𝑥
c) 2 √1 + 𝑠𝑖𝑛𝑥 d) 2 √1 − 𝑠𝑖𝑛𝑥
Ans(c)
2. 𝑥2
Q. Evaluate ∫ 1+ 𝑥 3 dx ?
1
a) log( 1 + x3 ) b) 3 log( 1 – x3 )
1
c) log ( 1 – x3 ) d) 3 log( 1 + x3 )
Ans(d)
3. 1 1
Q.Evaluate ∫ [ − ] dx ?
log 𝑥 (𝑙𝑜𝑔𝑥)2
𝑙𝑜𝑔𝑥 𝑥 𝑥−1 log 𝑥
𝑎) b) log 𝑥 c) 𝑙𝑜𝑔𝑥 d)
𝑥 𝑥−1
Ans(b)
𝜋
4.
Q. Evaluate ∫04 2 𝑡𝑎𝑛3 𝑥 dx ?
a)1 b) log 2 c) 1 – log2 d) 1 + log2
Ans(c)
5. 1
Q. Evaluate ∫ 1−𝑠𝑖𝑛𝑥 dx ?
a) secx – tanx b) tanx + secx c) tanx d) Secx
Ans(b)
ASSERTIONS AND REASONING QUESTIONS

In the following questions, a statement of Assertion (A) is followed by a statement of


Reason (R). Choose the correct answer out of the following choices.
A) Both Assertion (A) and Reason (R) true and Reason (R) is the correct explanation of
Assertion (A).
B) Both Assertion (A) and Reason (R) true and Reason (R) is not the correct explanation of
Assertion (A)
C) Assertion (A) is true but Reason (R) is false

D) Assertion (A) is false but Reason (R) is true

1 Assertion: ∫ sin x dx = −cos x + C.


Reason: sin x is an odd function and the integral of an odd function is -f(x) + C.

Solution: Assertion is true but reason is false. The integral of sin x is -cos x + C,
but sin x is not an odd function. Sin x is an even function.
Correct option: (C) Assertion is true but reason is false
𝜋
2
Assertion: ∫ 2𝜋 sin5 𝑥 𝑑𝑥 = 0

2
𝑎
Reason: If f(x) is odd function ∫−𝑎 𝑓(𝑥)𝑑𝑥 = 0

Answer: A) both assertion and reasoning are correct and reason is the correct
explanation
𝜋
3
Assertion: ∫ 2𝜋 𝑐𝑜𝑠 𝑥 𝑑𝑥 = 0

2

132
𝑎
Reason: If f(x) is odd function ∫−𝑎 𝑓(𝑥)𝑑𝑥 = 0

Answer: D) assertion is False and reasoning is correct


4 𝒙𝟑 𝟑𝒙𝟐
Assertion: ∫ 𝒙𝟐 − 𝟑𝒙 + 𝟐 𝒅𝒙 = − + 𝟐𝒙 + 𝒄
𝟑 𝟐
𝑎
Reason: The integral of a polynomial function 𝑎𝑥 𝑛 is 𝑥 𝑛+1 + 𝑐 , where 'C'
𝑛+1
is the constant of integration.

Answer: A) both assertion and reasoning are correct and reason is the correct
explanation
𝟐𝒙
5 Assertion: ∫ 𝒙𝟐+𝟏 𝒅𝒙 = 𝒍𝒐𝒈|𝒙𝟐 + 𝟏| + 𝒄
𝒇′ (𝒙)
Reason: ∫ 𝒅𝒙 = 𝒍𝒐𝒈|𝒇(𝒙)| + 𝒄
𝒇(𝒙)

Answer: A) both assertion and reasoning are correct and reason is the correct
explanation

6 𝒙 𝒔𝒊𝒏 𝟐𝒙
Assertion: ∫ 𝐬𝐢𝐧𝟐 𝒙 𝒅𝒙 = 𝟐 − 𝟒 +𝒄
Reason: 𝟏 − 𝐜𝐨𝐬 𝟐𝒙 = 𝟐 𝐬𝐢𝐧𝟐 𝒙

Answer: A) both assertion and reasoning are correct and reason is the correct
explanation
7 Assertion: ∫ 𝒆𝒙 (𝒄𝒐𝒔 𝒙 + 𝒔𝒊𝒏 𝒙 ) 𝒅𝒙 = 𝒆𝒙 𝒄𝒐𝒔 𝒙 + 𝒄
Reason: ∫ 𝒆𝒙 (𝒇(𝒙) + 𝒇′ (𝒙) ) 𝒅𝒙 = 𝒆𝒙 𝒇(𝒙) + 𝒄

Answer: D) assertion is False and reasoning is correct


3𝜋
8
Assertion: ∫𝜋4 sec 2 𝑥 𝑑𝑥 = −2
4
𝑏
Reason: If f(x) ≥ 0 on [a, b] then ∫𝑎 𝑓(𝑥) 𝑑𝑥 ≥ 𝟎

𝜋 𝜋 3𝜋
Solution: 𝑠𝑒𝑐 𝑥 is not defined at x = 2 in [4 , 4 ]
Answer: D) assertion is False and reasoning is correct

9 𝟏
Assertion: ∫ 𝒍𝒐𝒈(𝒍𝒐𝒈 𝒙) + 𝒍𝒐𝒈𝒙 𝒅𝒙 = 𝒙𝒍𝒐𝒈(𝒍𝒐𝒈𝒙) + 𝒄
Reason: ∫ 𝒆𝒙 (𝒇(𝒙) + 𝒇′ (𝒙) ) 𝒅𝒙 = 𝒆𝒙 𝒇(𝒙) + 𝒄

Answer: B) Both Assertion (A) and Reason (R) true and Reason (R) is not the
correct explanation of Assertion (A)

𝜋
10 Assertion: ∫(𝒔𝒊𝒏−𝟏 𝒙 + 𝒄𝒐𝒔−𝟏 𝒙)𝒅𝒙 = 2 𝒙 + 𝒄
𝜋
Reason: 𝒔𝒊𝒏−𝟏 𝒙 + 𝒄𝒐𝒔−𝟏 𝒙 = 2

Answer: A) both assertion and reasoning are correct and reason is the correct
explanation

133
2 MARK QUESTIONS

1. Evaluate ∫ 𝑥 . 𝑡𝑎𝑛−1 𝑥 . 𝑑𝑥 ?
Solution: ∫ 𝑡𝑎𝑛−1 𝑥. 𝑥 . 𝑑𝑥
Apply ∫ 𝑢. 𝑣 𝑑𝑥 formula
𝑥2 1 𝑥2
tan-1x. -∫ 2
. dx
2 1+ 𝑥 2
𝑥2 𝑥 𝑡𝑎𝑛−1 𝑥
= . tan-1x - + +C
2 2 2
2. Evaluate ∫ 𝑒 𝑥 ( 𝑡𝑎𝑛𝑥 + log 𝑠𝑒𝑐𝑥 )𝑑𝑥 ?
Solution: ∫ 𝑒 𝑥 ( log 𝑠𝑒𝑐𝑥 + 𝑡𝑎𝑛𝑥 )𝑑𝑥
Apply ∫ 𝑒 𝑥 ( 𝑓(𝑥) + 𝑓 1 (𝑥))𝑑𝑥 formula to get
𝑒 𝑥 𝑙𝑜𝑔 𝑆𝑒𝑐𝑥 + 𝐶

3. Evaluate ∫ √4 − 9𝑥 2 dx?
Solution: ∫ √22 − (3𝑥)2 dx
Apply ∫ √𝑎 − (𝑥)2 dx formula to get
𝑥 2 - 2sin-1 ( 3𝑥 )+C
. √4 − 9𝑥
2 3 2
4. 4
Evaluate ∫0 |𝑥 − 1| dx?
1 4
Solution: ∫0 (1 − 𝑥) 𝑑𝑥 + ∫1 (𝑥 − 1) 𝑑𝑥 =5

5. 2 + 𝑆𝑖𝑛2𝑥
Evaluate ∫ 𝑒 𝑥 . 𝑑𝑥 ?
1 + 𝐶𝑜𝑠2𝑥
2 + 2𝑆𝑖𝑛𝑥.𝑐𝑜𝑠𝑥 𝑥
Solution: ∫ 𝑒 . 𝑑𝑥
2 𝐶𝑜𝑠 2 𝑥
∫( 𝑆𝑒𝑐 2 𝑥 + 𝑇𝑎𝑛𝑥 ) 𝑒 𝑥 . 𝑑𝑥
Apply ∫ 𝑒 𝑥 ( 𝑓(𝑥) + 𝑓 1 (𝑥))𝑑𝑥 formula to get 𝑒 𝑥 . 𝑇𝑎𝑛𝑥 + C

6. 𝑆𝑒𝑐 2 (𝑙𝑜𝑔𝑥)
Evaluate ∫ dx?
𝑥
Solution: Substitute 𝑙𝑜𝑔 𝑥 = 𝑡 then
∫ 𝑆𝑒𝑐 2 𝑡. 𝑑𝑡 = 𝑡𝑎𝑛 (𝑙𝑜𝑔 𝑥 ) + 𝐶
7. 𝑑𝑥
Evaluate ∫ ?
𝑥 2 −4𝑥+8
1 1 𝑥−2
Solution: ∫ dx = 𝑡𝑎𝑛−1 ( )+C
( 𝑥−2 )2 + 22 2 2

𝜋
8.
Evaluate ∫02 𝑠𝑖𝑛2𝑥 . 𝑙𝑜𝑔𝑡𝑎𝑛𝑥 𝑑𝑥 ?
𝜋
𝜋 𝜋
Solution: I = ∫02 𝑠𝑖𝑛 2 ( 2 − 𝑥 ) log 𝑡𝑎𝑛 ( 2 − 𝑥 ) 𝑑𝑥
𝜋
= - ∫02 𝑠𝑖𝑛2𝑥 . 𝑙𝑜𝑔𝑡𝑎𝑛𝑥 𝑑𝑥 =>2I=0 =>I=0
9. 1 + 𝑐𝑜𝑡𝑥
Evaluate ∫ 𝑑𝑥 ?
𝑥 +log 𝑠𝑖𝑛𝑥
Solution: Substitute 𝑥 + 𝑙𝑜𝑔 𝑠𝑖𝑛𝑥 = 𝑡
=> log ( x + log sinx ) + C
𝑥
10. If f(x) = ∫0 𝑡 . 𝑠𝑖𝑛𝑡 . 𝑑𝑡 then find 𝑓 ′ (𝑥) ?
𝑥
Solution: f(x) = ∫0 𝑡 . 𝑠𝑖𝑛𝑡 . 𝑑𝑡
Apply ∫ 𝑢. 𝑣 𝑑𝑥 formula then
f(x) = [ −t . cost + sin t ]0𝑥

134
𝑓(𝑥) = −𝑥 𝑐𝑜𝑠𝑥 + 𝑆𝑖𝑛𝑥
𝑓 ′ (𝑥) = x. sinx

EXERCISE

1. 2 1 1
Evaluate ∫1 𝑒 𝑥 [ − ] 𝑑𝑥 ?
𝑥 𝑥2

2. Evaluate ∫ 𝑠𝑖𝑛𝑥 . √1 + 𝑐𝑜𝑠2𝑥 dx?

3. Evaluate ∫ 𝑒 𝑎𝑥 𝑠𝑖𝑛𝑏𝑥. 𝑑𝑥 ?

4. 𝑑𝑥
Evaluate ∫ ?
√15−8𝑥 2

5. 1
Evaluate ∫ dx
( 1+ 𝑥 )( 2 + 𝑥 )

3 MARK QUESTIONS

1. 𝑥
Evaluate ∫ dx?
( 𝑥+1)( 𝑥+2 )
𝑥 𝐴 𝐵
Solution:
( 𝑥+1)( 𝑥+2 )
= +
𝑥+1 𝑥+ 2
𝐴 = −1, 𝐵 = 2
( 𝑥+2)2
Ans : log ( )+C
( 𝑥+1 )

2. 𝑐𝑜𝑠 𝑥
Evaluate ∫ dx?
( 1− 𝑠𝑖𝑛𝑥 )( 2 −𝑠𝑖𝑛𝑥 )
Solution: Let cos x = t then
𝐶𝑜𝑠 𝑥 𝑑𝑡
∫ ( 1− 𝑠𝑖𝑛𝑥 )( 2 −𝑠𝑖𝑛𝑥 ) dx = ∫ ( 1−𝑡 )( 2−𝑡 )
Apply partial fraction method,
A = 1, B = -1
2 − 𝑠𝑖𝑛𝑥
I= log ( 1 − 𝑠𝑖𝑛𝑥 ) + C
3. 𝑥 3 + 𝑥+1
Evaluate ∫ dx?
𝑥2− 1
2
𝑥 + 𝑥+1 2𝑥+1
Solution: =𝑥+ ( Convert it into proper rational function)
𝑥2− 1 𝑥2− 1
2𝑥+1 2𝑥+1
Now, ∫ ( 𝑥 + ) dx = ∫ 𝑥 𝑑𝑥 + ∫ 𝑑𝑥
𝑥2− 1 𝑥2− 1
𝑥2 1 𝑥−1
= + log( 𝑥 2 − 1 ) + 2 log( 𝑥+1 ) + C
2
4. 1
Evaluate ∫ dx?
𝑥 ( 𝑥4− 1)
Solution: Multiplying num. and denom. by 𝑥 4-1
𝑥3
I=∫ dx
𝑥4( 𝑥 4− 1 )
Let x4 = t , then
1 𝑑𝑡
I = 4 ∫ 𝑡 ( 𝑡−1 ) dt

135
Apply partial fractions then
−1 𝑥4
I= log ( )+C
4 𝑥4− 1
5. 2𝑥
Evaluate ∫ dx
( 𝑥 2 + 1 )( 𝑥 2 + 3 )
Solution: Let 𝑥 2 = t, then
2𝑥 𝑑𝑡
∫ ( 𝑥 2 + 1 )( 𝑥 2 + 3 ) dx = ∫ ( 𝑡+1 )( 𝑡+3 )
Apply partial fractions then
1 𝑥2+ 1
I= 2 log( )+C
𝑥2 + 3
6. 6𝑥+7
Q. Evaluate ∫ 𝑑𝑥 ?
√( 𝑥 −5 )( 𝑥 −4 )
6𝑥+7 6𝑥+7
Solution: ∫ 𝑑𝑥 = ∫ 𝑑𝑥
√𝑥 2 − 9𝑥+20 √( 𝑥 −5 )( 𝑥 −4 )
𝑑
6x + 7 = A 𝑑𝑥 (𝑥 2 − 9𝑥 + 20 ) + B
A = 3 , B = 34 substitute the values then
9
I=6 √𝑥 2 − 9𝑥 + 20 + 34 log [ x - 2 +√𝑥 2 − 9𝑥 + 20 ] + C
7. 1
Evaluate ∫ d𝑥 ?
1+𝑐𝑜𝑡𝑥
1 𝑠𝑖𝑛𝑥 1 2𝑠𝑖𝑛𝑥
Solution: ∫ d𝑥 = ∫ d𝑥 = ∫ d𝑥
1+𝐶𝑜𝑡𝑥 𝑆𝑖𝑛𝑥 +𝐶𝑜𝑠𝑥 2 𝑆𝑖𝑛𝑥 +𝐶𝑜𝑠𝑥
1 ( 𝑠𝑖𝑛𝑥+𝐶𝑜𝑠𝑥)− ( 𝐶𝑜𝑠𝑥−𝑠𝑖𝑛𝑥 )
= ∫ d𝑥
2 𝑆𝑖𝑛𝑥 +𝐶𝑜𝑠𝑥
𝑥 1 𝐶𝑜𝑠𝑥−𝑠𝑖𝑛𝑥
= 2 - 2 ∫ 𝑆𝑖𝑛𝑥 +𝐶𝑜𝑠𝑥 dx
Put 𝑠𝑖𝑛𝑥 + 𝑐𝑜𝑠𝑥 = 𝑡 then
𝑥 1
I= 2 - 2 𝑙𝑜𝑔 (𝑠𝑖𝑛𝑥 + 𝑐𝑜𝑠𝑥) + 𝐶

8. 1
Evaluate ∫ 𝐶𝑜𝑠 ( 𝑥−𝑎 )𝐶𝑜𝑠 ( 𝑥−𝑏 ) d𝑥
1
Solution: ∫ d𝑥
𝐶𝑜𝑠 ( 𝑥−𝑎 )𝐶𝑜𝑠 ( 𝑥−𝑏 )
1 𝑆𝑖𝑛( 𝑎−𝑏 )
=
𝑆𝑖𝑛 ( 𝑎−𝑏 )
∫ 𝐶𝑜𝑠 ( 𝑥−𝑎 )𝐶𝑜𝑠 ( 𝑥−𝑏 ) d𝑥
1 𝑆𝑖𝑛[ ( 𝑥−𝑏)− ( 𝑥−𝑎 )]
= 𝑆𝑖𝑛 ( 𝑎−𝑏 ) ∫ 𝐶𝑜𝑠 ( 𝑥−𝑎 )𝐶𝑜𝑠 ( 𝑥−𝑏 ) d𝑥
1
= 𝑆𝑖𝑛 ( 𝑎−𝑏 ) [ ∫ 𝑡𝑎𝑛 ( 𝑥 − 𝑏) 𝑑𝑥 − ∫ 𝑡𝑎𝑛( 𝑥 − 𝑎 ) 𝑑𝑥
1 𝐶𝑜𝑠 ( 𝑥−𝑎 )
= 𝑆𝑖𝑛 ( 𝑎−𝑏 ) log ( 𝐶𝑜𝑠 ( 𝑥−𝑏 ) ) + C

9. 𝜋 𝑥
Find ∫0 d𝑥
1 + 𝑠𝑖𝑛𝑥
𝑎 𝑎
Solution: Apply the property ∫0 𝑓(𝑥)𝑑𝑥 = ∫0 𝑓(𝑎 − 𝑥)𝑑𝑥
and prove that I = 𝜋

10. Find ∫( 𝑠𝑖𝑛−1 𝑥 )2 d𝑥 ?


Solution: Let 𝑥 = sint then I = ∫ 𝑡 2 .𝑐𝑜𝑠𝑡 𝑑𝑡 , Apply ∫ 𝑢. 𝑣 𝑑𝑥 formula
I= 𝑥 . (sin-1 𝑥 )2 + 2 √1 − 𝑥 2 sin-1𝑥 -2𝑥 + C

136
EXERCISE

1. 𝐶𝑜𝑠 𝑥
Evaluate ∫ d𝑥
( 2+ 𝑠𝑖𝑛𝑥 )( 3+4 𝑠𝑖𝑛𝑥 )

2. 𝑒𝑥
Evaluate ∫ 𝑑𝑥 ?
√5−4𝑒 𝑥 − 𝑒 2𝑥

3. 𝑥.𝑒 𝑥
Evaluate ∫ d𝑥 ?
( 𝑥+ 1)2
4. 2
Evaluate ∫−1|𝑥 3 − 𝑥| d𝑥 ?

5. 1 − 𝑥2
Evaluate ∫ d𝑥 ?
𝑥 ( 1 − 2𝑥)

5 MARK QUESTIONS
𝜋
1. Evaluate ∫0 𝑙𝑜𝑔 ( 1 + 𝑐𝑜𝑠𝑥 )𝑑𝑥 ?
Solution: Apply the properties of definite integral and prove that
I = - 𝜋. 𝑙𝑜𝑔2
2. 𝜋 𝑥 .𝑡𝑎𝑛𝑥
Evaluate ∫0 𝑑𝑥 ?
𝑆𝑒𝑐𝑥 + 𝑡𝑎𝑛𝑥
𝑎 𝑎
Solution: Apply ∫0 𝑓(𝑥)𝑑𝑥 = ∫0 𝑓(𝑎 − 𝑥)𝑑𝑥
𝜋
I= 2 (𝜋−2)
3. 3𝑥 + 5
Evaluate ∫ d𝑥 ?
𝑥 3 − 𝑥 2 − 𝑥+1
3𝑥 + 5 3𝑥 + 5
Solution: ∫ 𝑥 3 − 𝑥 2 − 𝑥+1 d𝑥 = ∫ ( 𝑥−1 )2 ( 𝑥 +1) d𝑥
3𝑥 + 5 𝐴 𝐵 𝐶
= ( 𝑥−1 ) + +
( 𝑥−1 )2 ( 𝑥 +1) ( 𝑥 −1 )2 ( 𝑥+1 )
−1 1
A= , B = 4 and C = 2
2
1 𝑥 +1 4
I= 2 𝑙𝑜𝑔 𝑥 −1 - 𝑥−1 + C

EXERCISE

1. 𝑥 𝑑𝑥
Evaluate ∫ ?
𝑎2 𝐶𝑜𝑠 2 𝑥+ 𝑏2 𝑠𝑖𝑛2 𝑥

𝜋
2. 𝑠𝑖𝑛𝑥 + 𝑐𝑜𝑠𝑥
Evaluate ∫0 4 d𝑥 ?
9 +16 𝑠𝑖𝑛2𝑥

3. 𝑑𝑥
Evaluate ∫ ?
𝑥 3 + 𝑥 2 + 𝑥+1

137
CHAPTER: APPLICATION OF INTEGRALS
SYLLABUS: Applications in finding the area under simple curves, especially lines, circles/
parabolas/ellipses (in standard form only)

Definitions and Formulae:

Let f(x) be a function defined in [ a, b] , then the area bounded by the curve y = f(x) , x –
𝑏 𝑏
axis and the ordinates x = a and x = b is given by ∫𝑎 𝑓(𝑥)dx or ∫𝑎 𝑦 𝑑𝑥

Let 𝑔(𝑦) be a function defined in [ c, d] , then the area bounded by the curve 𝑥 = 𝑔(𝑦), y –
𝑑
axis and the ordinates 𝑦 = 𝑐 𝑎𝑛𝑑 𝑦 = 𝑑 is given by ∫𝑐 𝑔(𝑦)𝑑𝑦

If the curve y = f(x) lies below x- axis ,then the area bounded by the curve y= f(x) , x-axis
𝑏
and the ordinates x = a and x= b is –ve. So the area is |∫𝑎 𝑓(𝑥) dx|

138
MULTIPLE CHOICE QUESTIONS

Q.NO QUESTIONS AND ANSWERS


1 Find the area bounded by y = 𝑥 2 ,𝑥 𝑎𝑥𝑖𝑠 𝑎𝑛𝑑 𝑙𝑖𝑛𝑒𝑠 𝑥 = −1 𝑎𝑛𝑑 𝑥 = 1
−1 4
(a) sq. unit (b) sq. unit
3 3
2
(c) sq. unit (d) None of these
3
Solution:
1 1 1 2
Area of the required region = ∫−1 𝑥 2 dx = + =
3 3 3
2 Determine the area under the curve y =√𝑎 − 𝑥 2 included between the lines x
2
=0
and x = a
𝜋𝑎2 𝜋
(a) sq. unit (b) sq. unit
4 4
𝑎2 𝜋𝑎2
(c) sq. unit (d) sq.unit
4 12
Solution:
𝑎 𝑎2 𝜋 𝑎2
Required area = ∫0 √𝑎2 − 𝑥 2 𝑑𝑥 = x =𝜋
2 2 4
3 The area of the region bounded by the curve y = 𝑥 2 and the line y = 16 is
32 256
(a) sq. unit (b) sq. unit
3 3
64 128
(c) sq. unit (d) sq.unit
3 3
Solution:
4 4 64 256
Required area= 2 [∫0 16𝑑𝑥 − ∫0 𝑥 2 𝑑𝑥 ] =2[64 − ] =
3 3
4 Find the area bounded by y = sinx between x = 0 and x = 2𝜋.
(a) 4 sq.unit (b) 4𝜋sq. unit (c) 2 sq.unit (d) 1 sq.unit

Solution:
𝜋
Area = 2∫0 𝑠𝑖𝑛𝑥 𝑑𝑥= 4
5 Area lying in the first quadrant and bounded by the circle 𝑥 2 + 𝑦 2 = 4 and the
lines x =0
and x = 2 is
𝜋
(a) 𝜋 sq. unit (b) sq. unit
2
𝜋 𝜋
(c) sq. unit (d) sq. unit
3 4

Solution:
2
Area = ∫0 √4 − 𝑥 2 dx = 𝜋
6 The area of the region bounded by the straight line x = 2y + 3 , y axis and the lines
y = 1 and y = -1 is
3
(a) 4 sq. unit (b) 2 sq. unit (c) 6 sq. unit (d) 8 sq. unit

Solution:
1
Area = ∫−1 2𝑦 + 3dy = 1+3 –( 1- 3) = 6

139
CHAPTER VIDEO LINK FOR MCQs SCAN QR CODE FOR
VIDEO

APPLICATION OF INTEGRALS https://youtu.be/p4R2unWOT9Q

ASSERTION AND REASON QUESTIONS

The following questions consist of two statements – Assertion (A) and Reason(R), Answer
the questions selecting the appropriate option given below.

(a) Both A and R are true and R is the correct explanation for A.

(b) Both A and R are true and R is not the correct explanation for A.

(c) A is true and R is false

(d) A is false and R is true

1 Assertion: The area of the ellipse 2𝑥 2 + 3𝑦 2 = 6 is more than the area of the
circle 𝑥 2 +𝑦 2 - 2x +4 y + 4 =0
Reason: The length of the semimajor axis of ellipse 2𝑥 2 + 3𝑦 2 = 6 is more than
the radius of the circle 𝑥 2 +𝑦 2 - 2x +4 y + 4 =0
Answer:
Area of ellipse = √6 𝜋
Area of circie= 𝜋 .
A is true
Length of major axis is 2√3 .Radius of circle= 1.
R is true.
But R is not the correct explanation of A
Option b is correct
2 Assertion: Area enclosed by the circle 𝑥 2 +𝑦 2 = 36 is equal to 36𝜋 sq.unit
Reason: Area enclosed by circle 𝑥 2 +𝑦 2 = 𝑟 2 is 𝜋𝑟 2
Answer:
Area of the circle is 36𝜋 sq.unit Option ( a) is correct
3 Assertion: The area of the region bounded by y= cosx and the ordinates
𝑥 = 0 𝑎𝑛𝑑 𝑥 = 𝜋 𝑖𝑠 2𝑠𝑞. 𝑢𝑛𝑖𝑡
Reason: cos x is an increasing function in the first quadrant
Answer:
(c)

140
4 Assertion: The area of the region bounded by y = x+1 , x-axis and he lines x =2
5
and x=3 is 2 sq.units
Reason: The intercept made by the line on the x-axis and y axis is 1unit left of
zero and 1unit respectively.
Answer:
(d)
5 Assertion: The area bounded by the curve x =𝑦 2 , y-axis and the lines y =3 and
37
y= 4 is 2
𝑏
Reason: Area = ∫𝑎 𝑓( 𝑦) dy
Answer:
(a)
6 Assertion: Area bounded by 𝑦 = |𝑥 + 2| from x= -2 to x= 0 is 4 sq.unit
Reason: 𝑦 = |𝑥 + 2| is differentiable in R
Answer:
(c)
7 Assertion: The area of the region bounded by y = 𝑥 2 + x , x = 2 and x = 5
cannot be evaluated
Reason: Area of the unbounded region cannot be evaluated
Answer:
(d)
8 Assertion: The area of the region y =sin2x from 0 to 𝜋 will be more than that of
the curve y=sinx from 0 to 𝜋
Reason: 𝑥 2 > x if x > 1
Answer:
(d)

EXERCISE

The following questions consist of two statements – Assertion (A) and Reason( R) ,Anawer
the questions selecting the appropriate option given below

(a) Both A and R are true and R is the correct explanation for A.

(b) Both A and R are true and R is not the correct explanation for A.

(c) A is true and R is false

(d) A is false and R is true

1 Assertion:The region bounded by the curve y=√4 − 𝑥 2 is a semicircle above the


x-axis
Reason: area of the semicircle is half of the area bounded by the equation 𝑥 2 +
𝑦2 = 4

2 Assertion: Area enclosed by 𝑦 = 𝑥|𝑥| , x-axis and the ordinates


2
𝑥 = −1 𝑎𝑛𝑑 𝑥 = 1 is given by 3
Reason: f(x) = |𝑥| = x , x≥0 and –x , x< 0

141
3 Assertion: The region bounded by the curve y=√4 − 𝑥 2 is a semicircle above
the x-axis
Reason: area of the semicircle is half of the area bounded by the equation 𝑥 2 +
𝑦2 = 4
4 Assertion: The area bounded by the curve y = log 𝑒 𝑥 and x- axis and the
straight line x= e
Reason: The most approximate value of e =2.7
5 Assertion: The area between x-axis and y = cosx when 0 ≤ x≤ 2𝜋 is 4sq.unit
𝜋
2𝜋
Reason: Area= ∫0 𝑐𝑜𝑠𝑥 𝑑𝑥 = 4∫02 𝑐𝑜𝑠𝑥 𝑑𝑥
Solutions
1. (b) 2. (b) 3 .(c) 4. (d) 5. (a)

2 MARK QUESTIONS

1 Find the area of the region bounded by the line 𝑦 = 2𝑥 ,x-axis and x=2
Solution:
2
Area = ∫0 2𝑥 dx = 4 sq.unit.

2 Find the area bounded by 𝑦 = 𝑥 , 𝑡ℎ𝑒 𝑥 − 𝑎𝑥𝑖𝑠 and the ordinate 𝑥 = −1 , 𝑥 =


2.
Solution:
2 0 5
𝐴 = ∫0 𝑥 𝑑𝑥 + |∫−1 𝑥 𝑑𝑥 | = 2.

3 Find the area bounded by the circle 𝑥 2 + 𝑦 2 = 𝑟 2


Solution:
𝑟
A= 4× ∫0 √𝑟 2 − 𝑥 2 dx = 𝜋𝑟 2

142
4 𝑥2 𝑦2
Find area of the region bounded by the ellipse 2
+ =1
𝑎 𝑏2
Solution:
𝑏 𝑎
𝐴 = 4× ∫ √𝑎2 − 𝑥 2 dx = 𝜋ab
𝑎 0

5 Using integration find the area of the region bounded by the line 𝑦 – 1 = 𝑥 , x-
axis and the ordinates 𝑥 = −2 𝑎𝑛𝑑 𝑥 = 3

Solution:
−1 3 17
A = |∫−2 (𝑥 + 1) 𝑑𝑥 | + ∫−1(𝑥 + 1)𝑑𝑥 = 2

6 Using integration find the area of the region bounded between the line 𝑥 = 2, and
the parabola 𝑦 2 = 8x.
Solution:
2 32
A = 2∫0 √8𝑥 dx= sq.units
3

143
7 Draw the graph of 𝑦 = |𝑥 + 1| and find the area between
𝑥 − 𝑎𝑥𝑖𝑠 𝑥 = −4 𝑎𝑛𝑑 𝑥 = 2.
Solution:
−1 2
𝐴 = |∫−4 (𝑥 + 1) 𝑑𝑥 | +∫−1(𝑥 + 1) dx = 9 sq.units

𝜋
8 Draw the graph of 𝑦 = 𝑠𝑖𝑛𝑥 , 𝑦 = 𝑐𝑜𝑠𝑥 between 𝑥 = 0 and x = and also
4
𝜋
find the area between the curves between x = 0 and x = 4
Solution:
𝜋
A = ∫04 (𝑐𝑜𝑠𝑥 − 𝑠𝑖𝑛𝑥) dx = √2 - 1sq.units

EXERCISE

1 𝑎2
Calculate the area of the region bounded by 𝑦 2 = 4ax and the line y = mx is 12
sq. units. Find the value of m
2 Sketch the graph of y = |x+ 3| and evaluate the area under the curve y = |x+ 3|
above x- axis and between x = -6 and x = 0
3 Find the area bounded by 𝑦 2 = 4ax , latus rectum and x-axis
4 Find the area of the region bounded by 𝑦 2 = 4ax and x = 𝑎 , x = 2a, a > 0
Answers:
4 56𝑎2
1.m = 2 2. 9 sq.units 3. 𝑎2 sq.units 4. sq.units
3 3

144
3 MARK QUESTIONS

1 Find the area bounded by the curve y = sinx between 𝑥 = 0 𝑎𝑛𝑑 𝑥 = 2𝜋


Solution:
𝜋
∫0 𝑠𝑖𝑛𝑥 𝑑𝑥 = 2 𝑠𝑞. 𝑢𝑛𝑖𝑡𝑠.
2𝜋
And |∫𝜋 𝑠𝑖𝑛𝑥 𝑑𝑥 | =2 sq.units.
Required area = 4 sq.units

2 Find the area bounded by the curve 𝑦 = 𝑥|𝑥| and the ordinates x = -1 and x = 1.
Solution:
1 2
A = 2 ∫0 𝑥 2 𝑑𝑥 =
3

3 Find the area of the smaller part of the circle 𝑥 2 + 𝑦 2 = 𝑎2 cut off by the line
𝑎
𝑥=
√2
Solution:
𝑎 𝑎2 𝜋
A =2 ∫𝑎 √𝑎2 − 𝑥 2 dx = ( - 1 ).
√2
2 2

145
4 Area of the region bounded by the curve 𝑦 = | 𝑥 − 1| , 𝑦 = 1.
Solution:
2 1
𝐴 = ∫0 1 dx − 2 ∫0 (𝑥 − 1) 𝑑𝑥 = 1

EXERCISE

1 Find the area of the circle 4𝑥 2 + 4𝑦 2 = 1


2 Find the area of the region bounded by the ellipse 6𝑥 2 + 8𝑦 2 = 1
3 Sketch the area lying in first quadrant and bounded by y = 9 𝑥 2 , x = 0, y = 1
and y = 4 . Find the area of this region using integration.
4 Using integration, find the area of the triangle formed by +ve x-axis, tangent and
normal to the circle𝑥 2 + 𝑦 2 = 4 at (1, √3)
Answers:
5 𝜋 19
1) 8 sq. units 2).4√3 . sq. units 3). 4 sq.units 4).2√3
CASE STUDY QUESTIONS
I A student designs an open air honeybee nest on the branch of a tree, whose Plane
figure is parabolic, whose equation is 𝑦 2 = 2x and the branch of tree is given by
a straight line x- y = 4

Based on the above passage answer the following questions


1. Draw the rough diagram of parabola and straight line
2. Find point of intersection of the parabola and straight line
3. Find the area enclosed by the parabola and straight line

Solution:

146
1.

2) . Solve the equations 𝑦 2 = 2x and x- y =4 the point of intersection is


(2, -2) and ( 8,4)
4 𝑦2
3). ∫−2 𝑦 + 4 − dy = 18
2

II 𝑥2 𝑦2
A mirror is in the shape of an ellipse represented by + = 1was hanging on
𝑎2 𝑏2
the wall. Arun and his sister were playing with ball inside the house , even their
mother refused to do so . All of a sudden, ball hit the mirror and got a scratch in
𝑥 𝑦
the shape of line represented by + =1
𝑎 𝑏
Based on the above information answer the following questions.

1..Find the point of intersection of mirror and scratch


𝑏 𝑎
2. The value of
𝑎
∫0 √𝑎2 − 𝑥 2 dx
3. The area of the smaller region bounded by the mirror and the scratch
Solution:
𝑥2 𝑦2 𝑥 𝑦
1) Solve the equations + = 1 and + = 1 .
𝑎2 𝑏2 𝑎 𝑏
The points are (a,0) (0,b)
𝑏 𝑎 𝜋𝑎𝑏
2). ∫ √𝑎2 − 𝑥 2 =
𝑎 0 4
𝑏 𝑎 𝑏 𝑎 1 𝜋
3). ∫ √𝑎2 − 𝑥 dx - ∫0 𝑎 − 𝑥 dx = ( - 1 ).
𝑎 0 𝑎 2 2

III A child cut a pizza with a knife. Pizza is circular in shape which is represented by

147
𝑥 2 +𝑦 2 = 4 and knife represents x =√3𝑦, On the basis of the above information
answer the following.

1) Find the point of intersection of circle and straight line.


2) Find the area enclosed by the circle, line and x-axis.

Solution:
1. Point of intersection is (1,√3 ) and ( -1, - √3 )
1 2 2𝜋
2. Required area = √3 ∫0 𝑥 𝑑𝑥 + ∫1 √22 − 𝑥 2 dx = .
3
EXERCISE

I The location of three branches of a bank is represented by the three points A( -2,
0) , B( 1,4), C ( 2,3) . Based on this information solve the following questions.
1. Find the equations of line AB and BC
2. Find the area of triangle ABC.
II An insect moves on a curve represented by y = 𝑥 3 . It started from a point (-2,-8)
on the curve and as soon as it reached at a point (2,8) got tired and slept. The path
of its movement is given below. Based on this information answer the following
questions.

1. Find the area enclosed by the curve y= 𝑥 3 , the lines x=2 and x= -2

2. If it would have moved along the line represented by y = x what is the


area bounded by the curve y = 𝑥 3 and y = 6 x

Answers:
4 7
I) (1) y = (x +2), y = - x+3 (2). sq.units
3 2
II) (1). 8.sq. unit. (2)12 sq.unit

148
.

CHAPTER: DIFFERENTIAL EQUATIONS


SYLLABUS: Definition, order and degree, general and particular solutions of a differential
equation. Solution of differential equations by method of separation of variables, solutions of
homogeneous differential equations of first order and first degree. Solutions of linear
differential equation of the type:
𝑑𝑦
+py=q, where p and q are functions of x or constants.
𝑑𝑥

𝑑𝑥
+ px=q, where p and q are functions of y or constants.
𝑑𝑦

Definitions and Formulae:

Methods of solving First Order and First Degree Differential Equations

➤ Differential Equations with Variables separables

➤ Homogeneous differential equations

➤ Linear differential equations

Differential Equations with Variables separables

To solve the differential equation in variable separable form, write the differential equation as

(x terms) X dx = (y terms) X dy then integrate both sides.


𝑑𝑦 𝑓(𝑥)
• Let the differential equation 𝑑𝑥 = 𝑔(𝑦)
then g(y)dy = f(x)dx
then integrate on both sides ∫ 𝑔(𝑦)𝑑𝑦 = ∫ 𝑓(𝑥)𝑑𝑥

𝑑𝑦 𝑔(𝑦)
• Let the differential equation 𝑑𝑥 = 𝑓(𝑥)
𝑑𝑦 𝑑𝑥
then 𝑔(𝑦) = 𝑓(𝑥)
𝑑𝑦 𝑑𝑥
then integrate on both sides ∫ =∫
𝑔(𝑦) 𝑓(𝑥)

𝑑𝑦
• Let the differential equation 𝑑𝑥 = 𝑓(𝑥). 𝑔(𝑦)
𝑑𝑦
then 𝑔(𝑦) = f(x)dx
𝑑𝑦
then integrate on both sides ∫ 𝑔(𝑦) = ∫ 𝑓(𝑥) 𝑑𝑥

Homogeneous differential equations

❖ A function 𝐹(𝑥, 𝑦) is said to be homogeneous function of degree n if


𝐹(𝜆𝑥, 𝜆𝑦) = 𝜆𝑛 𝐹(𝑥, 𝑦)

149
𝑑𝑦
❖ A differential equation of the form 𝑑𝑥 = 𝐹(𝑥, 𝑦) is said to be homogeneous if F(x,y)
is a homogeneous function of degree zero

i.e. if 𝐹(𝜆𝑥, 𝜆𝑦) = 𝜆0 𝐹(𝑥, 𝑦)


𝒅𝒚 𝒚
Steps to solve the homogeneous differential equation of the type: 𝒅𝒙 = 𝒇(𝒙)

• Let 𝑦 = 𝑣𝑥
𝑑𝑦 𝑑𝑣
• = 𝑣 + 𝑥 𝑑𝑥
𝑑𝑥

𝑑𝑦 𝑑𝑣 𝑑𝑦 𝑦
• Substitute 𝑦 = 𝑣𝑥 and = 𝑣 + 𝑥 𝑑𝑥 in = 𝑓(𝑥 )
𝑑𝑥 𝑑𝑥

• Then use variables and separables in terms of 𝑦 and 𝑣 only


𝒅𝒙 𝒙
Steps to solve the homogeneous differential equation of the type: 𝒅𝒚 = 𝒇(𝒚)

• Let 𝑥 = 𝑣𝑦
𝑑𝑥 𝑑𝑣
• 𝑑𝑦
= 𝑣 + 𝑦 𝑑𝑦

𝑑𝑥 𝑑𝑣 𝑑𝑥 𝑥
• Substitute 𝑥 = 𝑣𝑦 and = 𝑣 + 𝑦 𝑑𝑦 in = 𝑓(𝑦)
𝑑𝑦 𝑑𝑦

• Then use variables and separables in terms of 𝑥 and 𝑣 only

Linear differential equation


𝒅𝒚
Steps to solve the Linear differential equation of the type:𝒅𝒙 + 𝑷(𝒙)𝒚 = 𝑸(𝒙)

𝑑𝑦
❖ + 𝑃(𝑥)𝑦 = 𝑄(𝑥)
𝑑𝑥

❖ 𝐼𝑛𝑡𝑒𝑔𝑟𝑎𝑡𝑖𝑛 𝐹𝑎𝑐𝑡𝑜𝑟 (𝐼𝐹) = 𝑒 ∫ 𝑝(𝑥)𝑑𝑥

❖ 𝑆𝑜𝑙𝑢𝑡𝑖𝑜𝑛 𝑖𝑠 𝑦. (𝐼𝐹) = ∫(𝐼𝐹). 𝑄(𝑥)𝑑𝑥


𝒅𝒙
Steps to solve the Linear differential equation of the type:𝒅𝒚 + 𝑷(𝒚)𝒙 = 𝑸(𝒚)

𝑑𝑥
❖ + 𝑃(𝑦)𝑥 = 𝑄(𝑦)
𝑑𝑦

❖ 𝐼𝑛𝑡𝑒𝑔𝑟𝑎𝑡𝑖𝑛 𝐹𝑎𝑐𝑡𝑜𝑟 (𝐼𝐹) = 𝑒 ∫ 𝑝(𝑦)𝑑𝑦

❖ 𝑆𝑜𝑙𝑢𝑡𝑖𝑜𝑛 𝑖𝑠 𝑥. (𝐼𝐹) = ∫(𝐼𝐹). 𝑄(𝑦)𝑑𝑦

150
MULTIPLE CHOICE QUESTIONS

S.NO QUESTIONS AND SOLUTIONS

1. Integrating factor for the differential equation


𝑑𝑦
(𝑥𝑙𝑜𝑔𝑥) 𝑑𝑥 + 𝑦 = 2𝑙𝑜𝑔𝑥 is
(a) 𝑙𝑜𝑔(𝑙𝑜𝑔𝑥) (b) 𝑙𝑜𝑔𝑥 (c) 𝑒 𝑥 (d) 𝑥

Ans (b)
𝑑𝑦 1 2
Equation is 𝑑𝑥 +xlogx.y = 𝑥
1
Here p(x) = 𝑥𝑙𝑜𝑔𝑥
1
∫𝑥𝑙𝑜𝑔𝑥𝑑𝑥
Integrating factor =𝑒 = 𝑒 log(𝑙𝑜𝑔𝑥) = 𝑙𝑜𝑔𝑥

2. If 𝑚 𝑎𝑛𝑑 𝑛, respectively, are the order and the degree of the differential
𝑑 𝑑𝑦 4
equation 𝑑𝑥 [(𝑑𝑥 )] = 0, then 𝑚 + 𝑛 =

(a) 1 (b) 2 (c) 3 (d) 4

Ans: (c)
𝑑 𝑑𝑦 4
The given differential equation is 𝑑𝑥 [(𝑑𝑥 )] = 0
Differentiate w.r.t x , weget
𝑑𝑦 3 𝑑2 𝑦
4(𝑑𝑥 ) 𝑑𝑥 2 = 0
Here, m = 2 and n =1
Hence, m + n = 3
3. 2
𝑑2 𝑦 𝑑𝑦
If 𝑝 𝑎𝑛𝑑 𝑞 are the degree and order of the differential equation (𝑑𝑥 2 ) + 3 𝑑𝑥 +
𝑑3 𝑦
= 4 , then the value of 2𝑝 − 3𝑞 is
𝑑𝑥 3

(a) 7 (b) −7 (c)3 (d) −3

Ans: (b)
degree p =1 and order q = 3 ∴ 2𝑝 – 3𝑞 = 2 − 9 = −7

4. Find the value of m and n, where m and n are order and degree of differential
3
𝑑2 𝑦
4( ) 𝑑3 𝑦
𝑑𝑥2
equation 𝑑3 𝑦
+ 𝑑𝑥 3 = 𝑥 2 − 1
𝑑𝑥3
(a) 𝑚 = 3, 𝑛 = 2 (b) 𝑚 = 2 , 𝑛 = 3

(c) 𝑚 = 2, 𝑛 = 2 (d) 𝑚 = 3, 𝑛 = 3
Ans (a)
3
𝑑2 𝑦
4( ) 𝑑3 𝑦
𝑑𝑥2
Solution(a) Given 𝑑3 𝑦
+ 𝑑𝑥 3 = 𝑥 2 − 1
𝑑𝑥3
3 2
𝑑2 𝑦 𝑑3 𝑦 𝑑3 𝑦
⇒ 4 (𝑑𝑥 2 ) +(𝑑𝑥 3 ) = (𝑥 2 − 1) 𝑑𝑥 3

𝑚 = 3, 𝑛 = 2

151
5. Differential equation 𝑒 𝑥 𝑑𝑦 = 3𝑦 2 can be solved using the method of
𝑑𝑥
(a) Separating the variables
(b) Homogenous equation
(c) Linear differential equation of first order
(d) None of these

Ans (a)

6. General solution of the differential equation


𝑑𝑦
log (𝑑𝑥 ) = 2𝑥 + 𝑦

1 1 1
(a) 𝑒 −𝑦 = 2 𝑒 2𝑥 + 𝐶 (b) 𝑒 𝑦 + 2 𝑒 2𝑥 + 𝐶
1 1
(c) −𝑒 −𝑦 = 2 𝑒 2𝑥 + 𝐶 (d) 𝑒 𝑦 = 2 𝑒 2𝑥 + 𝐶

Ans (c)
𝑑𝑦
= 𝑒 2𝑥+𝑦 = 𝑒 2𝑥 . 𝑒 𝑦
𝑑𝑥
⇒∫ 𝑒 −𝑦 𝑑𝑦 = ∫ 𝑒 2𝑥 𝑑𝑥
1
⇒−𝑒 −𝑦 = 2 𝑒 2𝑥 + 𝐶
7. The particular solution of the differential equation
𝑑𝑦
= 𝑦𝑡𝑎𝑛𝑥, given that y = 1 when x = 0 is
𝑑𝑥

(a) 𝑦 = 𝑐𝑜𝑠𝑥 (b) 𝑦 = 𝑠𝑒𝑐𝑥


(c) 𝑦 = 𝑡𝑎𝑛𝑥 (d) 𝑦 = 𝑠𝑒𝑐𝑥𝑡𝑎𝑛𝑥

Ans (b)
𝑑𝑦
Solution: ∫ 𝑑𝑥 = ∫ 𝑡𝑎𝑛𝑥 𝑑𝑥 ⇒ log|y| = log|𝑠𝑒𝑐𝑥| + 𝑙𝑜𝑔𝐶
⇒y =cosecx
Given y = 1, x = 0 ⇒1 = sec0 ⇒C = 1
Solution is 𝑦 = 𝑠𝑒𝑐𝑥
2
8. 𝑑3 𝑦 3
Degree of differential equation (𝑑𝑥 3 ) = 𝑥 is
(a) 1 (b)2 (c) 3 (d) Does not exist
Ans (b)
2
𝑑3 𝑦
Solution: (𝑑𝑥 3 ) = 𝑥 3

9. Integrating factor of the differential equation 𝑑𝑦 = 𝑐𝑜𝑠𝑦 is


𝑑𝑥 1−𝑥𝑠𝑖𝑛𝑦
(a) 𝑐𝑜𝑠𝑦 (b) −𝑠𝑒𝑐𝑦 (c) 𝑠𝑒𝑐𝑦 (d) 𝑡𝑎𝑛𝑦
Ans (c)
𝑑𝑦 𝑐𝑜𝑠𝑦
Solution: 𝑑𝑥 = 1−𝑥𝑠𝑖𝑛𝑦
𝑑𝑥 1−𝑥𝑠𝑖𝑛𝑦
⇒𝑑𝑦 = 𝑐𝑜𝑠𝑦
𝑑𝑥
⇒𝑑𝑦 + 𝑡𝑎𝑛𝑦. 𝑥 = 𝑠𝑒𝑐𝑦
Now, P(y) = tany; Q(y) = secy
I.F. = 𝑒 ∫ 𝑃 𝑑𝑦 = 𝑒 ∫ 𝑡𝑎𝑛𝑦 𝑑𝑦 = 𝑠𝑒𝑐𝑦

152
10. Differential equation 𝑥 𝑑𝑦 = 𝑦(𝑙𝑜𝑔𝑦 − 𝑙𝑜𝑔𝑥 + 1) can be solved using method
𝑑𝑥
of
(a) Separating the variables
(b) Homogenous equation
(c)Linear differential equation of first order
(d)None of these

Solution : (b)

CHAPTER VIDEO LINK FOR MCQs SCAN QR CODE FOR


VIDEO

DIFFERENTIAL EQUATIONS https://youtu.be/4T5yvAwh4dM

EXERCISE

1 The degree of differential equation


𝑑𝑦 3 𝑑𝑦 2
(1 + 𝑑𝑥 ) = (𝑑𝑥 )
(i) 1
(ii) 2
(iii)3
(iv) 4
Solution: (iii)
2 𝑑𝑦 𝑦
General solution of differential equation 𝑑𝑥 = 𝑥 is
(i) logy = Cx
(ii) y = Cx
(iii) xy = C
(iv) y = Clogx
Solution: (ii)
3 Integrating factor for the differential equation
𝑑𝑦
(𝑥𝑙𝑜𝑔𝑥) + 𝑦 = 2𝑙𝑜𝑔𝑥 is
𝑑𝑥
(i) log(logx)
(ii) logx
(iii)𝑒 𝑥
(iv) 𝑥
Solution: (ii)
4 Integrating factor for the differential equation
𝑑𝑦
𝑠𝑖𝑛2 𝑥 𝑑𝑥 + 𝑦 = cot 𝑥 is
(i) 𝑒 −𝑐𝑜𝑡𝑥

153
(ii) 𝑐𝑜𝑡𝑥
(iii)−𝑐𝑜𝑡𝑥
(iv) 𝑒 𝑐𝑜𝑡𝑥
Solution:(i)
5 Integrating factor for the solution of differential equation
(𝑥 − 𝑦 3 )𝑑𝑦 + 𝑦𝑑𝑥 = 0 is
1
(i) 𝑦
(ii) logy
(iii)y
(iv) 𝑦 2
Solution: (iii)

ASSERTION REASONING BASED QUESTIONS


In the following questions, a statement of assertion (A) is followed by a
statement of Reason (R). Choose the correct answer out of the following
choices.

(a) Both A and R are true and R is the correct explanation of A. (b) Both A and
R are true but R is not the correct explanation of A.
(c) A is true but R is false.
(d) A is false but R is true
1 Assertion (A): The solution of differential equation
𝑑𝑦 𝑦 𝑦 𝑦
= + tan is cos ( ) = 𝑥𝑐
𝑑𝑥 𝑥 𝑥 𝑥
𝑑𝑦 𝑦 𝑦
Reason (R): 𝑑𝑥 = 𝑥 + tan 𝑥 we can clearly see that it is an homogenous
equation substituting
Y=vx
𝑑𝑦 𝑑𝑣
⇒𝑑𝑥 = 𝑣 + 𝑑𝑥
𝑑𝑣
⇒ v + x𝑑𝑥 = 𝑣 + 𝑡𝑎𝑛𝑣
Separating the variables and integrating we get
1 1
∫ tan 𝑣 𝑑𝑣 = ∫ 𝑥 𝑑𝑥
Log(sinv) = logx + logC
Sin(v) =xC
𝑦
⇒sin(𝑥 ) = 𝑥𝐶
Is the solution where, C is constant.

Solution: (d)
2 Assertion (A): The degree of the differential equation given by
𝑑𝑦 𝑥 4 −𝑦4
= (𝑥 2 +𝑦2 )𝑥𝑦 is 1
𝑑𝑥
Reason (R): The degree of a differential equation is the degree of the highest
order derivative when differential coefficients are free from radicals and
fraction.

The given differential equation has first order derivative which is free from
radical and fraction with power = 1, thus it has a degree of 1.
Solution: (a)
3 Assertion (A): Solution of the differential equation
𝑑𝑦 3𝑥−2𝑦 2 −2𝑦
𝑒 2𝑦 𝑒 3𝑥 𝑥 2
=𝑒 +𝑥 𝑒 is = + +𝐶
𝑑𝑥 3 3 2
154
Reason (R):
𝑑𝑦
= 𝑒 3𝑥−2𝑦 + 𝑥 2 𝑒 −2𝑦
𝑑𝑥
𝑑𝑦
= 𝑒 −2𝑦 (𝑒 3𝑥 + 𝑥 2 )
𝑑𝑥
Separating the variables
𝑒 2𝑦 dy =(𝑒 3𝑥 + 𝑥 2 )𝑑𝑥
∫ 𝑒 2𝑦 𝑑𝑦 = ∫(𝑒 3𝑥 + 𝑥 2 )𝑑𝑥
𝑒 2𝑦 𝑒 3𝑥 𝑥2
= 3 + 2 + 𝐶.
2
Solution: (d)
4 Assertion (A): The order and degree of differential equation
𝑑2 𝑦 𝑑𝑦
√ = √𝑑𝑥 + 5 are 2 and 1 respectively
𝑑𝑥 2
Reason (R): The differential equation
𝑑𝑦 3 1
(𝑑𝑥 ) + 2𝑦 2 = 𝑥 is of order 1 and degree 3
Solution: (b)
5 𝑑𝑦
Assertion (A): Order of differential equation is 𝑑𝑥 + 4𝑦 = sin 𝑥 is 1
Reason (R): Since order of the differential equation is defined as order of the
highest derivative occurring in the differential equation, i.e., for nth derivative
𝑑𝑛 𝑦
if n = 1 then it’s order = 1.
𝑑𝑥 𝑛
𝑑𝑦
Given differential equation contains only 𝑑𝑥 derivative with variable and
constants.
Solution: (a)
6 𝒅𝒚 𝟑 𝒅𝟐 𝒚
Assertion (A): order of differential equation (𝒅𝒙) + 𝒅𝒙𝟐 = sinx is 1
Reason (R): Order of the differential equation is the order of the highest order
differential present in the equation

Solution: (d)
7 𝒅𝒚
Assertion (A): 𝒅𝒙 + 𝒙𝟐 𝒚 = 𝟓 is a first order linear differential equation
Reason (R): If Pand Q are functions of x only or constant then differential
𝒅𝒚
equation of the form 𝒅𝒙 + 𝑷𝒚 = 𝑸 is a first order linear differential
equation
Solution: (a)
8 𝒅𝒚 𝒙𝟑 −𝒙𝒚𝟐 +𝒚𝟑
Assertion (A): 𝒅𝒙 = is a homogeneous differential equation.
𝒙𝟐 𝒚−𝒙𝟑
𝒙𝟑 −𝒙𝒚𝟐 +𝒚𝟑
Reason (R): the function F(x,y)= is homogenous
𝒙𝟐 𝒚−𝒙𝟑

Solution: (a)
9 𝑑2 𝑦 𝑑𝑦 2
Assertion (A): The degree of the differential equation +3(𝑑𝑥 ) =
𝑑𝑥
𝑑2 𝑦
x²log(𝑑𝑥 2 ) is not defined.
Reason (R): If the differential equation is a polynomial in terms of its
derivatives, then its degree is defined.

Solution: (a)

155
EXERCISE
Assertion reasoning-based question
In the following questions, a statement of assertion (A) is followed by a
statement of Reason (R). Choose the correct answer out of the following
choices.

(a) Both A and R are true and R is the correct explanation of A. (b) Both A and
R are true but R is not the correct explanation of A.
(c) A is true but R is false.
(d) A is false but R is true
1 𝑑𝑦 𝑑𝑦
Assertion (A): Degree of differential equation 𝑑𝑥 + 𝑙𝑜𝑔 (𝑑𝑥 ) = 0 is not
defined.
Reason (R): Differential equation cannot be written as polynomial of
derivatives.

Solution: (b)

2 𝑑𝑦 𝑦
Assertion (A): The differential equation 𝑑𝑥 = 1 + 𝑥 is homogenous differential
equation.
𝑑𝑦 𝑑𝑦
Reason (R): For a homogenous equation 𝑑𝑥 = 𝑓 (𝑑𝑥 ).

Solution: (c)

3 𝑑𝑦
Assertion (A): To solve the differential equation 𝑑𝑥 = sin(𝑥 + 𝑦) we first
substitute x +y =t
𝑑𝑦 𝑑𝑡
Reason (R): If x +y = t, then 𝑑𝑥 = 𝑑𝑥 − 1

Solution: (a)

4 𝑑𝑦
Assertion (A): Integrating factor for the differential equation x 𝑑𝑥 − 𝑦 =
𝑥 2 𝑠𝑖𝑛𝑥 is -x.
𝑎
Reason (R): 𝑒 𝑎𝑙𝑜𝑔𝑥 = 𝑒 𝑙𝑜𝑔𝑥 = 𝑥 𝑎

Solution: (d)

5 𝑑𝑦 𝑥+√𝑦2 −𝑥 2
Assertion (A): The differential equation 𝑑𝑥 = is homogenous
𝑥
equation.
Reason (R): 𝑓(𝜆𝑥, 𝜆𝑦) = 𝑓(𝑥, 𝑦) for homogenous equation.

Solution: (a)
6 𝑑𝑦
Assertion (A): The differential equation 𝑥 2 = 𝑦 2 + 𝑥𝑦 𝑑𝑥 is an ordinary
differential equation.
Reason (R): An ordinary differential equation involves derivatives of the
dependent variable with respect to only one dependent variable.

Solution: (a)

156
7 𝑑2 𝑦
For the differential equation 𝑑𝑥 2 + y = 0, let its solution be y =
𝜋
∅1 (x) =sin (𝑥 + 4 ) .
Assertion (A): The function y = ∅1 (x) is called the particular solution.
Reason (R): The solution which is free from arbitrary constant, is called a
particular solution.

Solution: (a)

8 Assertion (A): The differential equation


𝑑𝑥 𝑑𝑥 −2𝑥
+x = cos 𝑦 𝑎𝑛𝑑 𝑑𝑦 + 𝑦 = 𝑦 2 𝑒 −𝑦
𝑑𝑦
are first order linear differential equations.
Reason (R): The differential equation of the form
𝑑𝑥
+ 𝑃1 𝑥 = 𝑄1
𝑑𝑦
Where, 𝑃1 and 𝑄1 are constants or functions of y only, is called first order linear
differential equations.

Solution: (a)

9 Assertion (A): The differential equation 𝑦 3 dy + (𝑥 + 𝑦 2 )dx = 0 becomes


homogeneous if we put 𝑦 2 = 𝑡
Reason (R): All differential equation of first order first degree becomes
homogeneous if we put y = tx.

Solution: (c)

10 Let a solution y = y(x) of the differential equation x√𝑥 2 − 1𝑑𝑦 −


2
𝑦√𝑦 2 − 1𝑑𝑥 = 0 satisfy 𝑦(2) =
√3
−1 𝜋
Assertion (A): y(x) = sec (sec 𝑥 − 6)
1 2√3 1
Reason (R): y(x) is given by 𝑦 = − √1 − 𝑥 2
𝑥

Solution: (c)
3
11 𝑑3 𝑦 𝑑2 𝑦 2
Assertion (A): The degree of the differential equation 𝑑𝑥 3 + 2 ( 𝑑𝑥 2 ) + 2y =
0 is zero
Reason (R): The degree of a differential equation is not defined if it is not a
polynomial eqn in its derivatives.

Solution: (d)

12 𝑑𝑦 𝑥 3 −𝑥𝑦 2 +𝑦3
Assertion (A): 𝑑𝑥 = is a homogeneous differential equation.
𝑥 2 𝑦−𝑥 3
𝑑𝑦 𝑥 3 −𝑥𝑦 2 +𝑦3
Reason (R): The function F(x,y) = 𝑑𝑥 = is homogenous.
𝑥 2 𝑦−𝑥 3

Solution: (a)

13 𝑑𝑦
Assertion (A): = 𝑑𝑥 + 𝑥 2 𝑦5 is a first order linear differential equation.

157
Reason (R): If P and Q are functions of x only or constant then differential
𝑑𝑦
equation of the form 𝑑𝑥 + Py = Q is a first order linear differential equation.

Solution: (a)

2 MARKS QUESTIONS

Q.NO QUESTIONS WITH SOLUTIONS


1 Question: Write the integrating factor of the differential equation:
𝑑𝑦
√𝑥 (𝑑𝑥 ) + y =𝑒 −2√𝑥
Solution: The given differential equation is:
𝑑𝑦 1 𝑒 −2√𝑥
+ ( 𝑥) 𝑦 =
𝑑𝑥 √ √𝑥
1
∫ 𝑑𝑥
I.F. = 𝑒 √𝑥 = 𝑒 2√𝑥

2 Question: Find the general solution of the differential equation


ydx – (x + 2𝑦 2 )𝑑𝑦 = 0
Solution: Given differential equation can be written as
𝑑𝑦 𝑑𝑥 1
y 𝑑𝑥 − 𝑥 = 2𝑦 2 or 𝑑𝑦 – 𝑦 . 𝑥 = 2𝑦
1
Integrating factor is 𝑒 −𝑙𝑜𝑔𝑦 = 𝑦
1
∴Solution is x. 𝑦 = ∫ 2𝑑𝑦 = 2y + C or x = 2y² + Cy

3 Write the order and degree of the differential equation


𝑑𝑦
y = px +√1 + 𝑝2 , where 𝑝 = 𝑑𝑥

solution: Order: 1, Degree: 2


4 𝑑𝑦
Solve the differential equation 𝑑𝑥 = 𝑒 𝑥−𝑦 + 𝑥 3 𝑒 −𝑦
𝑥4
Solution: ∫ 𝑒 𝑦 𝑑𝑦 = ∫(𝑒 𝑥 + 𝑥 3 )𝑑𝑥 ⇒𝑒 𝑦 = 𝑒 𝑥 + +𝐶
4

5 Find the general solution of differential equation


𝑑𝑦 𝑥 + 1
= , (𝑦 ≠ 2).
𝑑𝑥 2 − 𝑦

Solution: ∫(2 − 𝑦)𝑑𝑦 = ∫(𝑥 + 1)𝑑𝑥


𝑦2 𝑥2
⇒2𝑦 − 2 = 2 + 𝑥 + 𝐶 is the required solution.
6 Solve the differential equation xydy =(y + 5)dx, given that y(5) = 0
𝑦 𝑑𝑥
Solution: ∫ 𝑦+5 𝑑𝑦 = ∫ 𝑥
(𝑦+5)−5 𝑑𝑥
⇒∫ 𝑑𝑦 = ∫
𝑦+5 𝑥
5 𝑑𝑥
⇒ ∫{1 − 𝑦+5}𝑑𝑦 = ∫ 𝑥
⇒y – 5log|y + 5| = log|x| + C
Given when x = 5, y = 0
⇒0 – 5log5 = log5 +C
⇒C = -6log5

158
Substituting in (i), we get
y – 5log|y + 5| = log|x| - 6log5 is required solution.

7 𝑑𝑦
For the differential equation, √𝑎 + 𝑥 𝑑𝑥 + 𝑥 = 0
Find the general solution

Solution:
−𝑥 𝑥
dy = 𝑎+𝑥 𝑑𝑥 ⇒ ∫ 𝑑𝑦 = − ∫ 𝑑𝑥
√ √𝑎+𝑥
𝑎
⇒∫ 𝑑𝑦 = − ∫ (√𝑎 + 𝑥 − ) 𝑑𝑥
√𝑎+𝑥
3
2
⇒y = -3 (𝑎 + 𝑥) + 2𝑎 √𝑎 + 𝑥+C is the required solution.
2

𝑦
8 For the differential equation, 𝑒 𝑥 √1 − 𝑦 2 𝑑𝑥 + 𝑥 𝑑𝑦 = 0
𝑦
Solution: ∫ 𝑥𝑒 𝑥 𝑑𝑥 + ∫ 𝑑𝑦 = 0
√1−𝑦2
1
⇒𝑥𝑒 𝑥 − ∫ 1. 𝑒 𝑥 𝑑𝑥 − 2 𝑋 2√1 − 𝑦 2 = 𝐶
⇒x𝑒 𝑥 − 𝑒 𝑥 − √1 − 𝑦 2 =C is the required solution.
9 𝑑𝑦
Write the solution of differential equation 𝑑𝑥 = 2−𝑦
𝑑𝑦
Solution: Given 𝑑𝑥 = 2−𝑦 ⇒ ∫ 2𝑦 𝑑𝑦 = ∫ 𝑑𝑥
2𝑦
⇒ 𝑙𝑜𝑔 = 𝑥 + 𝐶 is the required solution
𝑒2

10 Find the general solution of the differential equation


(𝑥 − 𝑦 3 )𝑑𝑦 + 𝑦𝑑𝑥 = 0
𝑑𝑥 𝑦 3 −𝑥
Solution: =
𝑑𝑦 𝑦
1
𝑑𝑥 1 ∫𝑦𝑑𝑦
⇒𝑑𝑦 + 𝑦 . 𝑥 = 𝑦 2 ; 𝐼. 𝐹. = 𝑒 =𝑦
Solution is 𝑦. 𝑥 = ∫ 𝑦. 𝑦 2 𝑑𝑦 = ∫ 𝑦 3 𝑑𝑦
𝑦4
⇒yx = +𝐶
4
𝑦3 𝐶
⇒𝑥 = +𝑦
4

11. Find the general solution of the differential equation


𝑑𝑦 2
(𝑥 2 − 1) + 2𝑥𝑦 = 2
𝑑𝑥 𝑥 −1
𝑑𝑦 2𝑥 2
Solution: 𝑑𝑥 + 𝑥 2 −1 . 𝑦 = (𝑥 2 −1)2
2𝑥
𝑑𝑥 2 −1)
I.F. = 𝑒 ∫𝑥2−1 = 𝑒 𝑙𝑜𝑔(𝑥 = (𝑥 2 − 1)
2
Solution is (𝑥 2 − 1)𝑦 = ∫ 𝑥 2 −1 𝑑𝑥
𝑥−1
⇒(𝑥 2 − 1)𝑦 = 𝑙𝑜𝑔 |𝑥+1| + 𝐶

12. Find the particular solution of differential equation


𝑑𝑦
x𝑑𝑥 + 𝑦 = 𝑥 3 , given that y = 1, when x = 2

1
𝑑𝑦 1
solution: 𝑑𝑥 + 𝑥 𝑦 = 𝑥 2 , 𝐼. 𝐹. = 𝑒 ∫𝑥𝑑𝑥 = 𝑒 𝑙𝑜𝑔𝑥 = 𝑥

159
𝑥4
Solution x.y = ∫ 𝑥 3 𝑑𝑥 ⇒xy = 4 + 𝐶
given y = 1, x = 2 ∴2 = 4 + C ⇒C = -2
𝑥3 2
y = 4 − 𝑥 is the particular solution
13. Find the general solution of the differential equation
𝑑𝑥
+ 𝑥 = 1 + 𝑒 −𝑦 .
𝑑𝑦
𝑑𝑥
Solution: 𝑑𝑦 + 𝑥 = 1 + 𝑒 −𝑦 , 𝐼. 𝐹. = 𝑒 ∫ 1.𝑑𝑦 = 𝑒 𝑦 ,
Solution is 𝑒 𝑦 . 𝑥 = ∫ 𝑒 𝑦 (1 + 𝑒 −𝑦 )𝑑𝑦 = ∫(𝑒 𝑦 + 1)𝑑𝑦
⇒𝑒 𝑦 . 𝑥 = 𝑒 𝑦 + 𝑦 + 𝐶 is the required solution.

14. Question: Find the general solution of the differential equation: 𝑑𝑦 = 𝑡𝑎𝑛2 2𝑥
𝑑𝑥
𝑑𝑦
Solution: The given differential equation is 𝑑𝑥 = 𝑡𝑎𝑛2 2𝑥
On separating variable, we get:
dy = 𝑡𝑎𝑛2 2𝑥 dx
⇒ ∫ 𝑑𝑦 =∫ (sec² 2x − 1)dx
1
⇒y = 2 𝑡𝑎𝑛2𝑥 – 𝑥 + C
This is the required solution.

EXERCISE
1 For the differential equation, find a particular solution satisfying the given
condition
(1 + 𝑦 2 )(1 + 𝑙𝑜𝑔𝑥)𝑑𝑥 + 𝑥𝑑𝑦 = 0 given that y = 1 when x = 1.

(𝑙𝑜𝑔𝑥)2 π
Answer: Log|x| + 2 + tan−1 𝑦 = 4
2 Find the equation of curve passing through the point (-2,3) given that slope of the
2𝑥
tangent to the curve at point (x,y) is 𝑦 2 .

𝑦3
Answer: 3 = 𝑥 2 + 5
3 Solve the differential equation
𝑑𝑦
𝑥 𝑑𝑥 = 𝑦(𝑙𝑜𝑔𝑦 − 𝑙𝑜𝑔𝑥 + 1).

𝑦
Answer: log(𝑥 ) = 𝐶𝑥
4 Find the particular solution of the differential equation
𝑑𝑦 𝜋
+ 2𝑦𝑡𝑎𝑛𝑥 = 𝑠𝑖𝑥, given that y = 1, when x= 3
𝑑𝑥

Answer: y = cosx + C𝑐𝑜𝑠 2 𝑥

5 Find the general solution of differential equation


ydx - (𝑥 + 2𝑦 2 )𝑑𝑦 = 0

Answer: x = 2𝑦 2 + Cy

160
3 MARK QUESTIONS

Q.NO QUESTIONS WITH SOLUTIONS


1 Find the particular solution of differential equation
(1 + 𝑒 2𝑥 )𝑑𝑦 + (1 + 𝑦 2 )𝑒 𝑥 𝑑𝑥 = 0, given that when x=0,y =1

𝑑𝑦 𝑒𝑥
Solution: from differential equation ∫ 1+𝑦 2 = − ∫ 1+𝑒 2𝑥 𝑑𝑥
𝑒𝑥 1 𝑥
𝑙𝑒𝑡 𝑒 =𝑡
For ∫ 1+𝑒 2𝑥 𝑑𝑥 = ∫ 1+𝑡 2 𝑑𝑡 |⇒𝑒 𝑥 𝑑𝑥=𝑑𝑡

= 𝑡𝑎𝑛−1 𝑡 = −𝑡𝑎𝑛−1 𝑒 𝑥
From (i), we get
𝑡𝑎𝑛−1 𝑦 = −𝑡𝑎𝑛−1 𝑒 𝑥 + 𝐶
When x = 0, y= 1
⇒ 𝑡𝑎𝑛−1 1 = −𝑡𝑎𝑛−1 1 + 𝐶
𝜋 𝜋 𝜋
⇒C = + = (𝑒 0 = 1)
4 4 2
Substituting in (ii), we get
𝜋
𝑡𝑎𝑛−1 𝑦 = −𝑡𝑎𝑛−1 𝑒 𝑥 +
𝜋
2
⇒𝑡𝑎𝑛−1 𝑦 + 𝑡𝑎𝑛−1 𝑒 𝑥 = is the required solution
2

2 𝑑𝑦
Find the particular solution of the differential of the equation =1+x+y+xy,
𝑑𝑥
given that y=0, when x=1.
𝑑𝑦
Solution: Consider equation 𝑑𝑥 =1+x+y+xy
𝑑𝑦
⇒ 𝑑𝑥 =1(1+ x) +y(1+x)= (1+x)(1+y)
𝑑𝑦
⇒1+𝑦=(1+x)dx
on integrating both sides
𝑑𝑦
⇒ ∫ 1+𝑦 = ∫(1 + 𝑥) 𝑑𝑥
𝑥2
⇒log |1+y|= x+ 2 + C
Given y=0, when x=1
1 3
⇒log |1+0| =1+ 2+ C ⇒C =− 2
substituting in (i),we get
𝑥2 3
log |1+y|= x+ 2 − 2 is the required solution.
3 Find the particular solution of the differential equation
𝜋
𝑒 𝑥 tanydx+(2 − 𝑒 𝑥 )𝑠𝑒𝑐 2ydy=0, given that y = 4 when x=0.
Solution: consider equation 𝑒 𝑥 tanydx+(2 − 𝑒 𝑥 )𝑠𝑒𝑐 2ydy=0
⇒(2 − 𝑒 𝑥 )𝑠𝑒𝑐 2ydy=−𝑒 𝑥 tanydx
𝑠𝑒𝑐 2 y 𝑒𝑥
⇒ 𝑡𝑎𝑛𝑦 𝑑𝑦 = 𝑒 𝑥 −2 𝑑𝑥
Integrating both sides we get
𝑠𝑒𝑐 2 y 𝑒𝑥
∫ 𝑡𝑎𝑛𝑦 𝑑𝑦 = ∫ 𝑒 𝑥 −2 𝑑𝑥
⇒log|tany| =log(𝑒 𝑥 − 2) + 𝑙𝑜𝑔𝐶
= log|𝐶(𝑒 𝑥 − 2)|
⇒tany =𝐶(𝑒 𝑥 − 2)
𝜋
Given y = 4 when x=0.
𝜋
⇒tan4 =𝐶(𝑒 0 − 2)

161
⇒1= -C ⇒ C =-1
Subsituting in (i), we get
tany = -(𝑒 𝑥 − 2)
or tany =2 − 𝑒 𝑥 is particular solution

4 For the differential equation, find the particular solution satisfying the given
condition
(1 + 𝑠𝑖𝑛2 𝑥)𝑑𝑦 + (1 + 𝑦 2 )𝑐𝑜𝑠𝑥𝑑𝑥= 0, given that when x =
𝜋
,y=0
2
Solution:
𝑑𝑦 −𝑐𝑜𝑠𝑥
∫ = ∫ 𝑑𝑥
(1 + 𝑦 2 ) (1 + 𝑠𝑖𝑛2 𝑥)
On integrating , we get
𝑡𝑎𝑛−1 𝑦 = −𝑡𝑎𝑛−1 (𝑠𝑖𝑛𝑥) + 𝐶
[by substituting sinx=t]
𝜋
When x = 2 , y=0 0= - 4 +C C =
Substituting in (i), we get
𝜋
𝑡𝑎𝑛−1 𝑦 = −𝑡𝑎𝑛−1 (𝑠𝑖𝑛𝑥) + 4 is requires solution

5 Solve the differential equation


𝑑𝑦 𝑦
x𝑑𝑥 = 𝑦 − 𝑥𝑡𝑎𝑛 (𝑥 ).

𝑑𝑦 𝑦 𝑦
Solution: 𝑑𝑥 = 𝑥 − 𝑡𝑎𝑛 (𝑥 ), homogenous equation
𝑑𝑦 𝑑𝑣
Let y =vx ⇒𝑑𝑥 = 𝑣 + 𝑥 𝑑𝑥
𝑑𝑣
𝑣 +𝑥 = 𝑣 − 𝑡𝑎𝑛𝑣
𝑑𝑥
1 1
⇒∫ 𝑡𝑎𝑛𝑣 𝑑𝑣 = − ∫ 𝑥 𝑑𝑥
1
⇒∫ 𝑐𝑜𝑡𝑣 𝑑𝑣 = − ∫ 𝑥 𝑑𝑥
⇒𝑙𝑜𝑔|𝑠𝑖𝑛𝑣| = −𝑙𝑜𝑔|𝑥| + 𝑙𝑜𝑔𝐶
𝐶
⇒𝐿𝑜𝑔|𝑠𝑖𝑛𝑣| = 𝑙𝑜𝑔 |𝑥 |
𝑦
⇒𝑥𝑠𝑖𝑛 𝑥 = 𝐶 is the required solution.

6 Sole the differential equation


𝑥 𝑥
2y𝑒 𝑦 𝑑𝑥 + (𝑦 − 2𝑥𝑒 𝑦 ) 𝑑𝑦 = 0
𝑥 𝑥
Solution: 2y𝑒 𝑦 𝑑𝑥 + (𝑦 − 2𝑥𝑒 𝑦 ) 𝑑𝑦 = 0
𝑥
−(𝑦−2𝑥𝑒 𝑦 )
𝑑𝑥
⇒ 𝑑𝑦 = 𝑥 ….(i)
2y𝑒 𝑦
𝑑𝑥 𝑑𝑣
Let 𝑥 = 𝑣𝑦 ⇒𝑑𝑦 = 𝑣 + 𝑦 𝑑𝑦
Substituting in (i) we get
𝑑𝑣 −(𝑦−2𝑣𝑦𝑒 𝑣 ) −1+2𝑣𝑒 𝑣
𝑣 + 𝑦 𝑑𝑦 = =
2y𝑒 𝑣 2𝑒 𝑣
𝑑𝑣 −1+2𝑣𝑒 𝑣
⇒ y 𝑑𝑦 = −𝑣
2𝑒 𝑣
−1+2𝑣𝑒 𝑣 −2𝑣𝑒 𝑣 1
= = − 2𝑒 𝑣
2𝑒 𝑣

162
1 1 1
⇒𝑒 𝑣 𝑑𝑣 = − 2𝑦 𝑑𝑦 ⇒ ∫ 𝑒 𝑣 𝑑𝑣 = − 2 ∫ 𝑦 𝑑𝑦
1
⇒𝑒 𝑣 = − 2 log|𝑦| + 𝐶
𝑥
1
⇒𝑒 𝑦 = − 2 log|𝑦| + 𝐶 is the required equation.

7 Solve the differential equation


𝑦 𝑑𝑦
y+x.sin(𝑥 ) = 𝑥 𝑑𝑥
𝑑𝑦 𝑦 𝑦
Solution: 𝑑𝑥 = 𝑥 + 𝑠𝑖𝑛 (𝑥 )
𝑑𝑦 𝑑𝑣
Let y =vx ⇒𝑑𝑥 = 𝑣 + 𝑥 𝑑𝑥
𝑑𝑣 1 𝑑𝑥
⇒v+x𝑑𝑥 = 𝑣 + 𝑠𝑖𝑛𝑣 ⇒ ∫ 𝑠𝑖𝑛𝑣 𝑑𝑣 = ∫ 𝑥
𝑑𝑥
⇒∫ 𝑐𝑜𝑠𝑒𝑐𝑣 𝑑𝑣 = ∫ 𝑥
⇒log|cosecv – cotv| = log|x| + logC
⇒cosecv – cotv = xC
𝑦 𝑦
⇒(𝑐𝑜𝑠𝑒𝑐 𝑥 − 𝑐𝑜𝑡 𝑥 ) = 𝑥𝐶 is the required solution.

8 Solve the differential equation


𝑥 𝑥 𝑥
(1 + 𝑒 𝑦 ) 𝑑𝑥 + 𝑒 𝑦 (1 − ) 𝑑𝑦 = 0
𝑦
𝑑𝑥 𝑑𝑣
Solution: let x=vy ⇒𝑑𝑦 = 𝑣 + 𝑦 𝑑𝑦,
𝑑𝑣
We get (1 + 𝑒 𝑣 ) (𝑣 + 𝑦 𝑑𝑦) + 𝑒 𝑣 (1 − 𝑣) = 0
𝑑𝑣
⇒v+v𝑒 𝑣 + 𝑦(1 + 𝑒 𝑣 ) 𝑑𝑦 + 𝑒 𝑣 − 𝑣𝑒 𝑣 = 0
𝑑𝑣 −(𝑒 𝑣 +𝑣) (1+𝑒 𝑣 ) 1
⇒ = ⇒∫ 𝑑𝑣 = − ∫ 𝑑𝑦
𝑑𝑦 𝑦(1+𝑒 𝑣 ) 𝑒 𝑣 +𝑣 𝑦
𝑣
⇒ 𝑙𝑜𝑔|𝑒 + 𝑣| = − log|𝑦| + 𝑙𝑜𝑔𝐶
𝑥
𝑥 𝐶
⇒𝑒 𝑦 + 𝑦 = 𝑦
𝑥
⇒ 𝑦𝑒 𝑦 + 𝑥 = 𝐶 is the required solution.

9 Question: Find the general solution of the differential equation:


𝑑𝑦 5𝑥+3 7
= 2𝑦+7 ,y≠ − 2
𝑑𝑥
𝑑𝑦 5𝑥+3
Solution: = 2𝑦+7.This is a variable separable differential equation.
𝑑𝑥
On separation of variables, we get (2y+7)dy = (5x+3)dx
⇒ ∫(2y + 7) dy = ∫(5x + 3)dx
1 (5𝑥+3)²
⇒ 4(2𝑦 + 7)² + C₁ = + C₂
10
On multiplying throughout by 20, we get:
5(2y+7)² + 20C₁, =2(5x+3)² + 20C₂
⇒ 5(2y+7)² = 2(5x+3)² +20C₂ - 20C₁
⇒5(2y+7)² =2 (5x+3)² + C
This is the required solution.

10 Question: Solve the differential equation: (𝑥 + 𝑦 + 1) 𝑑𝑦 =1.


𝑑𝑥
𝑑𝑦
Solution:The given equation (𝑥 + 𝑦 + 1) 𝑑𝑥 = 1

163
𝑑𝑥
⇒ = 𝑥+𝑦+1
𝑑𝑦
𝑑𝑥
⇒ 𝑑𝑦 − 𝑥 =y +1(L.D.E)
∴ I.F. = 𝑒 ∫ −𝑑𝑦 = 𝑒 −𝑦
Now solution is
x 𝑒 −𝑦 = ∫(𝑦 + 1)𝑒 −𝑦 dy
(𝑦+1)𝑒 −𝑦 𝑒 −𝑦
⇒ x 𝑒 −𝑦 = − ∫ −1 𝑑𝑦
−1
⇒ x 𝑒 = −(𝑦 + 1)𝑒 −𝑦 − 𝑒 −𝑦 + C
−𝑦

⇒x = C 𝑒 𝑦 − (𝑦 + 2)

11 Question: Find the particular solution of the differential equation


𝑑𝑦 𝜋
+ 2ytanx = sinx, given that y=0 when x= 3
𝑑𝑥

Solution:
𝑑𝑦
+(2tanx)y = sinx
𝑑𝑥
IF = 𝑒 ∫ 2𝑡𝑎𝑛𝑥 𝑑𝑥 = 𝑒 2𝑙𝑜𝑔𝑠𝑒𝑐𝑥 = 𝑠𝑒𝑐 2 𝑥
y 𝑠𝑒𝑐 2 𝑥 = ∫ 𝑠𝑒𝑐 2 𝑥. 𝑠𝑖𝑛𝑥𝑑𝑥 + 𝐶
𝑠𝑖𝑛𝑥
=∫ 𝑐𝑜𝑠2 𝑥 𝑑𝑥 + 𝑐
= ∫ 𝑡𝑎𝑛𝑥𝑠𝑒𝑐𝑥𝑑𝑥 + 𝐶
y𝑠𝑒𝑐 2 𝑥= secx +C

12 Question: Find the general solution of the differential equation 𝑑𝑦


− 𝑦 = 𝑠𝑖𝑛𝑥
𝑑𝑥

Solution: Given differential equation is


⇒ IF = 𝑒 ∫(−1)𝑑𝑥 = 𝑒 −𝑥
∴Solution is
y. 𝑒 −𝑥 = ∫ 𝑠𝑖𝑛𝑥 . 𝑒 −𝑥 𝑑𝑥 = I1
⇒ I1 = −𝑠𝑖𝑛𝑥. 𝑒 −𝑥 + ∫ 𝑐𝑜𝑠𝑥𝑒 −𝑥 𝑑𝑥
= −𝑠𝑖𝑛𝑥. 𝑒 −𝑥 + [−𝑐𝑜𝑠𝑥𝑒 −𝑥 − ∫ 𝑠𝑖𝑛𝑥𝑒 −𝑥 𝑑𝑥]
1
⇒I1 = 2 [−𝑠𝑖𝑛𝑥 − 𝑐𝑜𝑠𝑥]𝑒 −𝑥
1
∴Solution is y. 𝑒 −𝑥 = 2 (-sinx-cos x) 𝑒 −𝑥 +c
1
Or y =− (sin x+ cos x) + c. 𝑒 −𝑥
2

EXERCISE
1 Solve the differential equation
𝑑𝑦 𝑑𝑦
(𝑦 − 𝑥 𝑑𝑥 ) = 𝑎 (𝑦 2 + 𝑑𝑥 ) .

Answer: y = C(x + a)(1 – ay)

2 For the differential equation, finf a particular solution satisfying the given
condition
𝑑𝑦
x(𝑥 2 − 1) 𝑑𝑥 = 1; 𝑦 = 0 when x = 2

1 1 1 3
Answer: y = 2 𝑙𝑜𝑔 |1 − 𝑥 2 | − 2 𝑙𝑜𝑔 4

164
3 Solve the differential equation
xdy -ydx = √𝑥 2 +𝑦 2 𝑑𝑥.

Answer: y + √𝑥 2 +𝑦 2 = C𝑥 2
𝑦
4 Show that the differential equation (𝑥𝑒 𝑥 + 𝑦) 𝑑𝑥 = 𝑥𝑑𝑦 is homogenous. Find the
particular solution of this differential equation, given that x = 1 when y = 1.
𝑦
1
Answer: log|x| + 𝑒 𝑥 = 𝑒

5 Find the particular solution of the differential equation


𝜋
dy = cosx(2 – ycosecx)dx, given that y = 2, when x =2 .

1 3
Answer: sinx.y = - 2 𝑐𝑜𝑠2𝑥 + 2

5 MARK QUESTIONS

Q.NO QUESTIONS WITH SOLUTIONS


1 𝑑𝑦
Question: Find the particular solution of the differential equation: 𝑑𝑥 = 1 +
𝑥 2 + 𝑦 2 + 𝑥 2 𝑦 2 given 𝑦 = 1 , 𝑤ℎ𝑒𝑛 𝑥 = 1

Solution: The given differential equation is


𝑑𝑦
= 1 + 𝑥2 + 𝑦2 + 𝑥2𝑦2
𝑑𝑥
𝑑𝑦
⇒𝑑𝑥 = (1 + 𝑥 2 ) + 𝑦 2 (1 + 𝑥 2 )
𝑑𝑦
⇒𝑑𝑥 =(1 + 𝑥 2 )(1 + 𝑦 2 )
It is the variable separable differential equation
𝑑𝑦
⇒∫ (1+𝑦 2 ) = ∫(1 + 𝑥 2 )𝑑𝑥
𝑥3 𝜋 4
⇒𝑡𝑎𝑛−1 𝑦 = 3 + x + C ⇒C=4−3
The particular solution of the given variable separable differential equation is
𝑥3 𝜋 4
𝑡𝑎𝑛−1 𝑦 = +𝑥+4−3
3

2 𝑑𝑦 2𝑙𝑜𝑔𝑥
Question: Solve: (xlog x)(𝑑𝑥 ) + y = 𝑥

𝑑𝑦 2𝑙𝑜𝑔𝑥
Solution: Give differential equation: (x log x)( 𝑑𝑥 ) + y = 𝑥
𝑑𝑦 1 2
⇒ 𝑑𝑥 + (𝑥𝑙𝑜𝑔𝑥) 𝑦 = 𝑥 2
𝑑𝑦
This is a linear equation of the form: 𝑑𝑥 + f(x)y = g(x)
1
∫ 𝑑𝑥
I.F. = 𝑒 𝑥𝑙𝑜𝑔𝑥 = 𝑒 log (logx) = log x
Now the solution of the differential equation will be:
y(I.F.) = ∫ g(x)(I.F.)dx
2logxdx
⇒ y(I.F) = ∫ 𝑥 2 = ∫(logx)( 2𝑥 −2 ) dx

165
Now on integrating RHS by parts, we get
2𝑥 −1 1 2𝑥 −1
ylogx= log x( ) - ∫𝑥( ) 𝑑𝑥
−1 −1
−2𝑙𝑜𝑔𝑥 −2
⇒ylog.x = + ∫ 2𝑥 dx
𝑥
−2𝑙𝑜𝑔𝑥 𝑥 −1
⇒ylogx = + 2 +𝐶
𝑥 −1
−2𝑙𝑜𝑔𝑥 2
⇒ylogx = −𝑥+𝐶
𝑥

3 Question: Find the particular solution of the differential equation


𝑒 𝑥 𝑡𝑎𝑛𝑦 𝑑𝑥 + (2 − 𝑒 𝑥 )𝑠𝑒𝑐 2 𝑦𝑑𝑦 = 0
𝜋
given that y = 4 when x = 0

Solution: 𝑒 𝑥 𝑡𝑎𝑛𝑦𝑑𝑥 = (𝑒 𝑥 − 2)𝑠𝑒𝑐 −2 𝑦 𝑑𝑦 = 0


𝑠𝑒𝑐 2 𝑦 𝑒𝑥
⇒∫ 𝑑𝑦 = ∫ (𝑒 𝑥 −2) 𝑑𝑥
𝑡𝑎𝑛𝑦
Put 𝑡𝑎𝑛𝑦 = 𝑡 ⇒ 𝑠𝑒𝑐 2 𝑥𝑦 𝑑𝑦 = 𝑑𝑡
𝑑𝑡 𝑒𝑥
⇒∫ = ∫ (𝑒 𝑥 −2) 𝑑𝑥
𝑡
Again put (𝑒 𝑥 − 2) = 𝑢 ⇒ 𝑒 𝑥 𝑑𝑥 = 𝑑𝑢
⇒ 𝑙𝑜𝑔𝑡 = 𝑙𝑜𝑔 𝑢 + 𝑙𝑜𝑔𝐶
⇒𝑙𝑜𝑔(𝑡𝑎𝑛𝑦) = 𝑙𝑜𝑔[(𝑒 𝑥 − 2)𝐶]
⇒tany =C(𝑒 𝑥 − 2)
𝜋
Put y = 4
𝜋
⇒tan4 =C(1-2)
⇒ C = -1
⇒tany = −(𝑒 𝑥 − 2)
4 Question: Solve the differential equation
(𝑡𝑎𝑛𝑥 −1 𝑥 − 𝑦 )𝑑𝑥 = ( 1+ 𝑥 2 )𝑑𝑦

Solution: Given differential equation can be written as


𝑑𝑦
(1+x²) 𝑑𝑥 + 𝑦 = 𝑡𝑎𝑛−1 x
𝑑𝑦 1 𝑡𝑎𝑛−1 x
⇒ 𝑑𝑥 + 1+𝑥 2 y = 1+x²
1
∫(1+x²)𝑑𝑥 𝑡𝑎𝑛−1 x
⇒I.F. = 𝑒 =𝑒
∴Solution is
−1 −1 1
y. 𝑒 𝑡𝑎𝑛 x = ∫ 𝑡𝑎𝑛−1 x. 𝑒 𝑡𝑎𝑛 x . 1+x2 𝑑𝑥
−1 −1
⇒ y. 𝑒 𝑡𝑎𝑛 x = 𝑒 𝑡𝑎𝑛 x . (𝑡𝑎𝑛−1x -1 ) + C
−1
Or y = (𝑡𝑎𝑛−1 x -1 ) + C . 𝑒 −𝑡𝑎𝑛 x

5 Question: Find the particular solution of the differential equation


𝑥 𝑥
2y 𝑒 𝑦 𝑑𝑥 + (𝑦 − 2𝑥𝑒 𝑦 ) 𝑑𝑦 = 0 given that 𝑥 = 0 𝑤ℎ𝑒𝑛 𝑦 = 1.

Solution: the given differential equation


𝑥 𝑥
2y 𝑒 𝑦 𝑑𝑥 + (𝑦 − 2𝑥𝑒 𝑦 ) 𝑑𝑦 = 0
On dividing Nr and Dr of RHS by y,we get
𝑥
𝑥 ( )
−[1−2( )𝑒 𝑦 ]
𝑑𝑥 𝑦
= 𝑥
𝑑𝑦 ( )
2𝑒 𝑦

166
It is homogenous differential equation of second type
𝑑𝑥 𝑑𝑣
∴Put x =vy and 𝑑𝑦 = 𝑣 + 𝑦 (𝑑𝑦)
𝑑𝑣 2𝑣𝑒 𝑣 −1
v + y(𝑑𝑦) = 2𝑒 𝑣
𝑑𝑣 2𝑣𝑒 𝑣 −1
⇒ y(𝑑𝑦) = −𝑣
2𝑒 𝑣
𝑑𝑣 1
⇒ y(𝑑𝑦) = − 2𝑒 𝑣
𝑑𝑦
⇒2 ∫ 𝑒 𝑣 dv = ∫ 𝑦
⇒ 2 𝑒 𝑣 = −𝑙𝑜𝑔|𝑦| + 𝐶
𝑥
⇒ 2 𝑒 𝑦 = −𝑙𝑜𝑔|𝑦| + 𝐶
Put x = 0 and y =1
2𝑒 0 = −log 1 + 𝐶
⇒ 2 = 0+ C ⇒C =2
∴Particular solution of given homogenous differential equation is
𝑥
⇒ 2 𝑒 𝑦 = −𝑙𝑜𝑔|𝑦| + 2

6 𝑑𝑦
Question: Solve: (1 + 𝑥 2 ) 𝑑𝑥 + 2xy = 4𝑥 2 subject to the initial condition y(0) =
0

Solution: The given differential equation:


𝑑𝑦
(1 + 𝑥 2 ) 𝑑𝑥 + 2xy = 4𝑥 2
𝑑𝑦 2𝑥 4𝑥 2
⇒ 𝑑𝑥 + (𝑥 2 +1) 𝑦= 𝑥 2 +1
It a linear differential equation of the form
𝑑𝑦
+ 𝑓(𝑥)𝑦 = 𝑔(𝑥)
𝑑𝑥
2𝑥 4𝑥 2
Where f(x) = 𝑥 2 +1 and g(x) = 𝑥 2 +1
2𝑥
)𝑑𝑥 2
I.F. = 𝑒 ∫ 𝑓(𝑥)𝑑𝑥 = 𝑒 ∫(𝑥2+1 = 𝑒 log(𝑥 +1)
⇒ I.F. = 𝑥 2 + 1
∴Solution of the given linear differential equation
y(I.F.) = ∫ 𝑔(𝑥)(I. F. )dx
4𝑥 2
y(𝑥 2 + 1) = ∫ 4𝑥 2 𝑑𝑥 = +𝐶
3
4𝑥 2
⇒ y(𝑥 2 + 1) = 3 + C
To find the value of C, put x=0 and y =0 in the above equation
4𝑋0
0(0+1) = 3 + 𝐶 ⇒ C = 0
4𝑥 2
⇒ y(𝑥 2 + 1) = 3

7 Question: Find the particular solution of the differential equation


𝑦
𝑒 𝑥 √1 − 𝑦 2 𝑑𝑥 + (𝑥 ) 𝑑𝑦 = 0 , 𝑥 = 0, 𝑦 = 0.

Solution: The given differential equation is:


𝑦
𝑒 𝑥 √1 − 𝑦 2 𝑑𝑥 + (𝑥 ) 𝑑𝑦 = 0
𝑦
⇒ (𝑥 ) 𝑑𝑦 = − 𝑒 𝑥 √1 − 𝑦 2 𝑑𝑥
𝑦
⇒( ) 𝑑𝑦 = −𝑥𝑒 𝑥 dx
√1−𝑦 2

167
𝑦
⇒∫ ( ) 𝑑𝑦 = − ∫ 𝑥𝑒 𝑥 𝑑𝑥
√1−𝑦 2
1 −2𝑦𝑑𝑦
⇒− ∫ = − ∫ 𝑥𝑒 𝑥 dx
2 √1−𝑦 2
⇒√1 − 𝑦 2 = 𝑥𝑒 𝑥 − 𝑒 𝑥 + 𝐶
To find the particular solution put 𝑥 = 0 𝑎𝑛𝑑 𝑦 = 0 in the above equation
√1 − 0 = 0𝑒 0 − 𝑒 0 + 𝐶 ⇒C=2
⇒the particular solution is
√1 − 𝑦 2 = 𝑥𝑒 𝑥 − 𝑒 𝑥 + 2

8 Question: Find the particular solution of the differential equation


(𝑥 2 − 𝑦𝑥 2 )𝑑𝑦 + (𝑦 2 + 𝑥 2 𝑦 2 )𝑑𝑥 = 0 ,given y =1, when x = 1

Solution: The given differential equation is


(𝑥 2 − 𝑦𝑥 2 )𝑑𝑦 + (𝑦 2 + 𝑥 2 𝑦 2 )𝑑𝑥 = 0
⇒𝑥 2 (𝑦 − 1)𝑑𝑦 = 𝑦 2 (𝑥 2 + 1)𝑑𝑥
(𝑦−1) (𝑥 2 +1)
⇒∫ 𝑑𝑦 = 𝑑𝑥
𝑦2 𝑥2
1 1 1
⇒∫ [𝑦 − 𝑦 2 ] 𝑑𝑦 = ∫ [1 + 𝑥 2 ] 𝑑𝑥
1 1
⇒logy + 𝑦 = 𝑥 − 𝑥 + 𝐶
To find the particular solution,
Put x = 1 and y = 1 in equation
1 1
log(1) + 1 = 1 − 1 + 𝐶
⇒ 0 +1 = C ⇒C =1
The particular solution of the given differential equation is
1 1
logy + = 𝑥 − + 1
𝑦 𝑥

9 𝑑𝑦
Question: Find the particular solution of the differential equation: log (𝑑𝑥 ) =

3𝑥 + 4𝑦,
given y =0, when x = 0

Solution: The given differential equation is


𝑑𝑦
log(𝑑𝑥 ) = 3𝑥 + 4𝑦
𝑑𝑦
⇒ = 𝑒 3𝑥+4𝑦 = 𝑒 3𝑥 . 𝑒 4𝑦
𝑑𝑥

⇒∫ 𝑒 −4𝑦 𝑑𝑦 = ∫ 𝑒 3𝑥 𝑑𝑦
𝑒 −4𝑦 𝑒 3𝑥
⇒ = +𝐶
−4 3
−1 𝑒 3𝑥
⇒ 4𝑒 4𝑦 = +𝐶
3
1 𝑒 3𝑥
⇒4𝑒 4𝑦 + =𝐶
3

To find the value of C ,put x = 0 and y = 0 in equation

168
1 𝑒0
+ =𝐶
4𝑒 0 3
1 1
⇒ + =𝐶
4 3
7
⇒ C =12
1 𝑒 3𝑥 7
Hence 4𝑒 4𝑦 + = 12 is the particular solution of given variable separable
3

equation.
10 Question: For the differential equation xy (𝑑𝑦) = (x+2)(y+2), find the solution
𝑑𝑥

curve passing through the point (1,-1)

𝑑𝑦
Solution: The given differential equation xy (𝑑𝑥 ) = (x+2)(y+2)

It is a variable separable differential equation.


𝑦 𝑥+2
∴ ∫ (𝑦+2) 𝑑𝑦 = ∫ ( ) 𝑑𝑥
𝑥
(𝑦+2−2) 𝑥+2
⇒ ∫( ) 𝑑𝑦 = ∫ ( ) 𝑑𝑥
𝑦+2 𝑥
2
⇒ ∫ (1 − 𝑦+2) 𝑑𝑦 = 𝑥 + 2𝑙𝑜𝑔𝑥 + 𝐶

⇒ y -2log(y+2) = x+2log(x) + C

This curve passing through a point (1, -1)


Put x =1 and y = −1 in it.
-1 -2log (1) = 1 + 2log(1) + C
⇒ -1 – 0 =1 + 2 × 0 + C
⇒ -1 = 1+ C ⇒ 𝐶 = −2
On putting 𝐶 = −2 in the curve equation:
𝑦 – 2𝑙𝑜𝑔|𝑦 + 2| = 𝑥 + 2𝑙𝑜𝑔|𝑥| − 2

EXERCISE

1 Find the general solution of differential equation


𝑑𝑦
(𝑥 3 + 𝑥 2 + 𝑥 + 1) = 2𝑥 2 + 𝑥
𝑑𝑥

1 3 1
Answer: y = 2 log|𝑥 + 1| + 4 𝑙𝑜𝑔|𝑥 2 + 1| − 2 tan−1 𝑥 + 𝐶

2 𝑑𝑦 𝑦2
Show that the differential equation = 𝑥𝑦− 𝑥 2 is homogenous and also solve it.
𝑑𝑥

169
𝑦
Answer: 𝑥 − log|𝑦| = 𝐶

3 Find the particular solution of the differential equation


𝑑𝑦 𝜋
tan 𝑥 . 𝑑𝑥 = 2𝑥𝑡𝑎𝑛𝑥 + 𝑥 2 − 𝑦; (tan 𝑥 ≠ 0) given that y = 0 when x = 2 .

𝜋2
Answer: 𝑠𝑖𝑛𝑥. 𝑦 = 𝑥 2 𝑠𝑖𝑛𝑥 − 4

CASE BASED QUESTIONS


Q.NO QUESTIONS AND SOLUTIONS
1 Polio drops are delivered to 50K children in a district. The rate at which polio
drops are given is directly proportional to the number of children who have not
been administered the drops. By the end of 2nd week half the children have
been given the polio drops. How many will have been given the drops by the
end of 3rd week can be estimated using the solution to the differential equation
𝑑𝑦
= 𝑘(50 − 𝑦)where x denotes the number of weeks and y the number of
𝑑𝑥
children who have been
given the drops.

(a) State the order of the above given differential equation.

(b) Which method of solving a differential equation can be used to solve


𝑑𝑦
= 𝑘(50 − 𝑦) ?
𝑑𝑥

𝑑𝑦
(c) the solution of the differential equation = 𝑘(50 − 𝑦) is given by,
𝑑𝑥

(i) 𝑙𝑜𝑔 |50 – 𝑦| = 𝑘𝑥 + 𝐶 (ii) − 𝑙𝑜𝑔 |50 − 𝑦| = 𝑘𝑥 + 𝐶


(iii) 𝑙𝑜𝑔 |50 − 𝑦| = 𝑙𝑜𝑔(|𝑘𝑥|) + 𝐶 (iv) 50 − 𝑦 = 𝑘𝑥 + 𝐶

(d) The value of c in the particular solution given that 𝑦(0) = 0 𝑎𝑛𝑑 𝑘 =
0.049 is

1
(i) log 50 (ii) log(50) (iii) 50 (iv) - 50

Solution:
(a) Order is 1
(b) (i), Variable separable method
(c) (ii), −𝑙𝑜𝑔|50 – 𝑦| = 𝑘𝑥 + 𝐶
1
(d) (ii), 𝑙𝑜𝑔 50

2 An equation involving variables as well as derivative of the dependent variable


with respect to only one independent variable is called an ordinary differential
equation.
𝑑𝑦 𝑑2 𝑦
e.g. + 𝑑𝑥 2 -2=0
𝑑𝑥

170
From any given relation between the dependent and independent variables, a
differential equation can be formed by differentiating it with respect to the
independent variable and eliminating arbitrary constants involved.

(a) The degree of the differential equation


𝑑𝑦 4 𝑑2 𝑦
(𝑑𝑥 ) + 3𝑦 𝑑𝑥 2 = 0 is

(i) 1 (ii) 3 (iii) 2 (iv) 4

(b) The order of differential equation


(i) 1 (ii) 2 (iii)4 (iv)3

(c) The number of arbitrary constants in general solution of differential equation


of third order is
(i) 0 (ii)2 (iii)3 (iv)4

(d) The degree of differential equation


2
𝑑2 𝑦 𝑑𝑦 4
𝑥 3 ( 𝑑𝑥 2 ) + 𝑥 (𝑑𝑥 ) = 0 is
(i) 1 (ii) 0 (iii) 4 (iv) Is not defined

𝑑2 𝑦
Solution: (a)-(i), The highest order derivative is 𝑑𝑥 2 whose
degree is one. So, degree of differential
equation is 1.

(b) - (iv), The highest order derivative present in the differential equation is 𝑦 ,,, .
Therefore, its order is 3.

(c) - (iii), We know that the number of arbitrary constant in the general solution
of a differential equation of order n is equal to its order.
Therefore, the number of constants in the general equation of the third-order
differential equation is three.

(d) - (iv), as differential equation can't be written as polynomial of derivative.

3 A Veterinary doctor was examining a sick cat brought by a pet lover. When it
was brought to the hospital, it was already dead. The pet lover wanted to find its
time of death. He took the temperature of the cat at 11.30 pm which was 94.6°F.
He took the temperature again after one hour; the temperature was lower than
the first observation. It was 93.4°F.The room in which the cat was put is always
at 70°F. The normal temperature of the cat is taken as 98.6°F when it was alive.
The doctor estimated the time of death using Newton law of cooling which is
𝑑𝑇
governed by the differential equation: 𝑑𝑡 ∝(T-70), where 70°F is the room
temperature and T is the temperature of the object at time t.
Substituting the two different observations of T and t made, in the solution of
𝑑𝑇
the differential equation 𝑑𝑡 = 𝑘(𝑇 − 70) where k is a constant of proportion,
time of death is calculated

a) State the degree of the above given differential equation.

171
(b) Write method of solving a differential equation helped in calculation of the
time of death?

(c) If the temperature was measured 2 hours after 11.30 pm, will the time of
death change? (Yes/No)

(d) Find the solution of the differential equation


𝑑𝑇
= 𝑘(𝑇 − 70)
𝑑𝑡

(e) If t = 0 when T is 72, then find the value of c

Solution: (a) Degree is 1, (b) Variable separable method, (c) No,


(d) 𝑙𝑜𝑔|𝑇 – 70| = 𝑘𝑡 + 𝐶, (𝑒) 𝑙𝑜𝑔2

4 A function of the form y = ∅(x) + C which satisfies given differential equation,


is called the solution of the differential equation. The solution which contains as
many arbitrary constants as the order of the differential equation, is called the
general solution of the differential equation.

A solution obtained by giving particular values to arbitrary constants in the


general solution of a differential equation, is called the particular solution.

(a) 𝑦 2 = 4ax is solution of differential equation

(i) y𝑦 ′ = xy𝑦 " + x𝑦 ′2 (ii) x𝑦 ′ = x𝑦 ′ + x𝑦 "


(iii)2x𝑦 ′ - y = 0 (iv) x𝑦 " + 𝑦 ′ = xy

(b)The solution of the differential equation 𝑠𝑒𝑐 2 𝑥. 𝑡𝑎𝑛𝑦𝑑𝑥 + 𝑠𝑒𝑐 2 𝑦𝑡𝑎𝑛𝑥𝑑𝑦 =


0 is

(i) 𝑡𝑎𝑛𝑥 = 𝐶 (ii) 𝑡𝑎𝑛𝑥. 𝑡𝑎𝑛𝑦 = 𝐶


𝑡𝑎𝑛𝑥
(iii) 𝑡𝑎𝑛𝑥 + 𝑡𝑎𝑛𝑦 = 𝐶 (iv) 𝑡𝑎𝑛𝑦 = C

(c) The genral solution of differential equation


tan−1 𝑦
(i) tan−1 𝑥 = 𝐶
(ii) tan−1 𝑥 . tan−1 𝑦 = 𝐶
tan−1 𝑥
(iii)tan−1 𝑦 = 𝐶
(iv) tan−1 𝑦 = tan−1 𝑥 + 𝐶

(d) The solution of differential equation


𝑒 𝑥 tan 𝑦 𝑑𝑥 + (1 + 𝑒 𝑥 )𝑠𝑒𝑐 2 𝑦𝑑𝑦 = 0

(i) tan 𝑥 = 𝐶(1 − 𝑒 𝑦 )


(ii) tan 𝑦 = 𝐶(1 − 𝑒 𝑦 )
(iii)tan 𝑦 = 𝐶(1 − 𝑒 𝑥 )
(iv) tan 𝑦 = 𝐶(1 + 𝑒 𝑥 )−1

Solution: (a) - (iii) ,(b) – (ii), (c) – (iii), (d) – (iv)

172
5 Friends are revising differential equations
𝑑𝑦 𝑦2
= 𝑥𝑦−𝑥 2 , answered. few questions are required to be
𝑑𝑥

(i) Which method is used to solve the given differential equation?


(ii) What is the degree of the given differential equation?
(iii) Find the general solution of the given differential equation.

Solution: (i) Method of homogenous differential equation,


𝑦
(ii) degree =1 ,(iii) 𝑥 − log|𝑦| = 𝐶

6 Two friend together are preparing for board examination and they were revising
differential equations. They were asking each other one by one concepts related
to differential equations, then one of the friends asked how to solve the
differential equation, (2x-5y+3)dx + (4x-10y-9)dy = 0. :
During conversation they came across the following questions

(i)Differential equations can be solved using

(a) seperating the variables


(b) method for solving homogeneous equations
(c) method for solving linear differential equations of first order
(d) using substitution method

(ii) We can start with

(a) separating variables


(b) substituting y = vx
(c) finding integrating factor
(d) substituting 2x - 5y = t
2𝑥−9
(iii) ∫ 3𝑥−1 𝑑𝑥 is equal to

2 25
(a) − 𝑙𝑜𝑔|3𝑥 − 1| + 𝐶
3 9
2 25
(b) 3 𝑥 − 𝑙𝑜𝑔|3𝑥 − 1| + 𝐶
9
2 25
(c) − 𝑙𝑜𝑔|3𝑥 − 1| + 𝐶
3 3
2 25
(d) 3 𝑥 − 𝑙𝑜𝑔|3𝑥 − 1| + 𝐶
3

(iv) ∫ 3 . 𝑑𝑥 is equal to

(a) 3 +C
(b) Log3 +C
(c) 3x +C
32
(d) +𝐶
2

(v) solution of differential equation is


2 25
(a) 9 (2𝑥 − 5𝑦) − 21 𝑙𝑜𝑔|18𝑥 − 45𝑦 − 3| = 𝑥 + 𝐶
(b) (2𝑥 − 5𝑦) − 𝑙𝑜𝑔|18𝑥 − 45𝑦 − 3| = 𝑥 + 𝐶

173
2
(c) 9 (2𝑥 − 5𝑦) − 𝑙𝑜𝑔|18𝑥 − 45𝑦 − 3| = 𝑥 + 𝐶
2 25
(d) 9 (2𝑥 − 5𝑦) − 27 𝑙𝑜𝑔|18𝑥 − 45𝑦 − 3| = 𝐶

Solution: (i)-(d), (ii) – (d), (iii) – (b), (iv) – (c), (v) – (a)
7 As during COVID-19 board examinations have been postpond, so friends are
revising the syllabus again and again and the topic in question is differential
equations and given differential equation is (1 + 𝑦 2 )dx = (tan−1 𝑦 − 𝑥)dy with
the above information answer the following:

(i) What is the degree of the differential equation?

(a) not defined


(b) 0
(c) 1
(d) 2

(ii) The differential equation can be solved using the method of solution by

(a) seperating the variables


𝑑𝑦
(b) using method for linear differential equation of type 𝑑𝑥 + P(x).y = Q(x)
𝑑𝑥
(c) using method for linear differential equation of the type 𝑑𝑦 + P(y).x = Q(y)
(d) using method for homogeneous equation

(iii) To solve the differential equation, it can be written as


𝑑𝑦 1+𝑦 2
(a) = tan−1 𝑦−𝑥
𝑑𝑥
𝑑𝑥 tan−1 𝑦−𝑥
(b) =
𝑑𝑦 1+𝑦 2
𝑑𝑥 1
(c) + 1+𝑦 2 𝑥 = tan−1 𝑦
𝑑𝑦
𝑑𝑦 𝑑𝑥
(d) = tan−1 𝑦−𝑥
1+𝑦 2

(iv) For the solution the differential equation integrating factor is

(a) tan−1 𝑦
−1
(b) 𝑒 tan 𝑦
1
(c) 𝑒 1+𝑦2
−1
(d) 𝑒 −tan 𝑦

(v) Solution is
−1
(a) x = tan−1 𝑦 − 1 + 𝐶. 𝑒 −tan 𝑦
−1
(b) y = tan−1 𝑥 − 1 + 𝐶. 𝑒 −tan 𝑥
(c) tan−1 𝑦 = 𝑙𝑜𝑔(tan−1 𝑦 − 𝑥) + 𝐶
(d) 𝑦 = tan−1 𝑥 + 𝐶

Solution: (i) – (c), (ii) –(c), (iii) – (b), (iv) – (b), (v) – (a)

174
CHAPTER: VECTORS
SYLLABUS: Vectors and scalars, magnitude and direction of a vector. Direction cosines
and direction ratios of a vector. Types of vectors (equal, unit, zero, parallel and collinear
vectors), position vector of a point, negative of a vector, components of a vector, addition of
vectors, multiplication of a vector by a scalar, position vector of a point dividing a line
segment in a given ratio. Definition, Geometrical Interpretation, properties and application
of scalar (dot) product of vectors, vector (cross) product of vectors.

Definitions and Formulae:

Scalar: those physical quantities which have only magnitude are called scalar

for example: time period, distance, work done, area, volume, mass, density, speed, temperature,
money voltage, resistance etc

Vector: Those physical quantities which are defined by both magnitude and direction are called
vector

For example: force, velocity, acceleration, displacement, weight, momentum, electric field
intensity etc

Zero vector: a vector whose initial and terminal points are same is called a zero vector or null
vector and is denoted by ⃗O

Co initial vectors: two or more vectors having the same initial point is called coinitial vectors

Collinear vectors: two or more vectors are said to be collinear if they are parallel to the same
line irrespective of their magnitudes

Equal vectors: two vectors 𝑎and𝑏⃗ are said to be equal, if they have the same magnitude and
direction regardless of the positions of their initial points and written as 𝑎 = 𝑏⃗

Negative of a vector: a vector whose magnitude is the same as that of a given vector but
direction is opposite to that of it is called negative of the given vector.

⃗⃗⃗⃗⃗
𝐵𝐴 is the negative of ⃗⃗⃗⃗⃗
𝐴𝐵

Important Properties

1. For any two vectors 𝑎 and 𝑏⃗ , 𝑎 + 𝑏⃗ = 𝑏⃗ + 𝑎 ( commutative property)


2. For any three vectors 𝑎 , 𝑏⃗ and𝑐 , (𝑎 + ⃗⃗⃗⃗
𝑏) + 𝑐 = 𝑎 + (𝑏⃗ + 𝑐 ) (associative property)
3. Triangle law of vector addition: A

B C

175
In the given triangle ⃗⃗⃗⃗⃗
𝐴𝐶 = ⃗⃗⃗⃗⃗ ⃗⃗⃗⃗⃗
𝐴𝐵 +𝐵𝐶

4. If 𝑟 = x𝑖̂ + y𝑗̂ +z𝑘̂ then the magnitude of 𝑟 is ⃒𝑟⃒ =√𝑥 2 + 𝑦 2 + 𝑧 2

5. Let 𝑎 be a given vector and λ a scalar. Then the product of the vector 𝑎 by the scalar λ is
denoted as λ𝑎 and ⃒λ𝑎⃒ = ⃒ λ⃒ ⃒𝑎⃒
𝑎⃗
6. The unit vector of 𝑎 is 𝑎̂ =
⃒𝑎⃒⃗
7. Direction ratios if 𝑟 = x𝑖̂ + y𝑗̂ +z𝑘̂ then 𝑥 , 𝑦 , 𝑧 are the direction ratios of vector 𝑟
8. Direction cosines: the direction cosines of 𝑟 are denoted by l, m, n (cosα, cosβ, cosγ)
𝑥 𝑦 𝑧
l= , m= ,𝑛 =
⃒ 𝑟⃒ ⃒ 𝑟⃒ ⃒ 𝑟⃒
9. The product of two vectors is commutative 𝑎. 𝑏⃗ = 𝑏⃗. 𝑎
10. If l ,m ,n are the direction cosines then l2+m2+n2 = 1
11. 𝑖̂. 𝑖̂ = 𝑗̂. 𝑗̂ = 𝑘̂. 𝑘̂ = 1
12. 𝑖̂ × 𝑗̂ = 𝑘̂ , 𝑗̂ × 𝑘̂ = 𝑖̂ , 𝑘̂ × 𝑖̂ =𝑗̂
13. 𝑗̂ × 𝑖̂ = −𝑘̂ , 𝑘̂ × 𝑗̂ = −𝑖̂ = 𝑖̂ × 𝑘̂ =−𝑗̂

14. 𝑖̂ × 𝑖̂ = 𝑗̂ × 𝑗̂ = 𝑘̂ × 𝑘̂ = 0

15. 𝑎. 𝑏⃗= |𝑎
⃗⃗⃗ ||𝑏⃗| 𝒄𝒐𝒔Ѳ
16. 𝑎×𝑏⃗= |𝑎 ⃗⃗⃗ ||𝑏⃗| 𝒔𝒊𝒏Ѳ𝑛̂ , where 𝑛̂ is a unit vector perpendicular to the plane containing
𝑎 𝑎𝑛𝑑 𝑏⃗ ,
such that 𝑎 , 𝑏⃗, 𝑛̂ form right handed system of coordinate axes

MULTIPLE CHOICE QUESTIONS

S.NO QUESTIONS AND ANSWERS


1 Which of the following is correct?
a) 𝑎×𝑏⃗ = 𝑏⃗×𝑎 ⃗⃗⃗ × ⃗⃗⃗𝑎 = |𝑎|2
b) 𝑎
c) 𝑎
⃗⃗⃗ × ⃗⃗⃗𝑎 = |𝑎| d) 𝑎×𝑏⃗ = −𝑏⃗×𝑎
Solution: d) 𝑎×𝑏⃗ = -𝑏⃗×𝑎
2 Two vectors 𝑎 and⃗⃗𝑏 are parallel if
a) 𝑎×𝑏⃗ = 0 b) 𝑎.𝑏⃗ = 0
c) |𝑎×𝑏⃗| = |𝑎 ⃗⃗⃗ ||𝑏⃗| d) None of the above
Solution: a)
𝑎×𝑏⃗= |𝑎 ⃗⃗⃗ ||𝑏⃗| 𝒔𝒊𝒏Ѳ
if Ѳ = 0 then,
𝑎×𝑏⃗= |𝑎 ⃗⃗⃗ ||𝑏⃗| sin0
⸫ 𝑎×𝑏⃗ = 0
3 The unit vector in the direction of 𝑎= 3𝑖̂ - 2𝑗̂ + 6𝑘̂
1 1
a) 49 (3𝑖̂ - 2𝑗̂ + 6𝑘̂) b) 7 (3𝑖̂ - 2𝑗̂ + 6𝑘̂)
c) 7(3𝑖̂ - 2𝑗̂ + 6𝑘̂) d)49(3𝑖̂ - 2𝑗̂ + 6𝑘̂)

Solution: b)

176
if 𝑎= 3𝑖̂ - 2𝑗̂ + 6𝑘̂
𝑎⃗ ̂
3𝑖̂ − 2𝑗̂ + 6𝑘
𝑡ℎ𝑒𝑛 𝑎̂ = =
⃒𝑎⃒⃗ √32 +(−2)2 +62
1
= 7 (3𝑖̂ - 2𝑗̂ + 6𝑘̂)
4 If 𝑎
⃗⃗⃗ is unit vector and if (𝑥 ⃗⃗⃗ - 𝑎
⃗⃗⃗ ). (𝑥
⃗⃗⃗ +𝑎
⃗⃗⃗ ) = 15 then |𝑥| is
a) 16 b) 6 c) 4 d)1

Solution: c)
⃗⃗⃗ - 𝑎
(𝑥 ⃗⃗⃗ ). (𝑥⃗⃗⃗ +𝑎
⃗⃗⃗ ) = 15
⸫ |𝑥 2
⃗⃗⃗ | - |𝑎 2
⃗⃗⃗ | =15
⃗⃗⃗ | – 1 =15 { since 𝑎
|𝑥 2
⃗⃗⃗ is unit vector}
⃗⃗⃗ |2=16
|𝑥
⃗⃗⃗ | = 4
|𝑥
5 The direction cosines of the vector -2𝑖̂+ 𝑗̂-5𝑘̂ are
−2 1 5 −2 −1 5
a) , , b) , ,
√30 √30 √30 √30 √30 √30
2 1 −5 −2 1 5
c) − , , d) , −√30 , −√30
√30 √30 √30 √30

Solution: c)
𝑥 𝑦 𝑧
The direction cosines of a vector 𝑟⃗⃗ =x𝑖̂ + y𝑗̂ + z𝑘̂ are : l = , m= ,𝑛 =
⃒ 𝑟⃒ ⃒ 𝑟⃒ ⃒ 𝑟⃒
−2 1
So the DC’s of -2𝑖̂+ 𝑗̂-5𝑘̂ are l = ,m= ,𝑛 =
√(−2)2 +12 +(−5)2 √(−2)2 +12 +(−5)2
−5
√(−2)2 +12 +(−5)2

𝟐 𝟏 −𝟓
=− , ,
√𝟑𝟎 √𝟑𝟎 √𝟑𝟎

6 If the vectors 2𝑖̂-3 𝑗̂+ 4𝑘̂ and a𝑖̂+ 6 𝑗̂- 8𝑘̂ are collinear then the value of a is
a) −4 b) 4 c) 2 d) −2
Solution: a)
if the vectors 2𝑖̂-3 𝑗̂+ 4𝑘̂ and a𝑖̂+ 6 𝑗̂- 8𝑘̂ are collinear then
2 −3 4
= 6 = −8
𝑎
2 −3
Or 𝑎 = 6
=> 𝑎 = −4
7 The position vector of the midpoint of the vector joining the points
𝑃(2,3,4) 𝑎𝑛𝑑 𝑄(4,1, −2) is
a) 6𝑖̂+ 4 𝑗̂- 6𝑘̂ b) 3𝑖̂+ 2𝑗̂- 2𝑘̂
c) 6𝑖̂+ 4𝑗̂+ 6𝑘̂ d) 3𝑖̂+ 2𝑗̂+ 𝑘̂
Solution: d)
The position vector of the midpoint of the vector joining the points 𝑃(2,3,4) and
2+4 3+1 4−2
Q(4,1,-2) is 2 𝑖̂ + 2 𝑗̂ + 2 𝑘̂
= 3𝑖̂+ 2𝑗̂+ 𝑘̂
8 The scalar components of the vector 𝐴𝐵 ⃗⃗⃗⃗⃗ with initial point 𝐴(2,1) and terminal
point 𝐵(1,0) are
a) 1 , 1 b) −1 , −1 c) 1 , −1 d) −1 , 1
Solution: b)
The vector ⃗⃗⃗⃗⃗
𝐴𝐵 with initial point A(2,1) and terminal point B(1,0) is
⃗⃗⃗⃗⃗
𝐴𝐵 = (1 − 2) 𝑖̂ + (0 − 1)𝑗̂
= -1𝑖̂ -1𝑗̂ so the scalar components of ⃗⃗⃗⃗⃗ 𝐴𝐵 are −1, −1
177
9 If 𝑎 = 6𝑖̂+ 𝑗̂ , 𝑏⃗ = 𝑖̂+ 2𝑗̂+ 𝑘̂ , 𝑐 = 𝑗̂- 𝑘̂ then 𝑎+ 𝑏⃗ - 𝑐 is
a) 6𝑖̂+ 4 𝑗̂- 𝑘̂ b)7𝑖̂+ 4 𝑗̂- 2𝑘̂
c) 7𝑖̂+ 2 𝑗̂+ 2𝑘̂ d) 7𝑖̂+ 4 𝑗̂+2𝑘̂

Solution: c)
if 𝑎 = 6𝑖̂+ 𝑗̂ , 𝑏⃗ = 𝑖̂+ 2𝑗̂+ 𝑘̂ , 𝑐 = 𝑗̂- 𝑘̂
⸫ 𝑎+ 𝑏⃗ - 𝑐 = 6𝑖̂+ 𝑗̂ + 𝑖̂+ 2𝑗̂+ 𝑘̂ - 𝑗̂ + 𝑘̂
= 7𝑖̂+ 2 𝑗̂+ 2𝑘̂
10 The value of (𝑖̂ × 𝑗̂). 𝑘̂ + 𝑖̂.𝑗̂- 𝑘̂.(𝑗̂ × 𝑖̂) is
a) 0 b) 1 c) −1 d) 2
Solution: d)
(𝑖̂ × 𝑗̂). 𝑘̂ + 𝑖̂.𝑗̂- 𝑘̂.(𝑗̂ × 𝑖̂)
= 𝑘̂ . 𝑘̂ + 0 + 𝑘̂. 𝑘̂ { ⸪ 𝑖̂ × 𝑗̂ = 𝑘̂ , 𝑗̂ × 𝑖̂ = −𝑘̂, 𝑖̂.𝑗̂ = 0}
= 1+1
=2

CHAPTER VIDEO LINK FOR MCQs SCAN QR CODE FOR


VIDEO

VECTORS https://youtu.be/t4jGZmgSilc

EXERCISE

1 if 𝑎 = 4𝑖̂ - 2𝑗̂- 𝑘̂ then the direction cosines of 𝑎 are


𝟒 𝟐 𝟏 4 −2 −1
a) , , b) , ,
√𝟐𝟏 √𝟐𝟏 √𝟐𝟏 √21 √21 √21
𝟒 −𝟐 −𝟏
c) , , d) None of the above
√𝟕 √𝟕 √𝟕
𝟒 −𝟐 −𝟏
Answer: (b) , ,
√𝟐𝟏 √𝟐𝟏 √𝟐𝟏

2 If 𝑎 = 𝑖̂ - 2𝑗̂ + 3 𝑘̂ and 𝑏⃗ = 4𝑖̂ - 8𝑗̂+ 12 𝑘̂ the vectors 𝑎 𝑎𝑛𝑑 𝑏⃗ are


a) Parallel b) Perpendicular
c) Co initial vectors d) None of the above
Answer: (a) Parallel
3 The magnitude of 𝑏⃗ = 𝑖̂ - 8𝑗̂+ 2 𝑘̂ is

a) √69 b) √63 c) √70 d) √11


Answer: (𝒂) √69

178
4 The vector components of 𝑏⃗ = 3𝑖̂ +𝑗̂+ 7 𝑘̂ are
a) 3,1,7 b) 3i , j , 7k
𝟑𝑖̂ 𝑗̂ ̂
7𝑘
c) 3𝑖̂ , 𝑗̂ , 7 𝑘̂ d) , ,
√𝟓𝟗 √𝟓𝟗 √𝟓𝟗
Answer:(𝒄) 3𝑖̂ , 𝑗̂ , 7 𝑘̂
5 Which of the following measure is vector?
a) Distance b) Time period
c) Volume d) Force
Answer: (d) Force
ASSERTION AND REASONING QUESTIONS

The following questions are of one mark each, two statements are given, one labelled
Assertion(A) and the other labelled Reason(R). Select the correct answer from the codes
(a),(b),(c),(d) as given below

a) Both Assertion(A) and Reason(R) are true and Reason(R) is the correct explanation of
the Assertion(A).
b) Both Assertion(A) and Reason(R) are true but Reason(R) is the not the correct
explanation of the Assertion(A).
c) Assertion(A) is true and Reason(R) is false
d) Assertion(A) is false and Reason(R) is true

1 Assertion(A): the magnitude of vector 𝑏⃗ = 3𝑖̂+ 2𝑗̂+ 6𝑘̂ is 7


Reason(R) : if 𝑟 = 𝑥𝑖̂+ y𝑗̂+ z𝑘̂ then │𝑟│= x2 + y2 + r2
Answers: c) Assertion(A) is true and Reason(R) is false

2 Assertion(A): the vector in the direction of 𝑎 = 𝑖̂ −2𝑗̂ with magnitude 7 units is


7 14
𝑖̂ − 𝑗̂
√5 √5
Reason(R) : the vector in the direction of 𝑟,
⃗⃗ which has magnitude d units is d.̂𝑟
Answers: a) Both Assertion(A) and Reason(R) are true and Reason(R) is the correct
explanation of the Assertion(A).

1 2 3
3 Assertion(A) : the direction cosines of 𝑖̂+ 2𝑗̂ + 3𝑘̂ are , ,
√14 √14 √14
Reason(R) : 𝑓𝑜𝑟 𝑎𝑛𝑦 𝑣𝑒𝑐𝑡𝑜𝑟 𝑟 = 𝑥𝑖̂+ y𝑗̂+ z𝑘̂ , │𝑟│ = √𝑥 2 +𝑦 2 +𝑧 2
Answers: b) Both Assertion(A) and Reason(R) are true but Reason(R) is not the
correct explanation of the Assertion(A)
4 Assertion(A) : the angle between 𝑎 𝑎𝑛𝑑 𝑏⃗ is 600 .if │𝑎│= √3,│𝑏⃗│=2, 𝑎. 𝑏⃗ = √3
Reason(R) : 𝑎 . 𝑏⃗ = │𝑎││𝑏⃗│sinѳ
Answers: c) Assertion(A) is true and Reason(R) is false

5 Assertion(A) : If either 𝑎 = 0 or 𝑏⃗ = 0 then 𝑎. 𝑏⃗ = 0


Reason(R) : if a,b,c are the direction ratios of the vector then a2+b2+c2 = 1
Answers: c) Assertion(A) is true and Reason(R) is false

6 Assertion(A): for any two vectors 𝑎 and 𝑏⃗ with │ 𝑎│≠0 ≠│𝑏⃗│we always have
│𝑎 + 𝑏⃗│≤│𝑎││𝑏⃗│
Reason(R): 𝑎 . 𝑎 = │𝑎│2

179
Answers: d) Assertion(A) is false and Reason(R) is true

7 Assertion(A): the direction cosines of a vector equally inclined to the axes OX, OY,
OZ are 1,1,1
𝑥 𝑦
Reason(R): the direction cosines of a vector 𝑟 = 𝑥𝑖̂+ y𝑗̂+ z𝑘̂ is l = ,m= ,𝑛 =
⃒ 𝑟⃒ ⃒ 𝑟⃒
𝑧
⃒ 𝑟⃒
Answers: d) Assertion(A) is false and Reason(R) is true

8 Assertion(A): (𝑎 + 𝑏⃗). (𝑎 + 𝑏⃗) = │𝑎│2 +│𝑏⃗│2 ,if and only if 𝑎, 𝑏⃗ are perpendicular,
𝑎 ≠ 0, 𝑏⃗ ≠ 0.
Reason(R) : 𝑎 + 𝑏⃗ = 𝑏⃗ + 𝑎
Answers: b) Both Assertion(A) and Reason(R) are true but Reason(R) is not the
correct explanation of the Assertion(A).
𝜋 𝜋
9 Assertion(A): the vector 𝑟 of magnitude 3√2 units, which makes an angle of 4 and 2
with y- and z- axes respectively is ± 3𝑖̂ + 3𝑗̂
Reason(R): 𝑎 . 𝑏⃗ = │𝑎││𝑏⃗│𝑐𝑜𝑠Ѳ
Answers: b) Both Assertion(A) and Reason(R) are true but Reason(R) is not the
correct explanation of the Assertion(A).

10 Assertion(A): the unit vector in XY-Plane, making an angle of 300 with positive
√3 1
direction of x-axis is 2 𝑖̂ + 2 𝑗̂
Reason(R): 𝑟̂ = cos300𝑖̂+ sin300𝑗̂
Answers: a) Both Assertion(A) and Reason(R) are true and Reason(R) is the correct
explanation of the Assertion(A).

EXERCISE

1 Assertion(A) : If 𝑎 = 3𝑖̂ +𝑗̂+ 7 𝑘̂ and 𝑏⃗ = 4𝑖̂ +2𝑗̂-2 𝑘̂ then 𝑎. 𝑏⃗ = 0


Reason(R) :𝑖̂. 𝑖̂ = 1 , 𝑗̂. 𝑗̂ = 1 , 𝑘̂. 𝑘̂ =1
Answer: Both Assertion(A) and Reason(R) are true and Reason(R) is the correct
explanation of the Assertion(A).
𝟏
2 Assertion(A) : the unit vector of 𝑖̂ +𝑗̂+ 𝑘̂ is 𝟏 (𝑖̂ +𝑗̂+ 𝑘̂)
𝑎⃗
Reason (R) :𝑎̂ = |𝑎⃗|
Answer: Assertion(A) is false and Reason(R) is true
3 Assertion(A) :𝑖̂ +3𝑗̂ − 𝑘̂ and 3𝑖̂ +9𝑗̂ − 3𝑘̂ are collinear vectors
Reason (R) : two collinear vectors are always equal in magnitude
Answer: Assertion(A) is true and Reason(R) is false
4 Assertion(A) : the unit vector in YZ-Plane, making an angle of 450 with positive
1 1
direction of x-axis is 𝑗̂ + 𝑘̂
√2 √2
Reason(R) :𝑟̂ = sin450𝑗̂+ cos450𝑖̂
Answer: Both Assertion(A) and Reason(R) are true and Reason(R) is the correct
explanation of the Assertion(A).
5 Assertion(A) : the vector joining the points A(1,0,-1) and B(2,1,0) is directed from

180
B to A is 𝑖̂ +𝑗̂- 𝑘̂
Reason (R): ⃗⃗⃗⃗⃗ ⃗⃗⃗⃗⃗⃗ − ⃗⃗⃗⃗⃗
𝑃𝑄 = 𝑂𝑄 𝑂𝑃
Answer: Assertion(A) is false and Reason(R) is true

2 MARK QUESTIONS

1 If 𝑎 = 4𝑖̂ - 𝑗̂+ 𝑘̂ and 𝑏⃗ = 2𝑖̂ - 2𝑗̂+ 𝑘̂ , then find the unit vector along the vector 𝑎 ×
𝑏⃗
Solution: 𝑎 = 4𝑖̂ - 𝑗̂+ 𝑘̂ and 𝑏⃗ = 2𝑖̂ - 2𝑗̂+ 𝑘̂
𝑖̂ 𝑗̂ 𝑘̂

𝑎 × 𝑏 = |4 −1 1|
2 −2 1
= 𝑖̂(−1 + 2) - 𝑗̂(4 − 2)+ 𝑘̂(−8 + 2)
= 𝑖̂ −2 𝑗̂ −6 𝑘̂
̂
𝑖̂−2 𝑗̂ −6 𝑘 ̂
𝑖̂−2 𝑗̂ −6 𝑘
the unit vector along the vector 𝑎 × 𝑏⃗ = =
√12 +(−2)2 +(−6)2 √41

2
2 If the vectors 𝑎 𝑎𝑛𝑑 𝑏⃗ are such that │𝑎│= 3,│𝑏⃗│= 3 and 𝑎 × 𝑏⃗ is unit vector then
find the angle between 𝑎 𝑎𝑛𝑑 𝑏⃗
⃗│
│𝑎⃗× 𝑏
Solution: Angle between 𝑎 𝑎𝑛𝑑 𝑏⃗ is Sin Ѳ = ⃗ │𝑎⃗││𝑏 │
1
Sin Ѳ = 2 { ⸪ │𝑎 × 𝑏⃗│ = 1}
3.
3
1
Sin Ѳ = 2
𝜋
Ѳ=6
3 Find the position vector of a point which divides the join of points with position
vectors (𝑎 − 2 𝑏⃗) and (2𝑎 + 𝑏⃗) externally in the ratio 2:1
⃗ )−2(2𝑎⃗+ 𝑏
1(𝑎⃗−2 𝑏 ⃗)
Solution: The required vector = 1−2
⃗ −4𝑎⃗− 2𝑏
𝑎⃗−2 𝑏 ⃗
= 1−2

−3𝑎⃗−4𝑏
= −1
= 3𝑎 + 4𝑏⃗
4 𝐼𝑓 𝑎 , 𝑏⃗ , 𝑐 are three non-zero unequal vectors such that 𝑎 . 𝑏⃗ = 𝑎 . 𝑐 , then find the
angle between 𝑎 𝑎𝑛𝑑 𝑏⃗ − 𝑐
Solution: Given 𝑎 . 𝑏⃗ = 𝑎 . 𝑐
𝑎 . 𝑏⃗ - 𝑎 . 𝑐 = 0
𝑎. (𝑏⃗ - 𝑐) = 0
⸫ 𝑎 𝑖𝑠 𝑝𝑒𝑟𝑝𝑒𝑛𝑑𝑖𝑐𝑢𝑙𝑎𝑟 𝑡𝑜 (𝑏⃗ - 𝑐 )
𝜋
So the angle between 𝑎 𝑎𝑛𝑑 𝑏⃗ − 𝑐 is 2
5 If 𝑎 = 2𝑖̂ + 𝑗̂+ 3𝑘̂ and 𝑏⃗ = 3𝑖̂ + 𝑗̂ − 2𝑘̂ , then find │𝑎 × 𝑏⃗│
𝑖̂ 𝑗̂ 𝑘̂
Solution: 𝑎 × 𝑏⃗ = |2 1 3 | = 𝑖̂(−2 − 3) − 𝑗̂(−4 − 9)+ 𝑘̂(2 − 3)
3 1 −2
𝑎 × 𝑏⃗ = −5 𝑖̂ + 13 𝑗̂ − 𝑘̂
│𝑎 × 𝑏⃗│ = √(−5)2 + (13)2 + (−1)2
= √195

181
6 Find │𝑎│ and │𝑏⃗│ if (𝑎 + 𝑏⃗ )(𝑎 - 𝑏⃗) = 8 and │𝑎│ = 8│𝑏⃗│
Solution: (𝑎 + 𝑏⃗)(𝑎 - 𝑏⃗ ) = │𝑎│2 − │ 𝑏⃗│2
2
8 = [8│𝑏⃗│] − │ 𝑏⃗│2
8 = 64│ 𝑏⃗│2 − │ 𝑏⃗│2
Or 63│ 𝑏⃗│2 =8
8
│𝑏⃗│ = √63
7 The position vectors of points A,B,C are λ𝑖̂ +3𝑗̂ , 12𝑖̂ +μ𝑗̂ and 11𝑖̂ − 3𝑗̂
respectively. If C divides the line segment joining A and B in the ratio 3:1, find the
values of λ and μ
3(12𝑖̂ +μ𝑗̂ )+1(λ𝑖̂ +3𝑗̂ )
Solution: 11𝑖̂ − 3𝑗̂ = 4
44 = 36 + λ ⇨ λ=8 ,
and -12 =3μ+3 ⇨ μ = -5

8 If │𝑎 + 𝑏⃗│= 60 and │𝑎 - 𝑏⃗│=40 and │𝑎│=22,then find │ 𝑏⃗│


Solution: │𝑎 + 𝑏⃗│2 +│𝑎 − 𝑏⃗│2 = 2[│𝑎 │2 + │ 𝑏⃗│2 ]
602 + 402 = 2[222 + │ 𝑏⃗│2 ]
│ 𝑏⃗│2 = 2600 − 484
│ 𝑏⃗│ = 46

9 If 𝑎 , 𝑏⃗ 𝑎𝑟𝑒 two vectors such that │𝑎 + 𝑏⃗│= │𝑎│, then prove that 2𝑎 + 𝑏⃗ is
perpendicular to 𝑏⃗
Solution: Given │𝑎 + 𝑏⃗│= │𝑎│
or │𝑎 + 𝑏⃗│2 = │𝑎│2
│𝑎│2 + │𝑏⃗│2 + 2𝑎 . 𝑏⃗ = │𝑎│2
│𝑏⃗│2 + 2𝑎 . 𝑏⃗ = 0
𝑏⃗ . 𝑏⃗+ 2𝑎 . 𝑏⃗ = 0
𝑏⃗ ( 𝑏⃗+ 2𝑎 ) = 0
⸫ 𝑏⃗ 𝑖𝑠 𝑝𝑒𝑟𝑝𝑒𝑛𝑑𝑖𝑐𝑢𝑙𝑎𝑟 𝑡𝑜 (𝑏⃗+ 2𝑎 )
10 If 𝑎 𝑎𝑛𝑑 𝑏⃗ are perpendicular vectors, │𝑎 + 𝑏⃗│=13 and │𝑎│ = 5, find the value
of │𝑎│
Solution: │𝑎 + 𝑏⃗│2 = │𝑎│2 + │𝑏⃗│2 + 2𝑎 . 𝑏⃗
132 = 52 + │𝑏⃗│2 + 0
169 – 25 = │𝑏⃗│2
│𝑏⃗│ = 12

EXERCISE

1 Find a vector in the direction of vector 𝑎 = -2𝑖̂ +𝑗̂ + 2𝑘̂ that has magnitude 9 units.

Answer: -6𝑖̂ +3𝑗̂ + 6𝑘̂


2 Find a unit vector in the direction of 𝑎+𝑏⃗ if 𝑎 =𝑖̂ +𝑗̂ − 2𝑘̂ and 𝑏⃗ =𝑖̂ + 3𝑗̂ + 2𝑘̂
Answer: 2𝑖̂ + 4𝑗̂

182
3 The position vectors of A,B,and C are λ𝑖̂ +3𝑗̂, 12𝑖̂ +μ𝑗̂and 11𝑖̂ −3𝑗̂respectively. If C
divides the line segment joining A&B in the ratio 3:1,find the values of λ and μ
Answer: λ =8 and μ = -5
4 If two vectors 𝑎 and𝑏⃗ are such that |𝑎|=3 ,|𝑏
⃗⃗⃗ |= 1, 𝑎.𝑏⃗=2 find (3𝑎+𝑏⃗).(2𝑎-3𝑏⃗)

Answer: 37
5 If 𝑎×𝑏⃗ = 𝑐⃗⃗ ×𝑑 and 𝑎×𝑐 = ⃗⃗⃗
𝑏 ×𝑑 , then show that (𝑎-𝑑 ) is parallel to 𝑏⃗ - 𝑐⃗⃗ , it is

Given that 𝑎≠𝑑 and 𝑏⃗≠𝑐⃗⃗


Answer: To show (𝑎-𝑑) × ( 𝑏⃗ - 𝑐⃗⃗ ) = 0
⃗⃗⃗

3 MARK QUESTIONS

1 Let 𝑎 = 4 𝑖̂ +5 𝑗̂ - 𝑘̂ , 𝑏⃗ = 𝑖̂ -4 𝑗̂ +5𝑘̂ and 𝑐 = 3𝑖̂ + 𝑗̂ - 𝑘̂. Find a vector 𝑑 which is


perpendicular to both vectors 𝑐 and 𝑏⃗ and 𝑑 .𝑎 =21
Solution: Since 𝑑 is perpendicular to both vectors 𝑐 and 𝑏⃗ so
𝑖̂ 𝑗̂ 𝑘̂

𝑑 = 𝜆(𝑐×𝑏) = λ|3 1 −1| { where λ is scalar}
1 −4 5
= 𝜆[𝑖̂(5 − 4) − 𝑗̂(15 + 1) + 𝑘̂ (−12 − 1)]
= 𝜆[𝑖̂ − 16𝑗̂ − 13𝑘̂]
𝑑 = 𝜆𝑖̂ − 𝜆16𝑗̂ − 𝜆13𝑘̂
given 𝑑.𝑎 =21
⸫ (𝜆𝑖̂ − 𝜆16𝑗̂ − 𝜆13𝑘̂). (4 𝑖̂ + 5 𝑗̂ − 𝑘̂) = 21
 4𝜆 − 80𝜆 + 13𝜆 = 21
1
 𝜆 = −3

1
⸫𝑑 = − 3 (𝑖̂ − 16𝑗̂ − 13𝑘̂ )

2 If 𝑎 = 𝑖̂ +2𝑗̂+𝑘̂ , 𝑏⃗ = 2𝑖̂+ 𝑗̂ and 𝑐 = 3𝑖̂ − 4𝑗̂ − 5𝑘̂. Find a unit vector perpendicular
to both vectors( 𝑎 − 𝑏⃗) 𝑎𝑛𝑑 (𝑐 − 𝑏⃗)
Solution: Given 𝑎 = 𝑖̂ +2𝑗̂+𝑘̂ , 𝑏⃗ = 2𝑖̂+ 𝑗̂ and 𝑐 = 3𝑖̂ − 4𝑗̂ − 5𝑘̂.
𝑎 − 𝑏⃗ = 𝑖̂ +2𝑗̂+𝑘̂ − 2𝑖̂ − 𝑗̂ = −𝑖̂ + 𝑗̂+𝑘̂

𝑐 − 𝑏⃗= 3𝑖̂- 4𝑗̂-5𝑘̂ − 2𝑖̂- 𝑗̂= 𝑖̂ − 5𝑗̂ − 5𝑘̂

𝑖̂ 𝑗̂ 𝑘̂
( 𝑎 − 𝑏⃗) ×(𝑐 − 𝑏⃗) = |−1 1 1|
1 −5 −5

= - 4𝑗̂ + 4𝑘̂

183
|( 𝑎 − 𝑏⃗ ) × (𝑐 − 𝑏⃗) | = √(−4)2 +42 = 4√2
̂
− 4𝑗̂ + 4𝑘 ̂
− 𝑗̂ + 𝑘
The required vector is = ± =±
4√2 √2

3 If 𝑟 = 𝑥𝑖̂ +y𝑗̂+ z𝑘̂ , find (𝑟×𝑖̂).(𝑟 × 𝑗̂) + xy


Solution: (𝑟×𝑖̂).(𝑟 × 𝑗̂) + xy
𝑟×𝑖̂ = (𝑥𝑖̂ +y𝑗̂+ z𝑘̂) × 𝑖̂ = 0 -y𝑘̂ + z 𝑗̂ = z 𝑗̂ − y𝑘̂
𝑟 × 𝑗̂ = (𝑥𝑖̂ +y𝑗̂+ z𝑘̂) × 𝑗̂ = 𝑥𝑘̂ + 0 - z𝑖̂ = 𝑥𝑘̂ - z𝑖̂
Now (𝑟×𝑖̂).(𝑟 × 𝑗̂) + xy = (z 𝑗̂ − y𝑘̂).(𝑥𝑘̂ - z𝑖̂)
= -xy + xy = 0

4 The magnitude of the vector product of the vector 𝑖̂ +𝑗̂+ 𝑘̂ with a unit vector along
the sum of vectors 2𝑖̂ +4𝑗̂-5𝑘̂ and 𝜆𝑖̂ + 2𝑗̂+ 3𝑘̂ is equal to √2 . find the value of λ .
Solution: Let 𝑎 = 𝑖̂ +𝑗̂+ 𝑘̂ , 𝑏⃗ =2𝑖̂ +4𝑗̂-5𝑘̂ , 𝑐 = 𝜆𝑖̂ + 2𝑗̂+ 3𝑘̂
Given 𝑏⃗ + 𝑐 = 2𝑖̂ +4𝑗̂-5𝑘̂ + 𝜆𝑖̂ + 2𝑗̂+ 3𝑘̂ = (2+ 𝜆)𝑖̂ + 6𝑗̂-2𝑘̂ = 𝑑 (say)
|𝑑 | = √(2 + 𝜆)2 +62 +(−2)2 = √(2 + 𝜆)2 + 40
𝑖̂ 𝑗̂ 𝑘̂
Now 𝑎 × 𝑑 = | 1 1 1 | = -8𝑖̂ +(4+ 𝜆)𝑗̂ + (4 − 𝜆)𝑘̂
2 + 𝜆 6 −2
𝑎⃗×𝑑
Given | |𝑑 | | = √2
̂
−8𝑖̂ +(4+ 𝜆)𝑗̂ +(4−𝜆)𝑘
=> | |= √2
√(2+ 𝜆)2 +40
√ (−8)2 +(4+ 𝜆)2 +(4− 𝜆)2
=> = √2
√(2+ 𝜆)2 +40
64+16+8𝜆+𝜆2 +16−8𝜆+𝜆2
=> =2
𝜆2 +4𝜆+44
96+2𝜆2
=> 𝜆2 +4𝜆+44 = 2
=> 8𝜆 = 8
=> 𝜆 = 1
5 let 𝑎 , 𝑏⃗ 𝑎𝑛𝑑 𝑐 be three vectors such that │𝑎│=3 ,│𝑏⃗│= 4 and │𝑐 │=5 and each one
of them being perpendicular to the sum of the other two, find │𝑎+ 𝑏⃗+𝑐 │
Solution:Given 𝑎.( 𝑏⃗+𝑐 ) = 0 , 𝑏⃗. (𝑎+𝑐 ) = 0 , 𝑐 . (𝑎+ 𝑏⃗ ) = 0
Now │𝑎 + 𝑏⃗ + 𝑐│2 = (𝑎 + 𝑏⃗ + 𝑐 )2
=(𝑎 + 𝑏⃗ + 𝑐 )(𝑎 + 𝑏⃗ + 𝑐 )
= 𝑎 . 𝑎+ 𝑎.( 𝑏⃗+𝑐 ) + 𝑏⃗. 𝑏⃗ + 𝑏⃗. (𝑎+𝑐 ) + 𝑐 . 𝑐 +
𝑐. (𝑎+ 𝑏⃗ )
Put 𝑎.( 𝑏⃗+𝑐 ) = 0 , 𝑏⃗. (𝑎+𝑐) = 0 , 𝑐 . (𝑎+ 𝑏⃗ ) = 0
⸫ │𝑎 + 𝑏⃗ + 𝑐 │2 = │𝑎│2 + 0 + │𝑏⃗│2 + 0 + │𝑐 │2 + 0
│𝑎 + 𝑏⃗ + 𝑐│2 = 32 + 42 + 52 {⸪│𝑎│=3 ,│𝑏⃗│= 4 and
│𝑐│=5}
│𝑎 + 𝑏⃗ + 𝑐│2 = 50
│𝑎 + 𝑏⃗ + 𝑐│ = √50 = 5√2

6 show that the points 𝐴(−2𝑖̂ +3𝑗̂+ 5𝑘̂) , B(𝑖̂ +2𝑗̂+ 3𝑘̂ ) and C(7𝑖̂ - 𝑘̂) are collinear
Solution:Given the points are 𝐴(−2𝑖̂ +3𝑗̂+ 5𝑘̂) , B(𝑖̂ +2𝑗̂+ 3𝑘̂ ) and C(7𝑖̂ - 𝑘̂)
⃗⃗⃗⃗⃗ = (𝑖̂ +2𝑗̂+ 3𝑘̂ ) − (−2𝑖̂ +3𝑗̂+ 5𝑘̂) = 3𝑖̂ - 𝑗̂ − 2𝑘̂
𝐴𝐵

184
⃗⃗⃗⃗⃗ = 7𝑖̂ - 𝑘̂ - (𝑖̂ +2𝑗̂+ 3𝑘̂ ) = 6𝑖̂ - 2𝑗̂ − 4𝑘̂
𝐵𝐶
𝐴𝐶 = (7𝑖̂ - 𝑘̂) −(−2𝑖̂ +3𝑗̂+ 5𝑘̂ ) = 9𝑖̂ - 3𝑗̂ − 6𝑘̂
⃗⃗⃗⃗⃗
│𝐴𝐵 ⃗⃗⃗⃗⃗ │ = √32 +(−1)2 + (−2)2 = √14
│𝐵𝐶 ⃗⃗⃗⃗⃗ │ = √6+(−2)2 + (−4)2 = 2√14
│𝐴𝐶 ⃗⃗⃗⃗⃗ │ = √92 +(−3)2 + (−6)2 = 3√14
Here │𝐴𝐶 ⃗⃗⃗⃗⃗ │ = │𝐵𝐶
⃗⃗⃗⃗⃗ │ +│𝐴𝐵
⃗⃗⃗⃗⃗ │
Hence the points A , B, C are collinear

7 if 𝑎 , 𝑏⃗ 𝑎𝑛𝑑 𝑐 are unit vectors such that 𝑎 + 𝑏⃗ + 𝑐 = ⃗0 , find the value of 𝑎 . 𝑏⃗ + 𝑏⃗. 𝑐
+𝑐 . 𝑎
Solution:Given │𝑎│=│𝑏⃗ │=│𝑐 │= 1 and also
𝑎 + 𝑏⃗ + 𝑐 = ⃗0
Or ( 𝑎 + 𝑏⃗ + 𝑐 )2 = 0
 │𝑎│2 +│𝑏⃗│2 +│𝑐 │2 + 2𝑎 . 𝑏⃗ +2 𝑏⃗. 𝑐 +2𝑐 . 𝑎 = 0

 12+12+12 + 2(𝑎 . 𝑏⃗ + 𝑏⃗. 𝑐 +𝑐 . 𝑎) = 0

 2(𝑎 . 𝑏⃗ + 𝑏⃗. 𝑐 +𝑐 . 𝑎) = −3
−3
 (𝑎 . 𝑏⃗ + 𝑏⃗. 𝑐 +𝑐 . 𝑎) = 2

8 if 𝑎 = 2𝑖̂ +2𝑗̂+3 𝑘̂ , 𝑏⃗ = - 𝑖̂ +2𝑗̂ + 3𝑘̂ , 𝑐 = 3𝑖̂ + 𝑗̂ are such that 𝑎 + 𝜆𝑏⃗ is


perpendicular to 𝑐,find the value of 𝜆
Solution: 𝐺𝑖𝑣𝑒𝑛 𝑎 = 2𝑖̂ +2𝑗̂+3 𝑘̂ , 𝑏⃗ = - 𝑖̂ +2𝑗̂ + 3𝑘̂ , 𝑐 = 3𝑖̂ + 𝑗̂
𝑎 + 𝜆𝑏⃗ = 2𝑖̂ +2𝑗̂+3 𝑘̂ + 𝜆(- 𝑖̂ +2𝑗̂ + 3𝑘̂ )
= (2- 𝜆) 𝑖̂ + (2+2 𝜆)𝑗̂ +(3+3 𝜆)𝑘̂
Given 𝑎 + 𝜆𝑏⃗ is perpendicular to 𝑐
⸫ (𝑎 + 𝜆𝑏⃗ ).𝑐 = 0
i.e [(2- 𝜆) 𝑖̂ + (2+2 𝜆)𝑗̂ +(3+3 𝜆)𝑘̂] .( 3𝑖̂ + 𝑗̂) = 0
=> 3(2- 𝜆) + (2+2 𝜆)=0 => 𝜆= 8

9 if 𝑎̂, 𝑏̂ , 𝑐̂ are mutually perpendicular vectors, then find the value of |2𝑎̂ + 𝑏̂ + 𝑐̂ |
Solution: Given 𝑎̂, 𝑏̂ , 𝑐̂ are mutually perpendicular vectors
⸫ 𝑎̂. 𝑏̂ = 0, 𝑏̂ . 𝑐̂ =0, 𝑎̂. 𝑐̂ =0
̂
│𝑎̂│=│𝑏 │=│𝑐̂ │ = 1
|2𝑎̂ + 𝑏 + 𝑐̂ |2 = 4│𝑎̂│2 +│𝑏̂ │2 +│𝑐̂ │2 + 4𝑎̂. 𝑏̂ + 𝑏̂ . 𝑐̂ + 𝑎̂. 𝑐̂
̂
= 4 .12 + 12 +12 + 0+0+0
= 4+1+1
=6
̂
⸫ |2𝑎̂ + 𝑏 + 𝑐̂ | = √6

10 If 𝑎 𝑎𝑛𝑑 𝑏⃗ are two vectors such that |𝑎 + 𝑏⃗| = |𝑎| , then prove that 2𝑎 + 𝑏⃗ is
perpendicular to 𝑏⃗ .
Solution: Given |𝑎 + 𝑏⃗| = |𝑎|
or |𝑎 + 𝑏⃗|2 = |𝑎|2

185
 |𝑎|2 + |𝑏⃗|2 + 2 𝑎 . 𝑏⃗ = |𝑎|2

 |𝑎|2 -|𝑎|2 + |𝑏⃗|2 + 2 𝑎 . 𝑏⃗ =0

 |𝑏⃗|2 + 2 𝑎 . 𝑏⃗ = 0

 . 𝑏⃗(. 𝑏⃗ + 2 𝑎 ) =

Hence 2𝑎 + 𝑏⃗ is perpendicular to 𝑏⃗ .

EXERCISE

1 If A, B, C, D are the points with position vectors 4𝑖̂ +5𝑗̂ + 𝑘̂, −𝑗̂ − 𝑘̂, 3𝑖̂ + 9𝑗̂ + 4𝑘̂
and - 4𝑖̂ +4 𝑗̂ + 4𝑘̂ respectively, then find 𝐴𝐵
⃗⃗⃗⃗⃗ .(𝐴𝐶
⃗⃗⃗⃗⃗ x 𝐴𝐷
⃗⃗⃗⃗⃗ )

Answers: 0
2 ⃗⃗⃗⃗⃗⃗ × 𝐴𝐶
Find |𝐴𝐵 ⃗⃗⃗⃗⃗ | If A(1,2,3) , B(2,-1,4) , C(4,5,-1) are three points in space.

Answers: √274
3 Find a unit vector perpendicular to both the vectors 𝑎+𝑏⃗ and 𝑎-𝑏⃗ where 𝑎 = 𝑖̂ +𝑗̂ +
𝑘̂ and 𝑎 = 𝑖̂ +2𝑗̂ + 3𝑘̂
−𝑖̂ 𝟐𝑗̂ ̂
𝑘
Answers: + -
√𝟔 √𝟔 √𝟔

4 If 𝑎 , 𝑏⃗ 𝑎𝑛𝑑 𝑐 are vectors such that 𝑎 + 𝑏⃗ + 𝑐 = ⃗0 and |𝑎|=5, |𝑏


⃗⃗⃗ |= 12 ,|𝑐⃗⃗ |=13, then
find the value of 𝑎. 𝑏⃗ + 𝑏⃗. 𝑐 +𝑐 . 𝑎

Answers: 𝑎. 𝑏⃗ + 𝑏⃗. 𝑐 +𝑐. 𝑎 = -169


5 If 𝑎 , 𝑏⃗ 𝑎𝑛𝑑 𝑐 are three mutually perpendicular vectors of equal magnitude, show
that 𝑎+𝑏⃗ + 𝑐 is equally inclined to 𝑎 , 𝑏⃗ 𝑎𝑛𝑑 𝑐 . also find the angle.

CASE BASED QUESTIONS

I Two boys Amit and Raju are playing in a park, they saw three points A(2,-1,3) ,
B(1,1,3) , C(3,2,0) and they decided to play a game Amit is moving from A to B
and Raju is moving from A to C

Based on the above information answer the following


186
1) The vector ⃗⃗⃗⃗⃗⃗
𝐴𝐵 is
a) 𝑖̂ +5𝑗̂ + 𝑘̂ b) −𝑖̂ +2𝑗̂ + 𝑘̂
c) −𝑖̂ +2𝑗̂ d) 𝑖̂ + 2𝑗̂
2) The vector 𝐴𝐶⃗⃗⃗⃗⃗ is
a) 𝑖̂ +3𝑗̂ + 3𝑘̂ b) 𝑖̂ + 2𝑗̂ + 𝑘̂
c) 𝑖̂ +3𝑗̂ − 3𝑘̂ d) 𝑖̂ + 3𝑘̂
3) The position vector of point A is
a) 2𝑖̂ - 𝑗̂ + 3𝑘̂ b) -2𝑖̂ - 𝑗̂ + 3𝑘̂
c) 2𝑖̂ -3𝑗̂ − 3𝑘̂ d) 2𝑖̂ + 3𝑗̂ − 3𝑘̂
4) The magnitude of ⃗⃗⃗⃗⃗𝐴𝐶 is
a) √9 b)√29 c) √18 d) √19
Answer:

1. c) −𝒊̂ +2𝒋̂ { ⸪𝐴𝐵 ̂}


⃗⃗⃗⃗⃗⃗ = (1−𝟐)𝒊̂ +(1+1)𝒋̂ + (3-3) 𝒌
2. c) 𝒊̂ +3𝒋̂ − 𝟑𝒌̂ { ⸪ ⃗⃗⃗⃗⃗⃗
𝐴𝐶 = (3−𝟐)𝒊̂ +(2+1)𝒋̂ + (0-3) 𝒌 ̂ }
3. ̂
a) 2𝒊̂ - 𝒋̂ + 𝟑𝒌 ̂}
{ The position vector of A is 2𝒊̂ - 𝒋̂ + 𝟑𝒌
4. d) √𝟏𝟗 { ⸪ |𝐴𝐶 ⃗⃗⃗⃗⃗ | = √𝟏𝟐 + 𝟑𝟐 + (−𝟑)𝟐 }

II The roof of a shop is in the shape of a trapezium with four points P, Q, R, S

The position vectors of P, Q, R and S are î +ĵ + 3k̂ , 2î +3ĵ + k̂ , î -3ĵ + 2k̂ , 4 î
+3ĵ − 4k̂ respectively, some ants moved on the boundary of the roof.
Based on the given information answer the following
1) The coordinates of Q are
a) (1,1,3) b) (2,3,1) c) (2,3,0) d) (1,2,0)
2) Two ants moved from the point P, one ant from P to Q and other ant from
P to S and the vectors made by the given condition are
a) Collinear vectors b) Negative vectors
c) Coinitial vectors d) Two collinear vectors with
unequal magnitudes
3) The direction ratios of ⃗⃗⃗⃗⃗
𝑆𝑅 are
a) 3,6, −3 b) 3,3,-2 c) 2,6,-3 d) - 3, - 6,
6
⃗⃗⃗⃗
4) 𝑃𝑆 and 𝑄𝑅 ⃗⃗⃗⃗⃗ are
a) Equal vectors b) Collinear vectors
c) With equal magnitudes d) None of the above
Answers:

1. b) (2,3,1) { ⸪ the position vector of Q is 2𝑖̂ +3𝑗̂ + 𝑘̂ }


2. c) coinitial vectors
3. 𝑆𝑅 = 𝑖̂-3𝑗̂ + 2𝑘̂ – (4𝑖̂ +3𝑗̂ − 4𝑘̂ ) = −3𝑖̂ - 6𝑗̂ + 6𝑘̂ }
d) - 3, - 6, 6 { ⃗⃗⃗⃗⃗⃗
4. d) none of the above

187
III Carrom or Karom is a game that has long been played throughout India and South
East Asia but the game has become increasingly popular throughout much of the
rest of the world during the last century.

A player is playing the carrom game, in the above picture suppose the striker is at
point A(1,1,3) , a white coin is at the point B(2,3,5) and the black coin at C(4,5,7
Based on the above information answer the following
1) If the striker hit the white coin then the vector is
a) 𝑖̂ +2𝑗̂ + 3𝑘̂ b)𝑖̂ +𝑗̂ + 2𝑘̂
c) -𝑖̂ -2𝑗̂ − 2𝑘̂ d) 𝑖̂ +2𝑗̂ + 2𝑘̂

2) The distance covered by the striker to the white coin is


a) √5 b) 3 c) 5 d) √3
⃗⃗⃗⃗⃗
3) The direction cosines of 𝐴𝐵 are
1 2 1 −1 −2 −2
a) 3 , 3 , 3 b) 3 , 3 , 3
𝟏 𝟐 𝟐 −1 1 2
c) , , d) , ,
𝟑 𝟑 𝟑 3 3 3
4) The direction ratios formed by the striker to the black coin is
a) 3,4,4 b) 3,3,4 c)4,4,3 d) -3,-3, -4
Answer:
1. d) 𝑖̂ +2𝑗̂ + 2𝑘̂ { ⸪ ⃗⃗⃗⃗⃗⃗
𝐴𝐵 = (2−1)𝑖̂ +(3-1)𝑗̂ + (5-3) 𝑘̂ }
2. b) 3 { |𝐴𝐶 ⃗⃗⃗⃗⃗ | = √12 + 22 + (2)2 = √9 = 3}
1 2 2 1 2 2
3. c) , , { Direction cosines of ⃗⃗⃗⃗⃗⃗
𝐴𝐵 are , , }
3 3 3 3 3 3
𝐴𝐶 = (4−1)𝑖̂ +(5-1)𝑗̂ + (7-3) 𝑘̂ = 3𝑖̂ + 4𝑗̂ + 4𝑘̂ } so the
4. a) 3,4,4 { ⃗⃗⃗⃗⃗⃗
direction ratios are 3,4,4

EXERCISE

I Three photos are placed on a wall at the points A, B, C

188
and the coordinates of these points are ( 2,1,3) , (-2,1,-3) and ( -3,3,2) respectively
based on the given information answer the following
1) The direction ratios of 𝐵𝐶⃗⃗⃗⃗⃗ is
a) 𝑖̂ +3𝑗̂ − 𝑘̂ b) 4𝑖̂ − 6𝑘̂
c) −𝑖̂ +2𝑗̂ + 5𝑘̂ d) 𝑖̂- 2𝑗̂ − 5𝑘̂
2) The direction ratios of 𝐵𝐶 ⃗⃗⃗⃗⃗ 𝑎𝑟𝑒
a) −1,2,5 b) 1, −2, −5 c) 4,0, −6 d) 1,3, −1
⃗⃗⃗⃗⃗
3) The magnitude of 𝐵𝐴 are
a) √44 b) √42 c) √54 d)√𝟓𝟐
4) The direction cosines of 𝐵𝐴 𝑎𝑟𝑒⃗⃗⃗⃗⃗
4 6 4 6
a) , ,0 b) ,0,
√52 √52 √52 √52
−4 6 4 −6
c) , ,0 d) , ,0
√52 √52 √52 √52
Answer:

𝟏) − 𝑖̂ +2𝑗̂ + 5𝑘̂
1) -1,2,5
2) √52
4 6
3) ,0,
√52 √52
II A class XII student Ravi is going to write the board examination and he was asked
to attempt the following question. Let 𝑎 , 𝑏⃗ , 𝑐 are three non-zero vectors
Based on the above information answer the following
1) If 𝑎 is perpendicular to𝑏⃗ then
a) 𝑎.𝑏⃗ = 1 b) 𝑎 . 𝑏⃗ = 0 c) 𝑎 × 𝑏⃗ = 0 d) 𝑎 × 𝑏⃗ = 1
2) If 𝑎= 𝑖̂ +3𝑗̂ − 𝑘̂ 𝑎𝑛𝑑 𝑏⃗ = 2𝑖̂ +3𝑗̂ + 𝑘̂ then 𝑎 . 𝑏⃗ is
a) 11 b) 10 c) 9 d) 4
̂ ⃗ ̂ ⃗
3) If 𝑎 = 𝑖̂ +3𝑗̂ + 𝑘 𝑎𝑛𝑑 𝑏 = 2𝑖̂ − 𝑘 then |𝑎 + 𝑏| is
a) 3√2 b) √19 c) 2√5 d) 2√2
4) If vectors 𝑎and𝑏⃗ are such that |𝑎 + 𝑏⃗| = |𝑎 − 𝑏⃗| then
a) 𝑎=𝑏⃗ b) 𝑎 is perpendicular to𝑏⃗
c) 𝑎 is parallel to 𝑏⃗ d) |𝑎| = |𝑏⃗|
Answer:
1) 𝑎 . 𝑏⃗ = 0 2) 10
2) 3√2
3) 𝑎 is perpendicular to𝑏⃗

189
CHAPTER: THREE - DIMENSIONAL GEOMETRY

SYLLABUS: Direction cosines and direction ratios of a line joining two points. Cartesian
equation and vector equation of a line, skew lines, shortest distance between two lines. Angle
between two lines.

Definitions and Formulae:

▪ Direction cosines of a line are the cosines of the angles made by the line with the
positive directions of the coordinate axes.
▪ Let a line makes the angles 𝛼, 𝛽, 𝛾 with 𝑥, 𝑦, 𝑧 axis respectively, then the Direction
Cosines of the line are 𝑙 = 𝑐𝑜𝑠𝛼, 𝑚 = 𝑐𝑜𝑠𝛽, 𝑛 = 𝑐𝑜𝑠𝛾
▪ If 𝑙, 𝑚, 𝑛 are the direction cosines of a line, then l2 + m2 +n2 = 1.
▪ Direction ratios of a line joining two points 𝑃(𝑥1 , 𝑦1 , 𝑧1 ) 𝑎𝑛𝑑 𝑄(𝑥2 , 𝑦2 , 𝑧2 ) are
𝑎 = 𝑥2 − 𝑥1 , 𝑏 = 𝑦2 − 𝑦1 , 𝑎 = 𝑦2 − 𝑦1
▪ If 𝑙, 𝑚, 𝑛 are the direction cosines and 𝑎, 𝑏, 𝑐are the direction ratios of a line then
𝑎 𝑏 𝑐
𝑙 = ± √𝑎2 , 𝑚 = ± √𝑎2 , 𝑛 = ± √𝑎2
+𝑏2 +𝑐 2 +𝑏2 +𝑐 2 +𝑏 2 +𝑐 2

▪ Direction cosines of a line joining two points P(x1 , y1 , z1 ) and Q(x2 , y2 , z2 ) are

x2 −x1 y2 −y1 z2 −z1


, , (where, PQ= √(𝑥2 − 𝑥1 )2 + (𝑦2 − 𝑦1 )2 + (𝑧2 − 𝑧1 )2 )
PQ PQ PQ

▪ Direction ratios of a line are the numbers which are proportional to the direction
cosines of a line.
▪ Skew lines are lines in space which are neither parallel nor intersecting. They lie in
different planes.
▪ Angle between skew lines is the angle between two intersecting lines drawn from any
point (preferably through the origin) parallel to each of the skew lines.
▪ If 𝑙1 , 𝑚1 , 𝑛1 𝑎𝑛𝑑 𝑙2 , 𝑚2 , 𝑛2 are the direction cosines of two lines; and 𝜃 is the acute
angle between the two lines; then 𝑐𝑜𝑠𝜃 = |𝑙1 𝑙2 + 𝑚1 𝑚2 + 𝑛1 𝑛2 |
▪ If 𝑎1 , 𝑏1 , 𝑐1 𝑎𝑛𝑑 𝑎2 , 𝑏2 , 𝑐2 are the direction ratios of two lines and 𝜃 is the acute angle

𝑎1 𝑎2 +𝑏1 𝑏2 +𝑐1 𝑐2
between the two lines; then 𝑐𝑜𝑠𝜃 = | |
√𝑎12 +𝑏12 +𝑐12 √𝑎22 +𝑏22 +𝑐22

▪ Vector equation of a line that passes through the given point whose position vector is
𝑎 and parallel to a given vector𝑏⃗ 𝑖𝑠 𝑟 = 𝑎 + 𝜆𝑏⃗ .
▪ Equation of a line through a point (𝑥1 , 𝑦1 , 𝑧1 ) and having direction cosines 𝑙, 𝑚, 𝑛 is
𝑥−𝑥1 𝑦−𝑦1 𝑧−𝑧1
= =
𝑙 𝑚 𝑛

190
▪ The vector equation of a line which passes through two points whose position vectors
are 𝑎 𝑎𝑛𝑑 𝑏⃗ 𝑖𝑠 𝑟 = 𝑎 + 𝜆(𝑏⃗ − 𝑎)
▪ Cartesian equation of a line that passes through two points (𝑥1 , 𝑦1 , 𝑧1 ) 𝑎𝑛𝑑 (𝑥2 , 𝑦2 , 𝑧2 )
𝑥−𝑥1 𝑦−𝑦1 𝑧−𝑧1
is =𝑦 =𝑧 .
𝑥2 −𝑥1 2 −𝑦1 2 −𝑧1

▪ If 𝜃 is the acute angle between 𝑟 = ⃗⃗⃗⃗ ⃗⃗⃗1 𝑎𝑛𝑑 𝑟 = ⃗⃗⃗⃗


𝑎1 + 𝜆𝑏 ⃗⃗⃗⃗2 , 𝑡ℎ𝑒𝑛
𝑎2 + 𝜆𝑏
⃗⃗⃗⃗ .𝑏
𝑏 ⃗⃗⃗⃗
𝑐𝑜𝑠𝜃 = ||𝑏⃗⃗⃗⃗1||𝑏⃗⃗⃗⃗2 ||
1 2

𝑥−𝑥1 𝑦−𝑦1 𝑧−𝑧1 𝑥−𝑥2 𝑦−𝑦2 𝑧−𝑧2


▪ If = = 𝑎𝑛𝑑 = = are the equations of two lines, then the
𝑙1 𝑚1 𝑛1 𝑙2 𝑚2 𝑛2

acute angle between the two lines is given by 𝑐𝑜𝑠𝜃 = |𝑙1 𝑙2 + 𝑚1 𝑚2 + 𝑛1 𝑛2 |.


▪ Shortest distance between two skew lines is the length of the line segment
perpendicular to both the lines.
⃗⃗⃗⃗ ⃗⃗⃗⃗
▪ Shortest distance between 𝑟 = ⃗⃗⃗⃗ ⃗⃗⃗1 𝑎𝑛𝑑 𝑟 = ⃗⃗⃗⃗
𝑎1 + 𝜆𝑏 ⃗⃗⃗⃗2 𝑖𝑠 |(𝑏1 ×𝑏2 ).(𝑎⃗⃗⃗⃗⃗2 −𝑎⃗⃗⃗⃗⃗1 )|
𝑎2 + 𝜇𝑏 ⃗⃗⃗⃗ ×𝑏
|𝑏 ⃗⃗⃗⃗ | 1 2

𝑥−𝑥1 𝑦−𝑦1 𝑧−𝑧1 𝑥−𝑥2 𝑦−𝑦2 𝑧−𝑧2


▪ Shortest distance between the lines: = = 𝑎𝑛𝑑 = = 𝑖𝑠
𝑎1 𝑏1 𝑐1 𝑎2 𝑏2 𝑐2

𝑥2 − 𝑥1 𝑦2 − 𝑦1 𝑧2 − 𝑧1
| 𝑎1 𝑏1 𝑐1 |
𝑎2 𝑏2 𝑐2
√(𝑏1 𝑐2 − 𝑏2 𝑐1 )2 + (𝑐1 𝑎2 − 𝑐2 𝑎1 )2 + (𝑎1 𝑏2 − 𝑎2 𝑏1 )2

⃗ ×(𝑎
𝑏 ⃗⃗⃗⃗⃗1 )
⃗⃗⃗⃗⃗2 −𝑎
▪ 𝑎1 + 𝜆𝑏⃗ 𝑎𝑛𝑑 𝑟 = ⃗⃗⃗⃗
Distance between parallel lines 𝑟 = ⃗⃗⃗⃗ 𝑎2 + 𝜇𝑏⃗ 𝑖𝑠 | |𝑏 |⃗
|

▪ Two lines 𝑟 = ⃗⃗⃗⃗ ⃗⃗⃗1 𝑎𝑛𝑑 𝑟 = ⃗⃗⃗⃗


𝑎1 + 𝜆𝑏 ⃗⃗⃗⃗2 are coplanar if (𝑎
𝑎2 + 𝜇𝑏 ⃗⃗⃗⃗2 − ⃗⃗⃗⃗ ⃗⃗⃗1 × ⃗⃗⃗⃗
𝑎1 ). (𝑏 𝑏2 ) = 0

MULTIPLE CHOICE QUESTIONS

Q.NO QUESTIONS AND SOLUTIONS


1 What is the distance of point (𝑎, 𝑏, 𝑐) from 𝑥 −axis?
𝐴. √𝑏 2 + 𝑐 2 B. √𝑎2 + 𝑐 2 C. √𝑎2 + 𝑏 2 D. a
Solution: Distance between ( a, b, c ) and (a, 0, 0) is √𝑏 2 + 𝑐 2 .
Correct Option: A
2 What is the angle between the lines 2𝑥 = 3𝑦 = −𝑧 and 6𝑥 = −𝑦 = −4𝑧?
A. 0° B. 30° C. 45° D. 90°
1 1
Solution: DRs of first line are 2 , 3 and -1.
1 1
DRs of second line are 6 , −1, and −4
1 1 1 1−4+3
𝑎1 𝑎2 + 𝑏1 𝑏2 + 𝑐1 𝑐2 = −3+4= = 0.
12 12

So the lines are perpendicular.

191
Correct Option: D
3 𝑥−5 𝑦+4 𝑧−6
The cartesian equation of a line is 3 = = . Write its vector form
7 2
A. 𝑟 = 5𝑖ˆ + 4𝑗ˆ +6𝑘ˆ + 𝜆(3𝑖ˆ + 7𝑗ˆ − 2𝑘ˆ)
B. 𝑟=1𝑖ˆ − 2𝑗ˆ +4𝑘ˆ + 𝜆(3𝑖ˆ − 7𝑗ˆ − 2𝑘ˆ)
C. 𝑟=4𝑖ˆ − 5𝑗ˆ +6𝑘ˆ+ 𝜆(3𝑖ˆ − 7𝑗ˆ + 2𝑘ˆ)
D. 𝑟=5𝑖ˆ − 4𝑗ˆ +6𝑘ˆ + 𝜆(3𝑖ˆ + 7𝑗ˆ + 2𝑘ˆ)
Solution: Vector equation of the above is
𝑟=5𝑖ˆ − 4𝑗ˆ +6𝑘ˆ + 𝜆(3𝑖ˆ + 7𝑗ˆ + 2𝑘ˆ)
Correct Option: D
4 Write the equation of a line passing through (2, −3, 5) and parallel to line
𝑥−1 𝑦−2 𝑧+1
= 4 = −1 .
3
𝑥−3 𝑦−4 𝑧−1 𝑥−2 𝑦+3 𝑧−5
A. = = B. = = .
1 2 1 3 4 −1
𝑥−1 𝑦−2 𝑧+1 𝑥−2 𝑦−3 𝑧−5
C. = = . D. = = .
2 −3 5 3 4 −1
𝑥−1 𝑦−2 𝑧+1
Solution: Drs of = = are 3, 4, and −1.
3 4 −1
𝑥−2 𝑦+3 𝑧−5
So the required equation is = = .
3 4 −1

Correct Option: B
5 𝑥−1 𝑦−3 𝑧−1 𝑥−2 𝑦+1 𝑧
What is the value of 𝜆 for which the lines 2 = 5 = 𝜆 and 3 = −2 = 2
are perpendicular to each other?
A. +2 B. – 2 C. ±2 D. 0
Solution: 𝑎1 𝑎2 + 𝑏1 𝑏2 + 𝑐1 𝑐2 = 0  6 − 10 + 2 𝜆 =0. So 𝜆 = 2
Correct Option: A
6 Write line 𝑟 = (𝑖ˆ − 𝑗ˆ) + 𝜆(2𝑗ˆ − 𝑘ˆ ) into cartesian form.
𝑥−1 𝑦−1 𝑧−1 𝑥−1 𝑦+1 𝑧−0
A. = = B. = =
2 0 1 0 2 −1
𝑥−1 𝑦+1 𝑧 𝑥+1 𝑦−1 𝑧
C. = = −1 D. = =1
0 2 0 2

Solution: The above line is passing through (1, -1, 0) and is with DRs 0, 2, −1.
𝑥−1 𝑦+1 𝑧−0
So, the cartesian form the line is = =
0 2 −1

Correct Option: B
7 If the direction ratios of a line are 1, −2, 2 then what are the direction cosines of
the line?
−1 2 −2 −1 2 −2
A. , , B. , ,
3 3 3 √8 √8 √8
1 −2 2 1 −2 2
C. , , D. , ,
3 3 3 √8 √8 √8
𝑎 𝑏 𝑐
𝐒𝐨𝐥𝐮𝐭𝐢𝐨𝐧: If DRs are a, b and c then DCs are , ,
√𝑎2 +𝑏 2 +𝑐 2 √𝑎2 +𝑏2 +𝑐 2 √𝑎2 +𝑏 2 +𝑐 2
1 −2 2
So DCs are 3 , , .
3 3

Correct Option: C
8 Write equation of a line passing through (0, 1, 2) and equally inclined to co-
ordinate axes.

192
A. x = y −1 = z − 2 B. x = y = z
C. x = y + 1 = z + 2 D. x – 1 = y + 2 = z + 3
Solution: If a line is inclined equally then its DRs are 1, 1, 1.
So the equation of a line passing through (0, 1, 2) is
𝑥 = 𝑦 −1 = 𝑧 − 2
Correct Option: A
9 A line is defined by 5x − 3 = 15y + 7 = 1 − 10z. Its direction cosines are ___
6 2 −3 ±6 ±2 ±3
A. −7 , −7 , −7 B. 7 , 7 , 7
−6 −2 −3 6 2 −3
C. , , D. 7 , 7 ,
7 7 7 7
3 7 1
𝑥− 𝑦+ 𝑧−
5 15 10
Solution: The equation of the given line is 1 = 1 = 1

5 15 10
1 1 −1 6 2 −3
Drs are 5, 15, . So DCs are 7 , 7 ,
10 7

Correct Option: D
10 Find the direction cosines of the normal to YZ plane?
A. 0, 0, 0 B. 1, 0, 0 C. 0, 1, 0 D. 0, 0, 1
Solution: The direction cosines of the normal to YZ plane are
1, 0, 0
Correct Option: B

CHAPTER VIDEO LINK SCAN QR CODE FOR


VIDEO

THREE DIMENSIONAL https://youtu.be/XimWUG-c_J0


GEOMETRY

EXERCISE
x+2 y−5 z+1
1 The co-ordinates of the point, where the line 1 = = cuts the YZ plane
3 5
are ____.
A. (0, 11, 9) B. (9, 11, 0) C. (0, 0, 0) D. (0, 3, 5)
x−5 y−1 z−6
2 What is the X coordinate of the point where the line = = crosses ZX-
−2 3 −5
plane
17
A. – 2 B. 5 C. 3
D. 3

3 What are the direction cosines of the Y axis?

193
A. 0, 0, 0 B. 0, b, 0 C. 0,1,0 D. 1, 0, 1
4 What is the cosine of the angle which the vector √2 iˆ + ĵ + 2 k̂ makes with y –
axis.
1 1
A. 0° B. cos C. D. –1
√7 √7

5 For what value of  are the vectors 𝑎 = 2 iˆ + 𝜆 ĵ + k̂ and 𝑏⃗= iˆ –2 ĵ + 3 k̂


perpendicular to each other?
5 2 5 1
A. − 2 B. 5 C. 2 D. 2
Answers:
17 1 5
1. (0, 11,9) 2. 3 3. 0, 1, 0 4. 5. 2
√7

ASSERTION REASONING QUESTIONS

The following questions consist of two statements, one labelled as ‘Assertion (A)’ and
the other labelled as ‘Reason (R)’. Select your answer to these items using the codes
given below and then select the correct option.
(a) Both A and R are individually true and R is the correct explanation of A
(b) Both A and R are individually true but R is not the correct explanation of A
(c) A is true but R is false
(d) A is false but R is true
𝜋
1 Assertion: If a line makes an angle of 4 with each of y and z- axes, then it makes
a right angle with X axis.
Reason: The sum of the angles made by a line with the coordinate axes is 1800
Solution: cos 2 𝛼 + cos2 𝛽 + cos 2 𝛾 = 1
1 1
So cos 2 𝛼 = 1 – ( )2 – ( )2 ; Hence cos 𝛼 = 0 , 𝛼 = 900
√2 √2
A is True and R is False
So Answer is ( c )


2 Assertion: The acute angle between the line r = 𝑖̂+ 𝑗̂ + 2𝑘̂ +  (𝑖̂ – 𝑗̂) and
𝜋
X- axis is 4
Reason: If 𝜃 is the acute angle between 𝑟 = ⃗⃗⃗⃗
𝑎1 + 𝜆𝑏⃗⃗⃗1 𝑎𝑛𝑑 𝑟 = ⃗⃗⃗⃗
𝑎2 + 𝜆𝑏 ⃗⃗⃗⃗2 ,
⃗⃗⃗⃗ .𝑏
𝑏 ⃗⃗⃗⃗
then 𝑐𝑜𝑠𝜃 = ||𝑏⃗⃗⃗⃗1||𝑏⃗⃗⃗⃗2 ||
1 2
Solution: Angle between vectors with DRs 1, -1, 0 (Given line) and 1, 0, 0 (X-
⃗⃗⃗⃗ .𝑏
𝑏 ⃗⃗⃗⃗ 𝜋
axis) is 𝑐𝑜𝑠𝜃 = ||𝑏⃗⃗⃗⃗1||𝑏⃗⃗⃗⃗2 || i.e 𝜃 = 4
1 2

So A is true and R is the right reason


Answer: (a)

3 Assertion: The distance of a point P (a, b, c) from Y axis is b


Reason: Point on Y axis is (0, b, 0)

194
Solution: The distance of a point P (a, b, c) from Y axis is
√(𝑎 − 0)2 + (𝑏 − 𝑏)2 + (𝑐 − 0)2 = √(𝑎)2 + (𝑐)2

A is false and R is true. So the answer is (d)


4 Assertion: The vector form of the line through the point (5, 2, −4) and which is
parallel to the vector 2 iˆ + ĵ –6 k̂ is 2 iˆ + ĵ –6 k̂ + 𝜆( 5 iˆ + 2 ĵ –4 k̂ )
Reason: Vector equation of a line that passes through the given point whose
position vector is 𝑎 and parallel to a given vector 𝑏⃗ 𝑖𝑠 𝑟 = 𝑎 + 𝜆𝑏⃗.
Solution: The vector form of the line through the point (5, 2, −4) and which is
parallel to the vector 2 iˆ + ĵ –6 k̂ is 5 iˆ + 2 ĵ –4 k̂ + 𝜆( 2 iˆ + ĵ –6 k̂ )
A is false and R is true. So the answer is (d)
1− 𝑥 7𝑦−14 𝑧−3 7−7 𝑥 𝑦−5 6−𝑧
5 If l1: = = l2: = =
3 2p 2 3p 1 5
70
Assertion: If l1 ⊥ l2 then p = 11
Reason: If two lines with DRs a1, b1, c1 and a2, b2, c2 are perpendicular then
𝑎1 𝑎2 + 𝑏1 𝑏2 + 𝑐1 𝑐2 = 0

2𝑝 −3𝑝
Solution: The Drs of the given lines are –3, , and 2 ; , 7, –5
7 7
70
And 𝑎1 𝑎2 + 𝑏1 𝑏2 + 𝑐1 𝑐2 = 0 ⇒ p = 11
Hence A is true and R is the right reason.
So the answer is ( a)

EXERCISE
𝑥+1 𝑦+4 𝑧−7 2𝑥+4 𝑦−5 2𝑧−7
1 Assertion : The lines 1 = 0 = 0 and = 0 = 0 are parallel
2
Reason : Two lines are parallel if their DRS are proportional
2 Assertion : The angle between the lines whose DRs are given by 2l − m + 2n = 0
and mn + nl + lm = 0 is 900
Reason : Two lines with DRs a1, b1, c1 and a2, b2, c2 are perpendicular if
𝑎1 𝑎2 + 𝑏1 𝑏2 + 𝑐1 𝑐2 = 0
3 Assertion : Skew lines are non - intersecting non - parallel lines
Reason : They exsist in 3D space only.
1
4 Assertion : The angle between the diagonals of a cube is cos -1 (3)
Reason : The DRs of the diagonals of the cube are proportional to
𝑎, 𝑎, 𝑎 𝑎𝑛𝑑 − 𝑎, 𝑎, 𝑎

5 Assertion : The image of (0, 2, 0) in X axis is ( 0, -2, 0)


Reason : X axis is perpendicular to Y axis and with reference to (0, 2, 0), ( 0, 0, 0)
is foot of the perpendicular on X axis
ANSWERS:
Q1. ( a ) Q2. ( a ) Q3. ( b ) Q4. ( a ) Q5. (a)

195
2 MARK QUESTIONS
1 Find the equation of a line parallel to x-axis and passing through the origin.
Solution: Since the line is parallel to the x-axis,
The direction ratio of a line is given by (a, 0, 0).
𝑥−𝑥 𝑦−𝑦
The equation of a line is 𝑎 1 = 𝑏 1
Here (x1, y1, z1) = (0, 0, 0) , (a, b , c) = (a, 0 , 0)
x y z x y z
Required equation of a line is = = or = = = k or x = k.
a 0 0 1 0 0
2 Find the equation of a line passing though (𝟐, 𝟎, 𝟓) and which is parallel to
line 𝟔𝒙 − 𝟐 = 𝟑𝒚 + 𝟏 = 𝟐𝒛 − 𝟐
1 1 1
Solution: Drs of 6𝑥 − 2 = 3𝑦 + 1 = 2𝑧 − 2 are 6, 3, 2
2 1 2
𝑥− 𝑦+ 𝑧−
6 3 2
As 6𝑥 − 2 = 3𝑦 + 1 = 2𝑧 − 2 can be written as 1 = 1 = 1
6 3 2
 Required equation of the line passing through the point (2, 0, 5) with
1 1 1 𝑥−2 𝑦−0 𝑧−5
DRs 6, 3, is 1 = 1 = 1
2
6 3 2
3 Find the equation of the line passing through the points (𝟐, 𝟑, −𝟒) and
(𝟏, −𝟏, 𝟑) and parallel to the 𝒙 −axis.
Solution: The direction ratios of the two points (2, 3, −4) 𝑎𝑛𝑑 (1, −1, 3) are
(−1, −4, 7)
𝑥+1 𝑦+4 𝑧−7
Hence, the equation of line is = =
1 0 0

4 Show that the line through the points (4, 7, 8), (2, 3, 4) is parallel to the line
through the points (– 1, – 2, 1), (1, 2, 5)
Solution: Drs of line through the points (4, 7, 8), (2, 3, 4) are 2, 4, 4
Drs of line through the points (– 1, –2, 1), (1, 2, 5) are also 2, 4, 4.
So the lines are parallel

5 𝒙−𝟏 𝒚−𝟐 𝒛−𝟑 𝒙−𝟏 𝒚−𝟏 𝒛−𝟔


If the lines −𝟑 = 𝟐𝐤 = 𝟐 and 𝟑𝐤 = 𝟏 = −𝟓 are perpendicular,
find the value of k.
Solution: If the lines are perpendicular then a1 a2 + b1b2 + c1 c2 = 0
−10
−3(3k) + 2k (1) + 2(−5) =0 ⟹ −9k + 2k −10 = 0. k= 7
6 The cartesian equation of a line is 3x + 1 = 6y−2 = 1 – z. Find a point on the
line, its DRs and also its vector equation.
−1 1
Solution: The point on the line is ( 3 , 3 , 1)
Direction Ratios are 2, 1, −6
 −1 1
Vector Equation is r = 3 iˆ + 3 ĵ + k̂ +  (2 iˆ + ĵ –6 k̂ )

7 Find the cartesian equation of the line passing through the point (2, -1, 3)

and parallel to the line r = ( iˆ + ĵ ) +  (2 iˆ + ĵ –2 k̂ )

196
Solution: The DRs of required line are 2, 1, –2
𝒙−𝟏 𝒚−𝟏 𝒛
Cartesian Equation of the required line is 𝟐 = 𝟏 = −𝟐
8 If the coordinates of the points A, B, C, D are ( 1, 2, 3), ( 4, 5, 7), (−4, 3, −6)
and (2, 9, 2), then find the angle between AB and CD.

Solution: The DRs of line AB are 3, 3, 4


DRs of line CD are 6, 6, 8
As DRs are proportional, The angle between the lines is 0.
9 Find the value of k so that the lines x =−y = kz and x – 2 = 2y + 1 = – z + 1
are perpendicular to each other.
𝟏
Solution: The DRs of the line 1 are 1, –1, 𝐤
𝟏
DRs of line 2 are 1, 𝟐, –1
Now that the lines are perpendicular a1a2 + b1b2 + c1 c2 = 0 , So k = 2.

10 Find vector equation of the line parallel to X axis and passing through origin.
Solution: DRs of X axis: 1, 0, 0 and given point is (0, 0, 0)
𝒙 𝒚 𝒛
So the required equation is 𝟏 = 𝟎 = 𝟎

EXERCISE
1 Find the projection of the line segment joining the points (−1, 0, 3) and (2, 5, 1) on
the line whose direction ratios are (6, 2, 3)
2 Find the direction ratios of a line perpendicular to the lines having direction ratios
(1, 3, 2) and (–2, 2, 4) respectively.
3 Find the vector equation of the line passing through the point (1, 2, –3) and
parallel to the vector 2i + 3j–4k.
4 Find the vector equation of the straight line passing through the Points (2, 1, –3)
and (5, –4, 1).
5 𝑥−1 2−𝑦 𝑧 𝑥−3 𝑦−4 2−𝑧
Find the angle between the lines: = = −4 and = =
2 2 1 3 1

6 Find the angle between the lines with direction ratios (2, 2, 1) and the line joining
(3, 1, 4) to (7, 2, 12).
7 Find the vector equation of the line passing through the point A (1, 2, –1) and
parallel to 5x – 25 = 14 – 7y = 35z
8 12 −3 −4 4 12 3 3 −4 12
Show that the lines with direction cosines 13, , ; , , ; , ,
13 13 13 13 13 13 13 13

are mutually perpendicular.

197
𝑥−1 7𝑦−14 𝑧−3 7−7𝑥 𝑦−5 6−𝑧
9 Find the value of  , if the lines = = and , = are at
−3 2𝜆 2 3𝜆 1 5

right angles.

10 If A, B are points (2, 3, -6) and (3, −4, 5), find the angle that OA makes with OB
where O is origin.

Answers:
22
Q1. Q2. 8, –8, 8 (Hint Find 𝑎 X 𝑏⃗ )
7

Q3. r = 1 iˆ + 2 ĵ –3 k̂ +  (2 iˆ + 3 ĵ –4 k̂ )
 2
Q4. r = 2 iˆ + 1 ĵ –3 k̂ +  (3i – 5j +4k) Q5. 0 Q6. cos-1 (3)

Q7. r = 1 iˆ + 2 ĵ – k̂ +  (7 iˆ – 5 ĵ + k̂ ) Q8. Hint: a1a2 + b1b2 + c1 c2 = 0
70 18√2
Q9. 𝜆 = 11 Q 10. cos 𝜃 = 35

3 MARK QUESTIONS

1 If a line makes angle 𝜶, 𝜷, 𝜸 with co-ordinate axes then what is the value of
𝐬𝐢𝐧𝟐 𝜶 + 𝐬𝐢𝐧𝟐 𝜷 + 𝐬𝐢𝐧𝟐 𝜸
Solution: sin2 𝛼 + sin2 𝛽 + sin2 𝛾 = (1– cos2 𝛼) + (1– cos 2 𝛽) + (1– cos2 𝛾 )
= 3 – ( cos2 𝛼 + cos2 𝛽) + cos 2 𝛾)
=3–1=2

2 Find the equation of a line passing through the point (𝟐, 𝟎, 𝟏) and parallel to
⃗ = (𝟐𝝀 + 𝟑)𝒊ˆ + (𝟕𝝀 − 𝟏)𝒋ˆ + (𝟑𝝀 + 𝟐)𝒌ˆ
the line whose equation is 𝒓
Solution: The DRs of the line 𝑟 = (2𝜆 + 3)𝑖ˆ + (7𝜆 − 1)𝑗ˆ + (3𝜆 + 2)𝑘ˆ
are 2, 7, 3
 Required equation of the line passing through the point (2, 0, 1) with
𝑥−2 𝑦−0 𝑧−1
DRs 2, 7, −3 is 2 = 7 = 3
3 Find the vector equation of a line passing through the point (1, 2, -4) and
𝒙−𝟖 𝒚+𝟏𝟗 𝒛−𝟏𝟎 𝒙−𝟏𝟓 𝒚−𝟐𝟗 𝒛−𝟓
perpendicular to two lines 𝑳𝟏 = 𝟑 = −𝟏𝟔 = 𝟕 and 𝑳𝟐 = 𝟑 = 𝟖 = −𝟓
Solution: DRs of the required lines can be obtained by calculating the determinant
𝑖 𝑗 𝑘
|3 −16 7 | which is equal to 24𝑖̂ + 36 ĵ +72𝑘̂
3 8 −5
 DRs are 24, 36, 72
i.e DRs 2, 3, 6
equation of line is 𝑟̂ = 𝑖̂ + 2𝑗̂ − 4𝑘̂ + 𝝀(𝟐𝑖̂ + 3𝑗̂ + 6𝑘̂ )

198
4 Find the angle between the lines whose direction ratios are a, b, c and
b – c, c – a, a – b.
𝑎(𝑏−𝑐)+𝑏(𝑐−𝑎)+𝑐(𝑎−𝑏)
Solution: cos  =
√𝑎2 +𝑏2 +𝑐 2 √(𝑏−𝑐)2 +(𝑐−𝑎)2 +⥂⥂(𝑎−𝑏)2
𝑎𝑏−𝑎𝑐+𝑏𝑐−𝑎𝑏+𝑎𝑐−𝑏𝑐
= =0  𝜃 = 𝜋⁄2
√𝑎2 +𝑏2 +𝑐 2 √(𝑏−𝑐)2 +(𝑐−𝑎)2 +⥂⥂(𝑎−𝑏)2

5 If the coordinates of the points A, B, C, D be (1, 2, 3), (4, 5, 7), (– 4, 3, – 6) and


(2, 9, 2) respectively, then find the angle between the lines AB and CD.

Solution: DRs of AB are 3, 3, 4 and DRs of CD are 6, 6, 8


as DRs of both the lines are proportional the angle between them is 0.
i.e. the lines are parallel.
6 The points A (4, 5,10), B (2, 3, 4) and C (1, 2, –1) are the three vertices of a
parallelogram ABCD. Find vector equation for the diagonal BD.
Solution: The coordinates of D be (x, y, z).
5 7 9 2+𝑥 3+𝑦 4+𝑧
Midpoint of AC is same as midpoint of BD. So ( 2 , 2 , 2 ) = ( , , )
2 2 2
So, D (x, y, z) is (3, 4, 5) and 𝑟 = 𝑎 + 𝜆(𝑏⃗ − 𝑎 )

∴ Equation of BD is r = (2 iˆ + 3 ĵ + 4 k̂ ) +  ( iˆ + ĵ + k̂ )
7 𝟐+𝒙 𝟏+𝒚 𝒛−𝟑
Find the point on the line 𝟑 = 𝟐 = 𝟐 at a distance of 3√𝟐 from the
point
P (1, 2, 3).
𝒙+𝟐 𝒚+𝟏 𝒛−𝟑
Solution : The general point Q on the given line 𝟑 = 𝟐 = 𝟐 is
x = 3  −2, y = 2  −1 z = 2  +3
Given PQ = 3√2 . ∴ √(3𝜆 − 2 − 1)2 + (2𝜆 − 1 − 2)2 + (2𝜆 + 3 − 3)2 = 3√2

𝟑𝟎 56 43 111
So 𝜆 = , And the required point P is (17 , 17 , )
𝟕 17

8 𝒙−𝟑 𝒚+𝟕 𝒛+𝟐


Find the distance from the point P(3, -8, 1) to the line 𝟑 = =
−𝟏 𝟓
Solution: Q (3, -7, -2) is a point on the given line.
𝑏⃗ = 3 iˆ - ĵ + 5 k̂ are the DRs of the above line.
|⃗b X ⃗⃗⃗⃗
PQ|
Then the distance ‘d’ from a point P(3, -8, 1) from the line is given by d = |⃗𝑏|
𝑖 𝑗 𝑘
⃗b X ⃗⃗⃗⃗⃗
PQ = |3 −1 5 | = 2 iˆ − 9 ĵ −3 k̂
0 1 −3
94
and |⃗b X ⃗⃗⃗⃗
PQ| = √94 , |⃗b | = √35 ∴ d = √35

9 Find the DRs of the line perpendicular to the lines passing through A (2 ,3, −4),
B ( −3, 3, 2) and C( −𝟏, 𝟒, 𝟐) D (3, 5, 1)
Solution: DRs of the line passing through A (2 ,3 , −4), B ( −3, 3, 2) are −5, 0, 6
DRs of the line passing through C( −1, 4, 2) D (3, 5, 1) are 4, 1, −1
DRs of the line perpendicular to AB and CD are
𝑖 𝑗 𝑘
|−5 0 6 | = –6i +19j –5k
4 1 −1

199
10 Find the angle between the lines whose direction cosines are given by
l + m + n =0 and l2+m2−n2 =0
Solution: Given l2 + m2 = n2 and l + m + n=0 and we know that l2 + m2 + n2 = 1
1
By solving above equations, we get n = ±
√2
1 1 1
For n = , then we get l = ± , m=0 or l=0, m = ±
√2 √2 √2
1 1 1 1
So the one possible set of DRs are ( , 0, ) and (0, , )
√2 √2 √2 √2

𝑎1 𝑎2 +𝑏1 𝑏2 +𝑐1 𝑐2 1
Let the angle between them be θ We have cos θ = | |=2
√𝑎12 +𝑏12 +𝑐12 √𝑎22 +𝑏22 +𝑐22
𝜋
So we get θ = 3

EXERCISE

1 Find the equation of the line joining (1, 2, 3) and (–3, 4, 3) and show that it is
perpendicular to z –axis.
2 Find the coordinates of the point where the line through the points A (3, 4, 10) and
B (5, 1, 6) crosses XY plane
3 Find the equation of a line which passes through (5, –7, –3) and is parallel to the
𝑥−2 𝑦+1 𝑧−7
line = =
3 1 9

4 𝑥+3 𝑦−1 𝑧−5 𝑥+1 𝑦−2 𝑧−5


Show that the lines = = and = = are coplanar.
−3 1 5 −1 2 5

Hint : Show (𝑎 ⃗⃗⃗1 × ⃗⃗⃗⃗


𝑎1 ). (𝑏
⃗⃗⃗⃗2 − ⃗⃗⃗⃗ 𝑏2 ) = 0
5 Find the equation of a straight line through (1, –2, 3) and equally inclined to the
axes.
6 Show that the line joining the origin to the point (2, 1, 1) is perpendicular to the
line determined by the points (3, 5, –1), (4, 3, –1).
7 𝑥−1 𝑦+1
Find the perpendicular distance of the point (1, 0, 0) from the line 2 = −3 =
𝑧+10
8

8 𝑥+2 𝑦+1 𝑧−3


Find the points on the line = = at a distance of 5 units form the point
3 2 2

P (1, 3, 3)
9 Find the points on the line through A(1, 2, 3) B (3, 5, 9) at a distance of 14 units
form the midpoint of line segment AB.
10 Find the equation of the line passing through A ( −2, 4, 7), B (3, −6, −8). Hence
show A, B and C (1, −2, −2) are collinear.
Answers:
Q1. Hint: DRs of line joining (1, 2, 3) and (–3, 4, 3) are –4, 2, 0. DRs of Z axis are 0, 0, 1
So the line and Z axis are perpendicular.
−7 𝑥−5 𝑦+7 𝑧+3
Q2. ( 8, 2 , 0) Q3. 3 = 1 = 9

200

Q5. r = iˆ – 2 ĵ + 3 k̂ +  ( iˆ + ĵ + k̂ ) Q6. Hint: 𝑎1 𝑎2 + 𝑏1 𝑏2 + 𝑐1 𝑐2 = 0
Q7. 2√6 Q8. ( -2, -1, 3) and (4, 3, 7)
Q9. (7,11,21) and (-1,-1,-3). Q10. AB = −2 iˆ + 4 ĵ +7 k̂ +  (5 iˆ –10 ĵ –15 k̂ )
5 MARK QUESTIONS

1 Find the shortest distance between lines


 
r = 6 iˆ + 2 ĵ + 2 k̂ +  ( iˆ – 2 ĵ +2 k̂ ) and r = –4 iˆ – k̂ +  (3 iˆ – 2 ĵ –2 k̂ ).
 
Solution: Let a1 = 6 iˆ + 2 ĵ + 2 k̂ a 2 = –4 iˆ – k̂
 
b1 = iˆ – 2 ĵ + 2 k̂ b2 = 3 iˆ – 2 ĵ –2 k̂
⃗ 1 ×𝑏
(𝑏 ⃗ 2 )⋅(𝑎⃗2 −𝑎⃗1 )
Shortest distance, d = | ⃗ 1 ×𝑏⃗ 2|
|
|𝑏
î ĵ k̂
⃗𝑏1 × 𝑏⃗2 = |1 −2 2 | = 8 iˆ + 8 ĵ + 4 k̂
3 −2 −2
 
( a 2 − a1 ) = –10 iˆ –2 ĵ – 3 k̂

(𝑏⃗1 X 𝑏⃗2 ) . (𝑎2 − 𝑎1 ) = –80 – 16 – 12  –108


108
|𝑏⃗1 × 𝑏⃗2 | = √64 + 64 + 16 = √144 = 12.  d = 12 = 9 unit.

2 𝒙 𝒚−𝟏 𝒛−𝟐
Find the image of a point P (1, 6, 3) with respect to the line = =
𝟏 𝟐 𝟑
Solution:
P (1, 6, 3)

A (0, 1, 2) M B (1, 3, 5)

Q
Let M be the foot of the perpendicular.
𝑥 𝑦−1 𝑧−2
Let 1 = 2 = 3 = 𝜆 ( say)
General point on the line AB is x = 𝜆 , y = 2 𝜆 + 1, z = 3 𝜆 + 2
DRs of PM = 𝜆 – 1, 2 𝜆 – 5, 3 𝜆 – 1
PM is perpendicular to AB. So a1 a2 + b1b2 + c1 c2 = 0
 1(𝜆 −1) + 2(2 𝜆 −5) + 3(3 𝜆 − 1) =0
 𝜆 −1 + 4 𝜆 -10 + 9 𝜆 −3=0
So 14 𝜆=14, and 𝜆 =1
∴ Point M is (1, 3, 5)
Let the image be Q (x3, y3, z3)
Now , using M as mid point of PQ
𝑥1 +𝑥3 1+𝑥3
= x 2 ⇒ = 1 and x3 = 1
2 2
𝑦1 +𝑦3 6+𝑦3
= y2 ⇒ = 3 and 𝑦3 = 0
2 2
𝑧1 +𝑧3 3+𝑧3
= z2 ⇒ = 5 and z3 = 7
2 2
∴ Image Q is (1, 0, 7)

201
3 𝒙−𝟏 𝒚−𝟐 𝒛−𝟑 𝒙−𝟒 𝒚−𝟏
Show that the line 𝟐 = 𝟑 = 𝟒 and 𝟓 = 𝟐 = 𝒛 intersect.
Also find their point of intersection.
Solution:
Part I : (Hint : Show that the shortest distance between the lines is 0 using
shortest
distance formula.)
Part II : For finding their point of intersection for first line.
𝑥−1 𝑦−2 𝑧−3
 = = =𝜆
2 3 4

 x = 2𝜆 + 1, y = 3 𝜆 + 2, z = 4 𝜆 + 3
Since, the lines are intersecting. So, Let’s put these values in equation of
another line.
2λ+1−4 3λ+2−1 4λ+3
Thus, = =
5 2 1
2λ−3 3λ+1 4λ+3
 = =
5 2 1
2λ−3 4λ+3
 = ⇒ 2 𝜆−3 = 20 𝜆+15 ⇒ 18 𝜆 = −18 = −1
5 1

So, the required point of intersection is


x = 2(−1) + 1 = −1, y = 3(−1) + 2 = −1, z = 4(−1) + 3 = −1
Thus, the lines intersect at (-1, -1,-1).
4 If the points (-1, 3 , 2), (-4, 2 , -2) and (5, 5,  ) are collinear, Use the concept of lines
to find the value of ‘  ’.
  
Solution: Let the position vectors of the points be a , b & c
  
 a = − iˆ + 3 ĵ + 2 k̂ b = −4𝑖̂ + 2 ĵ − 2 k̂ c = 5 iˆ + 5 ĵ +  k̂
 
The equation of a line passing through the points where position vectors a and b
  
is given by 𝑟 = a +  ( b - a )
𝑟 = − iˆ + 3 ĵ + 2 k̂ +  ( − 4iˆ + 2 ĵ − 2 k̂ + iˆ − 3 ĵ − 2 k̂ )
= − iˆ + 3 ĵ + 2 k̂ +  ( −3 iˆ − ĵ − 4 k̂ )

As it passes through c ,
5 iˆ + 5 ĵ +  k̂ = - iˆ + 3 ĵ + 2 k̂ +  ( −3 iˆ − ĵ −4 k̂ )
6 iˆ + 2 ĵ + (  − 2) k̂ =  ( −3 iˆ − ĵ − 4 k̂ )

6 = −3    = −2.
 − 2 = −4    = −4 (−2) + 2 ⇒  = 8+2 
 = 10.

EXERCISE

1 Find the shortest distance between the lines whose vector equations are
𝑟 = (1 − 𝑡)𝑖ˆ + (𝑡 − 2)𝑗ˆ + (3 − 2𝑡)𝑘ˆ and 𝑟 = (𝑠 + 1)𝑖ˆ + (2𝑠 − 1)𝑗ˆ − (2𝑠 + 1)𝑘ˆ
2 𝑥−1 𝑦+1 𝑧−1 𝑥−2 𝑦−1 𝑧+1
Show that the lines 3
= 2
= 5
& 4
= 3
= −2
do not intersect each
other.

202
3 Find the foot of the perpendicular drawn from the point A(1, 0, 3) to the join of
the points B(4, 7, 1) and C(3, 5, 3).
4 x +1 y + 3 z + 5 x − 2 y − 4 z − 6
Show that the lines = = = = intersect and find
3 5 7 1 3 5
their point of intersection.
5 x y −1 z − 2
Find the image of the point (1, 6, 3) in the line = = .
1 2 3
Find the distance between lines 𝑟 = 𝑖̂+ 2𝑗̂− 4𝑘̂ + 𝜆(2𝑖̂+ 3𝑗̂+ 6𝑘̂) and
𝑟 = 3𝑖̂+ 3𝑗̂− 5𝑘̂ + 𝜇(2𝑖̂+ 3𝑗̂+ 6𝑘̂).
Answers:
8 5 7 17
Q1. Q2. Hint: Show that shortest distance is not ‘0’. Q3. ( , , )
√29 3 3 3

1 −1 −3 √293
Q4. (2, , ) Q5. (1, 0, 7) Q6.
2 2 7

CASE BASED QUESTIONS

I The equation of motion of a rocket are: x = 2t, y = -4t, z = 4t, where the time 't' is
given in seconds, and the distance measured is in kilometers.

Based on the above information, answer the following questions.


(i) What is the path of the rocket?
(a) Straight line (b) Circle (c) Parabola (d) None of these
(ii) Which of the following points lie on the path of the rocket?
(a) (0, 1, 2) (b) (1, -2 , 2) (c) (2 , -2, 2) (d) None of these
(iii) At what distance will the rocket be from the starting point (0, 0, 0) in 10
seconds?
(a) 40 km (b) 60 km (c) 30 km (d) 80 km
(iv) If the position of rocket at certain instant of time is (3, -6, 6), then what will
be the height
of the rocket from the ground, which is along the xy-plane?

203
(a) 3 km (b) 2km (c) 4 km (d) 6 km

Answers: i) a ii) b iii) b iv) d

2 A butterfly is moving in a straight path in the space. Let this path be denoted by a
𝑥−1 2𝑦 + 2 3𝑧−3
line l whose equation is 2 = 4 = −6 say.

Using the information given above, answer the following with reference to the line
l:
(i) What are the direction ratios of the line?
(a) 2, 4, –6 (b) –2, 2, 2 (c) 2, –3, 4 (d) 2, 2, –2
(ii) If the z-coordinate of a point on this line is 7, then the x-coordinate of the
same point on
this line, is
(a) –5 (b) 5 (c) 0 (d) 1
(iii) The vector equation of the given line is
 
(a) r = (2 iˆ + 4 ĵ − 6 k̂ ) +  ( iˆ + 2 ĵ + 3 k̂ ) (b) r = ( iˆ − 2 ĵ + 3 k̂ ) +  (2 iˆ + 4
ĵ − 6 k̂ )
 
(c) r = ( iˆ + 2 ĵ − 4 k̂ ) +  (2 iˆ + 3 ĵ + 6 k̂ ) (d) r = ( iˆ − ĵ + k̂ ) +  (2 iˆ + 2 ĵ −
2 k̂ )
(iv) The unit vector in the direction of the vector parallel to the given line, is
1 1 1 1 1 1 1 1 1 1
(a) iˆ − ĵ + k̂ (b) iˆ + ĵ − k̂ (c) iˆ − ĵ − k̂ (d) iˆ +
√3 √3 √3 √3 √3 √3 √3 √3 √3 √3
1 1
ĵ + k̂
√3 √3

Answers: i) d ii) a iii) d iv) b

204
CHAPTER : LINEAR PROGRAMMING
SYLLABUS: Introduction, related terminology such as constraints, objective function,
optimization, graphical method of solution for problems in two variables, feasible and
infeasible regions (bounded or unbounded), feasible and infeasible solutions, optimal feasible
solutions (up to three non-trivial constraints.)

Definitions and Formulae:

1) Let R be the feasible region (convex polygon) for a linear programming problem and
let Z = ax + by be the objective function. When Z has an optimal value (maximum or
minimum), where the variables x and y are subject to constraints described by linear
inequalities, this optimal value must occur at a corner point* (vertex) of the feasible
region.
2) Let R be the feasible region for a linear programming problem, and let
Z = ax + by be the objective function. If R is bounded**, then the objective
function Z has both a maximum and a minimum value on R and each of these occurs
at a corner point (vertex) of R
Remark: If R is unbounded, then a maximum or a minimum value of the
objective function may not exist. However, if it exists, it must occur at a corner
point of R.
➢ Solving linear programming problem using Corner Point Method.
The method comprises of the following steps:
1. Find the feasible region of the linear programming problem and determine its corner
points(vertices) either by inspection or by solving the two equations of the lines
intersecting at the point.
2. Evaluate the objective function Z=ax+ by at each corner point. Let M and m,
respectively denote the largest and smallest values of these points.
3. (i) When the feasible region is bounded, M and m are the maximum and
minimum value of Z.
(ii) In case, the feasible region is unbounded, we have:
(a) M is the maximum value of Z , if the open half plane determined by
ax+ by >M has no point in common with the feasible region.
Otherwise, Z has no maximum value.
(b) Similarly, m is the minimum value of Z, if the open half plane determined by
ax+ by<m has no point in common with the feasible region.
Otherwise, Z has no minimum value.

205
MULTIPLE CHOICE QUESTIONS

Q.NO QUESTIONS AND ANSWERS


1 Solution set of the inequality 2x + y > 5 is
a) Half-plane containing origin
b) Half plane not containing origin
c) xy-plane except the points on the line 2x + y = 5
d) No solution
Solution: Origin does not satisfy this inequality.
Ans: b
2 Objective function of a LPP is
a) constant graph
b) A function to be optimized
c) Inequality
d) Quadratic function
Solution: Objective function is a function to be optimized.
Ans: b
3 In an LPP, if the objective function Z = ax + by has same maximum at two corner
points of the feasible region, then the number of points at which maximum value
of Z occurs is
a) 0 b) 1 c) 2 d) infinite
Solution: every point on the line joining these two corner points gives same
maximum.
Ans: d
4 The corner points of the feasible region determined by a system of linear
inequalities are (0, 0), (4, 0), (2, 4) and (0, 5). If the maximum value of Z = ax +
by where a,b > 0 occurs at both (2, 4) and (4, 0), then
a) a = 2b b) 2a = b c) a = b d) 3a = b
solution: Z(2, 4) = Z(4, 0) => 2a + 4b = 4a => a = 2b
Ans: a
5 A linear programming problem is as follows:
Maximize /minimize objective function Z = 2x – y + 5 subject to constraints
3x + 4y ≤60, x + 3y ≤ 30, x ≥ 0, y ≥ 0.
If the corner points of the feasible region are A(0, 10), B(12, 6), C(20, 0) and O(0,
0), then which of the following are true
a) Maximum value of Z is 40

206
b) Minimum value of Z is -5
c) Difference between maximum and minimum values of Z is 35
d) At two corner points the values of Z are equal
Solution: Z(0, 10) = 2x0 -10 + 5 = -5 is minimum value of Z Ans: b
6 A linear programming problem is as follows:
Minimize Z = 2x + y subject to constraints x ≥ 3, x ≤ 9, y ≥ 0, x – y ≥ 0, x + y ≤
14
The feasible region has
a) 5 corner points including (0, 0) and (9, 5)
b) 5 corner points including (7, 7) and (3, 3)
c) 5 corner points including (14, 0) and (9, 0)
d) 5 corner points including (3, 6) and (9, 5)
Solution: Five corner points as shown in the figure.

It includes (3,3) and (7, 7)


Ans : b
7 The objective function Z = ax + by of an LPP has maximum value 42 at (4, 6) and
minimum value 19 at (3, 2). Which of the following is true
a) a = 9, b = 1 b) a = 5, b = 2
c) a = 3, b = 5 d) a = 5, b = 3
solution: Z(4, 6) = 42 => 4a + 6b =42 Z(3, 2) = 19 => 3a + 2b = 19 solving we get
a = 3, b = 5
Ans:c
8 The corner points of the feasible region of a linear programming problem are (0,
4), (8, 0) and (20/3, 4/3). If Z = 30x + 20y is the objective function, then
(Maximum value of Z – Minimum value of Z) is equal to
a) 40 b) 96 c) 160 d) 136
Solution: Max. = Z(8, 0) =240 and Min. = Z(0, 4) = 80
Ans: c
9 The position of points O(0, 0) and P(2, -1) is-------, in the solution region of the
inequality 2y – 3x < 5
a) O is inside the region and P is outside the region

207
b) O and P both are inside the region
c) O and P both are outside the region
d) O is outside and P is inside the region
Solution: Both O and P satisfy the inequality.
Ans: b
10 If the corner points of the feasible region of an LPP are (0, 2), (3, 0), (6, 0), (6, 8)
and (0, 5), then the minimum value of the objective function Z = 4x + 6y occurs
at
a) (0, 2) only
b) (3, 0) only
c) The midpoint of the line segment joining the points (0, 2) and (3, 0)
only
d) Every point on the line segment joining the points (0, 2) and (3, 0)
Solution: If Z has same min.value at two points, then Z has same min. value at
every point on the line segment joining the two points.
Ans: d

CHAPTER VIDEO LINK SCAN QR CODE FOR VIDEO

LINEAR
PROGRAMMING https://youtu.be/Zx0yeGYW0kw

EXERCISE

1 The feasible region of constraints x+ y ≤ 4, 3x+ 3y ≥18, x ≥ 0, y ≥ 0 defines on


.........
a) bounded feasible region
b) unbounded feasible region
c) feasible region in first and second quadrants
d) does not exist
2 The maximum value of Z = 3x+ 4y subject to constraints x+ y ≤ 4, x ≥ 0, y ≥ 0
is

208
a) 16 b) 12 c) 0 d) not possible
3 Which of the following statement is correct?
(a) Every L.P.P. admits an optimal solution
(b) An L.P.P. admits a unique optimal solution
(c) If an L.P.P. admits two optimal solutions, it has an infinite number of
optimal solutions
(d) The set of all feasible solutions of a L.P.P. is not a convex set.
4 The shaded region in the given figure is the graph of.........

y
A(0,
3/2)

B(-
x
3/4,0)
O
(a) 4x– 2y≤3
(b) 4x– 2y≤–3
(c) 2x– 4y≥3
(d) 2x– 4y≤–3
5 Under the constraints x – 2y ≤ 6, x+ 2y ≥ 0, x ≤ 6, the maximum value of Z = 3x
+ 4y is
(a) 16 b) 17 c) 18 d) 19
ANSWERS:
1) d 2) a 3) c 4) a 5) c

ASSERTION AND REASONING QUESTIONS

The following questions consists of two statements-Assertion(A) and Reason(R).


Answer these questions selecting appropriate option given below.
a) Both A and R are true and R is the correct explanation for A
b) Both A and R are true and R is not the correct explanation for A
c) A is true but R is false
d) A is false but R is true

1 Assertion (A):The maximum value of Z = 5x + 3y, satisfying the conditions


x ≥ 0, y ≥ 0 and 5x + 2y ≤ 10, is 15

209
Reason(R): A feasible region may be bounded or unbounded
Solution: corner points are (0, 0), (2, 0) and (0, 5)
Zmax =5x0 + 3x5 =15 at (0, 5)

So, both A and R are true but R is not the correct explanation for A
Ans: b
2 Assertion (A):The max. value of Z = x + 3y subject to 2x + y ≤ 20, x + 2y ≤ 20
x ≥ 0, y ≥ 0 is 30

Reason(R): The variables that are present in the problem are called decision
variables.
Solution: corner points are (0, 0), (10, 0), (20/3, 20/3) and (0, 10)
Zmax. =x + 3y = 0 + 3x10 = 30

both A and R are true but R is not the correct explanation for A
Ans: b
3 Assertion (A): The feasible region represented by 2x + 5y ≥ 80, x + y ≤ 20, x ≥ 0,
y ≥ 0 is bounded.
Reason(R): A region is said to be convex if the line joining any two of its
points lies completely in the region.
Solution: There is no feasible region
=> A is false R is true
Ans: d
4 Assertion (A): The maximum value of Z = 11x + 7y subjected to 2x + y ≤ 6, x ≤
2, x ,y ≥ 0, occurs at (0, 6)
Reason(R): If the feasible region of an LPP is bounded, then maximum and
minimum
Value of the objective function occurs at corner points.
Solution: corner points are (0, 6), (3, 2), (3, 0)
=> Z is max. at (3, 2) A is false , clearly R is true
Ans: d
5 The corner points of the feasible region for an LPP are (60, 0), (120, 0), (60, 30)
and (40, 20). The objective function Z = ax + by, a, b > 0 has maximum value
600 at points (120, 0) and (60, 30)
Assertion (A): Minimum value of Z is 300
Reason(R): a = 5, b = 10
Solution: Z = ax + by maximum value 600 at points (120, 0) and (60, 30)

210
120a + 0 = 600 => a = 5

Also, 60a + 30b = 600 => 60x5 + 30b = 600 => b = 10


Z at (60, 0) = 5 x 60 + 0= 300 is min.
Ans: a
6 Assertion (A): If the feasible region of an LPP is bounded, then the objective
function Z = ax + by has both maximum and minimum values.
Reason(R): A feasible region of a system of linear inequalities is said to be
bounded if it can be enclosed within a circle.
Solution: conceptual /theory
Ans; b
7 Corner points of feasible region are (0, 0), (3, 0) and (0, 3) and the objective
function is Z = 4x + 3y
Assertion (A): Minimum value of Z is 9
Reason(R): Maximum value of Z is 21
Solution: Z is min. at (0, 3) => Z min. = 4x0 +3x 3 =9
Cleary, A is true and R is false
Ans: c
8 Assertion (A): The point (4, 2) does not lies in the half plane 4x + 6y – 24 < 0
Reason(R): The point (1, 2) lies in the half plane 4x + 6y – 24 < 0
Solution: Clearly A is true and R is false
Ans: c
9 Assertion (A): If the corner points of the feasible region for an LPP are (0, 4), (1,
4), (4, 1) and (12, -1), then the minimum value of the objective function Z = 2x +
4y is at (4, 1)
Reason(R): If the corner points of the feasible region for an LPP are (0, 4), (1,
4), (4, 1) and (12, -1), then the maximum value of the objective function Z = 2x +
4y is 20.
Solution: min = Z(4, 1) = 2x4 + 4x1 = 12, max.= Z(12, -1)= 2x12 + 4x(-1) = 20
Hence A is true and R is true but R is not correct explanation
Ans: b
10 The corner points of the feasible region for an LPP are (4, 0),(5, 0), (5, 3), (3, 5),
(0, 5) and (0, 4). The objective function Z = ax - by + 1900, a, b > 0 has maximum
value 1950 at (5, 0) and minimum 1550 at (0, 5).
Assertion (A): The value of Z at the point (5, 3) is 1740
Reason(R): a = 10, b = 70

211
Solution: Z max.at (5, 0) = 1950 => 5a – bx0 + 1900 = 1950 => a = 10
Zmin.at (0,5) = 1550 => ax0 – bx5 + 1900 =1550 => b = 70
Z(5, 3) = 10x5 – 70x3 + 1900 = 1740
Ans : a

EXERCISE

1 Assertion: All feasible regions are convex sets


Reason: A set is said to be convex set if the line segment joining any two points
of the set, is completely within the set
2 Assertion: Graphical method is not suitable for solving all Linear programming
problems
Reason: Graphical method is applicable only in case of LPP having two variables
3 Assertion: The objective function describes the purpose of formulating LPP
Reason: The objective function can be maximized or minimized
4 Assertion: The objective function is always non-negative
Reason: The variables involved in the objective function are non-negative due to
constraints

ANSWERS:
1) d 2) a 3) a 4) a

2 MARKS QUESTIONS

1 Minimise Z = 13x – 15y subject to the constraints x + y ≤ 7, 2x – 3y + 6 ≥ 0, x ≥


0, y ≥ 0
Solution:clearly Z is min. at C(0, 2) and Zmin.= 13x0 -15x2 = 30

B(3, 4)
C(0, 2)

A(7, 0)
O
2 Maximise Z = 80x + 120y subject to the constraints 3x + 4y ≤ 60, x + 3y ≤ 30,
𝑥, 𝑦 ≥ 0
Solution: The corner points are (0, 0), (20, 0), (12, 6), (0, 10)

212
y

C(0, 10)
B(12, 6)

A(20, 0) x
O
Corner points Z= 80x + 120y

(0,0) 0
(20, 0) 1600
(12, 6) 1680 maximum
(0, 10) 1200

3 Maximise Z = 100x + 120y subject to the constraints 5x + 8y ≤ 200, 5x + 4y


≤120, x,y≥0
Solution: Corner Z = 100x +
points 120y

(0, 0) 0
(0, 25) 3000
(24, 0) 2400
(8, 20) 3200

y
C(0, 25)

B(8, 20)

O A(24, 0) x
4 Maximise Z = 20x + 10y subject to the constraints 1.5x + 3y ≤ 42, 3x + y ≤ 24, x ,
y ≥0
Solution:

B(4, 12)

A(8, 0) x
O
213
Corner points Z = 20x + 10y

(0, 0) 0
(8, 0) 160
(0, 14) 140
(4, 12) 200 maximum
5 Find max. value of Z = 3x + 4y subject to the constraints x + y ≤ 1, x, y ≥ 0
Solution:
y
(0, 1)

(1, 0)
x
O
Z(1, 0) = 3 + 0 =3
Z(0, 1) = 0+ 4 = 4 maximum
6 Find Zmax.=3x + 2y subject to the constraints x + y ≤ 2, x, y ≥0
Solution:
y
(0, 2)

(2, 0)
x
O
Corner Z = 3x + 2y
points
(0, 0) 0
(2, 0) 6 Max.
(0, 2) 4

7 Let Z = ax + by has optimal value at two points (2, 3) and (5, 7), then find the
relation between a and b
Solution: Z(2, 3) = Z(5, 7)
=> 2a + 3b = 5a + 7b
=> 3a + 4b = 0

8 Feasible region for an LPP is shown in the figure. Maximize Z =5x + 7y

214
Solution:

C(0, 2)
B(3, 4)

A(7, 0) x
O

Corner points Z = 5x + 7y
(0, 0) 0
(7, 0) 35
(3, 4) 43 Maximum
(0, 2) 14
9 Maximise the function Z = 9x + 11y, subject to x ≤ 3, y ≤ 2, x, y ≥ 0
Solution:

x=3
y
(0, 2) (3, 2)
y=2

O (3, 0) x
Corner points Z =9x +11y

(0, 0) 0
(3, 0) 27 Maximum
(0, 2) 22
10 The feasible region for an LPP is shown in the figure. Find the min. value of Z =
11x +7y

O x
𝑥 + 3𝑦 = 9
𝑥 + 𝑦= 5

Solution:

215
Corner points Z = 11x + 7y

(0, 3) 21 Minimum
(3, 2) 44
(0, 5) 35

EXERCISE

1 Find the maximum of Z = 6x + 16y subject to x + y ≥ 2, x, y ≥ 0


2 Find the maximum value of Z = 3x + 2y where the corner points of the feasible
region are (0, 0), (0, 8), (2, 7), (5, 4) and (6, 0)
3 Solve the linear inequation -3x + 2y ≥ 6 graphically
4 Find the maximum value of Z = 4x + 3y subject to x + y ≤ 10, x, y ≥ 0
5 Is the feasible region represented by x + y ≥ 1, x, y ≥ 0 bounded? Justify your
answer
ANSWERS:
1) 32 2) 23 4) 40
5) Unbounded from the graph

O 𝑥 + 𝑦 =1

3 MARK QUESTIONS

1 Maximise: Z = 3x + 9y subject to x + y ≥ 10, x + 3y ≤ 60, x ≤ y, x, y ≥ 0


Solution: On solving x + y = 10 and y – x = 0 we get (5, 5)
Similarly solving y – x = 0 and x + 3y = 60 we get (15, 15)

y 𝑦– 𝑥 = 0
(0, 20)

(15, 15)
(0, 10)

(5, 5)
O𝑥 + 𝑦 = 10 𝑥 + x3𝑦 = 60

216
Corner points Z = 3x + 9y

(0, 20) 180 Maximum


(0, 10) 90
(5, 5) 60
( 15, 15) 180 Maximum
Zmax. = 180 at infinitely many
points on the line segment joining (0, 20) and (15, 15)
2 Maximise: Z = 5x + 10y subject to x + 2y ≤ 120, x + y ≥ 60, x -2y ≥ 0, x , y ≥ 0
Solution:
On solving x + y = 60 and x – 2y = 0 we get (40, 20)
On solving x + 2y = 120 and x – 2y = 0 we get (60, 30)

𝑥 – 2𝑦 = 0
(0, 60)
(60, 30)
(40, 20)
(120, 0)
(60, 0)
O x
𝑥 + 2𝑦 = 120
𝑥 + 𝑦 = 60
Corner points Z =5x + 10y

(40, 20) 400


(60, 0) 300 Minimum Zmin. = 300 at (60, 0)
(120, 0) 600
(60,30) 600

3 Maximise : Z = 300x + 190y subject to x + y ≤ 24, x + ½ y ≤ 16, x, y ≥ 0


Solution:
On solving x + y = 24, x + ½ y = 16 we get (8, 16)

y
C(0, 𝑥 + 1/2𝑦
24) = 16
B(8,
16)

O A(16, x
𝑥 + 𝑦
0)
= 24
Maximum value of Z is 5440 at (8, 16)

217
EXERCISE

1 Maximise :Z = 6x + 3y subject to 4x + y ≥80, 3x + 2y ≤ 150, x + 5y ≥ 15, x, y ≥ 0


2 Minimise: Z = 200x + 500y subject to x + 2y ≥ 10, 3x + 4 y ≤ 24, x, y ≥ 0
3 Maximise: Z = 20x + 10y subject to 1.5x + 3y ≤ 42, 3x + y ≤ 24 , x, y ≥0
ANSWERS:
1) Zmax.= 285 at (40, 15)
2) Zmin. = 2300 at (4, 3)
3) Zmax. = 200 at (4, 12)

5 MARK QUESTIONS

1 Maximise : Z = 70x + 40y subject to 3x + 2y ≤ 9, 3x + y ≤ 9, x, y ≥ 0


Solution:

y
3𝑥 + 2𝑦 = 9
(0, 9/2)

O (3, 0) x
Corner points Z =70x + 40y

(0, 0) 0
(3, 0) 210 maximum
(0, 9/2) 180

Maximum value of Z is 210 at (3, 0)


2 Maximise : Z = 60x + 40y subject to 5x + 6y ≤ 45, 3x + 2y ≤ 18 , x, y ≥ 0
Solution:
On solving 5x + 6y = 45, 3x + 2y = 18 we get (9/4, 45/8)

y
3𝑥 + 2𝑦 = 18

B(8, 16)

O 𝐴(6, 0) x
5𝑥 + 6 𝑦 = 45

218
Corner points Z =60x + 40 y

(0, 0) 0
(0, 15/2) 300
(9/4, 45/8) 360 Maximum
(6, 0) 360Maximum
Maximum value of Z is 360 at any
point on the line segment joining (6, 0) and (9/4, 45/8)

3 Minimise: Z = 5x + 7y subject to 2x + y ≥ 8, x + 2y ≥ 10 , x, y ≥ 0
Solution:
On solving 2x + y = 8, x + 2y =10 we get (2, 4)

y
(0,8)

(2, 4)

(10, 0)
O 2𝑥 + 𝑦 = 8 x
𝑥 + 2𝑦 = 10

Corner points Z = 5x + 7y

(10, 0) 50
(2, 4) 38 Minimum
(0, 8) 56

The minimum value of Z is 38 at (2, 4)

EXERCISE

1 Minimise : Z = 5x + 2y subject to x – 2y ≤ 2, 3x + 2y ≤12, -3x + 2y ≤ 3, x, y ≥ 0


2 Minimise : Z =x +2y subject to x + 2y ≥ 100, 2x – y ≤ 0, 2x + y ≤ 200, x, y ≥ 0
3 Minimise : Z = 3x + 5y subject to x + 2y ≥ 10, x + y ≥ 6, 3x + y ≥ 8, x, y ≥ 0
1) Zmin = 0 at (0 , 0)
2) Zmin = 100 at all points on the lin segment joining (0, 50) and (20, 40)
3) Zmin = 26 at (2, 4)

219
CASE BASED QUESTIONS

1 A dealer Ram Singh residing in a rural area opens a shop to start his business with
an investment of Rs.5760. He wishes to purchase ceiling fans and table fans. A
ceiling fan costs him Rs. 360 and table fan costs Rs.240
Based on the above information answer the following questions.
i) Ram Singh purchases x ceiling fans and y table fans. He has space
in his store for at most 20 items. Write its constraint
ii) If he sells ceiling fan at a profit of Rs. 22 and table fan for a profit
of Rs. 18, then express the profit Z in terms of x and y
iii) What is the maximum profit of selling all the fans
Solution: i) He has space in store for at most 20 items => x + y ≤ 20
ii) profit on ceiling fans = Rs.22, Profit on table fans = Rs18
Hence Z = 22x + 18y
iii)360x + 240y ≤ 5760 => 3x + 2y ≤ 48
also x +y ≤20 and x,y ≥ 0
on solving we get the corner points(0, 0), (16,0), (8, 12), (0, 20)
Maximum value of Z occurs at (8, 12)
Zmax. = 22x8 + 18x12 = 392
2 The students of class XII are asked to write linear inequalities in two variables.
They have written : 3x + 5y ≤ 15, 5x + 2y ≤ 10, x ≥ 0 and y ≥ 0
Based on the above information answer the following questions.
i) Draw the feasible region of above system of inequalities
ii) Find the corner points of the solution region
Solution: i)
y
C(0, 3)
B(20/19, 45/19)

O A(2, 0) x
(ii) corner points are (0, 0), (2, 0), (20/19, 45/19) and (0, 3)
3 Corner points of a feasible region of an LPP are (0, 0), (7, 0), (6, 2), (0, 5).
Let Z = 3x + 4y be the objective function
Based on the above information answer the following questions.
i) The minimum value of Z occurs at

220
a) (7, 0) b) (6, 2) c) (0, 5) d) (0, 0)
ii) The maximum value of Z occurs at
a) (7, 0) b) (6, 2) c) (0, 5) d) (0, 0)
iii) Maximum value of Z – Minimum value of Z is equal to
a) 26 b) 28 c) 21 d) 20
iv) The feasible solution of LPP belongs to
a) First and second quadrants b) First and third quadrants
c) Only second quadrant d) Only first quadrant
Solution:
Corner point Z = 3x + 4y
(0, 0) 0 Minimum
(7, 0) 21
(6, 2) 26 Maximum
(0, 5) 20

i) d ii) b iii) a iv) d

EXERCISE

1 In an LPP, the objective function Z =3x + 4y + 370 is to be optimized subjected to


the constraints : x + y ≥ 10, x + y ≤ 60, x ≤ 40, x, y ≥ 0
Based on the above information answer the following questions.
i) The maximum value of Z occurs at
a) (40, 0) b) (40, 20) c) (20, 40) d) (0, 40)
ii) The minimum value of Z is
a) 300 b) 400 c) 500 d) 600
iii) The value of Z at (40, 20) is
a) 490 b) 530 c) 550 d) 570
iv) Max. Z – Min. Z =
a) 190 b) 210 c) 230 d) 250

221
2 The feasible region of an LPP is shown in the figure.

y
C(0, 50)
B(30, 20)

A(40, 0) x
O
i) Equation of AB is
a) 2x + y = 80 b) x + y = 50 c) x + 2y = 50
d) x + y = 40
ii) Equation of BC is
a) 2x + y = 80 b) x + y = 50 c) x + 2y = 50
d) x + y = 40
iii) Constraints are
a) x + y ≤ 50, 2x + y ≤ 80, x ≥ 0, y ≥ 0
b) x + y ≤ 50, 2x + y ≥ 80, x ≥ 0, y ≥ 0
c) x + y ≥ 50, 2x + y ≤ 80, x ≥ 0, y ≥ 0
d) None of these
iv) The objective function Z = 10500x + 9000y is maximum at the point
a) A b) B c) C d) O

ANSWERS:
1) i) a ii) b iii) a iv) d

2) i) a ii) b iii) a iv) b

222
CHAPTER:PROBABILITY

SYLLABUS:

Conditional probability, multiplication theorem on probability, independent events, total


probability, Bayes’ theorem, Random variable and its probability distribution, mean of random
variable.

Definitions and Formulae:

Conditional Probability: If A and B are two events associated with any random experiment,
then 𝑃(𝐴/𝐵) represents the probability of occurrence of event A knowing that the event B
has already occurred.
𝑃(𝐴 ∩ 𝐵)
𝑃(𝐴/𝐵) = , 𝑃(𝐵) ≠ 0.
𝑃(𝐵)
𝑃(𝐵) ≠ 0, , means that the event B should not be impossible.

Multiplication Theorem on Probability: If the event A and B are associated with any
random experiment and the occurrence of one depends on the other, then
𝑃(𝐴 ∩ 𝐵) = 𝑃(𝐴). 𝑃(𝐵/𝐴), 𝑤ℎ𝑒𝑟𝑒𝑃(𝐴) ≠ 0

Independent Events:
When the probability of occurrence of one event does not depend on the occurrence /non-
occurrence of the other event then those events are said to be independent events.
Then 𝑃(𝐴/𝐵) = 𝑃(𝐴) 𝑎𝑛𝑑 𝑃(𝐵/𝐴) = 𝑃(𝐵)
So, for any two independent events A and B ,𝑃(𝐴 ∩ 𝐵) = 𝑃(𝐴). 𝑃(𝐵).

Theorem on total probability:


𝐼𝑓𝐸𝑖 (𝑖 = 1,2,3, . . . 𝑛) be a partition of sample space and all 𝐸𝑖 havenon-zero
roabability.Abeanyeventassociatedwiththesamplespace, which occurs with
𝐸1 𝑜𝑟𝐸2 𝑜𝑟𝐸3 𝑜𝑟. . . 𝑜𝑟𝐸𝑛 then
𝑃(𝐴) = 𝑃(𝐸1 )𝑃(𝐴/𝐸1 ) + 𝑃(𝐸2 )𝑃(𝐴/𝐸2 ) + 𝑃(𝐸3 )𝑃(𝐴/𝐸3 )+. . . . +𝑃(𝐸𝑛 )𝑃(𝐴/𝐸𝑛 )

Bayes’ Theorem:
"Let " S "be the sample space and " 𝐸1 , 𝐸2 , . . . , 𝐸𝑛 " be " 𝑛" mutuallyexclusive and
"exhaustive events associated with " a" random experiment." If A is any event
which occurs with 𝐸1 𝑜𝑟 𝐸2 𝑜𝑟, . . 𝑜𝑟 𝐸𝑛 𝑡ℎ𝑒𝑛
𝑃(𝐸𝑖 )𝑃(𝐴/𝐸𝑖 )
𝑃(𝐸𝑖 /𝐴) = 𝑛
∑𝑖=1 𝑃(𝐸𝑖 )𝑃(𝐴/𝐸𝑖 )

Random Variable: It is a real valued function whose domain is the sample space of random
experiment.

Probability Distribution: It is a system of number of random variable (X) such that


X X1 X2 …. Xn
P(X) P1 P2 …. Pn
𝑛
where P(Xi ) > 0 and ∑𝑖=1 𝑃(𝐸𝑖 ) = 1

223
Mean: Mean or Expectation of a random variable X is denoted by E(X)

E(X)= ∑𝑛𝑖=1 𝑥𝑖 𝑃𝑖

MULTIPLE CHOICE QUESTIONS:

Q. No QUESTIONS AND SOLUTIONS


1. If P(A/B)=0.3, P(A)= 0.4 and P(B)=0.8, then P(B/A) is equal to
(a) 0.6 (b) 0.3 (c) 0.06 (d) 0.4
Solution:
P(A  B) P(A  B)
P(A / B) = = = 0.3
P(B) 0.8
P(A  B) = 0.24
P(B / A) = 0.24 / 0.4 = 0.6
Ans: (a)
2. Ashima can hit a target 2 out of 3 times. She tried to hit the target twice. The
probability that the she missed the target exactly once is
(a) 2/3 (b) 1/3 (c) 4/9 (d) 1/9
Solution:
2 1
𝑃(𝐴) = , 𝑃(𝐴′) = (𝐴 − ℎ𝑖𝑡, 𝐴′ − 𝑛𝑜𝑡ℎ𝑖𝑡)
3 3
2 1 1 2
𝑃(𝑜𝑛𝑙𝑦𝑜𝑛𝑐𝑒ℎ𝑖𝑡) = . + . = 4/9.
3 3 3 3
Ans: (c)
3. 4 7
For any two events A and B, 𝑃(𝐴) = 5 𝑎𝑛𝑑 𝑃(𝐴 ∩ 𝐵) = 10,

then 𝑃(𝐵/𝐴)is
(a) 1/10 (b) 1/8 (c) 17/20 (d) 7/8
Solution:
7/10
𝑃(𝐵/𝐴) = = 7/8.
4/5
Ans: (d)
4. Five fair coins are tossed simultaneously. The probability of the events that at
least one head comes up is
(a) 27/32 (b) 5/32 (c) 31/32 (d) 1/32
Solution:
𝑃(𝑎𝑡𝑙𝑒𝑎𝑠𝑡𝑜𝑛𝑒𝐻) = 1 − 𝑃(𝑛𝑜𝑛𝑒𝑖𝑠𝐻)
= 1 − 1/32 = 31/32.
Ans: (c)

224
5. If A and B are two independent events such that P(A)=1/2 and P(B)=1/4, then
𝑃(𝐵′/𝐴) is
(a) 1/4 (b) 3/4 (c) 1/8 (d) 1
Solution:
𝑃(𝐵′ ∩ 𝐴) 𝑃(𝐵′)𝑃(𝐴)
𝑃(𝐵′/𝐴) = = = 𝑃(𝐵′) = 3/4
𝑃(𝐴) 𝑃(𝐴)
Ans: (b)
6. If the sum of numbers obtained on throwing a pair of dice is 9, then the probability
that number obtained on one of the dice is 4, is
(a) 1/9 (b) 4/9 (c) 1/18 (d) ½
Solution:
𝐴 = 𝑆𝑢𝑚9 = {(3,6), (4,5), (5,4), (6,3)}
𝐵 = 𝑜𝑛𝑒𝑑𝑖𝑒𝑠ℎ𝑜𝑤𝑠4 = {(1,4), (2,4), (3,4), (5,4), (6,4), (4,1), (4,2), (4,3), (4,5), (4,6)}
𝑃(𝐴) = 4/36𝑃(𝐵) = 10/36
𝑃(𝐵 ∩ 𝐴) 2/36
𝑃(𝐵/𝐴) = = = 1/2
𝑃(𝐴) 4/36
Ans: (d)
7. If A and B are two events such that P(A/B)=2.P(B/A) and P(A)+P(B)=2/3, then
P(B) is
(a) 2/9 (b) 7/9 (c) 4/9 (d) 5/9.
Solution:
𝑃(𝐴 ∩ 𝐵) 𝑃(𝐴 ∩ 𝐵)
=2
𝑃(𝐵) 𝑃(𝐴)
𝑃(𝐴) = 2𝑃(𝐵)
3𝑃(𝐵) = 2/3
𝑃(𝐵) = 2/9
Ans: (a)
8. If two events A and B, P(A-B)=1/5 and P(A)= 3/5, then P(B/A) is equal to
(a) ½ (b) 3/5 (c) 2/5 (d) 2/3
Solution:
𝑃(𝐴 − 𝐵) = 1/5, 𝑃(𝐴) = 3/5
𝑃(𝐴 ∩ 𝐵) = 3/5 − 1/5 = 2/5

𝑃(𝐴 ∩ 𝐵) 2/5
𝑃(𝐵/𝐴) = = = 2/3.
𝑃(𝐴) 3/5
Ans: (d)
9. If𝑃(𝐴 ∩ 𝐵) = 1/8and𝑃(𝐴̄) = 3/4,then𝑃(𝐵/𝐴)is equal to

225
(a) 1/2 (b) 1/3 (c) 1/6 (d) 2/3
Solution:
𝑃(𝐴) = 1 − 3/4 = 1/4
𝑃(𝐴 ∩ 𝐵) 1/8
𝑃(𝐵/𝐴) = = = 1/2
𝑃(𝐴) 1/4
Ans: (a)
10. For any two events A and B, 𝑃(𝐴’) = 1/2, 𝑃(𝐵’) = 2/3 and 𝑃(𝐴 ∩ 𝐵) = 1/4,
𝐴’
then 𝑃 (𝐵’) equals

(a) 8/9 (b) 5/8 (c) 1/8 (d) ¼


Solution:
(𝐴) = 1/2, 𝑃(𝐵) = 1/3
𝑃(𝐴𝑈𝐵) = 1/2 + 1/3 − 1/4 = 7/12
𝑃(𝐴′ ∩ 𝐵′) 1 − 𝑃(𝐴𝑈𝐵) 1 − 7/12 5/12
𝑃(𝐴′/𝐵′) = = = = = 5/8
𝑃(𝐵′) 𝑃(𝐵′) 2/3 2/3
Ans: (b)

CHAPTER VIDEO LINK SCAN QR CODE FOR


VIDEO

PROBABILITY https://youtu.be/YNifzbxLS5M

EXERCISE

MCQ -PRACTICE QUESTION


1. X and Y are independent events such that P(X  Y)=2/5 and P(X)=3/5.
Then P(Y) is equal to:
(a) 2/3 (b) 2/5 (c) 1/3 (d) 1/5
Answer:c
2. The probability that A speaks the truth is 4/5 and that of B speaking the truth is ¾.
The probability that they contradict each other in stating the same fact is
(a) 7/20 (b) 1/5 (c) 3/20 (d) 4/5

226
Answer:a
3. For any two events A and B, if P(A)=0.4 and P(B)=0.8 and P(B/A)=0.6, the
P(A  B) is :

(a) 0.24 (b) 0.3 (c) 0.48 (d) 0.96


Answer:d
4. The events E and F are independent. If P ( E ) = 0.3 and P(E  F)=0.5
then P(E/F)-P(F/E) equals:
(a) 1/7 (b) 2/7 (c) 3/35 (d) 1/70
Answer:d
5. If A and B are independent events such that P(A)=0.4 , P(B) =x
and P( A  B) = 0.5 then x is
(a)4/5 (b) 0.1 (c) 1/6 (d) None of these
Answer: c

ASSERTION-REASONING QUESTIONS

Select the correct answer from the codes (a), (b), (c) and (d) as given below.

(a) Both Assertion (A) and Reason (R) are true and Reason (R) is the correct explanation of
the Assertion (A)
(b) Both Assertion (A) and Reason (R) are true but Reason (R) is not the correct explanation
of the Assertion (A)
(c) Assertion (A) is true and Reason (R) is false.
(d) Assertion (A) is false and Reason (R) is true.

1. Assertion (A): Two coins are tossed simultaneously. The probability of getting two
heads, if it is know that at least one head comes up, is 1/3.
Reason (R) : Let E and F be two events with a random experiment, then
P(E  F )
P(F / E) = .
P(E)
Ans: (a)
For Assertion
F = {HH} E = {HH,HT, TH}
1/4
P(F / E) = =1/3
3/4
A is true and R is a correct formula and correct explanation.
2. Let A and B be two events associated with an experiment such that
P(A  B) = P(A).P(B)
Assertion (A): P(A/B)=P(A) and P(B/A)=P(B)
Reason (R): P(A  B)=P(A)+P(B)
Ans: (c) as A is correct but R is false.

227
3. For any two events A and B. P(A)=p and P(B)=q
Assertion (A): The probability that exactly one of the events A and B occurs is p+q-2pq
Reason (R): P(A  B)=P(A)+P(B)-P(A  B)
Ans: (b)
A is correct but R is not the correct explanation of A.
4. Assertion (A): Consider the experiment of drawing a card from a deck of 52 playing
cards, in which the elementary events are assumed to be equally likely.
If E and F denote the events the card drawn is a spade and the card drawn is an ace
respectively, then P(E/F)=1/4 and P(F/E)=1/13.
Reason (R): E and F are two events such that the probability of occurrence of one of them
is not affected by occurrence of the other. Such events are called independent events.
Ans: (b) as A is correct but R is not the correct explanation of A.

5. Consider that following statements:


Assertion (A): Let A and B be two independent events. Then P(A and B)= P(A)+P(B)
Reason (R): Three events A,B and C are said to be independent if
P(A  B  C ) = P(A).P(B).P(C )
Ans: (d) as P(A and B) = P(A) + P(B) − P(A).P(B) , hence A is false and R is True.

6. Assertion (A): In rolling a die, event A= {1, 3, 5} and event B = {2, 4} are mutually
exclusive events.
Reason (R): in a sample space two events are mutually exclusive if they do not occur at
the same time.
Ans: (a) A is true as P(A  B) =  and R is the correct explanation of A.
7. Let A and B be two independent events.
Assertion (A): If P(A)=0.3 and P(A  B)=0.8 then P(B) is 2/7
Reason (R) : P(E)=1-P(E), for any event E.
Ans : ( a ) as
P(AUB ') = P(A) + P(B ') − P(A).P(B ')
P(B ') = 5 / 7
P(B) = 2 / 7
Hence A is true and R is the correct explanation for A.
8. Assertion (A) : Let A and B be two events such that P (A) =1/5 and P (A or B) =1/2 then
P(B)=3/8 for A and B are independent events.
Reason (R) : For independent events P (A or B) = P (A) + P(B) - P(A).P(B).
Ans: (a) as A is true and R is the correct explanation for A.
For Assertion :
P(AUB) = 1 / 2
P(A) + P(B) − P(A).P(B) = 1 / 2
P(B) = 3 / 8 ( P(A) = 1 − 1 / 5 = 4 / 5)
R is correct exxplanation.
9. Assertion (A) : If A and B are mutually exclusive events with P (A’) =5/6 and P (B)=1/3.
Then P (A/B’)=1/4.
Reason (R) : If A and B are two events such that P (A)=0.2, P(B)=0.6 and P(A/B)=0.2
then the value of P(A/B’) is 0.2.
Ans: (b) as A is true and R is not the correct explanation for A.
𝐹𝑜𝑟𝐴𝑠𝑠𝑒𝑟𝑡𝑖𝑜𝑛 :
𝑃(𝐴) = 1/6

228
𝑃(𝐴 ∩ 𝐵′) 𝑃(𝐴) − 𝑃(𝐴 ∩ 𝐵) 1/6 − 0
𝑃(𝐴/𝐵′) = = = = 1/4
𝑃(𝐵′) 1 − 𝑃(𝐵) 1 − 1/3
𝑅𝑒 𝑎 𝑠𝑜𝑛:
𝑃(𝐴 ∩ 𝐵) = 0.12
𝑃(𝐴 ∩ 𝐵′) 𝑃(𝐴) − 𝑃(𝐴 ∩ 𝐵) 0.08
𝑃(𝐴/𝐵′) = = = = 0.2
𝑃(𝐵′) 1 − 𝑃(𝐵) 0.4

10 Assertion (A): If A and B are two independent events with P(A)=1/5 and P(B)=1/5, then
P(A’/B) is 1/5.
P(A ' B)
Reason (R ) : P ( A’ / B ) =
P(B)
Ans: (d)
P(A ' B) P(A ').P(B)
P ( A’ / B ) = = = P(A ')
P(B) P(B)
= 1 − P(A) = 4 / 5
So A is false and R is True.
EXERCISE

1. Assertion A: Two cards are drawn from a well shuffled pack of 52 playing cards without
replacement. Probability of getting 02 jacks is 16/169
Reason (R): For independent events A and B, P(A and B)=P(A).P(B)
Answer: d
2. Assertion (A): A can solve 80% of the problems in a book and b can solve 60%. then
probability that at least one of them will solve a problem is 0.92
Reason (R): P (at least one solve a problem) = 1- P(none of them solve it)
Answer: a
3. Assertion A: P(A) = 0.6 and P(B) = 0.4 then P(AUB) = 1 when A and B are mutually
exclusive events
𝑃(𝐴 ∩ 𝐵)
Reason (𝑅) : 𝑃(𝐴/𝐵) =
𝑃(𝐵)
Answer: b
4. The given below is a probability distribution table:
X 0 1 2 3
P k k/2 k/4 k/8
Assertion A: The value of k is 8/15
Reason R: Mean of X=  px
Answer: b
5. Assertion A: Three numbers are selected from first six natural numbers at random without
replacement. If X denotes the greatest of three numbers selected, then X={2,3,4,5,6}
Reason R: Random variable is a real valued function whose domain is a sample space of a
random experiment.
Answer: d

2 MARK QUESTIONS

1. A pair of dice is thrown. If the two numbers appearing on them are different, find the
probability that the sum of the numbers is 6.
Ans: A: Number appearing are different n(A)=30 (except (1,1),(2,2),(3,3),(4,4),(5,5)
and (6,6))
B: Sum of the numbers is 6.

229
P(A)=30/36
A and B={(1,5),(2,4),(4,2),(5,1)}
P(A and B)=4/36
P(A  B) 4 / 36
P (B / A ) = = = 4 / 30
P(A) 30 / 36
2. In a school, there are 1000 students, out of which 430 are girls. It is known that out of
430, 10% of the girls study in class XII. What is the probability that a student chosen
randomly studies in class XII given that the chosen student is girl?
Ans: A: Student of Class XII B: The student is a girl.
n(A & B)=10% of 430=43.
P(A  B) 43 1
P (A / B) = = =
P(B) 430 10
3. Two balls are drawn from a bag containing 2white,3 red and 4 black balls one by one
without replacement. What is the probability that at least one ball is red?
Ans: P(at lest one red ball)=1-P(none of the ball is red)
( that is 1st ball is non red and 2nd ball is non red.)
6 5 42 7
=1− . = =
9 8 72 12
4. If P(A)=3/8, P(B)=1/2 and P(A and B)=1/4, find P(A’/B’)
Ans:
P(A ' B ') P(AUB) ' 1 − P(AUB)
P(A '/ B ') = = =
P(B ') 1 − P(B) 1 − P(B)
P(AUB) = P(A) + P(B) − P(A  B)
5
= 3 / 8 +1 / 2 −1 / 4 =
8
5
1−
1 − P(AUB) 8 = 3/8 =3/4
P(A '/ B ') = =
1 − P(B) 1 1/2
1−
2

5. A committee of 4 students is selected at random from a group of 8 boys and 4 girls.


Given that there is at least one girl in the committee, calculate the probability that there
are exactly 2 girls in the committee.
Ans:
A: at least one girl in the committee.
B: exactly 02 girls in the committee.
P(A  B)
P(B / A) =
P(A)
8
C 70 85
P(A) = 1 − P(none is girl) = 1 − 12 4 = 1 − =
C4 495 99
C2 . 4 C 2 28.6 56
8

P(A  B) = P(2G and 2 B) = = =


12
C4 495 165
6. Events E and F are independent. Find P(F), if P(E) =0.35 and P(EUF)=0.6.
Ans:

230
P(EUF) = P(E) + P(F) − P(E  F)
= P(E) + P(F) − P(E).P(F)
0.6 = 0.35 + x − 0.35x
0.25 = 0.65x
25 5
x= =
65 13
7. A and B are two candidates seeking admission in a college. The probability that A is
selected is 0.7 and the probability that exactly one of them selected is 0.6. Find the
probability that B is selected.
Ans: E: Selecting A F: Selecting B
E and F are independent events.
P (E ) = 0.7,
P (E  F ')U(E ' F)  = 0.6
P(E).P(F ') + P(E ').P(F) − P (E  F ')  (E ' F)  = 0.6
P(E)(1 − P(F)) + (1 − P(E))P(F) = 0.6
P(E) + P(F) − 2.P(E).P(F) = 0.6
0.7 + x − 2(0.7).x = 0.6
0.1 = 0.4x
1
x = = P(F).
4
8. A bag contains 3 white, 4 red and 5 black balls. Two balls are drawn at random. Find
the probability that both balls are of different colours.
Ans:
P(both balls are of different colours) = 1 − P(both balls of same colour)
3
C 4
C 5
C 3 6 10 19
P(both balls of same colour) = 12 2 + 12 2 + 12 2 = + + =
C2 C2 C 2 66 66 66 66
19 47
P(both balls are of different colours) = 1 − =
66 66
9. An unbiased die is thrown thrice. Find the probability of getting at least 2 sixes.
Ans:
P ( at least 2 sixes ) = P(02 sixes) + P(03 Sixes)
1 1 5 1 1 1 16 2
= 3. . . + . . = =
6 6 6 6 6 6 216 27
10. A problem is given to A, B and C. The probabilities that they solve the problem
correctly are 1/3, 2/7 and 3/8 respectively. If they try to solve the problem
simultaneously, find the probability that exactly one of them solve the problem.
Ans:
P(Exactly one solve)=P(AB’C’)+P(A’BC’)+P(A’B’C)
1 5 5 2 2 5 2 5 3
= . . + . . + . .
3 7 8 3 7 8 3 7 8
25 + 20 + 30 75 25
= = =
168 168 56

231
EXERCISE

1. A die is tossed once. If the random variable X is defined as


1, if the die shows an even number
X=
0, otherwise
Find the mean of X.
2. There are 5 bags, each containing 5 white and 3 black balls. Also, there are 6 bags,
each containing 2 white balls and 4 black balls. A ball is taken at random from a bag.
Find the probability that it is a white ball.
3. The odds against a man who is 45 year old, living till he is 70 are 7:5 and the odds
against his wife who is now 40, living till she is 65 are 5:3. Find the probability that
the couple will be alive 25 years hence.
4. A coin is tossed thrice. Let E be the event, ‘first throw results in a head’ , and the event
F,
‘the last throw results in a tail’. Find whether the events E and F are independent.
5. In a class, 40% students study mathematics; 25% study biology and 15% study both
mathematics and biology. One student is selected at random. Find the probability that
he studies biology if it is known that he studies mathematics.
Answers: (1) 1/2 (2) 41/88 (3) 5/32 (4) Yes. E and F are independent (5) 3/8

3 MARK QUESTIONS

1. The probability distribution of a random variable X is given below:


X 1 2 3
P(X) k/2 k/3 k/6
(i) Find the value of k (ii) Find P(1  X) (iii) Find E(X), the mean of X.
Ans:
(𝑖) ∑ 𝑝 = 1

𝑘 𝑘 𝑘 12𝑘
+ + = =𝑘
2 3 6 12
𝑘=1
(𝑖𝑖)𝑃(1 ≤ 𝑥) = 𝑃(1) + 𝑃(2) + 𝑃(3) = 1/2 + 1/3 + 1/6 = 1
(𝑖𝑖𝑖)𝐸(𝑋) = ∑ 𝑝𝑥 = 1/2 + 2/3 + 3/6 = 5/3
2. 1 1
A and B are independent events such that P(A  B ) = and P(A  B) =
4 6
Find P(A) and P(B).
Ans:

232
P(A).P(B ') = 1 / 4 & P(A ').P(B) = 1 / 6
P(A) = x, P(B) = y
x(1 − y) = 1 / 4 and (1 − x)y = 1 / 6
on solving we get, x − y = 1 / 12
x = y + 1 / 12
On substituting, we get 12y 2 − 11y + 2 = 0
y = 1 / 4 or y = 2 / 3
Corresponding x = 1 / 3 or 3 / 4
P(A) = 1 / 3 P(B) = 1 / 4
(OR)
P(A) = 3 / 4 P(B) = 2 / 3
3. Two balls are drawn at random one by one with replacement from an urn containing equal
number of red balls and green balls. Find the probability distribution of number of red
balls. Also, find the mean of the random variable.
Ans:
X={0,1,2}-No. of red Balls.
P(0)=G and G =1/2.1/2=1/4
P(1)=GR +RG=1/2.1/2+1/2.1/2=1/2
P(2)=RR=1/2.1/2=1/4
X 0 1 2
P(X) 1/4 1/2 1/4
xP(x) 0 1/2 1/2
Mean=0+1/2+1/2=1
4. A and B throw a die alternately till one of them get a ‘6’ and wins the game. Find their
respective probabilities of winning, if A starts the game first.
Ans: S-Getting 6 F- not getting 6
P(S)=p=1/6 P(F)=q=5/6
P(A-Wins)=p+qqp+qqqqp+……
=1/6+(5/6)21/6+(5/6)41/6+….
1/6 6
= =
25 11
1−
36
6 5
P(B − Wins) = 1 − =
11 11
5. Two number are selected from first six natural numbers at random without replacement. If
X denotes the greater of two numbers selected, find the probability distribution of X.Also
find the mean of X
Ans: Total number of ways of selecting two numbers one by one without
replacement=6x5=30
X=the greater number obtained={2,3,4,5,6}
P(2)=2/30=1/15 (1,2),(2,1)
P(3)=4/30=2/15 (1,3),(2,3),(3,1),(3,2)
P(4)=6/30=3/15 (1,4),(2,4),(3,4),(4,1),(4,2),(4,3)
P(5)=8/30=4/15 similarly as above
P(6)=10/30=5/15
X 2 3 4 5 6
P(X) 1/15 2/15 3/15 4/15 5/15
X(P(X) 2/15 6/15 12/15 20/15 30/15

233
2 6 12 20 30 70 14
Mean =  XP(X) = + + + + = =
15 15 15 15 15 15 3

6. A fair coin and an unbiased die are tossed. Let A be the event, “Head appears on the coin”
and B is the event, “3 comes on the die”. Find whether A and B are independent events or
not.
Ans: S=((H,1),(H,2),(H,3),(H,4),(H,5),(H,6),(T,1),(T,2),(T,3),(T,4),(T,5),(T,6)}
A: H appears B: 3 on die
P(A)=6/12=1/2
P(B)=2/12=1/6
P(A and B)=1/12
P(A).P(B)=1/2.1/6=1/12=P(A and B)
Hence A and B are independent events.

7. A pair of dice is thrown simultaneously. If X denotes the absolute difference of numbers


obtained on the pair of dice, then find the probability distribution of X.
Ans: X={0,1,2,3,4,5}
X 0 1 2 3 4 5
P(X) 6/36 10/36 8/36 6/36 4/36 2/36
XP(X) 0 10/36 16/36 18/36 16/36 10/36
Mean=  PX = 70 / 36 = 35 / 18
8. There are two coins. One of them is a biased coin such that P(head): P(tail) is 1:3 and the
other coin is a fair coin. A coin is selected at random and tossed once. If the coin showed
head, then find the probability that it is a biased coin.
Ans:
A: Selecting Biased Coin B; Selecting fair coin C: Getting H
P(A)=P(B)=1/2, P(C/A)=1/4 (since ratio for head and tail is 1:3)
P(C/B)=1/2
P(C) =P(A).P(C/A)+P(B).P(C/B)=1/2.1/4+1/2.1/2=1/8+1/4=3/8
P (B ) .P ( C / B ) 1/4
P (B / C ) = = =2/3
P ( A ) .P ( C / A ) + P (B ) .P ( C / B ) 3 / 8
9. From a log of 30 bulbs which include 6 defective bulbs, a sample of 2 bulbs is drawn at
random one by one with replacement. Find the probability distribution of the number of
defective bulbs and hence find the mean number of defective bulbs.
Ans: X={0,1,2}
p-getting good 24/30=4/5 q=getting defective=6/30=1/5
P(X=0)=both good=4/5.4/5=16/25
P(X=1)=1 good and 1 def=4/5.1/5+1/5.4/5=8/25
P(X=2)=both defective=1/5.1/5=1/25
X 0 1 2
P(X) 16/25 8/25 1/25
XP(X) 0 8/25 2/25
Mean =  xP(x) =0 + 8 / 25 + 2 / 25 = 10 / 25 = 2 / 5
10. Two fair dice are thrown simultaneously. If X denotes the number of sixes, find the mean
of X.
Ans:
X={0,1,2}
P(0) =both non six=5/6.5/6=25/36
P(1)=one six and one non-six=2.(1/6.5/6)=10/36
P(2)=both six=1/6.1/6=1/36

234
X 0 1 2
P(X) 25/36 10/36 1/36
XP(X) 0 10/36 2/36
Mean =  xP(x) =0 + 10 / 36 + 2 / 36 = 12 / 36 = 1 / 3

EXERCISE

1. In a game, a man wins Rs.5 for getting a number greater than 4 and loses Rs.1 otherwise,
when a fair die is thrown. The man decided to throw a die thrice but to quit as and when he
gets a number greater than 4. Find the expected value of the amount he wins/loses.
2. A urn contains 3 white and 6 red balls. Four balls are drawn one by one with replacement
from the urn. Find the mean of the distribution of the number of red balls drawn.
3. A and B throw a pair of dice alternately, till one of them gets a total of 10 and wins the
game. Find their respective probabilities of winning, if A starts first.
4. A coin is biased so that the head is 4 times as likely to occur as tail. If the coin is tossed
thrice. Find the mean of the distribution of number of tails.
5. If A and B are two independent events, then prove that the probability of occurrence of at
least one of A and B is given by 1- P(A’).P(B’).
Answers: (1) Mean=57/27 (2) Mean= 8/3 (3) For A wins:12/23, B wins:11/23
(4) Mean=3/5

5 MARK QUESTIONS

1. In answering a question on a multiple choice test, a student either knows the answer or
guesses. Let 3/5 is the probability that he knows the answer and 2/5 be the probability that
he guesses. Assuming that a student who guesses at the answer will be correct with
probability 1/3. What is the probability that the student knows the answer, given that he
answered it correctly?
Ans: A: Knows the answer B: Guesses the answer E: Answered Correctly
P(A)=3/5, P(B)=2/5 P(E/A)=1, P(E/B)=1/3
By Bayes theorem,
P(A).P(E / A)
P ( A / E) =
P(A).P(E / A) + P(B).P(E / B)
3
.1
5 3
= =
3 2 1 11
.1 + .
5 5 3
2. A box contains 10 tickets , 2 of which carry a prize of Rs.8 each, 5 of which carry a prize of
Rs.4 each, and remaining 3 carry a prize of Rs.2 each. If one ticket is drawn at random, find
the mean value of the prize.
Ans:
X 8 4 2
P 2/10 5/10 3/10
XP 16/10 20/10 6/10
Mean=42/10=4.2 Rs. 4.2
3. There are three coins. One is a two-headed coin ( having head on both faces), another is a
biased coin that comes up heads 75% of the times and third is also a biased coin that comes
up tails 40% of the times. One of the three coins is chosen at random and tossed, and it shows
heads. What is the probability that it was the two-headed coin?

235
Ans:
A : two headed B : Biased(75%H) coin C : biased (40%T)
P(A) = P(B) = P(C) = 1 / 3
H : getting H
P(H / A) = 1 , P(H / B) = 75 / 100 , P(H / C) = 60 / 100
P(A).P(H / A) 100
P(A / H) = =
P(A).P(H / A) + P(B).P(H / B) + P(C).P(H / C) 235
EXERCISE

LONG ANSWER (LA)- PRACTICE QUESTIONS


1. A man is known to speak truth 3 out of 4 times. He throws a die and report that it is a more
than 4. Find the probability that it is actually more than 4.
2. Bag A contains 3 red and 5 black balls, while bag b contains 4 red and 4 black balls. Two
balls are transferred at random from bag A to bag B and then a ball is drawn from bag B at
random. If the ball drawn from bag B is found to be red, find the probability that two red
balls were transferred from A to B.
3. In a factory which manufactures bolts, machine A,B and C manufacture respectively 30% ,
50% and 20% of the bolts. Of their outputs, 3, 4 and 1 percent respectively are defective
bolts. a bolts is drawn at random from the product and is found to be defective. Find the
probability that this is not manufactured by machine B.
4. An electronic assembly consists of two sub-systems say A and B. From previous testing
procedures, the following probabilities are assumed to be known:
P(A fails)=0.2, P(B fails alone)=0.15, P(A and B fail)=0.15. Evaluate the following
probabilities. (i) P(B fails) (ii) P(A fails or B fails) (iii) P(A fails/B has failed)
(iv) P(A fails alone)
Answers: (1) 6/10 (2) 18/133 (3) 11/31. (4) (i) 0.30 (ii) 0.55 (iii) 0.5 (iv)0.05

CASE STUDY QUESTIONS

Case Study-04 Marks


1. There are different types of yoga which involve the usage of different poses of yoga
Asanas, Meditation and pranayama as show in fig. below.

236
The Venn diagram below represents the probabilities of three different types of Yoga A,
B and C performed by the people of a society. Further, it is given that the probability of a
member performing type C yoga is 0.44.

(i) Find the value of x. (ii) Find the value of y (iii) (a) Find P(C/B) (OR)
(a) Find the probability that a randomly selected person of the society does Yoga
type A or B but not C.
Ans: (i) x=0.44-0.21=0.23 (ii) y=1-0.96=0.04
P(C  B) 0.23 23
(iii) P ( C / B ) = = =
P(B) 0.36 36
(OR)
P(A OR B notC) = 0.32 + 0.09 + y = 0.41 + 0.04 = 0.45
2. Recent studies suggest that roughly 12% of the world population is left handed. Depending
upon the parents, the chances of having a left handed child are as follows:
A: When both father and mother are left handed:
Chances of left handed child is 24%
B: When father is right handed and mother is left handed:
Chances of left handed child is 22%
C: When father is left handed and mother is right handed:
Chances of left handed child is 17%
D: when both father and mother are right handed:
Chances of left handed child is 9%
Assuming that P(A)=P(B)=P(C)=P(D)=1/4 and L denote the event that child is left handed.
Based on the above information, answer the following questions:
(i) Find P(L/C) (ii) Find (L’/A)
(iii) (a) Find P(A/L) (OR) (b) Find the probability that a randomly selected child is
left handed given that exactly one of the parent is left handed.
Ans:
(i) P(L/C) =17/100
(ii) P(L’/A) =1-24/100=76/100
𝑃(𝐴∩𝐿)
(iii) (𝑎)𝑃(𝐴/𝐿) = 𝑃(𝐿)
24
𝑃(𝐿/𝐴) =
100
𝑃(𝐴 ∩ 𝐿) 24
=
𝑃(𝐴) 100
24 1
𝑃(𝐴 ∩ 𝐿) = . = 6/100
100 4

237
𝑃(𝐴 ∩ 𝐿) 6/100
𝑃(𝐴/𝐿) = = = 1/2
𝑃(𝐿) 12/100
(𝑂𝑅)
𝐿 𝑃[(𝐿) ∩ (𝐵𝑈𝐶)]
(𝑏)𝑃( )=
𝐵𝑈𝐶 𝑃(𝐵𝑈𝐶)
𝑃[(𝐿 ∩ 𝐵)𝑈(𝐿 ∩ 𝐶)]
=
𝑃(𝐵) + 𝑃(𝐶) − 𝑃(𝐵). 𝑃(𝐶)
𝑃(𝐿 ∩ 𝐵) + 𝑃(𝐿 ∩ 𝐶) − 𝑃(𝐿 ∩ 𝐵)𝑃(𝐿 ∩ 𝐶)
=
𝑃(𝐵) + 𝑃(𝐶) − 𝑃(𝐵). 𝑃(𝐶)
22 1 17 1 22 17
. + . − .
= 100 4 100 4 400 400
1 1 11
4 + 4 − 44
39 374
400 − 160000
=
1 1
2 − 16
= 0.217
3. An octagonal prism is a three-dim polyhedron bounded by two octagonal bases and eight
rectangular side faces. It has 24 edges and 16 vertices.

The prism is rolled along the rectangular faces and number on the bottom face (touching
the ground) is noted. Let X denotes the number obtained on the bottom face and the
following table give the probability distribution of X.
X 1 2 3 4 5 6 7 8
P(X) p 2p 2p p 2p P 2 P 7 P2+p 2 2

Based on the above information, answer the following questions:


(i) Find the values of p.
(ii) Find P(X>6)
(iii) (a) Find P(X=3m), when m is a natural number
(OR)
(a) Find the Mean E(X).
Ans:
(i) Sum of P(X)=1 , 9p+10p2=1 on solving p=1/10 or p=-1<0 (Rej) . p=1/10
(ii) P(x>6)=P(7)+P(8)=2p2+7p2+p=9/100+1/10=19/100
(iii) (a) P(X=3m)=P(x=3)+P(x=6)=2p+p2=2/10+1/100=21/100 (OR)
33 76 406
(b) E ( X ) =  xP(x) = 33p + 76p = + =
2

10 100 100

238
EXERCISE

1. A coach is training 3 players. He observes that the player A can hit a target 4 times in 5
shots, player B can hit 3 times in 4 shots and the player C can hit 2 times in 3 shots. From
this situation answer the following:

(i) What is the probability that B,C will hit and A will lose?
(ii) What is the probability that none of them hit the target?
(iii) (a) What is the probability that only one of them hit the target? (OR)
(b) What is the probability that at least two of them hit the target?

2. A building contractor undertakes a job to construct 4 flats on a plot along with parking
area. Due to strike the probability of many construction workers not being present for the
job s 0.65. The probability that many are not present and still the work gets completed on
time is 0.35. The probability that work will be completed on time when all workers are
present is 0.80.
Let E1: represent the even when many workers were not present for the job
E2: represent the event when all workers were present
E3: represent completing the construction work on time
Based on the above information, answer the following questions:
(i) What is the probability that all the workers are present for the job?
(ii) What is the probability that construction will be completed on time?
(iii) (a) What is the probability that many workers are not present given that the
construction work is completed on time?
(OR)
(b) What is the probability that all workers were present given that the
construction job was completed on time?
Answers: (1) (i) 1/10 (ii) 1/60 (iii) (a) 9/60 (b) 50/60
(2) 35/100 (ii) 5075/10000 (iii) (a) 2275/5075 (b) 2800/5075

239
SAMPLE QUESTION PAPER - 1
BLUE PRINT

CLASS XII MATHEMATICS


CHAPTERS MCQ A&R VSA(2M) SA LA(5M) CSQ(4M) TOTAL
(3M)
Relations &
Functions
Inverse
Trigonometric - 1 1 - 1 - 8
Functions
Matrices &
Determinants 5 - - - 1 - 10
Continuity &
Differentiability 2 - 1 - - - 4
Application Of
Derivatives - - 1 - - 2 10
Integrals 2 - - 3 - - 11
Application Of
Integrals - - - - 1 - 5
Differential
Equations 2 - - 1 - - 5

Vector Algebra
3 - 1 - - - 5
Three-
Dimensional
Geometry 1 1 1 - 1 - 9
Linear
Programming
Problem 2 - - 1 - - 5

Probability
1 - - 1 - 1 8
18(1M) 2(1M) 5(2M) 6(3M) 4(5M) 3(4M) 80 M

240
CLASS XII : MATHEMATICS
SAMPLE QUESTION PAPER - 1

SECTION – A
(Multiple Choice Questions)
Each question carries One Mark
(2𝑖−𝑗)2
1. Given a matrix A =[𝑎𝑖𝑗 ] of order 3 × 3 whose elements 𝑎𝑖𝑗 = , then
𝑖+𝑗
the element
𝑎32 of matrix A is :
16 15
a) 12 b) 18 c) d)
5 4
0 1
2. If 𝐴 = ( ), then 𝐴2 is equal to
1 0
0 1 1 0 0 1 1 0
a) ( ) b) ( ) c) ( ) d) ( )
1 0 1 0 0 1 0 1
3. 𝐼𝑓 𝐴 = (1 2 3), 𝑡ℎ𝑒𝑛 𝐴𝐴′ 𝑖𝑠 𝑒𝑞𝑢𝑎𝑙 𝑡𝑜
1
a) (1 4 9) 𝑏) (4) 𝑐) (14) d) (6)
9
sin 15° cos 15°
4. If 𝐴 = ( ) then the value of ⌈𝐴⌉ is
− sin 75° cos 75°
√3
a) 0 b) 1 c) −1 d) −
2

2  −3
5. If 𝐴 = (0 2 5 ) ,then 𝐴−1 exists if :
1 1 3
8 8
a) 𝜆 = 2 b) 𝜆 ≠ 2 c) 𝜆 ≠ d) 𝜆 ≠ −
5 5

241
𝜆(𝑥 2 − 2𝑥), 𝑖𝑓 𝑥 ≤ 0
6. For what value of , the function defined by 𝑓(𝑥) = {
4𝑥 + 1, 𝑖𝑓 𝑥 > 0
is continuous at 𝑥 = 0

a) 𝜆 = 1 b) 𝜆 = 2 c) No possible value exists d) 𝜆 = −2


𝑑𝑦
7. If 3𝑥 + 2𝑦 = 𝑠𝑖𝑛 𝑦, then is :
𝑑𝑥

3 sin 𝑦 − 3 2−sin 𝑦 2−cos 𝑦


a) b) c) d)
cos 𝑦−2 2 3 3

𝜋
8. The value of ∫02 𝑒 𝑥 (sin 𝑥 + cos 𝑥 ) 𝑑𝑥 is
𝜋 𝜋
−1
a) e b) 𝑒 2 c) 𝑒 2 d) 𝑒 2

9.The area bounded by the shaded figure is :

1 1 9 5
a) b) c) d)
4 4 4 4

10. If m and n are the order and degree of the differential equation
𝑑 𝑑𝑦 4
[( )] = 0, then m + n =
𝑑𝑥 𝑑𝑥

a) 1 b) 9 c) 3 d) 4
𝑑𝑦 1+𝑦
11. The integrating factor of +𝑦 = is
𝑑𝑥 𝑥

𝑒𝑥 𝑒 −𝑥
a) b) c) 𝑥𝑒 𝑥 d) 𝑥 2 𝑒 𝑥
𝑥 𝑥

12. The vector in the direction of the vector 𝑖̂ − 2𝑗̂ + 2𝑘̂ that has magnitude 9 is:
̂
𝑖̂−2𝑗̂ +2𝑘
a) 𝑖̂ − 2𝑗̂ + 2𝑘̂ b)
3

c) 3(𝑖̂ − 2𝑗̂ + 2𝑘̂) ̂ − 2𝑗̂ + 2𝑘̂)


d) 9(𝑖

242
13. If 𝜃 is the angle between the vectors, 𝑎 and 𝑏⃗ then 𝑎 •𝑏⃗ ≥ 0, only if
𝜋 𝜋
a) 0 < 𝜃 < b) 0 ≤ 𝜃 ≤
2 2

c) 0 < 𝜃 < 𝜋 d) 0 ≤ 𝜃 ≤ 𝜋

14. If |𝑎| = 10, |𝑏⃗| = 2, 𝑎 •𝑏⃗ = 12, then the value of |𝑎 × 𝑏⃗| 𝑖𝑠

a) 5 b) 10 c) 14 d) 16

15. If a line makes angles 𝛼, 𝛽, 𝛾 with the positive direction of the coordinate
axes, then the value of 𝑠𝑖𝑛2 𝛼 + 𝑠𝑖𝑛2 𝛽 + 𝑠𝑖𝑛2 𝛾 is :

a) 0 b) 1 c) 2 d) 3

16. The feasible solution for a LPP is shown in the given figure.

Let Z = 3x – 4y. Minimum of Z occurs at:

a) (0,0) b) (0 ,8) c) (5 ,0) d) (4, 10)

17. The graph of the inequality 2𝑥 + 3𝑦 > 6 is

a) Half plane that contains the origin


b) Half plane that neither contains the origin nor the points on the line
2𝑥 + 3𝑦 = 6
c) Whole XOY plane except the points on the line 2𝑥 + 3𝑦 = 6
d) Entire XOY plane
18. A die is thrown and a card is selected at random from a pack of 52 cards.

The probability of getting an even number and a spade card is:


1 1 3 17
a) b) c) d)
4 8 4 52

243
19. Assertion: The domain of the function 𝑠𝑒𝑐 −1 (2𝑥) is
1 1
(−∞ − )∪( ∞)
2 2
𝜋
Reason : 𝑠𝑒𝑐 −1 (−2) = −
4

𝑥+1 𝑦 𝑧+3 𝑥+3 𝑦+5 𝑧−4


20. Assertion:( A) The lines = = and = = are
2 5 4 1 2 −3

perpendicular
𝑥−𝑥1 𝑦−𝑦1 𝑧−𝑧1 𝑥−𝑥2 𝑦−𝑦2 𝑧−𝑧2
Reason:( R): The lines = = and = = are
𝑎1 𝑏1 𝑐1 𝑎2 𝑏2 𝑐2

𝑎1 𝑏1 𝑐1
parallel if = =
𝑎2 𝑏2 𝑐2

SECTION B
(Each question carries 2 marks)
21. Given a relation R on the set R, the set of real numbers as

R = {(𝑥, 𝑦), 𝑥 2 − 3𝑥𝑦 + 2𝑦 2 = 0}. Is R reflexive, symmetric?


𝑥 2 +3𝑥−10
22. Find the value of ‘k’ for which 𝑓(𝑥 ) = { 𝑥 ≠ 2 is
𝑥−2
𝑘 𝑥=2
continuous at 𝑥 = 2
23. Show that the function 𝑓(𝑥) = 𝑡𝑎𝑛−1 (sin 𝑥 + cos 𝑥) is decreasing for all
𝜋 𝜋
𝑥∈( )
4 2

(OR)
2
A particle moves along the curve 𝑦 = 𝑥 3 + 1. Find the points on the
3

244
curve at which the 𝑦-coordinate is changing twice as fast as the 𝑥-

coordinate

24. Find the area of the parallelogram whose one of the sides is 𝑖̂ − 𝑗̂ + 𝑘̂ and

diagonal is 4𝑖̂ + 5𝑗̂

25. Find the vector equation of the line passing through the point 𝐴(1, 1, −1)

and parallel to the line 5𝑥 – 25 = 14 – 7𝑦 = 35𝑧

(OR)

Find the direction ratios and direction cosines of the line whose equation is

6𝑥 – 12 = 3𝑦 + 9 = 2𝑧 – 2

SECTION C
(Each question carries 3 marks)
sin 𝑥
26. Evaluate: ∫ 𝑑𝑥
√1+sin 𝑥
(OR)
sin 𝑥
Evaluate: ∫
(1−cos 𝑥 )(2−cos 𝑥)
𝑑𝑥
𝜋
2 1+𝑥
27. Evaluate: ∫ 𝜋 cos 𝑥 𝑙𝑜𝑔 ( ) 𝑑𝑥
−2 1−𝑥
𝑐𝑜𝑠 −1 𝑥
28. Evaluate: ∫ 𝑑𝑥
𝑥2
𝑑𝑦 𝑦
29. Solve : 𝑥 = 𝑦 − 𝑥 𝑡𝑎𝑛 ( )
𝑑𝑥 𝑥
(OR)
𝑑𝑦 𝑥−𝑦 𝑐𝑜𝑠 𝑥
Solve : =
𝑑𝑥 1+ 𝑠𝑖𝑛 𝑥
30. Solve the Linear Programming problem graphically:

Min 𝑍 = 16 𝑥 + 20 𝑦,

subject to the constraints: 𝑥 + 2𝑦 ≥ 10, 𝑥 + 𝑦 ≤ 6, 3𝑥 + 𝑦 ≥ 8, 𝑥 ≥ 0, 𝑦 ≥ 0

31. From a lot of 10 bulbs, which includes 3 defectives, a sample of 2 bulbs are

drawn at random. Find the probability of the number of defective bulbs.

245
SECTION D
(Each question carries 5 marks)
𝑥 + 1, 𝑖𝑓 𝑥 𝑖𝑠 𝑒𝑣𝑒𝑛
32. Let 𝑓: 𝑊→𝑊 be defined by 𝑓(𝑥) = { . Show that 𝑓 is
𝑥 − 1, 𝑖𝑠 𝑥 𝑖𝑠 𝑜𝑑𝑑
bijective.
2 3 10
33. If 𝐴 = (4 −6 5 ), find 𝐴−1 . Using 𝐴−1 solve the system of
6 9 −20
equations:
2 3 10 4 6 5 6 9 20
+ + = 2; − + = 5; + − = −4
𝑥 𝑦 𝑧 𝑥 𝑦 𝑧 𝑥 𝑦 𝑧

34. Using integration find the area of the region

{(𝑥 , 𝑦): 9𝑥 2 + 𝑦 2 ≤ 36, 3𝑥 + 𝑦 ≥ 6}

(OR)

Find the area bounded by the region {(𝑥 , 𝑦): 𝑥 2 + 𝑦 2 ≤ 1 ≤ 𝑥 + 𝑦}

35. Find shortest distance between the following pairs of lines


𝑥−3 𝑦 −5 𝑧 −7 𝑥+1 𝑦+1 𝑧 +1
= = and = =
1 −2 1 7 −6 1

(OR)

Find the vector equation of the line which is perpendicular to the lines with
𝑥+2 𝑦−3 𝑧+3 𝑥−1 𝑦+1 𝑧 −3
equations = = and = = and passes through the
1 2 4 2 3 4

point (1, 1, 1). Also find the angle between the given lines

SECTION – E

(This section consists of three case study questions of 4 marks each)

36. A company is interested in making a new complex. It is planned to make an

open space the shape of rectangle and a restaurant in the form of semi-

circular portion is attached

along the breadth 2𝑟

𝑥
246
The total perimeter of the complex is 400 m.

Based on the above information answer the following questions

a) Derive the function for the total area of open space and the restaurant

b) What is the value of r for which the area of the rectangular region is

maximum

37. The Production function of a company is given by

𝑃(𝑥) = 𝑥 3 − 12𝑥 2 + 36𝑥 + 17

where 𝑥 is the years of production.

The company wants to decide the production rate for first 10 years

a) Find the range of years in which the production was increasing

b) Find the range of years in which the production was decreasing

38. In a hockey match both teams A and B scored same number of goals up to

the end of the match.

247
To decide the winner, the referee asked both captains to throw a die

alternatively and decided that the team, whose captain gets a six first, will

be declared winner.

a) If the captain of the team A was asked to start, find their respective
probabilities of winning the match
b) Show that the decision of the referee was not fair.

248
CLASS XII : MATHEMATICS - 1
SAMPLE PAPER
MARKING SCHEME
Q.No Answer Marks
𝟏𝟔 1
1 c)
𝟓
𝟏 𝟎 1
2 d) ( )
𝟎 𝟏
3 c) (𝟏𝟒) 1
4 b) 1 1
𝟖 1
5 d) 𝝀 ≠ −
𝟓
6 c) No possible value exists 1
𝟑 1
7 a)
𝒄𝒐𝒔 𝒚−𝟐
𝝅
8 b) 𝒆𝟐 1
𝟗 1
9 c)
𝟒
10 c) 3 1
𝒆𝒙 1
11 a)
𝒙
12 ̂)
c) 𝟑(𝒊̂ − 𝟐𝒋̂ + 𝟐𝒌 1
𝝅
13 b) 𝟎 ≤ 𝜽 ≤ 1
𝟐
14 d) 16 1
15 c) 2 1
16 b) (0 ,8) 1
b) Half plane that neither contains the origin nor 1
17 the points on the line 2𝒙 + 𝟑𝒚 = 𝟔

𝟏 1
18 b)
𝟖
19 (c) A is true but R is false 1
b) Both A and R are true but R is not the correct 1
20
explanation of A
Proving R is Reflexive 1
21
Proving R is not symmetric 1
Using the condition for continuity x = 2 1
22
Getting the value of k = 7 1
Find 𝒇′ (𝒙) 1
23 Checking the nature of 𝒇′ (𝒙) in the given interval 1
and writing the conclusion

249
Q.No Answer Marks
Find the derivative and using the condition 1
23 𝟓 𝟏 1
Find the points as (𝟏 ) and (−𝟏 𝟑)
𝟑
Getting the other side as ⃗𝒃 = 3𝒊̂ + 𝒋̂ + 𝟒𝒌
̂ 1
24
Getting the area as √𝟒𝟐 square units 1
Getting the direction ratios of the required line as 1
𝟕: −𝟓: 𝟏
25 Getting the required equation as 1
̂) + 𝝀(𝟕𝒊̂ − 𝟓𝒋̂ + 𝒌
⃗ = (𝒊̂ + 𝒋̂ − 𝒌
𝒓 ̂)
𝟏 𝟏 𝟏 1
Getting the direction ratios of the line as : :
𝟔 𝟑 𝟐
25 𝟏 𝟐 𝟑
Getting the direction cosines as , , 1
√𝟏𝟒 √𝟏𝟒 √𝟏𝟒
Splitting the integral 1
Getting the answer as 1+1
𝜋 𝑥 2 𝜋 𝑥 𝜋
26 −2√2 sin ( − ) + 𝑙𝑜𝑔 |sec ( − ) + tan ( −
4 2 √2 4 2 4
𝑥
2
)|+c

Converting into partial fractions using substitution 1


Getting the correct values of the constants at partial 1
26 fractions
Getting the answers as 1
𝒍𝒐𝒈(𝟏 − 𝒄𝒐𝒔 𝒙) − 𝒍𝒐𝒈(𝟐 − 𝒄𝒐𝒔 𝒙) + 𝑪
Proving f(x) is an odd function 2
27
Getting the answer as 0 by using the property 1
Converting the problem to integration by parts 1
using substitution
Integrating using integration by parts 1
28 Getting the answer as 1
cos −1 𝑥 1 + √1 − 𝑥 2
− + log | |+𝑐
𝑥 𝑥

Converting the given Differential Equation by using 1


substation y = vx
29 Applying the variable separable method and 1
separating the variables
𝒚 1
Getting the solution : 𝒍𝒐𝒈 (𝒔𝒊𝒏 ) = − 𝐥𝐨𝐠 𝒙 + 𝑪
𝒙
Writing in general form of Linear Differential 1
Equation
29
Find the Integrating factor as 𝟏 + 𝒔𝒊𝒏 𝒙 1
Getting the general solution as 1

250
𝒙𝟐
𝒚(𝟏 + 𝒔𝒊𝒏 𝒙) = +𝑪
𝟐
Q.No Answer Marks
Drawing the graph of lines 1
6 22 1
Getting the corner points as A( , ), B(1 , 5) ; C(2 ,
5 5
30
4), D(10 , 0)
6 22 1
Getting the minimum value as 107.2 at ( , )
5 5
Getting the probability for x=0,1,2 1
2
2
31 1
Expressing the distribution in tabular form
2
Proving one-one 2
32 Proving onto 2
Conclusion as bijection 1
Getting the value of |𝑨| = 𝟏𝟐𝟎𝟎 1
𝟕𝟓 𝟏𝟓𝟎 𝟕𝟓 2
−𝟏 𝟏
Finding 𝑨 = (𝟏𝟏𝟎 −𝟏𝟎𝟎 −𝟑𝟎)
𝟏𝟐𝟎𝟎
33 𝟕𝟐 𝟎 −𝟐𝟒
Expressing the given system in matrix form 1
2
Getting the values as 𝒙 = 𝟐 ; 𝒚 = −𝟑 ; 𝒛 = 𝟓 1
1+
2
For rough figure of the region 1
2
Finding the limits for integration 1
34
2
Calculating the area using integration 3
Getting the answer as 𝟑(𝝅 − 𝟐) 1
For rough figure of the region 1
2
Finding the limits for integration 1
34 2
Calculating the area using integration 3
𝝅 𝟏 1
Getting the answer as ( − )
𝟒 𝟐
Identifying the vectors for the formula 2
35 Using the formula to get the shortest distance 2
Getting the answer as 𝟐√𝟐𝟗 1

251
Q.No Answer Marks
35 Getting the direction ratios the given lines 1
Getting the direction ratios of the required line 2
using the given condition
Getting the equation of the line as 1
̂) + 𝝀(−𝟒𝒊̂ + 𝟒 𝒋̂ − 𝒌
⃗ = (𝒊̂ + 𝒋̂ + 𝒌
𝒓 ̂)
Getting the angle as 1
𝟐𝟒
𝒄𝒐𝒔−𝟏 ( )
√𝟔𝟎𝟗
36 a) Getting the total area as 2
𝟐 𝟐
𝝅𝒓𝟐
(𝟒𝟎𝟎 𝒓 − 𝝅𝒓 − 𝟐𝒓 ) +
𝟐
36 b) Finding the first and second derivative 1
𝟐𝟎𝟎 1
Proving area is maximum when 𝒓 =
𝝅+𝟐
37 a) Finding years in which increasing 2
37 b) Finding years in which decreasing 2
38 a) Calculating the probabilities 3
38 b) Drawing conclusion from the values of probabilities 1

252
SAMPLE QUESTION PAPER (2023-24) - 02
CLASS:XII MAX MARKS:80
SUBJECT : MATHEMATICS TIME : 3 HRS
BLUE PRINT
General Instructions :
1. This Question paper contains - five sections A, B, C, D and E. Each section is
compulsory. However, there are internal choices in some questions.
2. Section A has 18 MCQ’s and 02 Assertion-Reason based questions of 1 mark each.
3. Section B has 5 Very Short Answer (VSA)-type questions of 2 marks each.
4. Section C has 6 Short Answer (SA)-type questions of 3 marks each.
5. Section D has 4 Long Answer (LA)-type questions of 5 marks each.
6. Section E has 3 source based/case based/passage based/integrated units of
assessment (4 marks each) with sub parts.
CHAPTER MCQ’S & VERY SHORT LONG SOURCE TOTA UNIT
ASSERTI SHORT ANSWE ANSW BASED/CA L TOTA
ON- ANSW R ER SE BASED L
REASON ER (3 (5 (4 MARKS
(1 MARK) (2 MARK) MARK)
MARK)
RELATIONS & --- 1(2) * --- 1(5)* --- 2(7)
FUNCTIONS
INVERSE 1(1) (A R) --- --- --- --- 1(1) 3(8)
TRIGONOMETRI
C FUNCTIONS
MATRICES 1(1) --- --- --- --- 1(1) 6(10)
DETERMINANTS 4(4) --- --- 1(5) --- 5(9)
CONTINUITY & 2(2) 1(2) --- --- --- 3(4)
DIFFERENTIABI
LITY
APPLICATION --- 1(2) --- --- 2(8) 3(10)
OF 15(35)
DERIVATIVES
INTEGRALS 2(2) --- 1(3)*+2( --- --- 5(11)
6)
APPLICATION --- --- --- 1(5) --- 1(5)
OF INTEGRALS
DIFFERENTIAL 2(2) --- 1(3)* --- --- 3(5)
EQUATIONS
VECTOR 3(3) 1(2) --- --- --- 4(5)
ALGEBRA 8(14)
THREE 1(1)+1(1) 1(2) * --- 1(5)* --- 4(9)
DIMENSIONAL (A R)
GEOMETRY
LINEAR 2(2) --- 1(3) --- --- 3(5) 3(5)
PROGRAMMING
PROBABILITY 1(1) --- 1(3)* --- 1(4) 3(8) 3(8)

TOTAL 20(20) 5(10) 6(18) 4(20) 3(12) 38(80) 38(80)

* QUESTIONS WITH INTERNAL CHOICE

253
SAMPLE QUESTION PAPER (2023-24) - 02
CLASS:XII MAX
MARKS:80
SUBJECT : MATHEMATICS TIME : 3 HRS

General Instructions :
1. This Question paper contains - five sections A, B, C, D and E. Each section is
compulsory. However, there are internal choices in some questions.
2. Section A has 18 MCQ’s and 02 Assertion-Reason based questions of 1 mark
each.
3. Section B has 5 Very Short Answer (VSA)-type questions of 2 marks each.
4. Section C has 6 Short Answer (SA)-type questions of 3 marks each.
5. Section D has 4 Long Answer (LA)-type questions of 5 marks each.
6. Section E has 3 source based/case based/passage based/integrated units of
assessment (4 marks each) with sub parts.

SECTION A
(Multiple Choice Questions)
Each question carries 1 Mark
1. The number of all matrices of order 2 × 2 with each entry 2 or 3 is
a) 4 b) 8 c)16 d)32
𝛼 2
2. If 𝐴 = [ ] and |𝐴3 | = 27 , then the value of 𝛼 is
2 𝛼
𝑎) ± 1 b) ±2 c)±√5 d) ±√7
3. The angle between 𝑖̂ − 𝑗̂ and 𝑗̂ − 𝑘̂ is
𝜋 2𝜋 𝜋 5𝜋
a) b) c) d)
3 3 6 6
3𝑥 − 8, 𝑥 ≤ 5
4. If the function 𝑓(𝑥) = { is continuous , then the value
2𝑘, 𝑥 > 5
of 𝑘 is
2 7 3 4
a) b) c) d)
7 2 7 7
𝑥 1
5. ∫ 𝑒 (log√𝑥 + ) 𝑑𝑥 =
2𝑥
𝑒𝑥
𝑎)𝑒 𝑥 𝑙𝑜𝑔𝑥 + 𝑐 b) 𝑒 𝑥 log √𝑥 + 𝑐 c) +𝑐
2𝑥
d)𝑒 𝑥 log 𝑥 2 + 𝑐
6. The order and the degree of the differential equation
3
𝑑𝑦 3 𝑑3𝑦
(𝑑𝑥 ) + (𝑑𝑥 3 ) + 5𝑥 = 0 are
a) 3 ; 6 b) 3 ; 3 c) 3;9 d) 6;3

7. The graph of the inequality 2𝑥 + 3𝑦 > 6 is


a) Half plane that contains the origin

254
b) Half plane that neither contains the origin nor the points of the line
2𝑥 + 3𝑦 = 6
c) Whole XOY-plane excluding the points on the line 2𝑥 + 3𝑦 = 6
d) Entire XOY-plane
8. If 𝑎, 𝑏⃗ 𝑎𝑛𝑑 𝑐 are the position vectors of the points 𝐴(2,3, −4) ,
𝐵(3, −4, −5) and 𝐶(3,2, −3) respectively then |𝑎 + 𝑏⃗ + 𝑐 |=
a) √113 b) √185 c) √209 d) √203
𝜋
𝑑𝑥
9. ∫0 2 𝑑𝑥 =
1+√cot 𝑥
𝜋 𝜋 𝜋 𝜋
a) b) c) d)
3 2 6 4

6−𝑥 4
10. For what value of 𝑥 the matrix [ ] is a singular matrix
3−𝑥 1
a) 1 b) −1 c)2 d)−2
11.Based on the given shaded region as the feasible region in the graph ,
at which point(s) is the objective function 𝑧 = 3𝑥 + 9𝑦 maximum

a) Point B b) Point C c) Point D


d) every point on the line segment CD
2 4 2𝑥 4
12. If | |=| | , then 𝑥 =
5 1 6 𝑥
a) ±1 b) ±2 c)±√2 d) ±√3
3 1
13. If 𝐴 = [ ], then the value of |𝑎𝑑𝑗 𝐴| is
2 −3
a) 11 b)−11 c)9 d)−9
1
14.If A and B are two independent events such that 𝑃(𝐴) = and
3
1
𝑃(𝐵) = , Find 𝑃(𝐴′⁄𝐵′)
2
1 2 1 5
a) b) c) d)
2 3 6 6

𝑑𝑦
15. The integrating factor of the differential equation 𝑥 − 𝑦 = 𝑥 2 𝑐𝑜𝑠𝑥
𝑑𝑥
is
1
a) log 𝑥 b) − log 𝑥 c) 𝑥 d)
𝑥

255
𝑑𝑦
16. If 𝑒 𝑥 + 𝑒 𝑦 = 𝑒 𝑥+𝑦 , then =
𝑑𝑥
a) 𝑒 𝑦−𝑥 b) 𝑒 𝑥+𝑦 c) −𝑒 𝑦−𝑥
d) 2𝑒 𝑥−𝑦
17. The value of 𝑝 for which 𝑝(𝑖̂ + 𝑗̂ + 𝑘̂) is a unit vector is
1
a) 0 b) c) 1 d) √3
√3
18.The coordinates of the foot of the perpendicular drawn from the point
(2, −3,4) on the 𝑦 −axis is
a) (2,3,4) b)(−2, −3, −4) c)(0, −3,0) d)(2,0,4)
ASSERTION-REASON BASED QUSETIONS
In the following questions , a statement of assertion (A) is followed by as
statement of Reason ( R) . Choose the correct answer out of the following
choices .
a) Both A and R are true and R is the correct explanation of A.
b) Both A and R are true but R is not the correct explanation of A.
c) A is true but R is false.
d) A is false but R is true .
19.Assertion(A): sin−1 (0.76) is defined
Reason (R ) : sin−1 (0.76) is defined because it is defined for all real
numbers.
20. Assertion(A): 𝑎 = 𝑖̂ + 𝑗̂ + 2𝑘̂ is perpendicular to 𝑏⃗ = −𝑖̂ + 𝑗̂
Reason (R ) : Two vectors 𝑎 and 𝑏⃗ are perpendicular to each other if
𝑎. 𝑏⃗ = 0 is
SECTION B
This section comprises of very short answer type questions (VSA) of
2 marks each
21. Show that the signum function 𝑓: 𝑅 → 𝑅 given by
1 , 𝑖𝑓 𝑥 > 0
𝑓(𝑥) = { 0 , 𝑖𝑓 𝑥 = 0
−1 , 𝑖𝑓 𝑥 < 0
Is neither one-one nor onto
OR
13𝜋
Find the value of sin−1 [sin ( )]
7
𝑑2𝑦
22. If 𝑥 = 𝑎𝑡 2 , 𝑦 = 2𝑎𝑡 then find
𝑑𝑥 2

23. The radius of a right circular cylinder is increasing at the rate of


2cm/s and its height is decreasing at the rate of 8cm/s. Find the rate of
change of its volume, when the radius is 3 cm and height is 6cm.
24. Using vectors, find the area of triangle 𝐴𝐵𝐶 with vertices
𝐴(1, 1, 1) 𝐵(1, 2, 3) 𝑎𝑛𝑑 𝐶(2, 3, 1).

256
25. Find the angle between the lines ⃗⃗𝑟 = (2𝑗̂ − 3𝑘̂) + 𝜆(𝑖̂ + 2𝑗̂ + 2𝑘̂)
and ⃗⃗𝑟 = (2𝑖̂ + 6𝑗̂ + 3𝑘̂) + 𝜆(2𝑖̂ + 3𝑗̂ − 6𝑘̂)
OR
Find the value of k so that the lines 𝑥 = – 𝑦 = 𝑘𝑧 and 𝑥– 2 =
2𝑦 + 1 = – 𝑧 + 1 are perpendicular to each other.
SECTION C
(This section comprises of short answer type questions (SA) of 3
marks each)
26. Find ∫ sin−1 𝑥 𝑑𝑥.
4
27. Evaluate ∫−4|𝑥 + 2| 𝑑𝑥.
OR
3 √𝑥
Evaluate ∫1 𝑑𝑥
√𝑥+√4−𝑥
2𝑥
28. Find ∫ 𝑑𝑥
𝑥 2 +3𝑥+2
29. Find the general solution of the following differential equation
𝑦 𝑦
2𝑥𝑒 𝑥 𝑑𝑦 + (𝑥 − 2𝑦𝑒 𝑥 ) 𝑑𝑥 = 0
OR
Find the particular solution of the differential equation (2𝑥 2 +
𝑑𝑥
𝑦) = 𝑥 given that 𝑤ℎ𝑒𝑛 𝑥 = 1 , 𝑦 = 2
𝑑𝑦
30. Solve the following linear programming problem graphically
Maximize 𝑧 = 3𝑥 + 5𝑦 subject to 𝑥 + 𝑦 ≤ 5 ; 𝑥 ≥ 3 ; 𝑥 ≤ 4 ;𝑦 ≥ 0
31. A bag contains 19 tickets, numbered 1 to 19. A ticket is drawn at
random and then another ticket is drawn without replacing the first
one in the bag. Find the probability distribution of the number of even
numbers on the ticket. Also find the mean of the probability
distribution.
OR
Find the probability distribution of the number of successes in two tosses
of a die, when a success is defined as “number greater than 5”.Also find
the mean of the probability distribution.
SECTION D

(This section comprises of long answer-type questions (LA) of 5


marks each)
3 4 2
32. If 𝐴 = [0 2 −3] ; Find 𝐴−1
1 −2 6
Hence , solve the following system of equations
3𝑥 + 4𝑦 + 2𝑧 = 8
2𝑦 − 3𝑧 = 3
𝑥 − 2𝑦 + 6𝑧 = −2

257
33. Let N be the set of natural numbers and R be the relation on N × N
defined by (a, b) R (c, d) iff ad = bc for all a, b, c, d ∈ N. Show that R
is an equivalence relation.
OR
Show that the relation R on the set Z of all integers defined by
(𝑥, 𝑦) ∈ 𝑅 ⇒ (𝑥 – 𝑦) 𝑖𝑠 𝑑𝑖𝑣𝑖𝑠𝑖𝑏𝑙𝑒 𝑏𝑦 3 is an equivalence relation.
34. If the area between the curves 𝑥 = 𝑦 2 and 𝑥 = 4 divided into two
equal parts by the line 𝑥 = 𝑎 , then find the value of 𝑎 using
integration .
35. Prove that the line through A (0, –1, –1) and B(4, 5, 1) intersects the
line through C(3, 9, 4) and D(– 4, 4, 4).
OR
Find the vector and Cartesian equations of the line which is perpendicular
𝑥+2 𝑦−3 𝑧+1 𝑥−1 𝑦−2 𝑧−3
to the lines with equations = = and = =
1 2 4 2 3 4
and passes through the point (1, 1, 1).
SECTION E
(This section comprises of 3 case-study/passage-based questions of 4
marks each with two sub-parts. First two case study questions have
three sub parts (i) , (ii) ,(iii) of marks 1,1,2 respectively . The third
case study question has two sub parts of 2 marks each )
36.

Case-study 1 :Read the following passage and answer the questions


given below (2𝑥 and 2𝑦 are length and breadth of rectangular part)
The windows of a newly constructed building are in the form of a
rectangle surmounted by a semi circle . The perimeter of each window is
40m.
(i) Find the relation between 𝑥 and 𝑦
(ii) What is the area of the window in terms of 𝑥
(iii) Find the value of 𝑥 for which area of window will be maximum ?
OR
Find the value of 𝑦 for which area of window will be maximum ?
37. Case Study -2 : The total profit function of a company is given by

258
𝑃(𝑥) = −5𝑥 2 + 125𝑥 + 37500 where 𝑥 is the production of the
company
(i) Find the critical point of the function ?
(ii) Find the interval in which the function is strictly increasing ?
(iii) If 𝑃(𝑥) = −5𝑥 2 + 𝑚𝑥 + 37500 and 14 is the critical point , then
find
the value of 𝑚
OR
Find the absolute maximum for this value of 𝑚 in [0,16]
38. Case Study -3

An insurance company believes that people can be divided into two


classes: those who are accident prone and those who are not. The
company’s statistics show that an accident-prone person will have an
accident at sometime within a fixed one-year period with probability 0.6,
whereas this probability is 0.2 for a person who is not accident prone. The
company knows that 20 percent of the population is accident prone.
Based on the given information, answer the following questions.
(i)what is the probability that a new policyholder will have an accident
within a year of purchasing a policy?
(ii) Suppose that a new policyholder has an accident within a year of
purchasing a policy. What is the probability that he or she is accident
prone?
************

259
SAMPLE QUESTION PAPER (SESSION 2023-24) -02
CLASS:XII MAX
MARKS:80
SUBJECT : MATHEMATICS TIME : 3 HRS
MARKING SCHEME
Question Value Points Marks
No
1) c) 16 2) d) ±√7 3) b)
2𝜋
3
7
4) b) 5) b) 𝑒 𝑥 log √𝑥 + 𝑐
2
6) b) 3 ; 3
1 to 20 7) b) Half plane that neither contains the origin nor 1 Mark
the points of the line 2x+3y=6 for each
𝜋 correct
8) c) √209 9) d)
4
answer
10) c) 2
11 ) d) every point on the line segment CD
2
12) d) ±√3 13) b) −11 14) b)
3
1 𝑦−𝑥
15 ) d) 16) c) −𝑒
𝑥
1
17) b) 18) c) (0, −3,0)
√3
19) c) A is true but R is false.
20) a) Both A and R are true and R is the correct
explanation of A.

21 𝑓(1) = 𝑓(2) = 1 , So 𝑓 𝑖𝑠 not one one 1


as f(x) takes only 3 values (1, 0, or −1) there does not ½
exist any x in domain R such that f(x) = −2. ½
∴ f is not onto.
1
OR
13𝜋 𝜋 1
sin−1 [sin ( ) ] = sin−1 [sin (2𝜋 − ) ]
7 7
𝜋 𝜋
= sin−1 [sin (− ) ] = −
7 7
22 𝑑𝑦
𝑑𝑦 𝑑𝑡 2𝑎 1 1
= = =
𝑑𝑥 𝑑𝑥 2𝑎𝑡 𝑡
𝑑𝑡
2
𝑑 𝑦 1 𝑑𝑡 1 1 1 1
= − = − . = −
𝑑𝑥 2 𝑡 2 𝑑𝑥 𝑡 2 2𝑎𝑡 2𝑎𝑡 3
23 𝑑𝑟 𝑑ℎ ½
= 2 cm/s and =−8cm/s
𝑑𝑡 𝑑𝑡
𝑑𝑉 𝑑ℎ 𝑑𝑟
𝑉 = 𝜋𝑟 2 ℎ ⇒ = 𝜋𝑟 2 + 𝜋ℎ2𝑟 1
𝑑𝑡 𝑑𝑡 𝑑𝑡
260
𝑑𝑉 ½
(𝑎𝑡 𝑟 = 3 and ℎ = 6) = 0
𝑑𝑡
24 ½

25.
1

OR

1
26

27 4 −2 4

∫|𝑥 + 2| 𝑑𝑥 = ∫ −(𝑥 + 2) 𝑑𝑥 + ∫ −(𝑥 + 2) 𝑑𝑥


−4 −4 −2
−2 4
𝑥2 𝑥2
= −[ 2 + 2𝑥 ] + [2 + 2𝑥 ]
−4 −2 1½
= 20

½
1

261
27(OR)


½

28
½


½

29
½

OR
1

½
½
½

262
30

Correct
graph 1

The corner points of the feasible region are ½


𝐴(3,2),𝐵(3,0) ,𝐶(4,0) and 𝐷(4,1)
Corner Points 𝑍 = 3𝑥 + 5𝑦
𝐴(3,2) 19
𝐵(3,0) 9
𝐶(4,0) 12
𝐷(4,1) 17 1

Maximum of Z occurs at (3,2) and maximum of


½
Z =19
31 ½

½ +½+½

5 10 4
Mean of the distribution = 0 × +1× +2× =
19 19 19
18 1
19
OR
X =No of successes = No of times getting a number ½
greater than 5
X 0 1 2 ½+½+½
P(X) 5 5 25 1 5 10 1 1 1
× = 2× × = × =
6 6 36 6 6 36 6 6 36
25 10 1
Mean of the distribution =0 × +1× +2× =
36 36 36
12 1 1
=
36 3

263
32 ½

2
(1 mark
for 4
correct
cofactors)

33

OR
1

264

34

Correct
graph 1

35 𝑥 𝑦+1 𝑧+1 1
Equation of line AB is = =
4 6 2
½
Any point on this line is of the form (4𝜆, 6𝜆 − 1,2𝜆 − 1)
𝑥−3 𝑦−9 𝑧−4 1
Equation of line CD is = =
−7 −5 0
Any point on this line is of the form (−7𝜇 + 3, −5𝜇 + 9,4) ½
265
If line intersect then (4𝜆, 6𝜆 − 1,2𝜆 − 1) = (−7𝜇 + 3, −5𝜇 +
9,4)
5
⇒ 𝜆 = and 𝜇 = −1 1½
2
Also these values of 𝜆 𝑎𝑛𝑑 𝜇 gives the same set of
points .Hence lines intersect ½
OR
1

1
1
½
½

36 (i) 2𝑥 + 4𝑦 + 𝜋𝑥 = 40 1
𝜋𝑥 2 1
(ii) 40𝑥 − 2𝑥 2 −
2 2
(iii) Area is maximum when 𝑑𝑥𝑑 (40𝑥 − 2𝑥 2 − 𝜋𝑥2 ) = 0 ½
40
That is 𝑥 = 1
(𝜋+4)
Second derivative = −4 − 𝜋 < 0 ½
OR
Using in the equation related to 𝑥 and 𝑦 1
40
𝑦= 1
2(𝜋 + 4)
37 (i) 𝑥 = 12.5 1
(ii) (0,12.5) 1
(iii) For differntiating and equation to 0 1
𝑚 = 140 1
OR
For finding 𝑃(0), 𝑃(14), 𝑃(16) 1½
Absolute maximum = 38480 ½
38 Let E1 = The policy holder is accident prone.
E2 = The policy holder is not accident prone.
E = The new policy holder has an accident within a
year of purchasing a policy.
1
1

1
20 6
×
100 10 = 3 1
280 7
1000
266
SAMPLE QUESTION PAPER – 03
CLASS XII MATHEMATICS (041) 23—24
BLUE PRINT OF SAMPLE QUESTION PAPER (SQP)

Case based
Unit wise

( 4 marks)
questions
UNITS marks Chapters 1 Mark 2 marks 3 marks 5 marks

Either Or type

Either Or type

Either Or type

Either Or type
questions

questions

questions

questions

questions

questions

questions

questions
Single

Single

Single

Single
Relation and
Relations function
1(A&R) 1/1
and 8 Inverse
Functions Trigonometric 1/1
Functions
Matrices 1
Algebra 10
Determinants 4 1
Continuity and
Differentiability 2 1
Applications of
Derivatives
1(A&R) 2 1/1 1

Calculus 35 Integrals 1 1 1 1/1


Application of
the Integrals 1
Differential
Equations 2 1/1

Vectors and Vectors 3 1


Three-
14 Three
dimensional
Geometry dimensional 2 1/1
Geometry
Linear
Linear
Programmin 5
Programming 2 1/1
g
Probability 8 Probability 1 1 1
3
Total Total 18 + 2 5 6 4
80 Ques
Marks Questions Questions Questions Questions Questions
tions

Assertion Reason based questions

267
SAMPLE QUESTION PAPER - 03
MATHEMATICS (CODE – 041)
SESSION 2023-2024
Time: 3 hours Maximum
marks: 80

General Instructions:
1. This Question paper contains - five sections A, B, C, D and E. Each section is
compulsory. However, there are internal choices in some questions.
2. Section A has 18 MCQ’s and 02 Assertion-Reason based questions of 1 mark each.
3. Section B has 5 Very Short Answer (VSA)-type questions of 2 marks each.
4. Section C has 6 Short Answer (SA)-type questions of 3 marks each.
5. Section D has 4 Long Answer (LA)-type questions of 5 marks each.
6. Section E has 3 source based/case based/passage based/integrated units of
assessment of 4
marks each with sub-parts.

SECTION A
(Multiple Choice Questions )
Each question caries 1 mark
If A = [ 𝑎𝑖𝑗 ] is a skew-symmetric matrix of order n, then
Q1 1
a) 𝑎𝑖𝑗 = ∀ 𝑖, 𝑗 b) 𝑎𝑖𝑗 = 0 ∀ 𝑖, 𝑗 c) 𝑎𝑖𝑗 = 0, where i=j d) 𝑎𝑖𝑗 ≠ 0, where i =
𝑎𝑖𝑗
j
Given a relation R in set A = { 1, 2, 3}.Is relation R = {(1,1),(2,2), (2,3)}.
Q2 a) Reflexive b) Symmetric c) Transitive d) All the three
2𝑥 −1 3 0
Q3 If | |=| | then 𝓍 is
4 2 2 1
2 3 1
a) 3 b) 3 c) 2 d) − 4

A function f is said to be continuous for 𝓍 ∈ R , if


Q4 a) it is continuous at 𝓍 = 0 b) differentiable at 𝓍 = 0
c) continuous at two points d) differentiable for 𝓍 𝜖 R

The general point on the line 𝑟 = (2𝑖̂ + 𝑗̂ -4𝑘̂ ) + 𝜆(3𝑖̂ + 2𝑗̂-𝑘̂ ) is


Q5
a) (2, 1, -4) b) (3, 4, -1) c) (-1, 1, 3) d) (2+3λ, 1+2λ, -4-λ )
Integrating factor for the solution of differential equation (𝓍 − 𝑦 3 )𝒹𝓍 + 𝑦 𝒹𝓍= 0
Q6 is
1
a) 𝑦 b) log y c) y d) y2

The corner points of the bounded feasible region determined by a system of linear
Q7 constraints are (0,3), (1,1) and (3,0) . Let Z = px + qy, where p, q>0. The
condition on p and q so that the minimum of Z occurs at (3,0) and (1,1 ) is

268
𝑞
a) p = 2q b) p = 2 c) p = 3q d) p = q

The area of a triangle with vertices A, B, C is given by


Q8 1 1 1
a) | ⃗⃗⃗⃗⃗
𝐴𝐵 × ⃗⃗⃗⃗⃗
𝐴𝐶 | ⃗⃗⃗⃗⃗ × ⃗⃗⃗⃗⃗
b) 2 |𝐴𝐵 𝐴𝐶 | c) 4 | ⃗⃗⃗⃗⃗
𝐴𝐶 × ⃗⃗⃗⃗⃗
𝐴𝐵 | ⃗⃗⃗⃗⃗ × ⃗⃗⃗⃗⃗
d) 8 |𝐴𝐶 𝐴𝐵 |
𝑎 1 𝜋
If a is such that ∫0 𝑑𝑥 = , then the value of a is
Q9 1+4𝑥 2 8
1 1
a) 3 b) 2 c) 2 d) 1

𝑥 + 3𝑦 𝑦 4 −1
Q10 If [ ]=[ ] then x+y is
7−𝑥 4 0 4
a) 7 b) -1 c) 8 d) 6
The feasible region corresponding to the linear constraints of a linear programming
Q11 problem is given below.

Which of the following is not a


constraint to the given linear
programming problem?
a) 𝓍 + 𝑦 ≥ 2 b) 𝓍 + 2𝑦 ≤ 10
c) 𝓍 − 𝑦 ≥ 1 d) 𝓍 − 𝑦 ≤ 2

If |𝑎| = √3, |𝑏⃗ | = 2 and angle between 𝑎 and 𝑏⃗ is 60°, then 𝑎 . 𝑏⃗ is


Q12
1 1
a) √3 b) 2 c) 2 d) √3

Given a matrix A of order 3 × 3. If |A| = 3, then find |A.AdjA|.


Q13
a) 3 b) 27 c) 9 d) 81

A four digit number is formed by using the digits 1, 2, 3, 5 with no repetition. The
Q14 probability that
number is divisible by 5 is
1 1 1 1
a) 3 b) 4 c) 2 d) 6

Order of differential equation corresponding to family of curves y = 𝐴𝑒 2𝑥 + 𝐵𝑒 −2𝑥


Q15 is
a) 2 b) 1 c) 3 d) 4

If 𝑎̂ , 𝑏̂ and 𝑐̂ are mutually perpendiculars unit vectors, then the value of | 2𝑎̂ + 𝑏̂ +
Q16
𝑐̂ | is
a) √5 b) √3 c) √2 d) √6
The set of all points where the function f(x) = x + |x| is differentiable, is
Q17

269
a) (0,∞) b) (-∞, 0) c) (-∞,0) U (∞,0) d) (-∞,∞)
4−𝑥 𝑦 1−𝑧
Direction ratios of the line =6= are
Q18 2 3
a) 2, 6, 3 b) -2, 6, 3 c) 2, -6, 3 d) none of these

ASSERTION-REASON BASED QUESTIONS

In the following questions, a statement of Assertion (A) is followed by a


statement of Reason (R). Choose the correct answer out of the following
choices.

(a) Both (A) and (R) are true and (R) is the correct explanation of
(A).
(b) Both (A) and (R) are true but (R) is not the correct explanation of
(A).
(c) (A) is true but (R) is false.
(d) (A) is false but (R) is true.

ASSERTION (A) : The function y = [𝑥(𝑥 − 2)]2 is increasing in (0,1) U (2, ∞)


Q19 𝒅𝒚
REASON (R) : 𝒅𝒙 = 0, when x = 0, 1, 2

ASSERTION (A) : The relation f : { 1, 2, 3, 4} → {x, y, z, p} defined by f =


Q20 {(1,x), (2,y), (3,z)}
is a bijective function.
REASON (R) : The function f : { 1, 2, 3} → {x, y, z, p} defined by f = {(1,x),
(2,y), (3,z)} is
one-one.

SECTION - B
[This section comprises of very short answer type questions (VSA) of 2 marks
each]

13𝜋
Find the value of sin−1 [sin( )]
Q21 7
(OR)
1 2
Find the value of 𝑡𝑎𝑛2 (2 sin−1 3).

A man 1.6 m tall walks at the rate of 0.3 m/sec away from a street lightt is 4 m
Q22 above the ground .At what rate is the tip of his shadow moving? At what rate is
this shadow lengthening?

𝜋
Show that the function f(x) = log|𝑐𝑜𝑠𝑥| is strictly decreasing in ( 0, 2 ).
Q23
(OR)

270
Find the intervals in which the function f given by f(x) = 2𝑥 3 - 9𝑥 2 + 12𝓍 + 15 is
strictly
increasing or strictly decreasing.

−1 𝑥
𝑒 𝑚 tan
Q24 Evaluate ∫ 𝑑𝑥.
1+ 𝑥 2

Check whether the function f : ℝ → ℝ defined by f(x) = 𝑥 3 + 𝓍, has any critical


Q25 point/s or not ?
If yes, then find the point/s.

SECTION - C
[This section comprises of short answer type questions (SA) of 3 marks each]

e𝑥
Q26 Find ∫ (𝑒 𝑥 −1)2 (𝑒 𝑥 +2) 𝑑𝑥.

The random variable X has a probability distribution P(X)of the following form,
Q27 where ‘K’ is some real number:
𝑘, 𝑖𝑓 𝑥 = 0
2𝑘, 𝑖𝑓 𝑥 = 1
P(X) ={
3𝑘, 𝑖𝑓 𝑥 = 2
0, 𝑜𝑡ℎ𝑒𝑟𝑤𝑖𝑠𝑒
(i)Determine the value of k
(ii)Find P(X < 2)
(iii)Find P(X>2)

2
Evaluate ∫−1|𝑥 3 − 𝑥|dx.
Q28
(OR)
𝑥
Evaluate ∫ sin−1 √2+𝑥 dx.

Solve the differential equation: ydx +(x˗𝑦 2 )dy.


Q29 (OR)
Solve the differential equation: xdy ˗ ydx = √𝑥 2 + 𝑦 2 dx.

Solve the following linear programming problem (LPP) graphically.


Q30 Maximize Z = x + y
𝑥 𝑦
Subject to the constraints 25 + 40 ≤ 1; 2x + 5y ≤ 100 , x ≥ 0, y ≥ 0
(OR)
Solve the following linear programming graphically,
Maximize Z = 3x + 4y +370
Subject to the constraints,
y≥ 0

271
x + y ≤ 60
x ≤ 40
y ≤ 40
x + y ≥ 10

𝑑2 𝑦 𝑑𝑦
Q31 If y = A𝑒 𝑚𝑥 + B𝑒 𝑛𝑥 , prove that - ( m + n)𝑑𝑥 + mny =0
𝑑𝑥 2

SECTION - D
[This section comprises of Long answer type questions (LA) of 5 marks each]

Make a rough sketch of the region{(𝑥, 𝑦): 0 ≤ y≤ 𝑥 2 , 0≤ y ≤ x ,0 ≤ x ≤ 2} and find


Q32 the area of the region using integration.

Let ℕ be the set of all natural numbers and R be a relation on ℕ× ℕ defined by


Q33 (a, b)R(c, d)⇔ ad =bc for all (a, b), (c, d) ∈ ℕ×ℕ .Show that R is an equivalence
relation on
ℕ×ℕ. Also, find the equivalence class of (2, 6), i.e.,[(2,6)]
(OR)
𝑥
Show that the function 𝑓 : ℝ→{𝑥 ∊ ℝ ∶ −1 < 𝑥 < 1} defined by 𝑓(x) = 1+|𝑥| ,
x∊ ℝ is
One-one and onto function.

−4 4 4 1 −1 1
Q34 Determine the product [−7 1 3 ] [1 −2 −2] and use it to solve the
5 −3 −1 2 1 3
system of equations
x- y + z = 4; x ˗ 2y ˗2z = 9 ; 2x + y +3z =1

Find the shortest distance between the following two lines :


Q35
𝑟 = (1+λ)𝑖̂ + (2 ˗ λ)𝑗̂ + (λ+1)𝑘̂ ; 𝑟 = (2𝑖̂˗𝑗̂˗𝑘̂) +μ(2𝑖̂ +𝑗̂ +2𝑘̂)
(OR)
Find the equation of a line passing through the point P(2,˗1,3) and perpendicular
to the lines:
𝑟 = (𝑖̂+𝑗̂-𝑘̂)+λ(2𝑖̂-2𝑗̂+𝑘̂) and 𝑟 = (2𝑖̂˗𝑗̂˗3𝑘̂)+μ(𝑖̂+2𝑗̂+2𝑘̂)

SECTION – E
[ This section comprises of 3 case – study /passage based questions of 4 marks
each with sub parts. The first two case study questions have three sub

272
parts(i),(ii) of marks 1, 1, 2 respectively. The third case study questions has
two sub parts of 2 marks each.]
Case – Study: Read the following passage and answer the questions given below.
Q36

These days competitive examinations are online, a student has to go to a particular


place to give the examination at the given time. For this , company has to make
perfect arrangements and students are expected to be well prepared . But as a
human nature sometimes a student guesses or copies or knows the answer to a
multiple choice question with four choices each.
i) If the probability that a student makes a guess is 1/3 and that he copies the
answer is 1/6 .
what is the probability that he knows the answer?
ii) If answer is correct what is the probability that he guesses it?
III) What is the conditional probability that this answer is correct and he knew it?
OR
iii) what is the probability that he copied it given that his answer is correct?
Case – Study 2: Read the following passage and answer the questions given below.
Q37 A village panchayat wants to dug out a square base tank. For preparing fertilizers
and wants capacity to be 250 cubic meters. On calculations it was found tht cost
of the land is Rs. 50 per square meter and cost of digging increases with dept and
cost of the whole tank is Rs. 400 (Depth) . Tank is shown as

273
i) If the side of the square base is’ x’ m and the height of the tank is ‘h ‘m. then
establish relations between’ x ‘and ‘h ‘
ii) Find the cost C for digging the tank in terms of ‘x’ and’ h’
iii) Find the cost C in terms of ’ h ‘only
OR
iii) Find the value of ‘h’ for which cost C is minimum.

Case study- 3
Q38 Read the following passage and answer the questions given below

A building is to be constructed in the form of triangular pyramid, ABCD as shown


in the figure. Let its angular points are A(0,1,2), B(3,0,1), C((4,3,6) and D(2,3,2)
and G be the point of intersection of the medians of triangle ABCD
i)Find the coordinates of point G
ii)Find the length of vector

274
MARKING SCHEME
SAMPLE QUESTION PAPER -03
MATHEMATICS (CODE – 041)
SESSION 2023-2024
Time: 3 hours Maximum
marks: 80

General Instructions:
7. This Question paper contains - five sections A, B, C, D and E. Each section is
compulsory. However, there are internal choices in some questions.
8. Section A has 18 MCQ’s and 02 Assertion-Reason based questions of 1 mark each.
9. Section B has 5 Very Short Answer (VSA)-type questions of 2 marks each.
10. Section C has 6 Short Answer (SA)-type questions of 3 marks each.
11. Section D has 4 Long Answer (LA)-type questions of 5 marks each.
12. Section E has 3 source based/case based/passage based/integrated units of
assessment of 4
marks each with sub-parts.

Q. SECTION A Mark
No. (Multiple Choice Questions )
Each question caries 1 mark
(c), In a skew-symmetric matrix, the (i, j)th element is negative of the 1
Q1 (j,i)th element. Hence,
the (i, i)th element = 0.

(d), Relation R in set A is not reflexive as for a ∈ A, (a,a) ∉ R, e.g. (3,3) ∉ 1


Q2 R but 3 ∈ A.
Relation R in set A is not symmetric as for 𝑎1 , 𝑎2 ∈ A, (𝑎1 , 𝑎2 ) ∈ R,
(𝑎2 , 𝑎1 ) ∉ R but
(3, 2) ∉ R
Relation R in set A is not transitive as (2, 3) ∈ R, (2,2) ∈ R but (3, 2)
∉R

2𝑥 −1 3 0 1
Q3 (d), as | |=| |
4 2 2 1
⟹ 4𝓍 + 4 = 3 – 0
1
⟹ 𝓍 = -4

(d), as differentiable functions is continuous also. 1


Q4
(d), as given line is 𝑟 = (2𝑖̂ + 𝑗̂ - 4𝑘̂) + 𝜆 (3𝑖̂ + 2𝑗̂ - 𝑘̂) 1
Q5
⸫ position vector of a point through which line passes.
⸫ 𝑟 = (2 + 3𝜆)𝑖̂ + (1 + 2𝜆)𝑗̂ + (-4 – 𝜆)𝑘̂
⸫ General point is (2 + 3 𝜆, 1 + 2 𝜆, -4 – 𝜆)

275
(c), Equation is (𝓍 − 𝑦 3 ) 𝒹𝑦 + y 𝒹𝓍 = 0 1
Q6 𝑑𝑦 𝑥−𝑦 3 𝑑𝑥 1
⟹ =- ⟹ 𝑑𝑦 + 𝑦 . x =𝑦 2
𝑑𝑥 𝑦
1
∫𝑦𝑑𝑦
Integrating factor = 𝑒 = 𝑒 log 𝑦 = y

(b), Z = px + qy --------(i) 1
Q7 At (3,0) , Z = 3p ------(ii) and at (1,1), z = p + q --------(iii)
From (ii) & (iii), 3p = p + q ⟹ 2p = q

(b), The area of the parallelograms with adjacent sides AB and AC = |⃗⃗⃗⃗⃗⃗
𝐴𝐵 1
Q8
× ⃗⃗⃗⃗⃗
𝐴𝐶 |. Hence,
1
the area of the triangle with vertices A, B, C = 2 |⃗⃗⃗⃗⃗⃗
𝐴𝐵 × ⃗⃗⃗⃗⃗
𝐴𝐶 |.
𝑎 1 1
(b), as ∫0 𝒹𝓍 = [2 tan−1 2𝑥]𝑎0 1
Q9 1+(2𝑥)2
𝜋
=8
1 𝜋
tan−1(2𝑎) =8
2
𝜋
2a = tan 4
1
a =2

𝑥 + 3𝑦 𝑦 4 −1 1
Q10 (d) [ ]=[ ]
7−𝑥 4 0 4
x+3y=4 and y = -1 therefore x = 7
x+y= 7+(-1) = 6

(c) , We observe, (0,0) does not satisfy the inequality 𝓍 − 𝑦 ≥ 1 1


Q11 So, the half plane represented by the above inequality will not
contain origin
Therefore, it will not contain the shaded feasible region.

1
(a) as 𝑎.𝑏⃗ = |𝑎| |𝑏|
⃗⃗⃗ cos 60° = √3 × 2 × = √3 1
Q12 2
(b), as |A.Adj A| = |𝐴|3 = |3|3 = 27 1
Q13
(b), As total possibilities for four digit number is n(S) = 4! 1
Q14 Favourable possibilities for number to be divisible by 5, n(A) =
3! × 1 = 3!
𝑛(𝐴) 3! 1
⸫ Required probability = 𝑛(𝑆) = 4! = 4

(a) Two arbitrary constants present therefore order is 2 1


Q15
(d), as 𝑎̂, 𝑏̂ and 𝑐̂ are mutually perpendicular unit vectors. 1
Q16 ̂ = |𝑏|
̂ = |𝑐|
̂ =1
⸫ |𝑎|
Consider, |2𝑎̂ + 𝑏̂ + 𝑐̂ |2 = (2𝑎̂ + 𝑏̂ + ̂𝑐 )2
= 4𝑎̂ + 𝑏̂ 2 + 𝑐̂̂2 + 4𝑎̂. 𝑏̂ + 4𝑎̂.𝑐̂ + 2𝑏̂.𝑐̂
= 4|𝑎̂|2 + |𝑏̂|2 + |𝑐̂ |2 + 0 + 0 + 0

276
=4+1+1=6
⇒ |2𝑎̂ + 𝑏̂ + 𝑐̂ | = √6
2

(c) , Method 1: 1
Q17 2𝑥, 𝑥 ≥ 0
f(𝓍) = 𝓍 + |𝓍| = {
0, 𝑥 < 0

y = 2 𝓍, 𝓍 ≥ 0

y = 0, 𝓍<0
X' O X

Y
There is a sharp corner at 𝓍 = 0, so f(x) is not differentiable at 𝓍 =
0.
Method 2:
Lf ′(0) = 0& Rf '(0) = 2 ; so, the function is not differentiable at x
=0
For 𝓍 ≥ 0,f(𝓍) = 2𝓍 (linear function) & when 𝓍 < 0, f(𝓍) = 0
(constant function)
Hence f(𝓍) is differentiable when 𝓍 ∈ ( -∞, 0) U (0, ∞).
𝑥−4 𝑦 𝑧−1 1
(b) line =6= , dr’s are < -2,6,3 >
Q18 −2 −3

ASSERTION-REASON BASED QUESTIONS

In the following questions, a statement of Assertion (A) is followed


by a statement of Reason (R). Choose the correct answer out of the
following choices.

(e) Both (A) and (R) are true and (R) is the correct explanation of
(A).
(f) Both (A) and (R) are true but (R) is not the correct
explanation of (A).
(g) (A) is true but (R) is false.
(h) (A) is false but (R) is true.

(b), Both A and R are true but R is not the correct explanation of A. 1
Q19
(d), Assertion is false. As element 4 has no image under f, so relation f 1
Q20 is not a function.
Reason is true. The given function f :{1, 2, 3} → {𝔁, 𝒚, 𝒛, 𝒑} is one
– one, for each
a ∈ {1, 2, 3}, there is different image in {𝔁, 𝒚, 𝒛, 𝒑} under f.

SECTION - B

277
[This section comprises of very short answer type questions (VSA) of 2
marks each]

13𝜋 𝜋 1
sin−1[ sin( )] = sin−1[sin(2𝜋 − 7 )]
Q21 7

𝜋
= sin−1 [ sin(− 7 )]
𝜋
=-7 1

(OR)
−1 2 2 √5
Let 𝜃 = sin ⇒ sin 𝜃 = 3 , cos 𝜃 =
3 3
𝜃 𝜃
1 2 𝜃 𝑠𝑖𝑛2 2 2𝑠𝑖𝑛2 2
𝑡𝑎𝑛2 ( 2 sin−1 3 ) = 𝑡𝑎𝑛2 2 = 𝜃 = 𝜃
𝑐𝑜𝑠2 2 2𝑐𝑜𝑠2 2
√5
1−cos 𝜃 1− 1
3
= 1+cos 𝜃 = √5
1+
3
3− √5 ( 3−√5)2
= 3+ √5 = 32 − (√5)2 [ on Rationalisation]
(3−√5)2 9+5−6√5
= =
4 4
7−3√5
= 1
2

Let AB represent the height of the street light from the ground. At any
Q22 time t seconds, let the man represented as ED of height 1.6m be at a distance of
x from AB and the length of his shadow EC by y m.
Using similarity of triangles , we have
4 𝑥+𝑦
=
1.6 𝑦 1
3y = 2x
Differentiating both sides w.r.t ‘t ‘, we get

𝑑𝑦 𝑑𝑥
3 =
𝑑𝑡 𝑑𝑡
𝑑𝑦 2
= (0.3)
𝑑𝑡 3
𝑑𝑦
= (0.2)
𝑑𝑡
At any time t seconds, the tip of his shadow is at a distance of (x+y) m from
AB 1
The rate at which his shadow is lengthening = 0.2m/s

𝟏 𝜋 1
f '(𝓍) = 𝒄𝒐𝒔𝒙 . (-sin 𝓍) = - tan 𝓍.,tan 𝓍 > 0 for ( 0, 2 ).
Q23
1
f '(𝓍) < 0. Hence function is strictly decreasing.

(OR)

Consider f(𝓍) = 2𝑥 3 - 9𝑥 2 + 12 𝓍 + 15

278
f '(𝓍) = 6𝓍 2 - 18 𝓍 + 12 = 6(𝓍 2 - 3 𝓍 + 2)

= 6(𝓍 − 1) (𝓍 − 2)
1
For critical points f '(𝓍) = 0 ⇒ 𝓍 = 1, 2

-∞ ∞
-2 -1

𝔁<1 1< 𝔁 < 2 𝔁>2

6 + + +

𝓍−1 - - -

𝓍−2 - - +

f '( 𝓍) + - +
1
↑ ↓ ↑

Strictly increasing for (-∞, 1) U (2, ∞), strictly decreasing in (1, 2)

−1 𝑥
𝑒 tan 1
Q24 Consider ∫ 𝑑𝑥 = 𝑚 ∫ 𝑒 𝑡 𝑑𝑡
1+ 𝑥2
1 1 −1 𝑥
= 𝑚 𝑒 𝑡 + C = 𝑚 𝑒 𝑚 tan +c Let m tan−1 𝑥 = t

𝑚
⇒ 𝑑𝑥 = 𝒹𝑥
1+ 𝑥 2

f (𝔁) = 𝒙𝟑 + 𝒙, for all 𝔁 ∈ ℝ.


Q25 𝒅 1
(f(x)) = f '(x) = 3𝒙𝟐 + 1; for all 𝔁 ∈ ℝ, 𝓍 2 ≥ 0 ⇒ f '(x) > 0
𝒅𝒙
1
Hence, no critical point exits.

SECTION - C
[This section comprises of short answer type questions (SA) of 3 marks
each]

279
−1
𝑒 𝑚 tan 𝑥
Q26 ∫ 𝑑𝑥
1 + 𝑥2
let u = tan-1x
1 1
du = 2
𝑑𝑥
1+𝑥
−1
𝑒 𝑚 tan 𝑥 1
∫ 𝑑𝑥 =∫ 𝑒 𝑚𝑢 𝑑𝑢
1+ 𝑥 2
𝑒 𝑚𝑢
= +𝑐
𝑚
−1 1
𝑒 𝑚 tan 𝑥
= +𝑐
𝑚

Q27 We have i) ∑ 𝑃(𝑋 ) = 1


𝑖
k+2k+3k=1
6k=1
1 1
k=
6
ii)P(X<2)=P(X)=0+P(X)=1
k+2k=3k
1
=3.
6 1
1
=
2
iii) P(X>2) = 0
1
2
Consider ∫−1|𝑥 3 − 𝑥|dx,
Q28
Now 𝑥 3 ˗x =0 ⟹ x(𝑥 2 −1) =0 ⟹x(x+1)(x−1) =0
⟹x = 0,-1,1
For -1 < x< 0 , 𝑥 3 ˗ x is positive
For 0 < x <1 , 𝑥 3 ˗ x is negative
For 1 <x < 2 , 𝑥 3 ˗ x is positive
2 0 1 2 1
∴ ∫−1|𝑥 3 − 𝑥| dx = ∫−1(𝑥 3 − 𝑥)dx ˗ ∫0 (𝑥 3 − 𝑥)dx + ∫1 (𝑥 3 − 𝑥)dx
0 1 2
𝑥4 𝑥2 𝑥4 𝑥2 𝑥4 𝑥2 1
=| 4 − | ˗| 4 − | +|4 − |
2 −1 2 0 2 1
1 1 1 1 16 4 1 1
=(0-0) –( 4 - 2 ) – (4 - 2) + (0-0) + ( - 2 ) – (4 - 2) 1
4
1 1 1 3 11
=4+4+2+4⟹2+4 ⟹ 4
(OR)
𝑥
Consider ∫ 𝑆𝑖𝑛−1 √2+𝑥 dx.

Let x = 2𝑡𝑎𝑛2 𝜃 ⟹dx = 4tan 𝜃 𝑆𝑒𝑐 2 𝜃 d 𝜃 1

2𝑡𝑎𝑛2 𝜃
= ∫ 𝑆𝑖𝑛−1 √2+2𝑡𝑎𝑛2 𝜃 × 4tan 𝜃 𝑆𝑒𝑐 2 𝜃 d 𝜃

= 4∫ 𝑆𝑖𝑛−1 (𝑆𝑖𝑛𝜃) .(tan 𝜃 𝑆𝑒𝑐 2 𝜃 )d 𝜃.

280
= 4 ∫ 𝜃 (tan 𝜃 𝑆𝑒𝑐 2 𝜃 )d 𝜃. 1

𝑡𝑎𝑛2 𝜃
=4 [𝜃 𝑡𝑎𝑛2 𝜃 − ∫ 1. 𝑑𝜃] [∵
2
𝑡𝑎𝑛2 𝜃
∫ (tan 𝜃 𝑆𝑒𝑐 2 𝜃 )d 𝜃 = 2
]

= 2 [𝜃 𝑡𝑎𝑛2 𝜃 − ∫(𝑠𝑒𝑐 2 𝜃 − 1)𝑑𝜃]

= 2 [𝜃 𝑡𝑎𝑛2 𝜃 − 𝑡𝑎𝑛𝜃 + 𝜃] + C

𝑥 𝑥 𝑥 𝑥
= 2 [ 𝑡𝑎𝑛−1 √2 . − √2 − 𝑡𝑎𝑛−1 √2 ] + C 1
2

𝑥 𝑥
=(x ˗ 2 )𝑡𝑎𝑛−1 √2 ˗ 2 √2 + C

ydx +(x˗𝑦 2 )dy=0


Q29 Reducing the given differential equation to the form
𝑑𝑥 ½
+ Px =Q
𝑑𝑦

𝑑𝑥 𝑥
We get, 𝑑𝑦 +𝑦 = y
𝑥
∫𝑦 𝑑𝑦
I.F = 𝑒 ∫ 𝑝𝑑𝑦 = 𝑒
½

= 𝑒 𝑙𝑜𝑔𝑦 = y

The general solution is given by 1


x.I.F = ∫ 𝑄. 𝐼𝐹 𝑑𝑦
⇒ xy = ∫ 𝑦 2 dy
𝑦3
⇒ xy = + c,
3
Which is the required general solution 1

(OR)

x dy ˗ y dx = √𝑥 2 + 𝑦 2 dx.
It is a Homogeneous Equation as
½
𝑑𝑦 √(𝑥^2+𝑦^2 ) + 𝑦
=
𝑑𝑥 𝑥

𝑦 2 𝑦 𝑦
= √1 + (𝑥 ) +𝑥 = f(𝑥 )
Put y = vx
𝑑𝑦 𝑑𝑣
= v + x𝑑𝑥
𝑑𝑥

𝑑𝑣
v + x𝑑𝑥 = √1 + 𝑣 2 + v ½

Separating variables , and by integrating we get

281
log|𝑣 + √1 + 𝑣 2 | = log|𝑥| + log K, k>0 1
log|𝑦 + √𝑥 2 + 𝑦 2 | = log 𝑥 2 k
⇒ 𝑦 + √𝑥 2 + 𝑦 2 = ± K𝑥 2
⇒ 𝑦 + √𝑥 2 + 𝑦 2 = 𝐶𝑥 2
Which is the required general solution 1

Q30 2(graph
)

Points Z= x+ y Values
25+0 25
A(25,0)
50 40 30
50 40 +3
B( 3 , 3 ) 3

0+20 20
C(0.20)

Plotting the in equations as graph we notice shaded portion is feasible solution


50 40
.Possible points for maximum Z are A(25,0),B( 3 , ) and C(0,20).
3
50 40 50 40
Z is maximum at B( , 3 ) , i.e. x = ,y=
3 3 3
1

(OR)

Points Z = 3x + 4y +370 Values

30+0+370 400 2(graph


A(10,0)
120+0+370 490 )
B(40,0)
120+80+370 570
C(40,20)
60+160+370 590
D(20,40)
0+160+370 530
E(0,40)
0+40+370 410
F(0,10)

282
1

On plotting the in equations we notice shaded portion is feasible solution.


Possible points for minimum Z are
A(10,0),B(40,0),C(40,20),D(20,40),E(0,40),F(0,10)
Z is minimum for A(10,0),i.e, x = 10 , y = 0
We have, y = A𝑒 𝑚𝑥 + B𝑒 𝑛𝑥 …(i)
Q31 𝑑𝑦 1
⇒ = A𝑚𝑒 𝑚𝑥 + B𝑛𝑒 𝑛𝑥 …(ii)
𝑑𝑥
𝑑2 𝑦
and = A𝑚2 𝑒 𝑚𝑥 + B𝑛2 𝑒 𝑛𝑥 …(iii) 1
𝑑𝑥 2
𝑑2 𝑦 𝑑𝑦
Consider 𝑑𝑥 2 - (m + n) 𝑑𝑥 + mny
= A𝑚2 𝑒 𝑚𝑥 + B𝑛2 𝑒 𝑛𝑥 - (m + n)( A𝑚𝑒 𝑚𝑥 + B𝑛𝑒 𝑛𝑥 ) + mn(A𝑒 𝑚𝑥 + B𝑒 𝑛𝑥 )
[from (i), (ii) and (iii)]
= A𝑚2 𝑒 𝑚𝑥 + B𝑛2 𝑒 𝑛𝑥 - A𝑚2 𝑒 𝑚𝑥 - B𝑚𝑛𝑒 𝑛𝑥 - A𝑚𝑛𝑒 𝑚𝑥 - B𝑛2 𝑒 𝑛𝑥 + A𝑚𝑛𝑒 𝑚𝑥 1
+ B𝑚𝑛𝑒 𝑛𝑥

SECTION - D
[This section comprises of Long answer type questions (LA) of 5 marks
each]

Q32

The points of intersection of the parabola y=x2 and the line y=s are (0,0)
and ( 1,1) 1
1 2 1
Required area =∫0 𝑦𝑑𝑥 + ∫1 𝑦𝑑𝑥
parabola line

1+1

283
1 2
=∫0 𝑥 2 𝑑𝑥 + ∫1 𝑥𝑑𝑥 = 11/6 ( after integration and
applying the limits)
Let (a, b) be an arbitrary element of ℕ×ℕ. Then (a, b) ∈ ℕ×ℕ and a ,b∈ ℕ
Q33 We have ab = ba ;(as a ,b∈ ℕ and multiplication is commutative on ℕ)
⇒(a,b)R(a,b),according to the definition of the relation R on ℕ×ℕ 1
Thus (a, b)R(a, b),∀(a,b) ∈ ℕ×ℕ
So,R is reflexive relation on ℕ×ℕ.

Let (a, b),(c, d) be arbitrary elements of ℕ×ℕ such that (a,b)R(c,d)


Then (a,b)R(c,d) ⇒ ad=bc⇒bc =ad;(changing LHS and RHS)
⇒cb=da; (As a, b, c, d ∈ ℕ and multiplication is commutative on ℕ)
⇒( c, d) R (a, b); according to the definition of the relation R on ℕ×ℕ 1.5
Thus (a,b)R(c,d) ⇒ ( c, d) R (a, b) So R is symmetric relation on ℕ×ℕ

Let (a, b),(c, d), (e, f) be arbitrary elements of ℕ×ℕ such that (a,b)R(c,d) and (
c, d) R (e, f)
Then (a,b)R(c,d) ⇒ ad = bc
( c, d) R (e, f) ⇒ cf = de ⇒(ad) (cf) = (bc) (de) ⇒ af = be

⇒ (a,b) R (e,f) ; according to the definition of the relation R on ℕ×ℕ


Thus (a,b)R(c,d) and ( c, d) R (e, f) ⇒ (a,b) R (e,f)
So R is transitive relation on ℕ×ℕ 1.5
as the relation R is reflexive, symmetric and transitive so, it is equivalence
relation on ℕ×ℕ

[ ( 2, 6)] = { (x,y) ∈ ℕ×ℕ : (x,y) R (2,6)} 1


= { (x,y) ∈ ℕ×ℕ : 3x = y}
= {( x, 3x) : x ∈ ℕ } = { (1, 3), (2, 6), (3, 9),…………}
(OR)
𝑥
, 𝑥<0
we have 𝑓(𝑥) = {1−𝑥 𝑥
, 𝑥≥0
1+𝑥
Now we consider the following cases
𝑥
Case 1: when x≥ 0 we have f(x)=1+𝑥
Injectivity; Let x,y ∈R+U{0}
such that f(x)=f(y)
𝑥 𝑦
= 1+𝑦
1+𝑥 1
x + xy = y + xy
x=y
so, f is injective function.
𝑥
Surjective : When x≥0, f(x)=1+𝑥 ≥0
1
and f(x)=1 − 1+𝑥<1, as x≥ 0
𝑦
1
𝑦 1−𝑦
let y∈[0,1), thus for each y∈[0,1) there exists x=1−𝑦 ≥ 0 such that f(x)= 𝑦 =
1−
1−𝑦
y
so , f is onto function on [0,∞) 𝑡𝑜[0,1)

284
Case 2: when x<0
𝑥
we have f(x)=
1−𝑥
Injectivity; Let x,y ∈R- such that f(x)=f(y)
𝑥 𝑦
=
1−𝑥 1−𝑦 1
x - xy = y - xy
x=y
so, f is injective function

𝑥
Surjective : When x<0 f(x)=1−𝑥 <0
𝑥
and f(x)=1 − 1−𝑥
1
= 1 + 1−𝑥 > −1<1, -1< f(x) < 0 1
𝑦
let y∈ (−1,0), be an arbitrary real number and there exists x=1+𝑦 < 0 such
𝑦
1+𝑦
that f(x)= 𝑦 =y
1−
1+𝑦
𝑦
so , for y∈ (−1,0)there exists x=1+𝑦<0 such that f(x) = y
Hence f is onto function on (-∞, 0) 𝑡𝑜 (−1,0)

Case 3: (Injectivity) Let x>0 & y<0 such that f(x)=f(y)


𝑥 𝑦
=
1+𝑥 1−𝑦 1
x-xy=y+xy
x-y=2xy
LHS>0 but RHS <0 which is in admissible
hence f(x) ≠ 𝑓(𝑦)when x≠ 𝑦
so f is one-one and onto function

−4 4 4
Q34 Consider A = [−7 1 3]
5 −3 −1

1 −1 1
And B = [1 −2 −2]
2 1 3

−4 4 4 1 −1 1
AB = [−7 1 3 ] [1 −2 −2]
5 −3 −1 2 1 3

−4 + 4 + 8 4 − 8 + 4 −4 − 8 + 12
= [−7 + 1 + 6 7 − 2 + 3 −7 − 2 + 9 ]
5 − 3 − 2 −5 + 6 − 1 5+6−3

8 0 0
AB = [0 8 0] = 8I …(i)
0 0 8 2

285
Consider equations
𝓍 − 𝑦 + 𝑧 = 4; 𝑥 − 2𝑦 − 2𝑧 = 9; 2𝑥 + 𝑦 + 3𝑧 = 1
Corresponding matrix equation is
1 −1 1 𝑥 4
𝑦
[1 −2 −2] [ ] = [9]
2 1 3 𝑧 1
1
⇒ BX = C is matrix equation.

Its solution is X = 𝐵 −1 C

From (i), we have


1
AB = 8I ⇒ (8 A) B = I 1
1
⇒ 𝐵 −1 = 8 A
−4 4 4 4
1 1
⸫ From (ii), X = 8 AC = 8 [−7 1 3 ] [9 ]
5 −3 −1 1

𝑥 −16 + 36 + 4 24 3
1 1
⇒ [𝑦] = 8 [ −28 + 9 + 3 ] = 8 [−16] = [−2]
𝑧 20 − 27 − 1 −8 −1
1
⸫ 𝓍 = 3, 𝑦 = −2, 𝑧 = −1 is required solution.

Consider the line


Q35
𝑟 = (1+λ)𝑖̂ + (2 ˗ λ)𝑗̂ + (λ+1)𝑘̂
i.e. 𝑟 = (𝑖̂ + 2 𝑗̂ + 𝑘̂ ) + λ(𝑖̂ - 𝑗̂ + 𝑘̂)

𝑎1 = 𝑖̂ + 2 𝑗̂ + 𝑘̂ , ⃗⃗⃗
Here ⃗⃗⃗⃗ 𝑏1 = 𝑖̂ - 𝑗̂ + 𝑘̂……..(i) ½

Consider ,the line


𝑟 = (2𝑖̂˗𝑗̂˗𝑘̂) +μ(2𝑖̂ +𝑗̂ +2𝑘̂)
½
Here ⃗⃗⃗⃗
𝑎2 = 2𝑖 ̂ ˗ 𝑗̂ ˗𝑘̂ , ⃗⃗⃗⃗
𝑏2 = 2𝑖̂ + 𝑗̂ +2𝑘̂……….(ii)

Shortest distance between the lines ½


(𝑎 ⃗⃗⃗⃗ ×𝑏
⃗⃗⃗⃗⃗ ) .(𝑏
⃗⃗⃗⃗⃗ ˗𝑎 ⃗⃗⃗⃗ )
= | 2 |𝑏⃗⃗⃗⃗1 ×𝑏⃗⃗⃗⃗1 | 2 |
1 2

Now ⃗⃗⃗⃗
𝑎2 ˗⃗⃗⃗⃗⃗
𝑎1 =2𝑖 ̂ ˗ 𝑗̂ ˗ ̂𝑘-𝑖̂ -2𝑗̂ - 𝑘̂
½
= ̂𝑖 -3𝑗̂ -2𝑘̂ [from (i) and (ii)]

𝑖̂ 𝑗̂ 𝑘̂
⃗⃗⃗
𝑏1 × ⃗⃗⃗⃗
𝑏2 =|1 −1 1|
2 1 2
= 𝑖̂(-2-1) ˗ 𝑗̂(2 ˗ 2) + 𝑘̂(1+2) ½
= -3𝑖̂ + 3𝑘̂ ½
⃗⃗⃗1 × ⃗⃗⃗⃗
|𝑏 𝑏2 | = √9 + 9 = 3√2

286
̂ )(−3𝑖̂ + 3𝑘
(̂𝑖 −3𝑗̂ −2𝑘 ̂) 1
∴Shortest distance = | |
3√2
−3−6 3
=| | =
3√2 √2
3√2
= units
2 1

(OR)

Let line through point (2, -1,3) is 𝑟 = (2𝑖̂˗𝑗̂˗3𝑘̂) + 𝜆′ (a𝑖̂ + b𝑗̂ + c𝑘̂)……….(i)
If line (i) is perpendicular to lines
𝑟 = (𝑖̂+𝑗̂-𝑘̂)+λ(2𝑖̂-2𝑗̂+𝑘̂) and 𝑟 = (2𝑖̂˗𝑗̂˗3𝑘̂)+μ(𝑖̂+2𝑗̂+2𝑘̂)

Then (a𝑖̂ + b𝑗̂ + c𝑘̂). (2𝑖̂-2𝑗̂+𝑘̂) = 0 ⇒2a -2b +c = 0 1+1


And (a𝑖̂ + b𝑗̂ + c𝑘̂). (𝑖̂+2𝑗̂+2𝑘̂) =0 ⇒a +2b + 2c = 0

𝑎 −𝑏 𝑐 𝑎 𝑏 𝑐
⇒ −4−2 =4−1 i.e −6 = −3 =
4+2 6

⇒a; b:c is -6 :-3 ; 6 or 2 : 1: -2

From (i),line is 𝑟 = (2𝑖̂˗𝑗̂˗3𝑘̂) + 𝜆′ (2𝑖̂ + 𝑗̂ -2𝑘̂) 1


SECTION – E
[ This section comprises of 3 case – study /passage based questions of 4
marks each with sub parts. The first two case study questions have three
sub parts(i),(ii) of marks 1, 1, 2 respectively. The third case study
questions has two sub parts of 2 marks each.]

Q36 A: Guesses B: Copies, C= Knows, E= Correct

i) P(A)+P(B)+P(C)=1
1 1
+ + P( C) =1
3 6 1
1
P( C) =
2

ii) There are four choices for guesses 1


1
P( A ) =
4
iii) P(B/C) = 1 1

OR

iii) using Baye’s theorem


1 1
𝑋
6 8
P(B/E)= 29
48
1

287
1
=29

i) Volume = x.x.h = 250 1


Q37 𝑥 2 ℎ = 250
ii) Cost ( C ) = 50 X 𝑥 2 + 400(ℎ)2

=50𝑥 2 +400(h)2 1

250
iii) C = 50 +400h2 1

12500 1
= ℎ +400h2

OR 2

iii) h = 2.5M

i) Clearly , G be the centroid of the triangle ABCD, there fore coordinates 2


Q38 of G are

3+4+2 0+3+3 1+6+2


( , , ) = ( 3,2,3)
3 3 3

ii) Since A = (0,1,2) and G = ( 3,2,3)

⃗⃗⃗⃗⃗⃗⃗⃗ ̂+(3 − 2)𝑘̂


𝐴𝐺 = (3 − 0)𝑖̂ +(2 − 1)𝑗𝑖
= 3𝑖̂+𝑗̂ + 𝑘̂
2
⃗⃗⃗⃗⃗ | =32+12+12
|𝐴𝐺
=11
⃗⃗⃗⃗⃗ |=√11
|𝐴𝐺 2

ZIET MYSURU YOUTUBE CHANNEL


https://www.youtube.com/channel/UCFcMLspE4JTudI9T16JwqZQ

288

You might also like